• Shuffle
    Toggle On
    Toggle Off
  • Alphabetize
    Toggle On
    Toggle Off
  • Front First
    Toggle On
    Toggle Off
  • Both Sides
    Toggle On
    Toggle Off
  • Read
    Toggle On
    Toggle Off
Reading...
Front

Card Range To Study

through

image

Play button

image

Play button

image

Progress

1/233

Click to flip

Use LEFT and RIGHT arrow keys to navigate between flashcards;

Use UP and DOWN arrow keys to flip the card;

H to show hint;

A reads text to speech;

233 Cards in this Set

  • Front
  • Back
  • 3rd side (hint)
May 2011 Question 1

(a) What is the role of dexamethasone in the management of postoperative nausea and vomiting? (70%)

(b) What are the potential problems associated with its use? (30%)
PART A

Post Operative Nausea and Vomiting occurs in 25-30%, and 12-26% of surgical patients respectively. Dexamethasone is a synthetic glucocorticoid steroid, with established use in the treatment of Post operative Nausea and Vomiting (PONV)
- Dose:
o Paediatric 0.15mg/kg (max 8mg)
o Adult 4-10mg. At 8-10mg, NNT = 4 for nausea, but lower doses equally effective. NNT = 7 for vomiting. Minimal benefit < 2.5mg
- Used for prophylaxis of PONV
o Shown to be as effective as 8mg Ondansetron, and Droperidol
o No evidence for use in rescue treatment – not recommended agent per 2006 Consensus Guidelines (Anesth Analg)
- Timing
o Shown to be more efficacious if given at start of case, compared to at end.
o Peak effect after several hours
o Duration of action = 24hours
• Prolonged antiemetic action more pronounced than ondansetron in late post operative period
• More effective in treating delayed post-chemotherapeutic induced N&V
o Repeat dose can be given after 8hours (or TDS), though 2006 Consensus Guidelines suggest not useful
- Unknown mechanism of action
o Postulated potentially via prostaglandin antagonism, 5HT3 antagonism in GIT, or endorphin release
- Cost:
o Extremely cheap compared to 5HT3 antagonists
o As evidence to suggest as effective and as safe as other antiemetics, can be considered as first line choice for prophylaxis in adults.
- Multimodal administration
o Synergistic action with 5HT3 antagonists, or Droperidol – combination useful for high risk patients (3 or more RF’s for PONV)

PART B)

Problems:
- Perianal/perineal pain post injection (not issue if given post induction
- Hyperglycaemia
o In diabetic patients, or those with Glucose intolerance.
o Not significant in non-diabetic
- Potential Side Effects (though no evidence to support)
o Immunosuppression
o Wound Infection

- However good safety profile
o Dose used for antiemetic treatment equal or lower to that used commonly in ENT
o no evidence of significant steroid side effects with once off injection (as opposed to chronic steroid use)
May 2011 Question 2

A patient with known idiopathic pulmonary fibrosis (fibrosing alveolitis) presents for an open right hemicolectomy.

(a) What are the respiratory issues facing this patient with regard to their general anaesthetic? (70%)

(b) Explain your intraoperative ventilation strategy. (30%)
PART A
• Pulmonary fibrosis is a heterogenous group of disorders affecting lung parenchyma which share similar pathological, physiological, clinical, and radiographic features.
• Charaterised by acute or chronic inflammatory changes in the pulmonary interstitium leading to decreased lung compliance, due to progressive replacement of normal lung parynchema with fibrotic connective tissue.
• Leads to restrictive pattern of pulmonary function tests (reduction in all lung volumes)
• Low O2 reserve and low lung compliance are 2 major factors influencing anaesthetic management
o Low O2 reserve due to decreased FRC, and V/Q mismatch predisposes to rapid hypoxaemia.
• Further exacerbated by supine position (10-15%↓), tracheal intubation (mild ↓), GA (5-10%↓)
o Low lung compliance
• PPV in presence of low compliance necessitates higher inflation pressures to expand stiff lungs → risk barotrauma, volutrauma, high intrathoracic pressure can impede CVS preload
• Other respiratory issues include
o Secondary pulmonary hypertension, and Right Heart Failure
o Concommitant infections

Pre-operative considerations
• Review by respiratory physician to optimize patient pre-operatively
o Steroid administration, antibiotic treatment, other immunosuppressive agents (cyclosporine)
o Pre-operative physiotherapy
o Lung Ix: to assess severity, and baseline function
• CT Chest: degree of lung infiltrates
• RFTs: ↓TLC or FVC (<50% predicted = severe), ↓DLCO (<60% predicted = severe)
• Potential for mixed obstructive, restrictive disease
o Life expectancy from ILD (median survival time 3yrs from diagnosis in ILD), and severity of disease vs. life expectancy from bowel Ca – should operation be done if high perioperative mortality risk.
• consideration of thoracic epidural to improve postoperative pulmonary function

Intra-operative
• Ensure good preoxygenation prior to induction to minimise hypoxia. Semi-recumbent to improve FRC
• Intubate and PPV given will need muscle relaxation for laparotomy.
• Surgical factors: laparotomy, surgical retractors can further decrease FRC, and splint diaphragm.
Post-operative
• Risk of hypoxia post extubation due to low FRC. Sit up to improve.
• May require supplemental O2.
• CPAP post op useful to maximse FRC
• Potential need for HDU post-op, or ICU if difficulty with extubation.
• Adequate analgesia to reduce pain induced atelectasis

PART B
• Adoption of “lung protection strategies”
o avoidance of high tidal volumes to prevent volu and barotraumas – TV 6-8mls/kg
o minimization of plateau airway pressures < 30cmH2O.
o Aim RR 10-12b/min. Accept permissive hypercarbia (esp. if mixed obstructive/restrictive pattern), however can increase RR to improve MV and observe gas trapping and auto-PEEP
o Avoidance of unnecessarily high FiO2, which can cause acute exacerbation of pulmonary fibrosis. Aim O2Satn 88-94%
o Avoidance of high PEEP (>10cmH2O) → a/w lung overdistension, and worse outcome in ICU patients with ILD. Patients with ILD have little recruitable lung, and thus high PEEP not beneficial, and increases risk Ventilator induced Lung Injury (cf different to ARDS, where PEEP useful)
o High I:E ratio (>1) – increase I time allows for longer lung expansion to compensate for decreased compliance. However need to ensure no auto-PEEP.
May 2011 Question 3
Explain the professional attributes of an anaesthetist in specialist practice.
Scholar and teacher
• Values advances in scientific knowledge; identifies and appraises them critically for incorporation into contemporary anaesthesia practice.
• Enriches knowledge with wisdom.
• Maintains life-long learning.
• Acknowledges and learns from errors; values appraisal of performance.
• Contributes to the education and training of students, postgraduate trainees and other health professionals.

Professional
• Recognises that patient confidentiality is essential.
• Complies with the relevant policies, recommendations, and guidelines in professional practice as contained in ANZCA professional documents (see appendix).
• Exhibits appropriate personal and interpersonal professional behaviours.
• Values human diversity.
• Delivers high quality patient care in a way that is consistent with ethical and medicolegal obligations of a medical specialist.
• Conducts practice with integrity, honesty and compassion.
• Accepts peer determinations of clinical competence and professional capabilities.
• Recognises and deals with personal and professional limitations.
• Demonstrates exemplary practice as a member of a multidisciplinary team, by exercising flexible leadership, consultation and appropriate delegation.
• Shows respect for the expertise and concerns of other team members.
Professionalism and ethics
To commit to, and believe in the ethical and professional principles of:
o Altruism: the best care for the patient must be the principal driving force of practice.
o Patient autonomy: patients’ ability to determine their treatment.
o Beneficence: the principle of “doing good” to patients.
o Non-maleficence: the principle of not doing harm to patients.
o Fidelity: faithfulness to one’s duties and obligations. This principle underlies excellence in patient care, confidentiality, telling the truth, a commitment to continuing professional development and lifelong learning, and not neglecting patient care.
o Social justice: the right of all patients to be fairly treated.
o Utility: the principle of doing the most good for the greatest number of people.
o Duty to oneself in terms of personal health care and maintenance of competence to practise.
o Accountability: the anaesthetist is responsible for his or her actions.
o Honour and integrity in all conduct, including the generation and use of resources.
o Respect for others, including a responsibility to work as a team and to practise conflict resolution.
o Appropriate response to clinical error.
May 2011 Question 4

Evaluate the use of human albumin in perioperative volume replacement.

11% of candidates passed this question
Key components of a response to this question included:
o
oproperties of albumin and its role in volume replacement
o advantages and disadvantages of its use
o evidence (and its quality) for use over other solutions
o mention of SAFE study/meta-analyses
o
Many candidates wrote very little in answer to this question.
Consideration of the perioperative settings where volume replacement is required formed a reasonable starting point for good answers e.g. cardiac surgery, obstetrics, burns, liver transplant, trauma, burns, sepsis.
Use of albumin : an update
May 2011 Question 5

(a) How would you clinically assess a patient complaining of leg numbness the day after a spinal anaesthetic for an emergency caesarean section? (70%)

(b) How would you manage the situation? (30%)

77% of candidates passed this question

Key components of a good response to this question included:
Part (a)
o consideration of the likely causes (neurapraxia from various causes – obstetric or anaesthetic) as well as more rare causes (spinal haematoma/abscess/other)
o pre-existing neurological condition/contributory conditions (diabetes)
o obstetric history
o spinal insertion (difficulty/number of attempts/sterile technique interruption)
o history of labour
o evolution of symptoms/timing
o examination of drugs used/insertion site/vital signs/systemic symptoms
o goal-directed neurological examination to distinguish different diagnoses: e.g. lumbo-sacral neurapraxia, single nerve neurapraxia
Part (b)
o management depends on the diagnosis: history and clinical examination should identify a neurapraxia or point to a more serious cause
o neurapraxia should manifest progressive resolution that might take weeks
o MRI will identify neuraxial cause IF this is indicated by history and examination
o involvement of a neurologist and/or neurosurgeon depends on the diagnosis and its likely cause.
A structured approach was an important aspect of good answers for this question.
PART A
PART B
May 2011 Question 6

A 60-year-old man is booked for plating of a fractured ankle. He arrests on induction. His ECG shows ventricular fibrillation.

Outline the immediate management of his cardiac arrest with particular reference to current resuscitation guidelines.

55% of candidates passed this question

The current (2010) Australian or New Zealand resuscitation guidelines for this situation formed the basis for the answer to this question. A high standard was expected from Fellowship candidates.

A flow diagram that simplified the presentation of information for this question was used by some candidates.
Resusitation Guidelines
May 2011 Question 7
A patient is scheduled for posterior fossa surgery in the sitting position.
(a) Outline the precautions you would take to minimise the risk of venous air embolism. (70%)

(b) How would you recognise an air embolism intraoperatively? (30%)
PART A

VAE can occur whenever pressure within an open vessel is subatmospheric. Clinically significant VAE is unusual unless the surgical site is >20 cm above the level of the heart. Hence, VAE is a particular problem during surgery in the seated position. (They can also occur in the lateral or prone position.)
Most studies indicate that the incidence of VAE during posterior fossa procedures in the sitting position is 40% to 45%. (For seated cervical laminectomy or surgery in the prone or lateral positions, VAE occurs in approximately 10% to 15% of cases.)

Prevention of VAE

1) Positive venous pressure at surgical site
(a) Positioning : discussion with surgeon regarding tolerance of lower head positions
(b) Normovolaemia : Although hypovolemia has been proposed as a predisposing factor for VAE, evidence for a prophylactic effect of volume loading on the incidence of VAE and PAE is not strong enough to warrant its routine use. Adequate hydration is the goal.
(c) Avoidance of venodilation : drugs such as GTN
(d) PEEP : The use of PEEP to prevent VAE in the sitting position is controversial. High levels of PEEP (>10 cm H2O) are needed to increase venous pressure at the head, and studies are inconsistent as to whether PEEP decreases the incidence of VAE.
(e) jugular venous compression during periods of high risk
PEEP can, however, reduce venous return, cardiac output, and mean arterial blood pressure, and increase the risk of paradoxical VAE

2) Increasing transverse and sigmoid sinus pressure
(a) positioning
(b) mild hypoventilation : While some studies suggest that moderate hypoventilation may reduce the risk of VAE, hypoventilation also increases cerebral blood flow and cerebral blood volume, which may impair surgical exposure. Until the benefits of hypoventilation are confirmed, mild hyperventilation is the more common practice.

3) Surgical technique
(a) bone wax/ saline gauze and packs
(b) fastidious haemostasis
(c) venous circulation should be left open for the least amount of time

4) Early recogition
(a) bubbling at the sight : close communication between surgeon and anaesthetist
(b) monitoring : CVC/ direct arterial BP/ ETCO2 / TOE/ Doppler
May 2011 QUESTION 8

(a) Describe the anatomy, including surface landmarks, relevant to performing cricothyroidotomy. (50%) FIX INDENT

(b) What are the complications of this procedure? (50%) FIX INDENT
PART A
Cricothyroidotomy is one of the several emergency airway management techniques, used primarily for ‘can’t intubate, can’t ventilate’. It is a short term solution which provides oxygenation, not ventilation and is not a definitive airway.
In the adult the cricothyroid membrane is approximately 10mm in height and 22mm in width and is composed of mostly yellow, avascular elastic tissue. Attaching the posterior border of the thyroid cartilage to the superior border of the cricoid cartilage.

Adjacent structures include the overlying thyroid isthmus and superior thyroidal artery. The cricothyroid membrane is bordered by the cricothyroid muscle on either side. Lateral to the membrane are the venous tributaries from the inferior thyroid and anterior jugular veins. The vocal cords lie approximately 1 cm above the cricothyroid space, and therefore are not usually damaged during cricothyroidotomy.

Palpation of the cricothyroid membrane can be made easier by extending the head and neck, and it is felt as a slight dip below the thyroid cartilage. Run a finger down the front of the neck in midline and find the notch in the upper border of the thyroid cartilage. Below this is a depression between the thyroid and cricoid cartilages - the cricothyroid membrane.

PART B

Complications can be immediate, early or late.

IMMEDIATE
Hypercarbia
Hypoxia
Subcutaneous or medistinal emphysema
Needle displacement or kinking causing respiratory obstruction
Air embolus (insufflation into a vessel)
Haemorrhage
Oesophageal or medistinal perforation

EARLY
Hypercarbia
Pneumothorax
Vocal cord injury
Larygenal disruption
Aspiration

LATE
Tracheal and subglottis stenosis
Swallowing dysfunction
Tube obstruction
Tracheoesophageal fistula
Infection
Bleeding
Persistent stoma
Tracheomalacia
MAY 2011 QUESTION 9

You hear a cardiac murmur in a two-year-old child presenting for elective minor surgery.
(a) What are the features of the murmur that would differentiate an innocent from a pathological murmur? (50%)

(b) How would you evaluate this child’s fitness for anaesthesia from the cardiac perspective? (50%)
PART A

HISTORY INNOCENT
Asymptomatic : exercise, playing
Unremarkable perinatal history
No family history of cardiovascular problems, genetic conditions

HISTORY PATHOLOGICAL
Symptomatic
o Failure to thrive
o Feeding intolerance
o Syncope, spells
o SOB
o Playing
o Any positional symptoms
Perinatal history: premature, maternal diabetes, foetal distress, birth asphyxia
Syndromal
Family history of HOCM, congenital cardiac disease, sudden death
Respiratory symptoms, repeated chest infections

EXAMINATION INNOCENT
No signs of severity

Auscultation
-Early systolic
-soft
-crescendo-decrescendo
-changes with position

• Examples:
• Still’s murmur – most common, ‘musical or vibratory, groaning, squeaking, creaking, rasping’, 1 - 3/6, no thrill, inside apex or low parasternal, but may be over whole precordium, softer or disappear on standing, reappears on squatting
• Pulmonary ejection murmur – systolic, high-pitched ‘blowing’, second left interspace, may be heard at the apex, left sternal border, aortic area and neck, normal splitsecond heart sound during inspiration (not expiration)
• Supraclavicular arterial bruit (rare)
• Late systolic cardiorespiratory murmur (rare)
• Continuous murmur (venous hum) – systolic and diastolic hum, loudest in sitting during inspiration, disappears or diminishes in the supine position or with pressure over the supraclavicular

EXAMINATION PATHOLOGICAL
General examination
-thin, cachetic, cyanosed

SO2 indication of significant cynaosis of inadequate pulmonary blood flow, right to left shunt or mixing

Signs of severity
o Praecordial: thrills, heaves, displaced apex, added heart sounds.
o Loud
o Diastolic, pansystolic or late systolic
o Central: neck and chest
o Peripheral: never forget femoral and brachial pulses
• Radio-femoral delay
• Brachio-brachial delay

Palpation : precordial activity : ASD, moderate to large VSD, significant PDA

Murmur at lower left sternal border : VSD, TR, HOCM, subaortic stenosis

Murmur at left upper : pulmonary valve stensosis, ASD, PDA

Murmur at suprasternal notch : aortic stenosis

Murmur at apex : MR

Timing of brachial and femoral pulses should be equal in timing and intensity and pressure to exclude aortic coarctation

INVESTIGATIONS
Polycythemia
Coagulopathy
ECG : conduction defects, arrythmias, LVH, RVH
CXR : cardiomegalaly, pulmonary lung marking, consolidation
ECHO : HF, left to right shunt, right to left SHUNT

PART B
Michael Clifford’s flow chart for assessing murmurs in the child
• You hear a murmur
• If it is an emergency case then proceed. If not, is it diastolic?
• If yes, does the murmur go away when they lie down? If yes, proceed because not significant. If no then defer the case and refer to a paediatrician/cardiologist
• Other murmurs you need to exclude the 3 B’s
o Blue (cyanosis)
o Breathing fast (CCF)
o Bad signs to feel
• Pulse (femoral)
• Bounding (L -> R shunt)
• Low (LVOTO, stenosis)
• Low/Delayed (coarctation of the aorta)
• Praecordium
• Thrill (always indicates pathology, grades 4 - 6)
• Apex (displaced equals big heart and is a sign of severity)
• Peripheral
• Liver
• Peripheral and sacral oedema (kids lie down a lot)
• If there are no B’s and the child can play etc. then proceed
• Another tool is the S’: If the murmur was Short. Soft, heard over a Small area, without added Sounds, and the child is not Syndromal, then you can proceed Safely to Surgery.
• YOU SHOULD ALWAYS TRY TO GET A PAEDIATRIC/CARDIOLOGICAL CONSULT AND PERFORM BASIC INVESTIGATIONS PRIOR TO PROCEEDING. Put the kid on the end of the list, and get a CXR, ECG and arrange a consult. If you can’t get a consult or an echo that morning, you are probably safe to proceed as long as follow up is arranged.



• If the surgery is dirty then even innocent murmurs should have antimicrobial prophylaxis because it is so difficult to exclude a structural abnormality clinically
o Dental
• Amoxicillin (or clindamycin if allergic) at induction and 6 hours later
• High risk patients with prosthetic valves, recent courses of penicillin (resistance), PHx of endocarditis should have amoxicillin (2 doses) and gentamicin, vancomycin if allergic
o Upper airway instrumentation
• As above
• Some say that laryngoscopy produces quite a marked bacteraemia of its own
o Genitourinary
• Amoxicillin and gentamicin (must cover enterococcus)
o GIT
• As for prosthetic heart valves
May 2011 Question 10.

What factors contribute to acute kidney injury in the perioperative period? (70%)

Outline the efficacy of perioperative strategies to reduce acute kidney injury (30%).
PART A

Acute kidney injury (AKI) is a relatively common cause of perioperative morbidity and mortality and depends on a number of Patient, Anaesthetic and Surgical factors.

Patient
- Pre-existing renal impairment / Renal failure
- Hypovolaemia
- Anaemia
- Concurrent administration of nephrotoxic agents, eg. NSAIDs, diuretics, antibiotics, ACE inhibitors.
- Associated systemic diseases eg. DM, HTN, IHD, CCF, PVD.

Anaesthetic
- Fasting and inadequate volume assessment / resuscitation pre-operatively.
- Volatile anaesthetic agents reduce GFR and RBF (dose-dependant)
- Inadequate BP (eg. hypotension secondary to anaesthetic induction agents) and thus perfusion pressure.
- Inadequate replacement of blood loss to impair oxygen delivery.
- Use of nephrotoxic agents as above, also contrast agents, intra-operative antibiotics eg. Gentamicin, Vancomycin.
- Neuraxial Anaesthesia above T4 as reduces sympathetic tone to the kidney.

Surgical
- Direct trauma to Kidney or renal vasculature.
- Major surgery including major vascular and renal procedures.
- Aortic X-clamp

PART B

Strategies to reduce AKI:
1. Treat Risk Factors and causes – ensure adequate volume status, MAP, CVP. Optimise systemic disease and exclude obstruction.
2. Maintain O2 delivery to the Kidney via volume (CVP 10-12mmHg) and fluid balance, maintain cardiac outpute, PaO2 and Hct > 30%.
Intravascular volume repletion and augmentation of RBF by maintaining MAP and CO is the only effective therapy in abating perioperative renal insufficiency and is the most effective management of periop oliguria.
3. Avoid Nephrotoxins – reduce direct nephrotoxic effects.
4. Pharmacological Agents:
- Dopamine
- Mannitol
- Frusemide
- ANP
- Ca Channel Antagonists
- NAC – other than NAC for preventing contrast-induced nephropathy, no drugs have been shown to be clearly beneficial in preserving ReFn.
- Other: Surgical interventions eg shunt placement, Renal cooling.
May 2011 Q11.

Describe the clinical pharmacology of Codeine including an outline of its therapeutic use (70%)

Describe the influence of pharmacogenetics on the variability of patient response to Codeine (30%)
Codeine or 3-methylmorhpine is a weak opioid agonist (10 times less potent than morphine). It is an analgesic agent used for mild to moderate pain, with antitussive and antidiarrheal properties.

Codeine is available both as a sole agent, and in combination with other simple analgesics such as Paracetamol and NSAIDs. It is available in tablet form, elixir and in injectable form for subcutaneous and IM injection. Oral dose is 30-60mg up to 4 times per day.

Pharmacodynamics
Codeine is considered a prodrug as it is converted to active compounds morphine (5-10%) and codeine-6-glucuronide in vivo. The active components act via opioid receptors and G inhibitory proteins to hyperpolarize cells, reducing cAMP and inhibiting neurotransmitter release between cells and propagation of pain impulses.

Its primary use is as an analgesic agent but it is also present in cough mixtures as an antitussive and occasionally utilized for it’s anti-diarrhoeal properties.

Respiratory: potential respiratory depressant in high doses, or in combination with more potent analgesic agents. Reduces brainstem sensitivity to CO2 and dose-dependant reduction in respiratory rate. Potential for precipitating histamine release → bronchospasm.

The most frequent side effects of codeine include lightheadedness, dizziness, nausea, vomiting, shortness of breath, and sedation. Other side effects include allergic reactions, constipation, abdominal pain, rash and itching. Codeine is potentially addictive and mental and physical dependence can occur.

Pharmacokinetics
Codeine is well absorbed orally with a bioavailability of 50%. It is 90% plasma protein bound and metabolized to morphine (O-Demethylation) in the liver, cataylsed by Cytochrome P2D6. Cytochrome P3A4 produces Norcodeine and codeine, Norcodeine and Morphine are also broken down to their corresponding 3- and 6-Glucuronides. 5-15% is excreted unchanged in urine and Codeine has a half-life of 3 hours.

Approximately 6-10% of the Caucasian population, 2% of Asians are “poor metabolisers” as they possess little CyP2D6. Codeine is therefore a less or minimally effective analgesic in these patients. It will however provide some analgesic effect via conversion of codeine to codeine-6-glucuronide. Conversely, 0.5-2% of the population are "extensive metabolizers"; multiple copies of the gene for 2D6 produce high levels of CYP2D6 and will metabolize drugs through that pathway more quickly than others.
May 2011 Question 13

You are involved in the planning of a new Day Surgery Unit.

a) What systems would you put in place to reduce the likelihood of a power failure? (50%)

b) Outline a protocol for dealing with power failures. (50%)
PART A
Design including
- Multiple sources of power
o Regular power line/ Mains
o UPS (uninterruptible power supply)
• Provides near instantaneous power supply in case of failure of mains attached to back up battery or diesel generator.
o Back up generator

- 2 types of electrical outlets (color coded)
o Ordinary ones: connected to mains/ regular power supply
o Red face plates that are “emergency” electrical power outlets: connected to mains normally but also connected to UPS & back up emergency generator

- Sensor that detects fail in the mains/ regular power line and that can trigger both a transfer switch and activation of an internal emergency generator.

- Essential equipment e.g. ventilators, anaesthesia machines, patient monitors and infusion pumps should be connected to the emergency red, safety theoretically uninterruptible electrical outlets.

- System Education: should be included regarding the limitations of supply, duration, reliability, and for example to time back up generator will last.

PART B

Protocol for dealing with power failure
- Written protocol and education for staff regarding back – up capabilities and essential emergency contacts in case of power failure including:
o Prioritization of power use in OT
o Not starting any new/ elective cases whilst on back up generator
o Adequate lighting for general function
o Decision by operating team, surgeon, anaesthetists on whether to continue operating or cease operation as quick as possible
o Incase of failure of back up generator
• Prior training of staff for this scenario
• Flashlights available near every anaesthetic machine
• Awareness that the anaesthesia machine actually powers
• Readily available non electrical equipment e.g. manual BP cuff, stethoscope

- Intelligence regarding internal battery supply/ UPS usage and available supply
- 1 or 2 Expert in the department with deeper interest and have extended knowledge f the utility systems and power failure protocols whom will liaison with facility engineers.
- Designated lines of communication with all areas regarding the evolution/ resolution of power failure (e.g. power failure specific phones – in readily accessible areas – theatre/ main recovery)

- ANAESTHESIA MANAGEMENT
o Convert to spontaneous ventilation where possible
o Consider battery powered TIVA
o Knowledge amongst anaesthetists and trainees regarding independent power supply & alternatives to run essential equipment
May 2011 Question 12

(a) What are the complications associated with residual neuromuscular blockade? (30%)
(b) Evaluate the methods available to assess residual neuromuscular blockade. (70%)
55% of candidates passed this question
Key components of a response to this question included:
Part (a)
o complications such as airway obstruction/hypoxia and its causes/aspiration/increased length of stay in PACU/hospital
Part (b)
o clinical tests (head lift/grip strength/tidal volumes) and their reliability
o tactile/mechanical methods: levels of discrimination and limitations
o objective monitors – accelerometers/force transducers
o sensitivity of various methods
PART A
There is increasing evidence that residual neuromuscular block is common. A study by Debaene found that 45% of patients had residual curarization (TOF <0.9) (note: it is now thought that significant curarization is still present if TOF <0.9 rather than 0.7 which was previously though)

COMPLICATIONS
• Respiratory
o Airway obstruction
o Aspiration
o Hypoventilation (Increased PaCO2, Pulmonary HT, Increased ICP, Atelectasis/ hypoxia (impairs ventilatory response to hypoxia), Consolidation/collapse - pneumonia
o Increase PACU/hospital stay/ cost
o Increase hospital stay
o Distressed recovery staff : Psychological
o Distressing experience for patient
o Long term PTSD : Repeat reversal complications
o Potential for bradicardias with second neostigmine dose
(Note: no evidence that leads to increased mortality)

PART B

EVALUATION METHODS
Bedside clinical tests
• Head lift – sustained for 5 seconds, said to be most sensitive
• Hold tongue depressor though teeth
• Grip Strength
• Tidal volumes – not a very good measure as a substantial degree of paralysis may be present even with normal tidal volumes.

Tactile mechanical methods
• DBS (Double Burst Suppression):
2 short bursts at 50Hz at supra-maximal current last 0.2 s each and separated by 750ms – fade when second twitch less than first indicates residual blockade
Measured tactile – more sensitive than TOF measure tactile & better for small degrees of residual blockade.
• TOF (Train of Four):
4 supramaximal stimuli – frequency 2 Hz at 500ms apart. Look for decrement in height from T1 to T4, fade and TOF ratio T1:T4. Main use is for determining when patient is ready for reversal (TOF count at least 3) or depth of NM blockade intraoperatively rather than for residual blockade in recovery.

Overall these are crude – influenced my many other factors such as inability to follow command or residual sedation. Note even most experienced anaesthetists are able to detect TOF > 0.4

Disadvantage with all these nerve stimulation techniques is painful for patient in recovery!

Qualitative measure

Mechanomyography (MMG): measurement of evoked muscle tension. E.g. stimulate ulnar nerve at wrist and observe movement in adductor pollicus. Thumb stabilized with fixed tension and strain gauge transducer measures change in tension giving an electrical signal. This is gold standard & can be used for any pattern of nerve stimulation (including Double Burst ,TOF, PTC and single twich)
Advantage:
• Gold standard, most accurate
Disadvantages:
• cumbersome & impractical in operating theatre
EMG (electromyography): Evoked action potentials are a measurement
of electrical changes that occur in muscle during stimulation;
it is assumed that these are equivalent to the muscular contraction that occurs after excitation–contraction coupling.

Advantages:
• Equipment not as bulky as MMG
• Arm does not need to be fixed as rigidly
Disadvantages:
• Prone to interference (esp diathermy)
• Hand temperature and movement effect results more so than with MMG

Accelerometers (acceleromyography):
Similar to MMG but acceleration rather than force of contraction is measured. Developed as a more convenient method to MMG that can be used in theatre.

Advantages:
• More convenient for use in theatre (less bulky)
Disadvantages:
• Can only be used for TOF or PTC (not DBS)
May 2011 Question 14

a) Describe the abnormality on this electrocardiogram. (30%)

b) What are the implications of this abnormality for anaesthesia? (70%)
PART A

Prolonged QT – not accounted for by the prolongation of the QRS complex duration.

QT interval: Measure of time between the start of the Q wave and the end of the T wave. Varies with HR (lengthening with bradycardia and shortening with tachycardias)

Corrected QT (QTc): QTc= QT/√ RR
Normal = 0.3 to 0.44 (>044 = prolonged.)

PART B

LONG QT ANAESTHESIA IMPLICATIONS
Anaesthesia is potentially hazardous as they are at significant risk of developing malignant ventricular arrhythmias including polymorphic VT (torsades de pointes) or VF.
Long QT may also be unmasked by anaesthesia
Pre-operative:
- Assessment – history: including episodes of syncope, pseudoseizures, family history of sudden death & drugs that may increase QTc
- Cardiology review:
o consider ß-blockers
o need for ICD – if difficult to control
o 12 lead ECG – where ideally a valsalva manouvre should not illicit any increase in QTc if adequately beta blocked.
- Check all electrolytes & correct and Mg2+, Ca2+, or K+ abnormalities.
Intra-operative:
- Emergency equipment & drugs for cardiac resuscitation ready – including defibrillator, pads & trans-venous pacing
- Full invasive monitoring and 5-lead ECG for more major surgery
- Avoid Excessive sympathetic stimlation
o Consider pre-operative sedative
o Blunt pressor response to intubation with opiod
o Avoid hypercarbia & it’s associated sympatho-adrenal response
o Avoid sympathomimetic drugs where possible including Ketamine
- Beware drugs that prolong QTc
• General
• E.g. Droperidol, Amiodarone, Sotalol, Flecainide, Erythrocmycin
• Volatiles:
• Note: although volatiles (E.g. sevolurane) have been shown to increase QTc – they have no effect on Tp-e interval (interval from peak to end of T wave – which is a much better predictor of drug torsadogenity) therefore highly unlikely to precipitate torsades
• Consider TIVA
• Muscle relaxants
• Avoid Suxamethonium as may prolong QTc
• Atracurium and Vecuronium safe
• Reversal (avoid where possible)
• Anticholinergics: atropine & glycopyrrolate both prlong QTc as does neostigmine-glycopyrrolate comination
- Avoid Hypothermia
• Prolongs the QTc
• Forced air warmer/ warm fluids and monitor temperature
- Avoid High intra-thoracic peak pressures
• As may increase QTc in inadequately beta blocked patients
- Avoid Hypotension
• Consider judicious preloading
- Regional
o Epidural and spinal are safe

Post operative
- ECG monitoring for 24 hours for more major surgery
- Adequate analgesia
- Nurse in calm quiet environment
May 2011 Question 15

a) How would you identify a patient with autonomic neuropathy associated with diabetes? (50%)

b) What are the anaesthetic implications from a cardiovascular perspective? (50%)
PART A

Important to recognize Diabetic Autonomic Neuropathy is a multi-system disorder & identify patients based on a history, examination, reflex test approach
Systems affected are:
- Gastro-intestinal system:
o Gastroparesis, aspiration risk, may be asymptomatic
o GORD
o Constipation/ diahrroea/ faecal incontinence
- Metabolic:
o hypoglycaemia unawareness
o hypoglycaemia unresponsiveness
- Cardiovascular system:
o Postural hypotension – history of fainting, measure postural BP (>30mmHg = abnormal)
o Asymptomatic myocardial ischemia/infarction
o Exercise intolerance
• Due to impaired sympathetic and parasympathetic responses that normal augments CO
o Abnormal HR
• Lack of HR variability during deep breathing or exercise (experation:inspiration raito > 1.17 is abnormal)
• Resting tachycardia (early sign)
• Prolonged corrected QT – indicates imbalance between right and left sympathetic innervation
- Thermo-regulatory:
o Reduced thermal sensitivity (very early marker of diabetic neuropathy
o Heat intolerance
- Neurological:
o Other neuropathies
• Charcot’s joints, neuropathic ulcers
• Symmetrical anhydrosis (inability to sweat)
• Hyperhydrosis
o Abnormal pupils
• Decreased diameter of dark adapted pupils
• Argyll- Robertson type pupil
- Genitourinary:
o Erectile dysfunction
o Retrograde ejeculation
o Atonic bladder

Use of provocation/reflex tests:
- Valsalva manouvre: forcible exhalation against mouthpiece with pressure of 40mmHg for 15 seconds. A ratio of longest to shortest R-R interval of less than 1.2 is abnormal.
- Tilt table testing
- cystometry
Diabetic neuropathy is a diagnosis of exclusion and other differentials of autonomic dysfunction should be ruled out – including cancer, drug/alcohol use , HIV, amyloidosis.

PART B

Cardiovascular anaesthetic implications
Beware of:
- poor cardiac function
- impaired reserve
- masked cardiac ischaemia
- exaggerated HR/ BP responses to stimuli/ drugs
- prolonged QTc

Pre operative
- in addition to assessment above consider tests:
o TTE – looking for impaired cardiac function
o Cardiac angiography / stress test if concerned about ischamia
Intraoperative
- Induction:
o Monitoring – consider arterial line, 5 lead ECG
o Avoid pressor response of intubation (opioid)
o Emergency drugs available
o Notify senior staff
- Intraoperative
o Beware unreliable, exaggerated HR/BP response to stimuli & drugs and where possible use of other indicators rather than heart rate/BP for monitoring: volume status, pain, depth of anaesthesia
• E.g. BIS, IDC to monitor UO, keep well on top of pain
• CVC for measuring fluid balance

Anaesthesia – careful of cardiac function
- Arterial line
- Central line
- Close monitoring
- 5 lead ECG
Q250 1997A12.

A 75 year old man requiring transurethral resection of the prostate takes a diuretic and beta blocker for hypertension. Spinal anaesthesia with 0.5% bupivacaine (plain) results in a sensory level of T10.

After 70 minutes of surgery the patient becomes restless. Explain your management.
Most likely cause is water intoxication syndrome but must exclude:
1 Drugs (i.e. XS sedation either pre- or intraop, premedication, alcohol, or recreational drugs).
2 Hypoxia (because head down, ? pulm oedema from over infusion therefore hypoxia)
3 Hypotension induced decrease in cerebral perfusion therefore confusion.
4 Block has worn off or patchy block, in pain; excision past prostatic capsule therefore pain
5 Posture related discomfort; ? chronic back pain
6 Hypothermia
7 Pre-op limited neurological function i.e. dementia pre-op
If above excluded and treated as appropriate then assuming H2O intoxication - best management is prevention i.e. via
1 Limited duration of resection <60min
2 Limited by diastolic pressure <50-70cm H2O of irrigation fluid
3 Appropriate irrigation fluid (1.5 % glycine is hypotonic)
4 Experienced surgeon
5 Absorption of fluid is 10-30 ml/min of irrigation fluid

Management
1 Notify surgeon - cease surgery as soon as possible
2 O2 to patient
3 Decrease height of irrigation fluid
4 Reposition patient to neutral
5 Send blood for Na+ and K+ and osmolarity

If patient not too restless and stable CVS and respiratory parameters - await results of biochem.

If > 130 mmol - N/Saline and cease operation

If Na+ 121 mmol - 130 mmol - use N/Saline, ceasing surgery and frusemide 10 - 20 mg will suffice, and observation.

If Na+ 110 - 120 mmol - use 3% N/Saline max. 3 ml/kg important give slowly because of risk central pontine myelinolysis syndrome (no ³ 0.5 meq/l per hour change). Use frusemide, O2 and monitor. If profound confusion - secure airway with ETT and treat as above.
October 2010 Question 1

(a) What are the clinical consequences of hypothermia to 34⁰C in adults? (50%)

(b) How can you manage body temperature in a multi-trauma patient? (50%)
PART A
Physiological changes include:
1. behavioural changes (eg adding clothing, moving to a warmer environment etc)
2. vasoconstriction
3. shiverin
4. impaired immune function and platelet dysfunction.

Clinical consequences include :
1. impaired wound healin
2. altered drug metabolism (eg prolonged muscle relaxation),
3. possible cerebral protection in certain circumstances
4. increased sympathetic tone with hypertension and tachycardia
5. the consequences of shivering (eg on metabolic rate and myocardial oxygen demands, difficulties with monitoring, haemodynamic instability and patient discomfort) particularly in the postoperative period.


PART B

Measures to limit heat loss include
1. preoperative “prewarming”, increasing the operating room temperature
2. forced air warming blankets
3. warming fluids
4. wrapping/covering body parts
5. limiting wound evaporative losses
6. heat and moisture exchangers (HMEs) or humidification of the breathing circuit
October 2010 Question 2.

(a) Describe the arterial blood supply of the spinal cord. (50%)

(b) Why is spinal cord function at risk during openrepair of a thoracic aortic aneurysm and what measures are available to reduce this risk? (50%)
PART A

Blood is supplied to the spinal cord via THREE arteries:
1) One Anterior [median] spinal artery
a. Located along the anterior midline of the cord
b. Supplies most (75%) of the arterial blood supply to spinal cord

2) Two smaller Posterior spinal arteries
a. Lie on each side of the cord posteriorly
b. Supplies only 25% of the arterial blood supply to posterior region of spinal cord

* Arteriolar sized vessels from both the anterior spinal & both posterior spinal arteries encircle the spinal cord forming a fine pial plexus —> these vessels constitute the vasocorona.
* The vaso corona provide a limited anastomosis between the longitudinal vessels, however if the major anterior radicular artery is occluded, there will not be sufficient blood supply to the distal two-thirds of the spinal cord
* Various Radicular arteries supply the anterior & posterior spinal arteries along the spinal canal:

Cervical Region:
· spinal arteries receive blood from:
1)vertebral arteries
2)posterior inferior cerebellar arteries
3)segmental branches

Thoracic & Lumbar region
· spinal arteries receive blood from Radicular arteries
· Radicular arteries are the anterior & posterior Radicular branches of the spinal branch of the dorsal branch of the posterior intercostal artery
· In this region, have only a few radicular arteries supplying the cord vessels (ant & post spinal arteries), therefore reserve blood supply is precarious. Between C8 & T9 only two small radicular branches supply this long segment of the cord.
· The arteria radicularis magna (major anterior radicular artery) is the principle arterial blood supply of the lower two-thirds of the spinal cord (usually found between T11 & L3).

Venous Drainage of the Spinal Cord
* This is by 6 irregular, plexiform channels.
-3 anterior and 3 posterior which run longitudinally
* There is one along:
1. The anterior and posterior midlines;
2. Along the line of attachment of the dorsal roots of each side;
3. Along the line of attachment of the ventral roots of each side.

* These are drained by the radicular veins.
* Each, in turn empty into the epidural venous plexus.
*communicates superiorly with the venous sinuses of the cranium and drains by multiple communications into the vertebral and ascending lumbar veins and thence into the azygos and hemiazygos veins.

PART B
(b) Aetiology of risk to cord function and measures to reduce risk:
 Aortic cross-clamping during thoracic aortic surgery potentially sacrifices the segmental arteries arising from the descending aorta
 Methods to reduce this risk include:
o minimisation of cord ischaemia (eg minimisation of cross clamp time, institution of lower body cardiopulmonary bypass or reimplantation of segmental arteries)
o increasing tolerance to ischaemia (eg mild systemic hypothermia, deep hypothermic circulatory arrest, epidural cooling or pharmacological neuroprotective measures)
o augmentation of spinal cord perfusion (eg lumbar CSF drainage or deliberate hypertension)
o spinal cord monitoring with evoked potentials with a plan for intervention if abnormalities are detected
October 2010 Question 3.
A 45-year-old man with a longstanding history of alcoholism is booked for upper gastrointestinal endoscopy and banding of oesophageal varices following an episode of haematemesis.

(a) How is the severity of this patient's liver disease assessed? (50%)


(b) How do these findings influence your evaluation of this patient‟s perioperative risk? (50%)
TBA
October 2010 Question 4.
A 68-year-old man in hospital awaiting definitive surgery for a supraglottic squamous cell carcinoma of the larynx has worsening stridor at rest.

(a) How might his symptoms be improved in the preoperative period? (30%)

(b) Describe your evaluation of his airway and how this will influence your intraoperative airway management plan. (70%)
a) Pre-op measures to relieve fixed airway obstruction
• Position head up/sit up
• Reduce oedema associated with tumour by nebulised adrenaline (5mls 1:1000); steroids (IV/neb)
• Give Heliox (79% He, 21% O2) = low density gas, decreases turbulence of air flow past obstruction
• Treat potential superimposed infection
• CPAP keeps open small airways behind obstruction

b) Evaluation of airway and plan
Hx:
• Stridor implies airway diameter reduced by 50% (or <4-5mm)
• Duration/onset of stridor, exercise tolerance
• Mass effect Sx – worse supine (sleeping), swallowing, phonation
• Treatment (surgery, radiotx)
• Previous anaes chart: Grade intubation, problems intra-op/recovery
Ex:
• Resp distress at rest, ability to speak (exhaustion/obtunded = emergency!)
• Routine airway Ax: Mouth opening, dentition, MP score, neck extension
• If visible or palpable, assess mass/lymph nodes – size, mobility, location
Ix:
• Nasendoscopy by surgeon – visualised cords/larynx?
• CT/MRI/neck or CXR – size, location (supra/glottis/sub), involvement of other structures
Mx:
Induction:
Issues in securing airway access are:
1) Worsening airway obstruction by lying flat/instrumenting larynx/GA (all techniques!)
2) Anatomical distortion (esp supraglottic) – can be hard to identify laryngeal inlet
3) Severe stenosis – hard to pass tube despite good view (sub/glottis tumour)
Options:
PROS CONS
Deep inhalational anaesthetic + direct laryngoscopy

Maintains spontaneous ventilation Risk airway obstruction at laryngoscopy if too light
Have NPA ready rather than oral airway
If difficult anatomy or v narrow on laryngoscopy, withdraw and surg trach
AFOI + LA ??Awake patient maintains own airway Many problems:
• Anxious patient!
• Difficult to anaesthetise with LA
• Potentially dislodges tumour/bleeding
• Scope only 4mm, poor view
• If stenotic, scope may obstruct airway completely –hypoxemia
Planned tracheostomy + LA (or GA)
-Avoid sedation! Safest for subglottic, large, fixed mass, esp if nasendoscopy not able to visualise cords! More invasive
Pt discomfort during dissection with no sedation
Surgical skill/availability

• Use of IV induction agents/NMB before airway is secured risks can’t intubate/can’t ventilate scenario
• No one technique is definitely the best – final decision depends on collaboration between ENT surgeon, anaesthetist, skills/experience
• Whichever technique have range of equipment available – laryngoscopes, cricothyroidotomy kit, sizes of ETT, LMA
• If complete airway obstruction occurs – emergency needle cricothyroidotomy
Maintenance/Emergence
• Use remifentanil if planning to extubate patient – simultaneous return of cough reflex and consciousness (avoids coughing with tube in situ)
• Intraoperative dexamethasone
• Extubate awake; Cook airway exchange catheter if concerns re oedematous larynx
Post-op:
• Extended recovery or HDU for monitoring of potential airway obstruction
• Tracheostomy needs humidification, CPAP +/- IPPV
October 2010 Question 5.

A 6-month-old boy presents with an acute abdomen. He is diagnosed with intussusception and booked for laparotomy after a failed attempt at reduction. His heart rate is 160bpm and BP is 75/45 mmHg.

His electrolyte profile is as shown:

Na⁺ 132 mmol/l (135 – 145)
K⁺ 2.7 mmol/l (3.5 – 5.5)
Cl⁻ 106 mmol/l (95 – 110)
Urea 3.3 mmol/l (3.5 – 8.5)
Creatinine 86 μmol/l (60 – 110)
Lactate 4.5 mmol/l (1.0 – 1.8)

(a) How would you determine his degree of dehydration and how severe is it likely to be? (40%)

(b) Describe your perioperative fluid management. (40%)

(c) When would you proceed to surgery and why? (20%)
PART A
Clinical assessment of dehydration can be difficult, especially in young infants, and rarely predicts the exact degree of dehydration accurately.
* The most useful individual signs for predicting 5% dehydration in children are an abnormal capillary refill time, abnormal skin turgor and abnormal respiratory pattern.
* Combinations of examination signs provide a much better method than any individual signs in assessing the degree of dehydration.
* Of the clinical indicators used, the pinch test (skin turgor) has been shown to be the most reliable in several studies but is still not a reliable test when used without other clinical indicators.
* Clinical assessment therefore comprises some of the following indicators of dehydration:
o Loss of body weight:
+ Normal: no loss of body weight.
+ Mild dehydration: 5-6% loss of body weight.
+ Moderate: 7-10% loss of body weight.
+ Severe: over 10% loss of body weight.
o Clinical features of mild-to-moderate dehydration; 2 or more of:
+ Restlessness or irritability.
+ Sunken eyes (also ask the parent).
+ Thirsty and drinks eagerly.
o Clinical features of severe dehydration; 2 or more of:
+ Abnormally sleepy or lethargic.
+ Sunken eyes.
+ Drinking poorly or not at all.
o Pinch test (skin turgor):
+ Skin turgor is assessed by pinching the skin of the abdomen or thigh longitudinally between the thumb and the bent forefinger.
+ The sign is unreliable in obese or severely malnourished children.
# Normal: skin fold retracts immediately.
# Mild or moderate dehydration: slow; skin fold visible for less than 2 seconds.
# Severe dehydration: very slow; skin fold visible for longer than 2 seconds.
o Other features of dehydration include dry mucous membranes, reduced tears and decreased urine output.
o Additional signs of severe dehydration include circulatory collapse (e.g. weak rapid pulse, cool or blue extremities, hypotension), rapid breathing, sunken anterior fontanelle.

Investigations
* Urinalysis: ketones and glucose in diabetic ketoacidosis (DKA).
* Urine specific gravity: may be elevated (but diabetes insipidus causes the urine to be dilute).
* Serum sodium: hyponatraemia and hypernatraemia require specific management.
* Potassium may be raised (e.g. congenital adrenal hyperplasia, renal failure) or low (e.g. pyloric stenosis, alkalosis).
* Bicarbonate: causes of reduced bicarbonate include DKA and diarrhoea.
* Chloride: may be low in pyloric stenosis.
* Blood glucose: may be low as a result of poor intake or grossly elevated in diabetic ketoacidosis.
* Blood urea and creatinine: raised in renal impairment.
* ECG: monitor for cardiac arrhythmias caused by electrolyte disturbance.
* Electrolyte analysis of any fluid that is lost, e.g. urine, stool, gastric fluid.

PART B
Whenever possible use the enteral route.
*Initial bolus to correct intravascular depletion :10-20 ml/kg of NS which may be repeated
-observe for normalisation of vitals
-after second bolus of cystalloid consider blood

Maintainance fluids
-4-2-1 rule
-6/12 would be roughly 10kg, therefore 40ml/hour
Fluid of choice
-0.9% NaCl (for boluses)
-0.9% NaCl with 5% glucose or 0.45% NaCl with 5% glucose
-DO NOT USE 0.18% NaCl with 4% glucose with KCl 20mmol/L (or 4% and 1/5 NS, or 5% and ¼ NS) in unwell children

Replace blood for blood

Monitoring
* All children on IV fluids should be weighed prior to the commencement of therapy, and daily afterwards.
* Children with ongoing dehydration/ongoing losses may need 6 hourly weights to assess hydration status
* All children on IV fluids should have serum electrolytes and glucose checked before commencing the infusion (typically when the IV is placed) and again within 24 hours if IV therapy is to continue.
* For more unwell children, check the electrolytes and glucose 4-6 hours after commencing, and then according to results and the clinical situation but at least daily.
* Pay particular attention to the serum sodium on measures of electrolytes.
* Children on iv fluids should have a fluid balance chart documenting input, ongoing losses and urine output.

PART C
Decision to proceed to OT
Depends on discussion with the surgeon regarding the urgency of OT vs time for fluid/electrolyte resuscitation.
Preferable to aim for normovolemia, normal electrolytes, increasing urine output, improving pH before surgery
Regular monitoring whilst giving fluid resuscitation.
October 2010 Question 6.

How would you critically appraise a paper published in a journal?
Critical appraisal is a systematic process used to identify strengths and weaknesses of a research article in order to assess usefulness and validity.
–Young JM & Solomon MJ How to critically appraise a journal article Nature Clin Prac Gast Hep (2009) 6; 82-91

This process provides a basis for decisions on whether to use the results in clinical practice (usefulness of study, trustworthiness of results)

Components of this include:
- Evaluating appropriateness of study design for the question asked
- Assessing methodology
- Suitability of statistical analysis
- Conflict of interest
- Relevance of research to own practice

Evaluate:
• Type of article (editorial, research paper, case report)
• Presentation
• Relevance of the study question:
• Has this issue previously been investigated?
• Application to your patient population
• Type of question: about treatment efficacy or event frequency?
• Methodology and design
• Valid hypothesis and outcome measures
• Appropriate study design to answer question (frequency of events usually requires observational studies eg anaphylaxis to NMB; ethically inappropriate to conduct RCT!)
• Ethics approval
• Study performed according to original protocol
• Correct statistical analysis (eg intention to treat analysis in RCTs preferred unless very large crossover btw groups; maintains randomisation)
• Measures to reduce bias; randomisation, blinding
• Does data justify conclusions?
• overemphasis on statistically significant findings with minimal clinical difference; large difference NOT significant due to small sample size (underpowered)
• statistically significant association is NOT cause/effect!
• Conflict of interest declared? (open disclosure)
o Study validity
October 2010 Question 7.

(a) Describe the common classification code for permanent pacemakers. (30%)

(b) Outline the principles involved in the perioperative management of patients with a permanent pacemaker. (70%)
PART A
The common classification code for permanent pacemakers was developed originally by the international conference on heart disease and later modified by the NASPE/BPEG (North American Society of Pacing and Electrophysiology/ British Pacing and Electrophysiology Group). The NAPSE/BPEG code consists of a 5 position system using a letter in each position to describe the programmed function of a pacing system.
1st letter : chamber being paced (O=none, A=atrium, V=ventricle, D=dual A+V)
2nd letter : chamber being sensed (O=none, A=atrium, V=ventricle, D=dual A+V)
3rd letter : response to sensing (O=none, I=inhibited, T=triggered, D=dual I+T)
4th letter : programmability (O=none, C=communicating, P=simple, M=multi programmable, R=rate modulation
5th letter : antitachyarrythmia functions (O=none, P=pacing, S=shocks, D=dual P+S)

PART B
Preoperative evaluation (in addition to routine anaesthetic assessment)
1. Establish whether patient has a pacemaker : history, examination, Identification Card, ECG (turn off filter to display pacing spikes), CXR to locate position
2. Define the type of device, when was it inserted, its function, pacemaker dependent function : interrogate the pacemaker with a technician
3. Detailed cardiac history : including history of IHD, arrhythmias, syncope, indication for insertion of pacemaker.
4. Contact manufacture/cardiologist to define function and advice for anaesthetic
i. Reprogram as required (AOO, VOO, DOO)
ii. All AICD need to be reprogrammed. Disabling tachytherapy and defibrillation function in AICD
5. Consider bipolar electrocautery or ultrasonic harmonic scalpel to minimize effect
6. Ensure temporary pacing and defibrillation equipment

Intraoperative
1. Diathermy : pacemakers are extremely resistant to damage, but diathermy can result in inhibition of pacemaker output or revert to an asynchronous mode. Generally, diathermy performed on limbs will be safe. The ground should be distal to the thorax and the line of current flow should not pass the pacemaker. Bipolar should be considered
2. Magnet : used in emergency situations if reprogramming is not available. This will inhibit sensing and allow for asynchronous pacing. Indicated for pacemaker dependant patients. Diathermy should be in short bursts and observed to reassure that asynchronous pacing continuous.
3. All AICD need to be switched off, diathermy will be detected as VF resulting inappropriate shocks. A magnet can be placed in emergency situations which will suspend detection, however it will not affect the pacemaker function of the AICD. This is suboptimal.
4. Pulse oximetry or IAL should be used to ensure an output, as ECG can be uninterpretable during diathermy.
5. Consider invasive blood pressure monitoring : balance between complexity of surgery and patients cardiovascular stability.
6. Patients at high risk of arrythmias (eg VF/VT) should have external defibrilllation pads attached.

Post operative
1. Close observation in recovery, continue cardiac monitoring and external defibrillation pads
2. Reinterrogate pacemaker in recovery, turn AICD back on.
3. Consider transfer to ICU/HDU if required
October 2010 Question 8.

How and why is cardiopulmonary resuscitation modified for the pregnant patient at term compared with the non-pregnant patient?
Cardiac arrest is a rare event (1:30,000 late pregnancies). There are a number of physiological changes in pregnancy which necessitate a modified cardiopulmonary reuscitation. Attention to lateral displacement of the uterus, more aggressive airway management, and early consideration of emergency cesarean delivery are major modifications in the management of maternal arrest.

1. Aorto-caval occlusion
The vena cava is completely occluded in 90% of supine pregnant patients, resulting in reduced venous return and preload.

During cardiac arrest, to minimize the effects of a gravid uterus on venous return and cardiac output, a maternal pelvic tilt to the left of >15deg. However, the tilt needs to be less than 30deg for effective closed chest compressions to take place. This should be initiated immediately before chest compressions

Delivery of the fetus during cardiac arrest will reduce the oxygen demands on mother and restore vena caval flow to normal. Therefore, greatly increasing the chances of successful resuscitation.

Urgent delivery of fetus should be considered if CPR is unsuccessful in restoring spontaneous circulation within 4 minutes.

2. Lung function
Increases likihood of hypoxia due to 20% reduction of FRC (due to gravid uterus and enlarged breasts) and 20-50% increase in metabolic requirements (to service the growing fetus and the uterus). Leading to a rapid decline in oxygen saturation during apnea.

Furthermore weight gain can interfere with adequate ventilation. During cardiac arrest, adequate bag-mask ventilation can be difficult. Intubation should be considered early to maintain oxygenation.

3. Aspiration risk
The pregnancy‐associated increase in levels of progesterone relaxes sphincter tone of the lower esophagus. During labor, gastric emptying is delayed, increasing the risk for aspiration during mask ventilation and intubation.

During cardiac arrest, intubation should be consider early to protect the lungs. Intubation with cricoid pressure should be considered as a standard of care.

The aspiration risk may be exacerbated by gastric distention from air insufflation during bag-mask ventilation.

4. Difficult airway
Edema of the upper airway, increased breast size, and generalized weight gain can interfere with adequate ventilation and intubation during maternal resuscitation.

There is an increased incidence of difficult airway. A senior anaesthetist should be present to secure the airway early.

5. Effective forces for chest compressions

There is reduced chest wall compliance due to enlarged breasts.

The reduced chest wall compliance, necessitates more vigorous CPR (requiring deeper compressions). Furthermore, the gravid uterus displaces the thoracic contents up, meaning compressions should be done higher in the chest.

Available evidence suggests that defibrillation energy requirements do not change significantly with pregnancy; thus, positioning and dosing do not need to be modified for cardiac defibrillation. Defibrillation does not carry any increased risk for the fetus, but the fetal monitor should be removed prior to defibrillation. The same protocols for pharmacologic management of CPR should be used for pregnant and nonpregnant patients.
October 2010 Question 9.

(a) List the predisposing factors for pain persisting for more than three months postoperatively. (50%)

(b) Outline the interventions that have been demonstrated to be efficacious in the prevention of persistent postoperative pain. (50%)
PART A
Pain persisting for more than 3 months post operatively is chronic pain. It is one of the most common complications of surgery. Pain is the a psychological sensory experience.
1. Genetic susceptibility
• Possible heritable response although no gene responsible have been identified
• Polymorphisms of catecholamine-O-methyltranferase
• Red hair and fair skin
2. Preceding pain
• Past history of chronic pain, eg phantom limb pain have more intense and enduring pain to subsequent operations
• History of backache, IBS or headache
3. Psychological factors
• Expectation of pain, fear, past memories, social environment, work, and levels of physical activity
• Catastrophising
• Perceived level of social support
• Preoperative anxiety
4. Age and sex
• Older patients have reduced risk of developing chronic pain
• Women have higher postoperative pain than men
5. Surgical operation
• Not correlated with size of incision, but instead type and duration of surgery (>3 hours) and experience of surgeon
• Repeat surgery
• Amputation : 30-50% incidence of chronic pain
• Thoracotomy : 20-30%
• Mastectomy : 30-40%
• Inguinal hernia : 10%
• Coronary bypass : 30-50%
• Caesarian section : 10%
6. Post operative factors
• Radiotherapy
• Neurotoxic chemotherapy




PART B
1. Surgical technique : opt for less invasive techniques, more careful incision and dissection
2. Optimizing pre operative pain management
3. Preoperative psycho-social support and optimization, counseling as required
4. Aggressive and multimodal pain management
5. Pre-emptive analgesia
• Regional analgesia commenced before surgical incision
i. Little benefit for prevent CP, but beneficial in acute pain outcome measures in thoracotomy
ii. Benefits have been found with spinals for caesarian section and epidural analgesia in abdominal surgery
iii. Reduction in phantom limb pain NNT6
iv. Paravertabal block for mastectomy patients reduced incidence of pain
• Local anaesthetic would infiltration : positive-premptive effects especially in the setting of amputation
• Systemic NMDA antagonsists
• Systemic opiods : not shown to be benefitial
• Systemic NSAIDs : positive pre-emptive effect

6. Protective and preventive analgesia
• Protective : technique that reduces measures of sensitization such as hyperalgesia
• Preventative : persistence of analgesic treatment effeicacy beyond expected duration
• NMDA receptor antagonists : ketamine and dextromethorphan produced significant preventive analgesia
• Ketamine bolus followed by infusion for 72 hours reduced the incidence of severe phantom limb pain
• Low dose ketamine 3mg/hour shown to reduce pain in thoracotomy
October 2010 Question 10.

An adult patient who was intubated for tonsillectomy is noted to have an upper central incisor tooth missing in the Recovery Room after extubation.

(a) List the predisposing factors for perioperative dental damage. (50%)

(b) What is your management of this situation? (50%)
PART A

Predisposing factors include:
1. Patient factors:
o Vulnerable teeth (loose, isolated, capped teeth, veneers, crowns)
o Gum pathology; hyposalivation (eg from previous radiotherapy)
o Anatomy causing difficulty in intubation (eg poor mouth opening or neck extension)

2. Anaesthetic/surgical factors
o Use of general anaesthesia and an endotracheal tube
o Poor technique at laryngoscopy
o Surgical gags/retractors
o Inadequate anaesthesia or relaxation allowing biting
o Poor care with suctioning or oral airway (eg Guedel) use
o Postoperative shivering


PART B
Management includes
1. Review the records to assess the patient‟s prior dental condition
2. Review the case, if necessary with other team members, to determine the most likely timing and cause of the dental damage
3. Find the tooth, with imaging (eg CXR) if necessary
4. Consult a dentist for assessment and further management
5. Document the events and findings
6. Notify medical defence organisation and local quality assurance mechanisms (eg for departmental morbidity meeting)
October 2010 Question 11.

A 78-year-old female presents for fixation of a displaced femoral fracture. She has longstanding mitral regurgitation and is known to have a mean pulmonary artery pressure of 60mmHg. She reports orthopnoea but is not short of breath at rest.

(a) What are the issues of concern in your preoperative assessment? (50%)

(b) How would you manage pulmonary vascular resistance perioperatively? (50%)
TBA
October 2010 Question 12.

What are the advantages and disadvantages of general versus local anaesthesia for carotid endarterectomy?
There is demonstrated benefit of CEA for symptomatic patients with more than 70% carotid artery stenosis. However as the surgical technique involves clamping of the carotid artery above & below the effected area, there is a risk of ipsilateral cerebral hypoperfusion & CVA. Shunts can be put in place but these are associated with their own risk of microembolic strokes. Overall CEA is a high risk procedure with an overall peri-operative stroke/mortality risk of 2-5%.


LA advantage:
- Less requirements for shunts (down to 10%) and hence less of the associated morbidity with shunts – e.g. risk of micro-embolic stroke
- Avoid all risks of GA
o General risks: Anaphylaxis, aspiration dental injury etc..
o Specific for CEA:
• Hypotension on induction or intra-operatively: CVA
• Rough emergence/ extubation – resulting in coughing and potentially blowing carotid repair.
- Gold standard for monitoring for cerebral hypo-perfusion by means of awake patient which allows for continual observation of motor, speech & higher cerebral functions & subsequent treatment by insertion of shunt or pharmacological intervention if indicated.

LA disadvantages:
- Patient anxiety
o May result in tachycardia and hypertension which may make haemostatis more chanllenging as well as risk myocardial ischemia
o ↑ stress = ↑ CMRO2 = ↑ O2 demand at time when O2 supply is limited.
- Patient discomfort during long surgery
- Poor Patient compliance during long surgery - moving?
- Need to convert to GA e.g. due to poor block, patient, surgical reason = more complicated – practical problems for surgeon and anaesthetist.

GA Advantages:
- Controlled conditions
o Patient immobility, secure airways
- Avoidance of LA block complication
o General LA risks: LA toxicity, inadequate block (results in conversion to GA & a patient that has been exposed to both set of risks), neuropraxia
o Specific LA risks to deep cervical block (less so superficial cervical block)
• Inadvertent injection into the dural cuff (epidural, intrathecal
• Injection into vertebral artery
• Phrenic nerve block, laryngeal nerve palsy, brachial plexus block
• Pneumothorax
• (and local infiltration)
• Bleeding – carotid haematoma
o Local Infiltration:
• Distortion of anatomy for surgeons (I have heard some complain about this)
- Patient preference to be asleep (less PTSD if particularly traumatic experience)
- Possible provision of cerebral protection
o Volatiles: Reducing CMRO2 and increasing cerebral blood flow

GA disadvantages
- More difficult to monitor adequacy of cerebral perfusion. Techniques used include:
o EEG/ somato-sensory evoke potentials
o Transcranial Doppler of MCA
o Near- Infrared spectroscopy

Note: Recent GALA trial results: comparing LA & GA for CEA. Failed to show any difference in CVA & mortalilty rates between two groups however possibly some long-term benefit with LA resulting in lower levels of long term cognitive impairment.
October 2010 Question 13

Outline the principles of an initial management plan for diabetic ketoacidosis, having regard to the physiological derangements involved.
DKA is a potentially life threatening condition where there is insufficient insulin resulting a severe dehydration, metabolic acidosis and hyperkalaemia.

DKA Management principles:
Patho-physiological reason:
1. Aggressive IV fluid resuscitation:
- Use Isotonic solution e.g. normal saline
- Replace losses & maintenance over 48 hours

To correct the severe dehydration that occurs secondary to renal losses due to the hyperglycaemic osmotic diuresis

2. Electrolyte replacement
- Regularly check K+ & replace it
- If K+ >5.5mmol/L then monitor
- Mg2+ replacement may help in treating resistant hypokalaemia

Hypokalemia due to several mechanisms including:
- Vomiting
- Renal losses: osmotic diuresis
- Hyperaldosteronism
- Metabolic acidosis (can mask hypokalemia by acidosis causing extracellular shift of intracellular K+)
- Insulin treatment: pushes K+ into cells

3. Beware pseudohyponatraemia

Corrected sodium = measured Na + 0.4(glucose in mmol/L)
October 2010 Question 14

You are on the interview panel appointing new Assistants for the Anaesthetist. What are the educational requirements and the practical responsibilities expected of the applicants?
(Refer: College document PS 8)

Educational requirements:
- Completion of year 12/VCE equivalent
- Course with a minimum requirement of:
• 150 hours of lectures
• internal assessments
• practical work (registered in logbook)
• assignments
- Duration as trainee anaesthesia assistant with study work combination:
o No previous hospital experience: 3 years
o Division 2 nurse: 2 years
o Division 1 nurse: 1 year
- Course content should include:
o Basic sciences:
• Physics, chemistry, pharmacology, anatomy, physiology, clinical measurements, microbiology
o Anaesthesia
• In depth knowledge re-inforced by experience in:
• Anaesthesia equipment
o Anaesthesia equipment, monitors, ventilators, airway devices including fibre-optic instruments, intra vascular devices
o Cleaning and sterilization of equipment
o Infection control
• Operating room safety
o Electrical, gas cylinders, staff & patient safety
• Anaesthesia techniques
o All perioperative areas including preparation, monitoring, induction, securing airways, maintenance & recovery
• Invasive techniques
o Arterial, central and pulmonary arterial line insertion
• Regional anaesthesia/ ultrasound
• Fluid & drug preparation
• Crisis management
o Familiar with algorithms for management of CPR, Failed intubation, anaphylaxis, MH, Massive haemorrhage etc..
• Post operative Pain management
o Management aspects:
• Rostering, budget, incident monitoring, health & safety, legal responsibilities, communication


The practical responsibilities expected of the applicants are:
- Assisting in the safe and efficient conduct of anaesthesia
- Immediately available throughout preparation and induction of anaesthesia as well as conclusion of anaesthesia until that level of assistance no longer required.
- Be available at short notice during the rest of the case.
- Be exclusively available for that case.
- Responsible for the preparation and application of anaesthetic monitoring and ancillary equipment.
- Ensuring anaesthesia equipment properly maintained and checked before use e.g. anaesthesia machine, airway equipment, pumps, infusion devices, etc.
- Cleaning, decontamination and sterilisation of equipment in compliance with best practice.
- Restocking / ordering equipment, drugs, etc
- Ensuring appropriate quality assurance measures met.
- To work within a team in an effective and professional manner.
October 2010 Question 15 PART A

(a) picture 1
(i) Identifying the key features, what pattern of disorder is demonstrated by these tests?
(ii) What are the possible causes?
(b) What are the implications of general anaesthesia for an adult patient with Curve B presenting for a knee arthroscopy?
Part (a)
i.) The pattern is restrictive.
a. The reason for this is normal FEV1/FVC ratio however there is a reduction in all lung volumes including FVC, TLC & RV.
ii.) Restrictive lung disease causes
a. Parenchymal
i. Auto-immune
1. Rheumatoid Arthritis
2. Scleroderma
ii. Drugs
1. Amiodarone
2. Chemotherapeutics
iii. Dust/ allergen exposure
1. Asbestosis
2. Bird fanciers Lung, Farmer’s Lung
3. Coal workers
iv. Idiopathic pulmonary fibrosis
v. Intersitial pulmonary fibrosis
vi. Cystic fibrosis
b. Extraparenchymal
i. Chest wall deformity
1. Kyphoscolosis
2. Ankolising spondylitis
ii. Severe obesity
iii. Abdominal splinting e.g. LBO
October 2010 Question 15 PART B

What are the implications of general anaesthesia for an adult patient with Curve B presenting for a knee arthroscopy?
These patients have rapid, shallow breathing to minimise their work of breathing. They like to sit up (as lying supine further ↓their lung compliance and volumes

- ↓ O2 reserve as ↓ FRC
o Prone to hypoxia with apnoeic episodes∴ adequate pre-oxygenation vital
- V/Q mismatch – hypoxia/ atelecatasis
o worst in supine position
o worst with opioids
o Control over ventilation with IPPV important → avoid spont ventilation
- Stiff non- compliant lungs → higher inflation pressures
o Prone to:
• barotraumas/ volutrauma
• Excessive inflation pressure could ↓ VR → ↓CO
o Ventilation strategy
• IPPV (Pressure control) with higher frequency 10-14 & ↓ Tidal Volumes (6- 8ml/kg)
• Avoid pressure > 30mmHg to avoid barotrauma
• Mild PEEP/ CPAP is good as puts them on steeper part of compliance curve (avoid excessive PEEP > 10cmH2O)
• High I:E ratio: ↑ I-time allows for longer expansion of lungs
• Adjust FiO2 aim sats near baseline 88-94% (no need to over oxygenate to achieve sats of 98 -100%)
- Secondary pulmonary HT/ Right heart failure
o Could be present, important to assess for this if suspicious (Hx, Ex Ix)
- Concomitant infection
o Be aware and treat if present
- Consider regional anaesthesia where possible
October 2010 Question 15 PART C

(i) Describe the abnormalities on this capnograph.
(ii) What is your differential diagnosis?
(iii) How would you identify the likely cause in the intraoperative setting?
(i) The abnormalities at the onset of a steep expiratory upstroke and apparent drop in end-tidal carbon dioxide level.

(ii) Causes include: (generally due to obstruction to expiration whether that be patient related or mechanical.
a. Patient factors
i. Bronchospasm
1. Asthma
2. Histamine release: e.g atracurium
3. anaphylaxis
ii. Exacerbation of COAD
b. Equipment factors
i. Mechanical obstruction to ventilation
1. ETT foreign body/ kink
2. Kinked circuit tubing
3. Circuit expiratory valve stuck
4. APL valve obstruction
5. Soda lime reservoir

(iii) How identify likely cause of this obstruction?
a. Check patient breathing
i. Any obvious obstruction, foreign body, sputum in endotracheal tube or patient
ii. Auscultate the chest for wheeze, crepitations, check air entry to both sides
iii. Check airway pressures & switch over to manual hand ventilation to check for ease of ventilation and any possible broncospasm
b. Check the circuit
i. Check circuit from ETT – circuit tubing, valves, APL valve, sodalime etc.. looking for kinks, obstruction with blood clots, foreign body, mucus etc..
ii. If necessary – switch over to manual ventilation whilst diagnosing the fault in the circuit
May 2010 Question 1

a. List the complications associated with the use of limb tourniquets during surgery. (60%)

b. How can these complications be minimised? (40%)
Q1) Tourniquets are commonly used to produce a bloodless field.

The complications associated with the use of limb tourniquets during surgery include:

1) Nerve and muscle injury due to mechanical pressure and ischaemia as well as poor application techniques.
2) Pressure sores and skin injury due to poor application or moisture under padding.
3) Haemodynamic changes associated with inflation and exsanguination (increased SVR and CVP, tachycardia, hypertension, fluid overload and LVF possible so avoid bilateral thigh tourniquets)
4) Metabolic and haemodynamic changes associated with deflation – post-ischaemic reactive hyperaemia→ decreased MAP / hypotension, decreased CVP, decreased temperature due to blood volume shifts, increased K+/lactate/CO2, decreased saturations.
5) Haemodynamic changes associated with prolonged tourniquet time and tourniquet pain (after 30-60 minutes, tachycardia, hypertension, sympathetic activation)
6) Possible embolisation during exsanguinations (dissemination of tumour)
7) Intraoperative tourniquet failure causing bleeding, iv regional anaesthesia leakage causing toxicity.
8) Post deflation uncontrolled bleeding due to inadequate haemostasis or temporary increased fibrinolytic activity.
9) DVT/PE (case reports)
10) High risk tourniquet complications – peripheral vascular disease (increased risk ischaemia and vascular injury/plaque rupture), DVT (propagation of clot), sickle cell disease (sickling of red blood cells under anoxic conditions causes thrombosis), poor skin integrity, known peripheral neuropathy.

These complications can be minimized by:
1) Minimising tourniquet time, 2 hour maximum if greater need to ensure breaks for reperfusion of 10-15mins.
2) Only pneumatic tourniquets should be used as mechanical tourniquets can cause areas of unpredictable high pressure in the underlying tissues.
3) Choosing appropriate sites of application eg upper arm and thigh where there is sufficient muscle bulk.
4) Choosing appropriate cuff width (20% greater than diameter upper arm, 40% greater than circumference of thigh) and length (should exceed circumference of extremity by 7-15cm) helps ensure that the pressure in the underlying central artery will be equal to that in the cuff (minimises excessively high pressures). Use of widest possible cuff.
5) Protect tissues with cotton wool padding.
6) Ensure appropriate pressures. Upper limb SBP + 50mmHg, lower limb twice SBP or SBP + 100-150mmHg. Tailor to individual patient risk factors and use patient’s usual awake baseline SBP.
7) Anticipate tourniquet deflation and ensure adequate fluid status +/- bolus, use of vasopressors, hyperventilation to control CO2, vigilance for dysrhythmias and myocardial ischaemia, use FiO2 100% for deflation, be prepared for reinflation if necessary.
8) Use of high and low pressure alarms with audiovisual pressure and time alarms.
9) Tourniquet pain is often difficult to manage and resistant to opioids or increasing depth of anaesthesia. The use of clonidine may be useful or low dose iv ketamine 0.1-0.25mg/kg.
10) Post-operative vigilance of cuff site, for nerve injury and ischaemia, ensure early assessment and intervention for post-operative problems.
May 2010 Question 2

a. List the hazards to the patient associated with the prone position under general anaesthesia.
(60%)

b. How can these hazards be minimised? (40%)
Hazards Prevention / Minimising risk

1. Obtaining prone position
2. Pressure Factors
3. physiologic effects of abdominal/ thoracic pressure
4. monitoring
5. limited accessibility
6. embolus risks

1. OBTAINING THE PRONE POSITION
• Requires minimum of 4 people to turn and presence of surgeon
• Anaesthetist specifically responsible for control of airway/cervical spine/ head
• Aim to avoid disconnection of ETT/ IV cannula/ foley catheter/ arterial line/ CVC


2. PRESSURE FACTORS
1. Avoid thoracic/ abdominal compression
• Apply parallel (foam/ towel) chest rolls
• Elevates chest preventing cephalad movement of diaphragm which may cause impaired diaphragmatic excursion and obstruction of aorta/ IVC
2. Pressure Care
attention to…
• head forehead, eyes, ears, nose, face
• upper limbs elbows, radial/ ulnar nerve at wrist
• lower limbs knees, hips, ankles
• male genitalia
• breasts

specifically
a. head
• can remain face down in cushioned holder ie do nut or horse shoe or can rotate head within normal range of movement
• extreme rotation may cause decrease cerebral venous drainage/ CBF
b. pressure necrosis particularly of ears/ nose/ chin/ elbows/ breasts/ male genitalia
• ensure adequate padding
• hourly movement of head/ limbs avoids pressure necrosis and venous pooling
c. nerve injury
• avoid brachial plexus injury by correct positioning
o upper limbs not >90 degree abduction shoulder
o elbows flexed to < or equal to 90 deg
o no pressure on axilla/ brachial plexus
o palms up


3. PHYSIOLOGIC EFFECTS OF ABDOMINAL/ THORACIC PRESSURE
A increased intra-thoracic pressure
*Decreases thoracic and total lung compliance
*Increases basal atalectasis requiring increased inflation pressures
*(also inc work of breathing in spont breathing patient not relevant in this scenario)
*ensure adequate control of ventilation prior to surgery commencing
B. compression of IVC/ veins
1. Dec VR==> dec preload==> dec CO==> dec BP
• Pre-empt hypotension with adequate position, volume status and early treatment
• Particularly important here where hx of cor a disease as need to maintain adequate DBP for coronary perfusion
• If ischaemia should progress to cardiopulmonary arrest positioning/ access would make external cardiac compression and defibrillation impossible without rotating patient
• Concept of controlled hypotension may not be suitable in this patient with hx of cor a disease
2. Venous engorgement
• inc risk of bleeding
• furthermore 2 level spinal fusion carries high risk of significant bleeding
1. cross match blood avail
2. consider cell saver
3. close monitoring of haemodynamics for early intervention
4. local weak adr solution


4. MONITORING
• IABP/ CVP/ Regular Hb check via ABG/ IDC
• To avoid hypovolaemia/ hypotension/ anaemia
• Ideally obtained prior to prone positioning because of access limitation

5. LIMITED ACCESIBILITY
• secure endotracheal tube and check after rolling
• 2 x large IV cannula and extension tubing
• invasive monitors discussed
• temperature measurement to monitor adequate warming

6. EMBOLUS
• be aware of risk of air embolus
• aim for positive venous pressure to avoid entrainment of air
• increased risk in underfilled patient
May 2010 Question 3
A 20 year old female with a body mass index of 48 kg/m2 presents for an elective diagnostic
laparoscopy for endometriosis. She has no other medical conditions. Describe the potential
problems associated with anaesthetising this patient.
Providing anaesthesia for laparoscopy in obese patients can be challenging. Potential problems include:

Preoperative
1) Thorough history and examination – elicit associated comorbidities eg diabetes, obstructive sleep apnoea (may be undiagnosed), reflux, fasting status. Assess airway / increase risk of difficult intubation.

Intraoperative
1) Airway – rapid sequence induction (ETT) due to increased aspiration risk in obesity secondary to reflux. Ensure adequate preoxygenation and ideal positioning with pillows, availability of bougie and difficult intubation trolley.
2) Breathing – Difficulties with pneumoperitoneum and Trendelenburg on ventilation. Ventilate with PEEP to minimize atelectasis, aim to keep plateau pressures < 30cmH20 to prevent barotrauma (pressure controlled ventilation). Allow permissive hypercapnoea, ensure adequate muscle relaxation to help with ventilation.
3) Circulation – ensure large bore iv access (may present with difficult iv access), iv fluids (pneumoperitoneum will decrease VR and CO with possible hypotension), BP monitoring (consider arterial line), ECG
4) Monitoring difficulties – difficulty measuring NIBP / may be inaccurate due to inappropriate cuff size (arterial line provides continuous and more accurate BP measurements).
5) BIS monitor – helps to guide appropriate depth of anaesthesia
6) Pressure care with appropriate padding, ensure on non-slip mat.
7) Avoid hypothermia with forced air warmer.
8) Pain management – multimodal analgesia eg paracetamol, NSAIDs help to minimize opioid use and associated sedation (increased sensitivity to opioids in obese).
9) Prior to extubation – ensure fully reversed, assess for facial / airway oedema if prolonged head down position, extubate awake and sitting up.
10) Surgical difficulties – access often improved with adequate muscle relaxation.
11) Pharmacological DVT prophylaxis to start intraoperatively (after discussion with surgeons) as high risk for DVT/ PE. Pneumatic calf compressors should be used intraop.

Post-operative
1) Ensure adequate analgesia to aid deep breathing and minimize respiratory complications (atelectasis) (multimodal – paracetamol, NSAIDs, tramadol +/- PCA if extensive excision of endometriosis performed).
2) Encourage early mobilization and DVT prophylaxis continued in post-operative period.
3) Consider close monitoring in HDU if associated OSA.
May 2010 Question 4

a. Describe the pathophysiological changes associated with a haemoglobin of 75 g/L. (50%)

b. Outline the patient factors that would indicate the need for a perioperative red blood cell
transfusion in a patient with a haemoglobin of 75 g/L. (50%)
Anaemia
- Defined as a haemoglobin concentration < 130 g/L (male) or < 120 g/L (female)
- Causes
- Decreased production
- Bone marrow failure - tumour or drugs
- Deficiencies in Iron, B12, folate
- Increased destruction / decreased survival
- Blood loss, acute or chronic
- Haemolysis - inherited (eg thal), acquired (eg drugs) physical (DIC, AVR)
- Sequestration - hypersplenism
- Both
- Renal failure / Rheumatoid arthritis / hypothyroidism
- Dilution
- Infusion of crystalloid

- Haemoglobin essential for delivery of oxygen to tissues
- 200 mL/L with Hb 10, SpO2 100%, PaO2 100 mmHg
- ↓ 25% at Hb 75 with same parameters

- Absolute level of anaemia is less important that then rate of change
- chronic mild anaemia may be asymptomatic due to increased oxygen extraction
- Impossible for tissue with already high extraction - Myocardium & Brain

- Major changes
- ↓ Oxygen delivery
- Fatigue, headache, confusion, syncope
- Children: Neurological / developmental
- ↑ Cardiac output
- Palpitations, angina, claudication, syncope
- ↑ 2,3 DPG
- ↑ offloading of oxygen to periphery
- β adrenoreceptor stimulation - Shunting
- Vasoconstriction of splanchnic and peripheries

- Goals for transfusion
- ↓ end organ hypoxia
- Symptomatic relief
- Adequate wound healing

- Transfusion triggers are guides - not end points - patients at risk of end organ damage need to be considered for transfusion earlier

- TRICC trial (NEJM 1999) - ICU patients Hb < 90 in 1st 72 hours of admission
- ‘restrictive’ group: trigger Hb < 70. Target Hb 70-90
- ‘liberal’ group: trigger Hb < 100. Target Hb 10-12
- 30 & 60 mortality was the SAME in both groups
- restrictive group ↓ mortality if the patient was YOUNGER or LESS UNWELL
- Difficulty extrapolating this to context of perioperative transfusion.

- Patients I would consider transfusion @ 75 g/L
- Vasculopathy / Risk of critical ischaemia
- Acute / Chronic Ischaemic heart diasease
- CVA / TIA
- Renal artery stenosis
- Peripheral vascular disease
- Age > 65
- Early severe sepsis
- ie ‘goal directed therapy’ demonstrated benefit
- Respiratory disease
- Hypoxic respiratory failure
- Bleeding diathesis
- Known / developing coagulopathy
- must of course be treated with appropriate factors / components rather than RBC concentrates
- eg Duchenne’s muscular dystrophy
- Burns

- Surgical factors (outside scope of question)
- involving vascular anastomosis
- Cardiac surgery
- Vascular surgery
- Limb reattachment surgery
- Likely significant blood loss or previous resuscitation
May 2010 Question 5

A 26 year old woman with subclinical myotonic dystrophy presents to the high risk obstetric
clinic. She is 25 weeks pregnant in her first pregnancy and otherwise well. She hopes for a normal vaginal delivery.

Describe and justify your recommendations for the management of her analgesia for labour and the perioperative management of any potential operative delivery.
A 26 year old woman with subclinical myotonic dystrophy presents to the high risk obstetric clinic. She is 25 weeks pregnant in her first pregnancy and otherwise well. She hopes for a normal vaginal delivery.
Describe and justify your recommendations for the management of her analgesia for labour and the perioperative management of any potential operative delivery.

Myotonic dystrophy
• Most common inherited myopathy in adults
• Multisytem disorder presenting with:
o Myotonia (incomplete relaxation, labor can trigger)
o Wasting
o Respiratory insufficiency : diaphragmatic weakness and expiratory muscle involvement, restrictive lung pattern (compounded by effect of gravid uterus)
o Bulbar palsy, aspiration risk (compounded by GORD of pregnancy)
o Cardiac conduction abnormalities
o Cardiomyopathy : usually dilated
o Septal and valvular defects : MV prolapse in 20%
o Endocrine dysfunction : hypothryoidism, diabetes,
o Intellectual impairment
Preoperative assessment
• Focused history/examination looking for features of above. Particularly: current medications (eg. phenytoin, procainamide), any exacerbations, hospitalisations, need for ventilation/intubation. Swallowing/breathing difficulties. Exercise tolerance with features of cardiac failure/respiratory insufficiency.
• Investigations:
o ECG and TTE (LV function, MVP?) to exclude conduction defects and myocardial dysfunction and valvular defects
o CXR/spirometry and ABG to assess respiratory system (? restrictive deficit, evidence of consolidation/collapse.)
o EUC, TFTs, blood glucose.
• Requires early anaesthetic plan, high risk

Labour plan
• Increased risk of respiratory depression/oversedation with opioids- early epidural analgesia preferable.
• Alerting anaesthetist when admitted. Senior consultant involvement early.
• Myotonia may be precipitated by pain/labour- early epidural desirable to reduce (but will not completely remove risk)
• Extra monitoring during labour should include CTG (risk of hypertonic uterine contractions) and pulse ox.

Operative issues
• Increased risk of bleeding- dual large bore IV access. Blood crossmatched and available.
• Risk of severe respiratory insufficiency if high block- planned GA may be preferable if severe respiratory involvement although balanced against risk below:
• GA issues:
o Invasive blood pressure monitoring and 5L ECG for significant CVS impairment
o suxamethonium- precipitates myotonia, K+ efflux. Avoid.
o RSI advisable- bulbar palsy additional to usual pregnancy issues.
o Non-depolarisers safe but may not lead to muscle relaxation. Rocuronium 1.2 mg/kg RSI with sugammadex available but have backup plan in case of inadequate muscle relaxation (eg. proseal)
o Increased response to sedatives/hypnotics.
o Volatiles have greater myocardial depression
o Diathermy may precipitate myotonia also.
o If spasm- direct infiltration of LA or phenytoin/quinidine
o Temperature control

Post-op
• Observe in HDU/ICU environment
• Increased post-partum bleeding risk
• Keep warm- shivering may precipitate myotonia
• Analgesia if possible by regional or local block, best to avoid opiods due to increased sensitivity to depressant effects
May 2010 Question 6

A 40 year old man with hypertrophic obstructive cardiomyopathy (HOCM) presents for elective laparoscopic cholecystectomy.

a. Describe the principles of intraoperative haemodynamic management for this patient. (40%)

b. How would you manage hypotension post induction of general anaesthesia in this patient?
(60%)
Hypertrophic obstructive cardiomyopathy is a condition where there is dynamic obstruction of the left ventricular outflow tract during systole. Main feature is asymmetric hypertrophy of the interventricular septum, obstructing LVOT when it contracts.

Ventricular systole associated with movement of the anterior mitral valve leaflet towards the septum (systolic anterior motion) causing further outflow tract obstruction may also occur. This may cause mitral regurgitation in some patients.

HOCM results in a pressure overload of the LV leading to LVH and diastolic dysfunction.

Intraoperative haemodynamic management is based on the severity of the condition and the consequences of the procedure.
• Assessment of severity of disease – symptoms, functional exercise tolerance, past and current treatment, recent investigations including echocardiograms.
• Treatment is often with beta-blockers or verapamil (negatively ionotropic).
• Dual chamber pacing or AICD may be required as patients are prone to arrhythmias (AF/VT/VF) which are refractory to medical treatment and a cause for sudden death.

Principles of intraoperative haemodynamic management include minimising dynamic obstruction by having:
• Adequate volume loading / full ventricle
• High preload
• High normal SVR (alpha agonists eg metaraminol)
• Slow heart rate and sinus rhythm (crucial for maintaining ventricular filling)
• Low ventricular contractility (avoid ionotropes)
This requires invasive monitoring with arterial line and measurement of CVP or use of an oesophageal Doppler to help guide volume status.

Consideration of the surgery specific needs and physiological consequences include:
• Pneumoperitoneum (depending on pressures (>20mmHg) will decrease venous return (IVC compression) and cardiac output and BP.) Pneumoperitoneum may increase SVR directly as well as indirectly through increase sympathetic stimulation / increased catecholamines → increases LVOT.
• Reverse Trendelenburg position (decreased preload, CO and BP)
• Carbon dioxide absorption (increases catecholamine levels → tachycardia and increased contractility; may lead to vasodilatation and decreased SVR)

b. Hypotension post induction can be multifactorial and differential diagnoses include
• Cardiac causes – decreased preload, decreased SVR, exacerbation of LVOT obstruction, myocardial ischaemia, arrhythmia (AF/VT/VF)
• Non-cardiac causes – anaphylaxis, effects of pneumoperitoneum (increased vagal tone, decreased cardiac output), effects of reverse Trendelenburg, venous gas embolism, pneumothorax/pneumomediastinum, haemorrhage due to incorrect trocar insertion

Management of hypotension:
• Confirm BP, feel pulse
• Simultaneous diagnosis and treatment of hypotension depending on severity
• Further volume loading 20mls/kg and iv metaraminol 0.5mg titrations (alpha agonist)
• Review monitors and quick check of ABC, in particular ECG – if arrhythmia eg new AF with haemodynamic compromise or VT/VF → defibrillate and CPR.
• Optimise myocardial oxygen supply and demand by ensuring adequate analgesia with opioids to minimise sympathetic stimulation and ionotropy. Increase BP and decrease HR (increases diastole time and coronary perfusion).
• Consider beta blockade if LVOT obstruction due to increased contractility suspected and high HR.
• Review stage of surgery – if hypotension related to onset of pneumoperitoneum inform surgeons and decrease pressure or change position from reverse Trendelenburg to Trendelenburg.
• If severe hypotension and immediately life-threatening, call for help and alert surgeons, ensure ABC optimised with 100% O2 and discontinue anaesthetic.
• If available review oesophageal Doppler monitoring to assess volume status (aim for FTC > 300) and consider on table TTE/TOE to delineate between LVOT obstruction, hypovolaemia and ischaemia.
May 2010 Question 7

A 43 year old female with a Grade 1 subarachnoid haemorrhage is scheduled for coiling of her
middle cerebral artery in the radiology suite.

Discuss the important issues to consider when providing anaesthesia for this patient.
Main issues:
• Remote site anaesthesia
• Complications of SAH- cardiovascular, neurological, respiratory
• Neuro-anaesthetic, with attention to issues of ICP, haemodynamic stability
• Prevention of post-operative complications (incl. vasospasm, re-bleeding)

Pre-operative
• Adequate history: onset, headache, focal neurological deficit, CVS complications eg. chest pain, palpitation, SOB- pulm. oedema, routine anaesthetic history (past anaesthetics, co-morbidities, allergies, medications, fasting status, GORD)
• Examination- GCS (Grade 1 SAH of WFNS scale = GCS 15 with no neurological deficit), focal neurological deficit (document pre-op neurological deficits to compare to post-procedure), signs pulmonary oedema/RHF, airway examination
• Investigations- FBE, UEC (electrolyte derangement- hypoNa and hypoK common), coags, G&H. ECG (strain pattern, ischaemic changes), CXR (neurogenic APO), CTB (hydrocephalus, raised ICP, Fisher grading of SAH to stratify risk of vasospasm/delayed ischaemic neurologic deficit):
o Grade 1 (no SAH blood)- vasospasm risk 0%
o Grade 2 (diffuse layer SAH blood)- risk 60%
o Grade 3 (focal clot)- risk 90%
o Grade 4 (intraventricular blood)- risk 0%

Intra-operative
• Remote site- issues include cold environment, unfamiliar location, lack of skilled assistance, lack of equipment (eg. difficult airway trolley, syringe drivers), cramped environment., poor lighting
• Environment- cramped, radiation hazards (lead, limit exposure in room), C-arms limit access to airway
• Monitoring- as per ANZCA PS 18, incl. IABP, 5 lead ECG, NMS as profound blockade crucial. BIS monitor not possible (interferes with image acquisition). Temperature monitoring.
• Propofol/remifentanil TCI good option for minimal cerebral autoregulatory disruption.
• Normotension, normothermia with normocapnia goals. Avoid haemodynamic perturbations at induction and intubation (eg. bolus lignocaine 1.5 mg/kg), adequate remifentanil dose prior to intubation. Vasopressor to hand to maintain BP.
• Heparinisation required- ACT machine nearby. Aim ACT 2-3 x normal. 70 U/kg when radiologist requests.
• Normoglycaemia important for cerebral protection- hourly BSLs, may require insulin
• If aneurysmal rupture- quick decision ?can manage interventionally or require urgent transfer to neurosurgical theatre. Reverse heparin, keep BP at pre-rupture levels.

Post-operative
• Low grade SAH can probably be extubated (ie. in this case) if routine coiling and criteria for extubation met and cared for on neuro-HDU
• Transfer to recovery in appropriate fashion (PS 20).
• Risk of vasospasm- peaks 4-10 days post. Nimodipine 60mg qid orally/NGT should be started (ARR 5%, NNT = 20). Triple H therapy largely unproven but still widely adopted. Aim CVP 8-12, HCt 30-33%, SBP 180 mmHg in secured aneursym. Surveillance includes clinical (decreased GCS, focal deficit) and monitoring (EEG, trans-cranial doppler, CT angiography)
• Risk of re-bleeding
• Risk of puncture site haemorrhage- reversal of heparin (protamine) and pressure
• Risk of femoral artery damage- dissection, aneurysm, distal ischaemia
Question 8
a. Describe the anatomy of the eye relevant to a sub-Tenon's eye block. (40%)

b. Discuss the potential advantages and disadvantages of this technique for providing regional anaesthesia for eye surgery. (60%)
The sub-Tenon’s block involves obtaining surface anaesthesia, gaining access to the sub-Tenon’s space, insertion of a cannula and subsequent administration of local anaesthetic agent into the sub-Tenon’s space. Therefore, knowledge of globe anatomy, especially Tenon capsule and the surrounding structures is particularly important before embarking on this technique.

• The orbit is an irregular 4-sided pyramid with its apex pointing posteriomedially (known as the Annulus of Zinn) and its base facing anteriorly
• The base is formed by the surface of the cornea, the conjunctiva and the lids
• Globe movements are contoled by the rectus muscles (inferior, lateral, medial and superior) and the oblique miscues (superior and inferior), they arise from the Annulus of Zinn and interior anterior to the equator forming an incomplete cone
• The Annulus contains
o Optic Nerve (II)
o Oculomotor Nerve (III) : superior branch supplies the superior rectus and levator palpebrae; inferior branch supplie medial rectus, inferior rectus and inferior oblique muscles
o Abducens nerve (VI) : supplies the lateral rectus
o Nasociliary nerve (branch of V)
o Ciliary ganglion and vessels
• Trochlear nerve (IV) : supplies the superior oblique and is outside the Annulus. Retained activity of this muscles frequently observed as failed flock.
• Subtenon's capsule is a fascial sheath that envelopes the globe and separates it from the orbital fat
• Anteriorly it attaches to the limbus of the eye, posteriorly it is pierced by the extra-ocular muscle sheaths before fusing with the dura around the optic nerve
• Space between Tenon's capsule and sclera is the "episcleral" space, or "Sub-Tenon's" space
• Anaesthesia is produced by blockade of short ciliary nerves as they traverse the Sub-Tenon's space to supply the globe
• Akinesia is produced by flow of local anaesthetic directly into muscle sheaths
• Visual blockade is produced by direct flow of LA into dura invaginating optic nerve posteriorly
• Sensation to cornea, perilimbal conjunctiva and superonasal quadrant of conjunctiva is is via the nasociliary (V1) nerve. The remainder of the peripheral conjunctiva is supplied by the lacrimal (V1), frontals (V1) and infraorbital (V2) nerves.


PART B

Advantages
• Greater akinesia than topical or subconjunctival local anaesthetics
• Rapid onset of analgesia
• Good and reliable akinesia over 5-15 minutes
• Consistently reproducible results
• Less pain compared to peribulbar or retrobulbar
• No sharp needle placed within orbital cavity
o Reduced risk of globe perforation
o CNS injection, subarachnoid injection is improbable
o Unlikely intravascular or intraneural injection
• Method of choice for anticoagulated patients
o Less likely to cause bleeding/retrobulbar haemorrhage
o No need to cease anti-coagulation (therapeutic warfarinisation for relevant condition)
• Can perform block in any of four quadrants

Disadvantages
• More invasive than subconjunctival or topical application of local anaesthetics
• Skilled technique- requires
• Requires dedicated equipment (Moorfield's forceps, specific cannula). Expense and lack of availability may prevent block.
• Common to cause chemosis of conjunctiva (rarely interferes with surgery)
• Cosmetically less appealing
o Subconjunctival haemorrhage in 32%
o Easily managed by surgical diathermy
• Reflux of local anaesthetic on injection
o Ballooning of conjunctiva heralds incorrect tissue plane of local anaesthetic
• Difficult to detect end point of dissection (plane continues to optic nerve)
o Comes with experience
• More difficult in myopic patients
o Longer globe with staphyloma
• Contraindicated in scleral disease with possible scarring/friability of sclera
o Previous retinal detachment surgery
o Scleral buckles, adhesions may hinder spread of solution
o Increased risk of globe perforation in quadrant dissected
• May interfere with glaucoma surgery
• Some patients may require sedation to tolerate
May 2010 Question 9

a. Describe the factors that influence emergence delirium in children. (50%)

b. How would you manage emergence delirium in a 3 year old child having had myringotomy tubes inserted under general anaesthesia? (50%)
PART A
Emergence delierium is a dissociated state of consciousness in which the child is irritable, uncompromising, uncooperative, incoherent, and inconsolably crying, moaning, kicking, or thrashing.
Usually occurs within 30 minutes of recovery with resolution within 5-15 minutes but has been documented to last 2 days.
Incidence 12-13% in children, 6% in adults
Anaesthetic Related Factors
• Rapid Emergence
o Postulated that rapid awakening after use of volatiles, especially with sevo=des>iso>halothane
o However, recovery from propofol which is also rapid, is smooth and pleasant : Protective
o Delayed emergence by a slow, stepwise reduction in sevo concentration did not reduce the incidence
• Intrinsic characteristics of an anaesthetic
o Sevo : pro-convulsive metabolites, irritant CNS properties, epileptiform EEG
o Des : no proconvulsant properties, but similar rates of delerium
• Adjunct medication
o Protectie effect from fentanyl, clonidine, dexmedetomidine, or propofol bolus at the end of surgery
o No effect from midazolam ?hyperstimulation
o Contributory effects on post-op delerium with droperidol, antocholinergics, barbituates
Surgical Related Factors
• Pain
o Possible confounder
o Inadequate pain relief especially after short procedures
o Preemptic analgesia reduces incidence, fentanyl 1-2mcg/kg, clonidine 2mcg/kg
o However, can occur after non painful procedures such as MRI
• Type of surgery
o Surgical procedures that involve the tonsils, thyroid, middle ear and eye, head and neck surgery
Patent Related Factors
• Age
o Peak incidence 2-5 years
o Thought to be psychological immaturity coupled with rapid awakening in a strange environment
• Preoperative anxiety
o Intense preoperative anxiety in both children and their parents
• Temperament
o More impulsive, emotional, less social, less adaptable to environmental changes

PART B
• Prevention-
o Premed with oral ketamine, clonidine or fentanyl
o TIVA with propofol
o Switching sevo/des to iso before emergence
o Pre-emptive analgesia : caudal
o bolus of propofol (1 mg/kg), fentanyl (1 mcg/kg) or clonidine (0.5-1 mcg/kg) prior to emergence
• Assessment
o Rule out other causes (eg. hypoxia, pain, hunger, thirst, too cold/hot, anxiety). Baseline obs including HR, O2 sats. Treat reversible causes (eg. supplemental O2, feed, analgesia, parental presence)
o Rating scales eg. PAED scale may help differentiate ("Paediatric Anaesthesia Emergence Delirium"). 5 psychometric items.
• Environment
o Protect the child from self injury : Padding over rails/hard surfaces, may require physical restraints
o Minimise stimuli (noise, handling, light). Quiet, darkened corner of PACU appropriate
• Parental reassurance- common, self limiting phenomenon with no adverse effects
• If severe/patient at significant risk of harm, consider treatment:
o Clonidine 0.5-1 mcg/kg
o Fentanyl 1-2 mcg/kg IV
o Propofol 0.5-1 mg/kg IV
o Dexmedetomidine 0.5 mcg/kg
May 2010 Question 10

a. Describe the pathophysiological effects of an inhalational injury following a house fire. (60%)

b. What implications would this have for anaesthesia one week after the injury? (40%)
1. Inhalational injury
-History: entrapment, LOC,
-Exam: sooty nostrils/mouth/sputum, stridor, hoarse, ʻbrassyʼ cough, tachypnoea
-Hot gas will damage mostly ABOVE the larynx (chemicals BELOW)

2. Thermal injury may coexist - much more obvious
-Disruption of cutaneous integumentary system ie barrier function
-blood loss, fluid loss, hypothermia, infection

3. Toxins
*Carbon monoxide
-Prevents Hb O2 carriage (causes cytotoxic hypoxia) despite normal pO2
-disrupts pulse oximetry, need 100% O2 via rebreather to decrease CO-Hb
-Higher CO-Hb correlates with neurological signs
-15%, Smoker, 20% = confusion, 40% convulsions; > 60% = death
*Cyanide, phogene, HCl, complex inorganic compounds from burnt plastics
*Bronchospasm, pneumonitis & V/Q mismatch may progress to ARDS
*Inflammatory storm: Histamine, PGs, leukotrienes, TxA2 etc released
-↑ vascular permeability = generalised oedema & ↓ IV albumin

Early management of serious burn
-Assess airway, breathing (circulatio) GCS (?intoxicated)
-immediate intubation - ETTs in various sizes, uncut (massive facial oedema)
-Early NGT and invasive lines helpful before oedema worsens
-Fluid management - parkland formula crystalloid
-> 10% burn = 2 ml/kg/TBSA 1st 8 hours, further 2 ml/kg/TBSA next 16 hours

Transfer to burns unit if:
->10% TBSA adult, >5% TBSA child or full thickness
-Face, hands, feet, genitalia, perineum or major joints
-Electrical, chemical burn, Inhalational injury
-Circumferential
-Extremes of age
-Pre-existing brittle medical disease

Management of Burn one week after injury/Issues
Airway
1. may still be intubated, tracheostomy,
2. difficult airway : tracheal strictures, distorted anatomy, reduced mouth opening
3. opiods : ?gastric statisis

Breathing
1. Hypermetabolic : requiring higher ventilation
2. Reduced chest wall compliance
3. ARDS? ALI?

Circulation
1. access : difficult, may have long lines
2. fluid balance : assessment and monitoring, UO>1ml/kg/hr
3. inotropes
4. blood products

Other
1. Analgesia
2. Neuromuscular blockade
3. Temperature regulation
May 2010 Question 11

A 34 year old, opioid-­dependant woman is complaining of severe pain on the day after a first
metatarsal osteotomy. The nurses are concerned she is drug-­seeking.

a. How would you assess this patient? (60%)

b. Outline your pain management plan. (40%)
Statement : I would take the patient’s complaint of pain seriously and assess the patient immediately .
History
• Review of surgery and anaesthetic record : intraoperative analgesia administered ?multimodal ?opiod sparing techniques
• Review of ward drug chart
• Characterise pain : intensity, VAS, site, radiation, FAS, etc
• Illicit drug history : timeline, drugs, last known abuse, current abuse ?drowsy, absenteeism, vistors ?observed tampering with IV lines, PCA or Drug infusions
• Collateral history from primary prescriber/drug and alcohol : compliance, drug dosing
Examination :
• Evidence of opioid overdose: pinpoint, bradypnea
• rule out surgical causes/complications
• bleeding/haematoma
• ischemia/infarction
• DVT
• Nerve damage
• infection

Pain management
• Explain & negotiate pain management plan with patient and nursing staff
• Multi-disciplinary approach : involve specialist care : APS, CPMS, D&A and surgeon
• Biopsychosocial approach
• Multimodal analgesia
o Opiod PCA for 24 hours
o NSAIDs eg ibuprofen 7.5-10 mg/kg tds ac (care with IHD, renal dysfunction, PUD)
o Tramadol
o Paracetamol 15 mg/kg/dose - max 4 g /day, care > 7 days & hepatic dysfunction
o Ketamine 0.05-0.1 mg/kg/hr IVI
o Clonidine 1-2 mcg/kg po/IV (bd / tds / nocte)
• Regular APMS review
• Early discharge planning : back to primary prescriber, D&A
May 2010 Question 12

a. Describe the aims of a quality assurance program. (40%)

b. Outline the steps you would take to set up a quality assurance program for your anaesthesia
department. (60%)
Definition:
ʻan organised process that assesses and evaluates health services to improve
practice or quality of careʼ
Aim is to maintain (or establish) acceptable standards - often outlined by relevant
policy documents outlined by the responsible authority (ie ANZCA)
-all anzca trainees and fellows should be involved in QA

Process - 4 steps - repeat continually for ongoing QA
1.Planning (QA design, preparation)
-ie topic being evaluated, data collected, methods to collect, analyse
2.Implementation (Collection & review of data)
-Monitor & evaluate quality
-Identify areas of deficiency or risk
-Implement changes & continue to collect/review data (monitor effects of changes)
3.Review
! - Monitor outcomes of changes implemented - ʻcloses the loopʼ
4.Setting Standards
! - Incorporating changes into official policy

Examples include
*Anaesthesia service structure & performance (compare with equivalent services)
-Staff - numbers, qualifications, process and selection for appointment, workload,
supervision, education activities

*Physical facilities - equipment, compliance with standards, maintenance &
replacement; service space; facilities for teaching, education & research.

*Management - budgets, expenditure and cost effectiveness

*Audit
-Performance evaluation compared to reported outcomes of peer groups

*Clinical guidelines & protocols
-Check for compliance to published guidelines and recommendations

*Critical incidents

*Risk management

*Peer reviews of performance
-Evaluation of communication with patients / relatives, patient selection,
anaesthesia techniques
-Methods: M&M, randomly selected case reviews, direct practice review by a peer

*Patient surveys

*Root cause analysis

*Reporting to national / state bodies including:
-VCCAMM, Adverse reaction committees, sentinel events, critical incidents etc

*Audit of QA
Example of QA ʻright operation, right patient, right sideʼ
-Introduce ʻtime outʼ to decrease critical events.

*Planning
-Do official forms exist for the time out / consent?
-Are staff aware that they are mandatory to be completed before operation?
-Working party to design form, who will write on it?
-Literature review, collect data on incorrect operation, patient or side
-hospital or state authority - ʻpre-interventionʼ complication rate

*Implementation
-period of education and ʻroll outʼ forms on a trial basis
-Research nurses collect forms in recovery and collate data - completion
-Did ʻright operation, patient, sideʼ occur? Form completed correctly?

*Review
-random unannounced spot checks of all theatres
-with research nurses in anaesthetic room / theatre
-observe checks being made
-assesses individual compliance with time out (surgeon, anaesthetist, nurse)
-Does rate of compliance change with random spot check review
-may take weeks to months
-Does incidence of incorrect operation, side, or patient decrease with
implementation? - may take months to years

Setting standards
-How can the forms/process be improved
-ʻtrialʼ form improved to ʻpermanentʼ
-state wide / national / profession wide form?
-Policy: eg transport of patients from holding bay to anaesthetic rooms / theatre
BLOCKED until form complete
-implementation to prevent incomplete/incorrect form
May 2010 Question 13

a. What do you understand by the term "Universal Precautions"? (40%)

b. Describe how you apply these precautions in your daily anaesthesia practice. (60%)
a) Universal precautions are infection control techniques that are recommended to minimize the risk of transmission of blood borne infections(disease carried by blood or other body fluids) by avoiding contact with patients’ bodily fluids.
- recommended for all staff who may come in contact with patients or bodily fluids directly or indirectly.
- aim is to protect staff and patients
- every patient is treated as potentially infectious

Standard precautions is a new term used for an expansion of universal precautions recognizing that any body fluid may hold contagious organisms. They are still primarily designed to prevent the spread of bloodborne disease so in essence the term is used interchangeably with universal precautions.

Infectious bodily fluids are considered to be:
- blood
- semen/vaginal secretions
- cerebrospinal fluid
- amniotic fluid
- synovial fluid
- pleural/peritoneal/pericardial fluid


Bodily fluids that do not require such precautions:
- faeces
- urine
- vomitus
- nasal secretion/sputum
- saliva
- sweat

b) Universal precautions:
- Immunization (Hepatitis B with regular booster, Influenza) of staff
- Hand washing – before and after coming in contact with patients. Appropriate disinfectant and hand washing technique important.
- Barrier technique – gloves when handling bodily fluids, face shields when there is danger of fluids splashing onto mucous membranes
- Sterile procedures – sterile gloves, gown , mask and eyewear – eg: epidural insertion/ CVC insertion
- Correct sharps handling – do not recap needles, direct disposal into puncture resistant container, transfer sharps in tray if needed
- Correct disposal of soiled waste/waste with blood – appropriate containers provided
- Environmental control including design and maintenance of premises, cleaning and spill management
- Education of all staff in correct hygiene practices
- Identification of high risk patients
- Communication between staff members re: high risk patients (Eg: high viral load)
- Protocol in case of needlestick/contact with bodily fluid – wash area with water, immediate testing of staff member and patient (including three months later for HIV seroconversion), prophylactic antivirals for HIV.

Anaesthesia related:
- ensure compliance of all other staff
- disinfection of re-usable items eg: laryngoscopy blades, masks
- appropriate disinfection of surfaces contaminated with bodily fluids eg: anaesthetic machine and trolley
- use if disposable items eg: guedel airways, LMA, ETT
- single use of drug ampoules, vials and syringes
- placing drugs on sterile tray
- capping syringes containing drugs

Additional precautions:
Used in addition to universal precautions for patients who are known or suspected to have an infectious condition. Vary depending on the infection control needs of that patient.
These additional precautions are NOT needed for blood borne infections, unless there are complicating factors.
Conditions requiring additional precautions:
- Prion disease (eg: using mainly disposable equipment)
- Air borne disease (eg:TB – masks, patient isolation)
- Droplet borne disease (Eg:mumps, influenza, rubella)
- Direct contact with dried skin (Eg: MRSA)
May 2010 Question 14

A 58 year old man presents for tonsillectomy for a tonsillar tumour. He has a 2 year history of
intermittent palpitations. His electrocardiogram at diagnosis shows the following

a. What is the diagnosis? Describe the electrocardiographic changes that support your diagnosis. (30%)

Following the administration of neostigmine and atropine for reversal of neuromuscular
blockade, you see the following rhythm on your monitor.

b. What is this rhythm? How would you manage this situation? (70%)
A) Wolff Parkinson White (WPW) syndrome
- Incidence: 0.1-3%
- WPW can lead to a type of reentrant (narrow complex)tachycardia due to aberrant accessory pathway between the atria and ventricles (called the Bundle of Kent). This pathway bypasses the AV node hence slowing of the atrial initiated rhythm does not occur.
- Most patients are asymptomatic – some may have intermittent palpitations, dizziness, shortness of breath or syncope during tachycardia.

ECG changes:
When in normal sinus rhythm:
- short PR interval
- Widened QRS (but still less than 0.12 secs)
- slurred upstroke of QRS – delta wave
- ST – T wave changes – non specific

Electrical activity generated in the SA node travels through the accessory pathway prematurely (no AV node slowing) which causes the short PR interval. The impulse via the accessory pathway first activates the ventricles followed immediately by the AV nodal pathway – causing the slurred upstroke. Changes in repolarisation cause the ST T wave changes.

B) Ventricular fibrillation (some people confused it with torsades de pointes but think simple during an exam)

This is a medical emergency – Inform surgeon and staff and press emergency button to summon more help – activate emergency response
Team effort – allocate roles to each available person
Shockable rhythm in witnessed VF arrest – priority: defibrillation

1) Ask for defibrillator and commence effective CPR immediately -
- 100 compressions/min
- depth of at least 5cm ie, 1/3-1/2 of patient’s chest
- allow complete recoil of chest post each compression
- avoid interruptions to compressions and avoid excessive ventilation
- rotate compressors every 2 mins
- if ETCO2<10mmHg or diastolic BP<20mmHg – improve CPR quality

2) Airway and Breathing
- patient still intubated – hence airway secure. Check ETCO2 trace or bilateral chest movement
- Fio2 of 1.0. Can IPPV (8 breaths/min) or assist ventilation if patient spontaneously breathing

3)Attach defibrillator – assess rhythm – VF
Single shock: 360J monophasic; 200J biphasic

4)Continue CPR immediately for 2mins (5 cycles of 30:2)
Verify IV access

5)Check rhythm – if still VF
- repeat shock
- continue CPR immediately for 2 mins
- adrenaline 1mg IV – repeat every 3-5mins (NB: can replace adrenaline with vasopressin 40units if 2nd dose not effective)

6) Check rhythm – if still VF
- repeat shock
- continue CPR immediately for 2 mins
- amiodarone 300mg IV (NB:second dose is 150mg)

7) Go back to step 3. If nonshockable rhythm develops then continue CPR for 2 mins without shock and reassess again. Also continuous assessment of return of spontaneous circulation – BP and pulse

8)Whilst performing above
- Check electrode position
- Correct reversible causes
- Hypoxemia
- Hypovolemia - ?blood loss
- Hypothermia/hyperthermia
- hypokalemia/hyperkalemia/ other elect abnormalities
- Tamponade (unlikely)
- Tension pneumothorax
- Thromboembolism – AMI/PE
- Toxins/ drugs ?atropine induced tachycardia
- antiarythmic
- magnesium 1.5mg/kg
May 2010 Question 15

a) What is the physiological basis of preoxygenation?

b) Describe your method of preoxygenation including how you assess its adequacy.
Physiology of preoxygenation:

During apnoea, oxygen stores are limited to the oxygen in the lungs and in the blood.
Amount of oxygen in the blood depends on blood volume and Hb concentration.
Amount of oxygen in the lungs depends on alveolar concentration of oxygen and the FRC. (Functional residual capacity is the volume of air that remains in the lung at the end of expiration)

Whilst breathing room air, total stores of O2 in blood and lungs are small.
FRC – 450ml
Blood (Hb) – 850ml
Dissolved or bound in tissues – 250ml
Oxygen consumption is 250ml/min – hence the amount of oxygen in the lungs would not be sufficient to meet tissue oxygen demands in periods of apnoea- will lead to rapid destauration.

Preoxygenation involves breathing 100%oxygen for 3 minutes through an anaesthetic circuit. This is the time taken to replace nitrogen in the FRC to oxygen using normal tidal ventilation.
Whilst breathing 100%O2,
FRC – 2500 -3000ml (rapid increase in oxygen stores)
Blood – 950ml
Dissolved – 300ml
Although the FRC falls during induction of anaesthesia, the extra oxygen contained within the FRC provides an essential store of oxygen for periods of apnoea that may occur during RSI or difficult intubation.
Patients with a small FRC (infants, pregnancy,obesity) or low Hb conc and therefore smaller oxygen stores desaturate rapidly and preoxygenation is especially indicated in these patients.


To achieve adequate preoxygenation:

- Adequate tight seal with good fitting mask
- Technique
1) Deep breathing for 1.5 or 2mins at 10L/min FGF. Drop in ETCO2 noted.
2) Tidal volume breathing for 3-5mins at 5L/min. Increasing FGF from 5 to 10L/min does not enhance preoxygenation by much.
- Use capnography - Good ETCO2 trace to suggest good seal and normal ETCO2 value (30-40mmHg)
- High flow 02 – minimum 10L/min. Achieve FiO2 of >0.95
- Adequate oxygenation – aim for ETO2 of more than 75-80%
October 2009 Question 1

There is a 70-­year-­old female on your emergency list for an urgent laparotomy. She was involved in a
motor vehicle accident this morning and sustained multiple trauma. Her medications include clopidogrel to cover the insertion of bare metal stents into her coronary arteries 2 months ago.

1. Describe the mechanism and duration of action of clopidogrel. (30%)

2. What are the major considerations for the perioperative period in view of the patient's stent?
(70%)
Clopidogrel is an antiplatelet agent (thienopyridine) that acts by blocking the ADP receptor on the platelet surface irreversibly, resulting in inhibition of platelet aggregation. The duration of action is from 5-10 days and is unrelated to the elimination half life of the drug.

Major considerations for the perioperative period in view of the patient’s stent are:

1) Increased risk of bleeding in a multi-trauma patient with multiple injuries who is on an irreversible antiplatelet agent.
2) The risk of stent thrombosis in the perioperative period especially if the anti-platelet effects are reversed or allowed to wear off.
3) The considerations for the management of multitrauma patients according to EMST guidelines with particular attention to this patient’s age and comorbidities such as IHD.

Management
Preoperative
1) Ensure primary, secondary and tertiary survey completed and issues addressed.
2) Thorough history (context of stent insertion 2 months ago eg recent infarct, comorbidities eg cardiac failure, respiratory disease, other injuries, medications) and examination (haemodynamic stability, assess volume status, cardiovascular including assessing for fluid overload, respiratory, airway, cervical spine clearance).
3) Investigations – Preop ECG (compare with old ECG to ascertain if any new changes), FBE (Hb), UEC (electrolyte disturbance, renal impairment), cardiac enzymes if indicated, Coags (to exclude other coagulation abnormalities given multitrauma and potential blood loss). CXR – to exclude other chest injuries, cardiomegaly / cardiac failure.
4) Ensure availability of blood products. Preop platelet transfusion indicated if bleeding (give immediately prior to surgery)
5) ACC/AHA guidelines 2007 suggest that after a bare metal stent, patients should remain on clopidogrel for a minimum of 1 month and ideally up to 12 months unless the patient is at increased risk of bleeding. In this case given the multitrauma with high risk of bleeding, the clopidogrel should be ceased perioperatively. According to ACC/AHA guidelines (Dec 2009 update), in patients with a BMS or DES for ACS, clopidogrel should be given for at least 12 months. If the risk of morbidity because of bleeding outweighs the benefit of thienopyridine, earlier discontinuation should be considered.

Intraoperative
1) Monitoring – arterial line for continuous BP monitoring, 5 lead ECG (to monitor for ischaemia), consider CVC for accurate fluid status assessment, multiple large bore iv canulaes, BIS (high risk for awareness given multitrauma patient), temp probe. Regular monitoring of Hb (aim Hb > or equal to 100).
2) Rapid sequence induction and relaxant general anaesthetic
3) Avoid tachycardia and hypotension, optimise haemodynamics and oxygenation to avoid cardiac ischaemia.
4) Use of fluid warmer and Bair Hugger to avoid hypothermia and further platelet dysfunction
5) Pressure care and careful positioning
6) Planning of analgesia – TAP blocks and PCA for laparotomy

Postoperative
1) Needs monitoring in HDU/ICU given recent coronary stent and multitrauma with increased risk of cardiac ischaemia and bleeding
2) Close liasing with surgeons and cardiology unit re: recommencement of clopidogrel or supplementation with aspirin if appropriate.
October 2009 Question 2

List the advantages and disadvantages of tight glycaemic control perioperatively in a diabetic patient on
insulin. (30%)

How would you manage the glycaemic control for such a patient having a minor procedure under
general anaesthesia? (70%)
Tight glycaemic control is defined as control of blood glucose to between 4.5-6mmol/L.

Advantages
1) avoidance of hyperglycaemia and its complications such as risks of dehydration, diuresis, ketosis, glycosuria
2) decreased risk of infection, evidence in cardiac surgery of decreased sternal wound infections with good diabetic control

Disadvantages
1) increased risk of potentially dangerous hypoglycaemia
2) increased mortality in ICU patients: NICE-SUGAR trial (NEJM 2009) demonstrated that in the tight control group BSL (4.5-6mmol/L) there was increased mortality and increased incidence of severe hypoglycaemia vs conventional control BSL <10mmol/L. There was no significant difference in the length of ICU stay, length of hospital admission, septic shock, median number of days on mechanical ventilation or renal replacement therapy.
3) Potential increased need for equipment and nursing (eg need for insulin infusion for tight control).

The aim of perioperative diabetic management is to maintain physiological blood glucose levels.

Perioperative management of a diabetic patient on insulin having a minor procedure under GA includes:
1) History and examination including assessment of regular preoperative insulin therapy (type and dose) and control of diabetes (assessment of whether type 1 or 2 diabetes, BSL diary and number of hypoglycaemic episodes, DKA). If poor control consider referral to endocrinologist.
2) Associated comorbiddities and complications of diabetes and ensuring stabilisation of these conditions eg IHD, vascular disease (cerebral, peripheral), systematic review
3) Assess previous response to anaesthesia and surgery and likely magnitude of proposed surgery and post-op fasting.
4) Investigations – BSL, test urine for ketones and glucose, ECG (silent infarcts), FBE (exclude infection), UEC (renal impairment), HbA1C (assess control).
5) Appropriate timing of surgery (first on list ideally to avoid long fasting times).
6) An explicit perioperative insulin regimen that covers the patient’s basal insulin requirements while avoiding hypoglycaemia in the fasting period. This will need to be tailored to each patient depending on their usual insulin. For example a patient on a mixed long and short acting insulin eg novomix 30/70 bd having morning surgery should fast from midnight and present to the day surgery unit at 0700, check BSL and take a half dose of intermediate acting insulin with iv dextrose infusion and regular hourly BSL monitoring.
7) A monitoring regimen with a plan for abnormal results eg anaesthetist / doctor to be notified if BSL < 3.5mmol/L or > 10mmol/L, regular BSL monitoring every hour. This would be acceptable control for this patient.
8) An appropriate post-operative plan to recommence regular therapy once eating and drinking with monitoring of BSL. Given that this is minor surgery, it is likely that the patient will be able to resume oral intake post-op. If this is not the case, they will need their basal insulin requirement with a short acting sliding scale insulin or an insulin infusion. If fasting and on insulin, need iv dextrose.
October 2009 Question 3

A 49-­year-­old woman has just arrived in the Recovery Room following a total abdominal hysterectomy under general anaesthesia. She is agitated and complaining of difficulty breathing.

1. List your differential diagnoses. (40%)

2. How would you determine if this was caused by residual neuromuscular blockade? (40%)

3. What is the role of sugammadex in the treatment of residual neuromuscular blockade? (20%)
Differential Diagnosis
1) Pain
2) Upper airway obstruction (retained throat pack)
3) Lower airway obstruction
4) Lung disease (pneumothorax / pneumomediastinum (especially if procedure was laparoscopic)
5) Chest wall or diaphragmatic dysfunction including inadequate neuromuscular reversal
6) Myocardial ischaemia or infarction
7) Pulmonary oedema / cardiac failure
8) Abnormalities of circulation or oxygen delivery (eg anaemia)
9) Neurological dysfunction including delirium or anxiety
10) Anaphylaxis

To determine the cause it is important to perform a –
a) relevant history (peruse anaesthetic chart for comorbiddities, timing and dosing of any recently administered drugs eg muscle relaxant, concurrent drug administration (eg gentamicin), allergy, reversal given)
b) examination (assessment of airway, presence of stridor, ventilation, air entry, muscle power sustained head lift >5s or strong hand grip > 5s, use of a nerve stimulator TOF assessment).

An acceptable recovery from block occurs when the TOF ratio has reached 0.9 or greater using accelerometry. TOF is when four supramaximal stimuli are applied over 2s. The TOF ratio is the ratio of amplitude of the 4th to 1st response. These patterns of stimulation are usually assessed by visual or tactile means but can be assessed more objectively with mechanomyography, electromyography or accelerometry.
a) In mechanomyography the monitored muscle is fixed to ensure a constant preload and the force of muscular contraction is measured following supramaximal nerve stimulation.
b) Accelerometry/acceleromyography comprises a small mass suspended on a strain gauge within a box attached to the accelerating object to be studied, the acceleration is derived from the force exerted on the strain gauge.
c) Piezoelectric methods of assessing neuromuscular blockade are also used. Piezoelectric crystals produce a charge when compressed or distorted, thumb movement is converted into an electrical signal (piezoelectric sensor in clip placed on patient’s thumb and index finger eg Datex-Ohmeda NMT module).

Clinical signs of reversal are generally not as reliable compared to neuromuscular monitoring. In particular the ability to breathe is not a good indicator of adequate reversal, assessment of sustained muscle contraction is better, eg head lift for >5s.

In patients who are found to be inadequately reversed, give a further dose of reversal.
Sugammadex is a cyclodextrin that binds to aminosteroid neuromuscular blocking agents and inactivates them. It has no effect on other muscle relaxants. The role of sugammadex is limited to the treatment of residual neuromuscular blockade in patient’s who have had rocuronium or to a lesser extent vecuronium and pancuronium only. It has a rapid onset of effect suitable for use when neuromuscular blockade is life-threatening or symptomatic, particularly when neostigmine has reached its ‘ceiling’ effect or is unlikely to work eg with profound blockade. It is costly and allergic reactions have been reported.
Sugammadex dosing moderate block (2 twitch) – 2mg/kg
Deep block (1-2 PTC) – 4mg/kg
Immediate reversal (3 mins post 1.2mg/kg roc) – 16mg/kg
October 2009 Question 4

A 70 year old female had a cardiac arrest after arriving in the Recovery Room following open fixation of
a femoral fracture.

This arterial blood gas was taken after intubation and several minutes of CPR.
NO QUESTION IN THIS STEM TO ANSWER???

59% of candidates passed this question
The following were key components of an answer required to pass this question:
• Recognition of a severe mixed respiratory and metabolic acidosis
• The most likely cause of this abnormality is lactic acidosis from hypoperfusion to the peripheral tissues, in conjunction with absent or hypo-ventilation from inadequate perfusion to the respiratory centre of the brain stem. Artificial ventilation is inadequate or has had inadequate time to remove the accumulated carbon dioxide, and external cmoardiac compression has been too late or inadequate to prevent anaerobic metabolism in the peripheral tissues
• Many possible causes of both respiratory and metabolic acidosis were acceptable including dual pathology, but any aetiology had to explain BOTH components

Comments about how this question was answered
• This question related to basic knowledge of core material that was not evident in many responses
• Most candidates scored well in the first part of the question but failed to answer the second two parts adequately, if at all
• Organised answers scored very well, however poorly organised answers scored very badly even if many of the “facts” appear to have been present in the answer, since a demonstration of their understanding was lacking
October 2009 Question 5

A woman who is 10 weeks pregnant presents to ED with a closed tibial shaft fracture.

1. Classify the drugs used in pain management according to their safety to use at this stage of pregnancy (40%).

2. What are the options available for perioperative pain management in this patient? (30%)

3. What would you recommend? Justify your choice. (30%)
• Analgesic drugs given to pregnant women almost always cross the placenta
• Most drugs are safe, but there are times of concern – period of organogenesis (Weeks 4 to 10 – this patient) and just before delivery
• Where possible, non-pharmacological treatment options should be considered before analgesics are used
• Liaise with obstetrician as required

ADEC Categories:

Drugs that might be prescribed during pregnancy have been categorized according to fetal risk by the Australian Drug Evaluation Committee (ADEC):
• Category A –
o Drugs that have been taken by a large number of pregnant women and women of childbearing age without any proven increase in the frequency of malformations or other direct or indirect harmful effects on the fetus being observed
o Examples – Paracetamol, Bupivacaine, Lignocaine
• Category B –
o Drugs that been taken by a limited number of pregnant women and women of childbearing age, without an increase in the frequency of malformations or other direct or indirect harmful effects on the fetus
o The below subcategories are based on available animal data:
• Cat B1 - Animal studies have not shown evidence of an increased occurrence of fetal damage (Eg Ropivacaine)
• Cat B2 – Animal studies are inadequate or lacking, but available data show no evidence of an increased occurrence of increased fetal damage
• Cat B3 – Animal studies have shown evidence of an increased occurrence of fetal damage, the significance of which is considered uncertain in humans (Example – Celecoxib)
• Category C –
o Drugs that, owing to their pharmacological effects, have caused or may be suspected of causing, harmful effects on the human fetus or neonate without causing malformations.
o These effects may be reversible
o Examples – All opioids, except Codeine (Cat A), nsNSAIDs, Parecoxib, Tramadol, SSRIs, TCAs
• Category D –
o Drugs that have caused, are suspected or may be expected to cause, an increased incidence of human fetal malformations or irreversible damage
o Examples – Anticonvulsants (Phenytoin, Carbamezapine, Valproate, Lamotrigine)
• Category X –
o Drugs that have such a high risk of causing permanent damage to the fetus that they should not be used in pregnancy or when there is possibility of pregnancy

Analgesia options in this patient:
• This patient is at a high risk of developing Compartment Syndrome – increasing analgesic requirement or poorly responsive pain should cause suspicion

Non-pharmacological:
• Psychological interventions:
o Provision of information
o Hypnosis and relaxation
o Attentional techniques
o Cognitive behavioural interventions
• TENS
• Acupuncture
• Heat / cold therapies

Pharmacological:
• Paracetamol:
o Analgesic of choice
• NSAIDS:
o Relatively safe in early and mid pregnancy, but can precipitate fetal cardiac and renal complications in late pregnancy (avoid after 32 weeks)
o Fetal exposure has been associated with persistent pulmonary hypertension in the neonate, and increased risk of premature closure of ductus arteriosus
• Opioids:
o Codeine safest
o Neonatal Abstinence Syndrome requiring treatment occurs in 60-90% of infants exposed to opioids in utero
o Short-term use to treat pain appears safe, at a minimal dose
• Epidural analgesia:
o Provides better pain relief than systemic opioids
o A combination of local anaesthetic and opioid solution has synergistic effect
o Risk of complications with epidural insertion, catheter related complications, and motor blockade
• Regional analgesia:
o Continuous Peripheral Nerve Blockade (CPNB) techniques provide better postoperative analgesia than and reduce adverse effects of opioid analgesia
o This patient would require combined Femoral Nerve and Sciatic Nerve blockade – doubling the incidence of complications and failure rates
o Technically cumbersome and requires trained ward staff and regular followup by prescribing anaesthetist

Recommendation:
• Non-pharmacological techniques may be useful supplementally, but unlikely to be sufficient as sole analgesia
• Regular:
o Paracetamol 1g QID orally – safest systemic analgesic for parturients
o Diclofenac 50mg tds orally – safe in early pregnancy, significant opioid sparing effect
• As required:
o Oxycodone 5 to 10 mg 3 hourly orally, OR
o Intravenous Morphine PCA 1mg with 5 min lockout
• On an as required basis to minimize opioid consumption
• If still significant pain despite above:
o Consider diagnosis of compartment syndrome
o Lumbar epidural catheter placement, with continuous infusion of 0.125% Bupivacaine with Fentanyl 2mcg/ml (Rate ranges dependent on patient body weight)
October 2009 Question 6.

Identify the structures labeled A to H on this normal chest X-ray. (40%)

Describe the arterial blood supply and venous drainage of the myocardium. (60%)
A. Ascending aorta, trachea, superior vena cava, tracheobronchus, right paratracheal stripe (NOT carina)
B. Right pulmonary artery branch, right pulmonary vessels/trunk/vasculature
C. Right atrium
D. Left subclavian artery, left innominate
artery, left brachiocephalic artery, (NOT
brachiocephalic trunk)
E. Aortic knob/arch/notch
F. Left pulmonary artery
G. Atrial appendage
H. Left ventricle

Myocardial Arterial Supply:
• The myocardium is supplied by the Right and Left Coronary Arteries, which arise from the aortic root, behind the right and left cusps of the aortic valve respectively.
o No nutritional blood supply directly from cardiac chambers
• Dominance corresponds to the artery which supplies the AV node and inferior part of the Left Ventricle – Right Coronary Artery dominant in 90%
• Right Coronary Artery:
o Runs vertically downwards on anterior surface in atrioventricular groove
o Gives off vessels to supply principally the Right Ventricle and Right Atrium
o If dominant, also supplies the inferior wall of the Left Ventricle and posterior 1/3 of the septum (via the Posterior Descending Artery).
o Marginal branch – supplies Right Ventricle at inferior border of heart
• Left (Main) Coronary Artery:
o Left Main Coronary Artery divides near its origin into Left Anterior Descending (LAD) and Left Circumflex (LCx) Arteries
o LAD – runs in anterior interventricular groove towards apex and gives off septal perforators to anterior 2/3 of septum, Bundle of His as well as Left and Right Bundle branches. It also gives off diagonal branches to supply anterior Left Ventricular wall.
o LCx – travels along left Atrioventricular groove and gives off obtuse Marginal branches to Left Atrium, and posterolateral branches to Lateral and Posterior walls of Left Ventricle. It also gives off a branch to supply the SA node in <50%.

Myocardial Venous Drainage:
• Coronary Sinus – after passage through capillary beds, ~90% of blood returns to Right Atrium through the coronary sinus.
• Anterior Coronary Veins – also drain in the Right Atrium
• Arteriosinusoidal Vessels – small arteries or arterioles that lose their arterial structure as they penetrate the chanber walls and divide into irregular endothelium lined sinuses, which anastomose with capillaries and with one another to drain into the cardiac chambers.
• Arterioluminal Vessels – small arteries or arterioles that open directly into the atria and ventricles.
• Thebesian Vessels – small veins that connect capillary beds directly with the cardiac chambers and also communicate with cardiac veins and other Thebesian veins.
October 2009 Question 7.

A 27 year old man presents with a glioblastoma for a craniotomy. As part of your anaesthetic techniquie, you decide to use a remifentanil infusion.

1. Discuss the characteristics of remifentanil with respect to its use as an infusion (50%)

2. What are the advantages and disadvantages of using effect site calculations to guide remifentanil infusions? (50%)
Remifentanil is a potent synthetic phenylpiperidine opioid characterised by its non-organ dependent metabolism and rapid offset.
• Plasma esterase metabolism of remifentanil is indepent of plasma cholinesterase levels
• Non-organ dependent metabolism makes offset reliable in disease states
• Rapid equilibration of plasma-effect site makes onset rapid and predictable
• Easily titratable due to rapid onset/offset
• Context sensitive half time is stable at about 3 minutes regardless of infusion duration (as compared with other opioids/volatiles with saturable kinetics) (CSHT = the time taken for plasma concentration to decrease by half after cessation of an infusion, in the context of the infusion duration).
• Inter-individual variability is thus reduced
• Potent and profoud blockade of sympathetic responses to nociception is ideal to minimise haemodynamic changes during neurosurgery
For these reasons, remifentanil is an ideal drug for use as an infusion for a long operation

Advantages of effect site modelling:
• Accounts for not just variations in age and lean body weight (as do TIVA models based on plasma concentration), but also differences in time to reach equilibration between plasma and effect site levels
• This results in less variability in drug effect (ie. more predictable effect from a given dose)
• Ideal drug for neurosurgical case- potent opioid thus can be used as sole analgesic for maintenance phase.

Disadvantages:
• Some inter-individual variability still exists which is mainly pharmacodynamic (ie. the same modelled effect site concentration will not always correlate with the same pharmacodynamic effect in different individuals)
• Thus calculations are only a guide. Clinical effect must still be utilised.
• No prolonged analgesia- must provide for towards end of case (eg. morphine 5-10mg 20-30 mins before end of case). Harder to titrate analgesic requirements prior to emergence
October 2009 Question 8.

Outline the factors that determine oxygen delivery to the tissues (30%)

How might you increase the oxygen delivery to tissues in the anaesthetised patient (40%)

How does a hyperbaric chamber influence oxygen delivery to the tissues (30%)
1. Oxygen flux equation determines oxygen delivery to tissues:
Oxygen delivery/min = [ CO x [Hb] x SaO2 x 1.34 ] + [ 0.003 x paO2]

Thus the important determinates are:
1. Cardiac output. The oxygen content of blood is of no importance if it is not delivered to tissues.
2. Haemoglobin concentration. Each 1g of Hb can bind 1.34 mL of O2
3. Oxygen saturation
4. paO2. Dissolved oxygen content is increased by increasing partial pressure of O2.

This governs global oxygen delivery. Regional oxygen delivery to tissues is also influenced by:
• Sympathetic tone (ie. increased vasoconstriction will decrease blood flow)
• pH- acidaemia will right shift O2-Hb dissociation curve, with more oxygen given up to tissues
• Autoregulation. Regional hypoxaemia/hypoperfusion will release vasoactive mediators increasing perfusing flow (organ-regulation of own blood flow).
In room air (pAO2 around 100mmHg) the dissolved oxygen content (0.3mLs) is only about 4% of the whole (96% bound to Hb).

2. Increasing oxygen delivery to tissues in anaesthetised patient:
• Increase FiO2. At 100% oxygen, pAO2 will be 663 mmHg. This increases dissolved oxygen to 2mLs. Also will increase O2 saturation.
• Transfusion RBCs. Increased haemoglobin concentration increases O2 carrying capacity (optimal Hb may be 10g/dL- balance between O2 carrying and optimal viscosity to improve microvascular flow)
• Increase CO. Fluid loading (increased preload), inotropes (increase contractility) and optimal SVR (optimise afterload).
• Optimise regional distribution of oxygen delivery
o Temperature control. Prevents vasoconstriction with poor O2 delivery
o Organ perfusion. Avoid abdominal hypertension (eg. excessive pneumoperitoneum pressures)
o Caution with vasoconstrictors (eg. in flap surgery)
o Minimise compartment pressures (eg. ICP control to increase cerebral O2 delivery- avoid barriers to venous drainage, mannitol etc.)

3. Hyperbaric chamber increases oxygen delivery mainly by increasing dissolved oxygen:
• At 3 atm at 100% O2, pO2 = 2280 mmHg. After accounting for pACO2 and water vapour, 54 mLs O2/L of blood is dissolved. Thus all oxygen needs (250 mLs/min) can be delivered by dissolved oxygen (ie. no Hb required)
October 2009 Question 9.

What are the indications for prophylaxis against perioperative bacterial endocarditis? (50%)

Justify your choice of antibiotics (50%)
Patient indications (high risk patients) :
• Prosthetic valves or other prosthetic material used for cardiac valve repair
• Previous infective endocarditis
• Congential heart defect
o Unrepaired CHD including palliative shunts/conduits
o Completely repaired CHD with prosthetic material or device, whether placed by surgery or catheter intervention, during the first 6 months after the procedure
o Repaired CHD with residual defects at the site or adjacent to the site of a prosthetic patch or prosthetic device
• Cardiac transplant recipient with valvulopathy (structurally abnormal valve)

Surgical indications
• Dental procedures involving manipulation of gingival tissue/periapical region of teeth, or penetration of oral mucosa
• Actively infected GI or GU tract prior to procedure
• Procedures involving breach of respiratory mucosa (eg. Tonsillectomy, adeniodectomy)
• Prophylaxis is no longer recommended for prevention of infective endocarditis for GI or GU procedures, including diagnostic esophagogastroduodenoscopy or colonoscopy
• However, in high-risk patients with infections of the GI or GU tract, it is reasonable to administer antibiotic therapy to prevent wound infection or sepsis.
• For high-risk patients undergoing elective cystoscopy or other urinary tract manipulation who have enterococcal urinary tract infection or colonization, antibiotic therapy to eradicate enterococci from the urine before the procedure is reasonable.

The latest AHA/ACC Focused Update on Valvular Heart Disease (2008) made important changes:
• Prior patient indications no longer included: (eg. MV prolapse, isolated ASD secundum, prior valve repair, HOCM)
• Prior surgical indications no longer included: (such as diagnostic procedures involving respiratory/GI tract- TOE, bronchoscopy, gastroscopy)

The rationale for these revisions is based on the following:
• IE is more likely to result from frequent exposure to random bacteremias associated with daily activities than from bacteremia
• Prophylaxis may prevent an exceedingly small number of cases of infective endocarditis (if any)
• The risk of antibiotic-associated adverse effects exceeds the benefit (if any) from prophylactic antibiotic therapy.
• Maintenance of optimal oral health and hygiene may reduce the incidence of bacteremia

Antibiotic choice:
• Should be based around likely infective organisms (eg. oral flora- Strep Viridans, skin flora- Staph. species, urinary tract- gram negative/enterococci)
• Should be administered ideally 30-60 mins prior to procedure
• Oral amoxycillin 2g (children 50mg/kg)
• If no oral route or for theatre: Ampicillin 2g IV (children 50mg/kg)
• If penicillin allergy, oral: Cephalexin 2g (children 50mg/kg) OR Clindamycin 600mg (children 20mg/kg) OR Azithromycin/clarithromycin 500mg (children 15mg/kg)
• If allergic to penicillins and IV: Cefazolin/Ceftriaxone 1g IM/IV (children 50mg/kg) OR Clindamycin 600mg IV (children 20mg/kg)
• (Do not use cephalosporins if history of anaphylaxis/angioedema or urticaria to penicillins)
October 2009 Question 10

1. Describe the differences between biphasic and monophasic manual external cardiac defibrillators. (50%)

2. What is the "synchronise" button for? When would you use it? (20%)

3. List the potential hazards of defibrillation. (30%)
PART 1. Differences between monophasic & biphasic
*Direction of the current
A monophasic waveform defibrillator, the standard of care over the past 40 years, provides a shock with current flow in one direction (from one electrode or paddle to the other).
Biphasic waveform defibrillators incorporate two-way current flow in which electrical current first flows in one direction, then reverses and flows in the opposite direction.

*Amount of energy required is controversial
-↑ success with higher energy levels (Joules) - ie area under the curve
-↑ myocardial damage proportional to Peak current
-↑ shock duration (up to 20 ms) allows same Energy delivery for ↓ Peak current

*Monophasic 360 J is recommended for VF or pulseless VT
-Insufficient evidence for/against biphasic shocks.
-Current evidence suggests that biphasic are at least as effective as monophasic for the termination of ventricular arrhythmias ? better side effect profile

termination of ventricular arrhythmias ? better side effect profile

PART 2.
Synchronise function
-Elective cardioversion to prevent delivery of current during repolarisation and precipitating a ventricular arrhythmia
-prevent R on T
-If accidentally activated for VF - failure to shock will occur

PART 3. Hazards of defibrillation
-Burns
-Biphasic
-less chance of burns, myocardial damage - ʻpeak current flowʼ
-Animal studies: myocardial damage was worse in higher-energy shocks.
-Safety
-Avoid patient contact with metal fixtures (ie bed rails) increases risk
-Do not charge unless paddles/pads on victims chest
-Good contact/placement - prevents sparking
-Paddle placement crucial - away from
-GTN patches and oxygen may ignite, PPM/ICD, central line
insertion sites
-out-of-hospital - hazards eg petrol, water - do not defibrillate
-Care with oxygen
-leave oxygen attached if ETT in situ (0.5 m from defib may ignite)
-DO NOT let oxygen from resuscitator flow onto patients chest
-Muscle & Nerve damage
-Lethal arrhythmia
-May convert SVT / VT to VF
-Failure to shock (ie no motor response to shock)
• Check that synchronise has not been activated (VF it wont shock)
• Recommence CPR with oxygen
• Check paddle or electrode position - is the test load still connected?
• Check that there is adequate skin contact.
• (Clipping/shaving body hair under paddle/pads may be required).
• ? changing the defibrillator pads.
• ? A-P chest placement (so that max current traverses myocardium).
• ? flat battery, charge dump or lead fracture (get another defib)
-Hazard to health care workers
-State loudly your intention to shock with “Stand clear” AND visual check
-ensure no one is in direct contact with the patient or the bed (inadvertent shocking
October 2009 Question 11

What are the key objectives of ethical review of a research project?
Ethical review should be made up by a number of professionals across different domains
-(eg medical, scientist, nursing, lay person, ethicist)

o Methodological validity
-That is, what is being measured is actually reflects reality/fact/truth
-Is there a tool that can measure what can be sought?
-Troponin 12 hours, ICU charts for ventilator-free days etc
-Identify & eliminate biases (confounders, selection, information)
-Numerical / binary data studies can be analysed readily
-cf: free text - how do you feel? not able to be analysed easily
-Are multiple tests / questionairres required for follow up - is this feasible?
-ie transient populations
-Peer review prior to HREC application / may increase this
-Should be an expert, not directly involved with the project

o Justifiable in terms of contribution to knowledge
-Tangible beneficence to patients / improvements in care
-ie NNT 500 for a rare complication is not helpful to the wider community
-Non-repetition of previous studies, ʻcommon knowledgeʼ
-Resources - time, money, staff hours - not wasted

o Respect for persons/patients has been kept
-Beneficence, non-malfeasance, autonomy & informed consent are upheld

o Voluntary informed consent has been obtained
-informed consent (as a concept) should be explained to the subject / guardian
-Provision of information in layman terms able to be understood by subject
-Belief that subject is able to make autonomous decision to ʻopt-inʼ
-Non-coercion - not being forced in any way to take part
-Opportunity to ask questions, clarification if unclear
-provided orally & in writing - so concerns can be brought up later
-Consent may be withdrawn by subject at any time

o Privacy and confidentiality have been respected
-Personal information is kept confidential - strategy for implementing this
-Subjects anonymity preserved by a coded identifiers (NOT a UR/bradmar)
-master list secure and accessible only by chief investigators

o Any potential harm has been minimised (non-malfeasance)
-Interventional vs observational trial
-Particularly relevant to Drug & Device trials
-Observational trial still has potential to harm by not offering subjects interventions
that may prove to be beneficial
-Also important when considering
-RCT - control group may be harmed if a trial is not stopped early (due to
unforeseen benefit)
-Vulnerable / Minority groups (May be confounders, informed consent difficulties)
-Databank or register creation (implications on confidentiality)
-Implications of genetic testing / stem cells (health insurance ramifications)
October 2009 Question Q12

You are asked to give a practical tutorial on paediatric airway management to Emergency Department registrars at a large hospital.

What are the important aspects of paediatric airway management that you would present to them?
Anatomy
-Larynx is higher (C1-4) than adult (C4-7)
-Floppy U shaped epiglottis & prominent arytenoids
-Large head, occiput, Large tongue
-Cricoid, not the cords is the narrowest part of the airway

Physiology
-Prone to hypoxia - FRC < closing capacity = airway collapse
-Higher metabolic rate / kg = higher ventilatory requirement
-Difficulty tolerating mask, applying seal with pre-oxygenation
-NB: Awake fibreoptic / regional technique to avoid direct laryngoscopy not an option

Assessing the airway in a child
-Unco-operative - often unable to assess
-Drooling, swollen tongue, lips
-Syndromes associated with difficult airway important
-Pierre-robin, treacher-collins, goldenhaarʼs, mucopolysaccharidosis, Downʼs etc
-Breathing work may show - IC recession, accessory mm, tracheal tug, head bobbing

Basic airway manoeuvres
-Neutral head position (pillow only required if patient conscious / uncomfortable)
-Open mouth, tongue off roof of mouth if possible
-Jaw thrust more helpful than chin lift/neck extension (also better if neck precautions)
-Airway equipment
-Any device should have available one size bigger and smaller than anticipated
-Guedel airway - sized - corner of mouth to earlobe: NPA - sized nostril to earlobe
-LMA sizes
-0 - 5 kg = 1, 5-10 kg = 1.5, 10-20 kg = 2, 20-30 kg = 2.5, 30-50 kg = 3
-reluctant to use < 1.5 except in extreme contraindication to intubation
-ETT sizes (ID)
-neonate 2.5 (1 kg) - 4.0 (> 3kg), infant/<2 years 3.5-5.0
-over 2 years: based on weight (age/4 +4) approximate size of little finger
-stylet / bougie helpful depending on familiarity with those devices
-ETT Depth
-2-3 cm beyond cords ample (still able to see some of the ʻblack lineʼ above)
-over 2 years: oral age/2 + 12 // nasal age/2 +16
-Laryngoscope blades size 1 & 2, may include curved and straight blades

Intubation equipment and technique in paediatric resuscitation
-Skilled assistant, equipment, environment, drugs etc
-Do i need senior specialist help? should i be doing this in ED ? is OT appropriate ?
-IV very helpful, may need interosseous if difficult to cannulate
-Important drugs in correct doses are available (est Age x 2 + 9 = kgs)
-Propofol 4 mg/kg (in extremis 1 mg/kg may be enough)
-Suxamethonium 1.5-2.0 mg/kg
-Atropine 10 mcg/kg - antisialogogue & suxamethonium bradycardia
-Alternatively Ketamine 5 mg/kg induction IV
-Self inflating bag / ventilator with appropriate settings (PCV - WATCH Volume ++)
-Airway procedures including the surgical airway in a child
-Retrograde wire techniques
-Cricothyroidotomy
-Formal tracheostomy - trachea may be difficult to access

Common paediatric airway scenarios
-Croup
-Upper Airway Foreign Body / Lower airway foreign body
-Epiglottitis
-Anaphylaxis
-Trauma
-Altered conscious state (sepsis, seizure, hypoglycaemia etc
-Always consider the cause of the airway problem - prompt treatment may render
airway support unnecessary
October 2009 Question 13

You see a 28-­year-­old woman at the pre-­admission clinic who is 32 weeks pregnant. She weighs 150kg
and has gestational diabetes. She is hoping to have a normal vaginal delivery at term.

1. What are the issues you would discuss with her during the appointment? (50%)

2. What would you recommend for her management when she goes in to labour? (50%)
PART 1)

HIGH RISK patient due to obesity and associated gestational diabetes
(hyperinsulinemia and insulin resistance common in obesity). Multiple
issues related to both that require discussion:
Assessment: on history and examination
- comorbidities related to obesity
- GORD
- hypertension, CVS disease — IHD, CVA
- obstructive sleep apnoea
- congestive cardiac failure — maternal cardiac output increases by
50ml/min for every 100g of fat gain. Tachycardia causes decreased
myocardial perfusion
- gestational diabetes — level of control, daily BSL levels, meds
- Fatty liver
- poor exercise tolerance
- previous anaesthetic history — grade of intubation, epidurals,
complications
- obstetric history — previous pregnancies and outcome
- Investigations: ECG, Bloods including extended cross match and FBE, U+E,
LFT, coags
Issues during pregnancy:
- gestational diabetes — management with diet, regular meals, glucose
monitoring +/- insulin. Increased risk oftype 2 diabetes later.
- Pre-eclampsia risk increased
- Risk of pregnancy induced hypertension increased
Issues during birth:
- Increased risk of fetal macrosomia related to obesity and gestational
diabetes
- Increased risk of
- failure to progress
- failed induction of labour
- elective and emergency caesarian sections
- shoulder dystocia
- third and fourth degree tear requiring surgery
- VBAC success reduced by 50%
- Difficulty with providing satisfactory analgesia during labour — failed
epidurals common. Options include: N20, IM opioids, early epidural,
opioid PCA.
- Increased risks related to caesarian section
- difficulty moving and positioning patient in an emergency
- difficult IV access
- difficult spinal — may need multiple attempts
- difficult ventilation — due to reduced FRC, obstructive sleep apnoea
(neck circumference) — more likely to desaturate, obstruction risk at induction
and extubation
- difficult intubation
- aspiration pneumonitis more common
- accurate BP monitoring difficult — may require arterial line, extra
monitoring
- longer procedure due to technical difficulty
- difficulty with positioning — risk of nerve compression
Post operative issues:
Increased risk of
- airway obstruction (OSA)
- post operative atelectasis and pneumonia
- thromboembolism (3-4-X increased risk with gestational diabetes)
- wound infection and endometritis
- post parptum haemmorhage (increased by 30-70%)
- prolonged hospital stay
- postnatal depression
2) This patient should be flagged as a high risk patient and preparations
should be made for every possible outcome even if she intends to have a
normal vaginal delivery.
- planned induction of labour such that delivery can be made possible
during daylight hours
- management in a large obstetric centre recommended highly
- Bring CPAP machine if patient has one at home
- On arrival to labour ward — IV access (may need anaesthetist +U / S) early
with early bloods and cross match sent.
- Analgesia options: early epidural recommended for possibility of an
emergency caesarian section (spinal difficult and may not last long
procedure)
- If planned IOL — fasting from midnight before, if possible only small sips
of water intake
- If emergency caesarian required — oral sodium citrate and effervescent
ranitidine. Attempt to proceed with regional anaesthesia and avoid GA.
Should be prepared for difficult intubation though.
- Glucose monitoring — no insulin the morning of the IOL. Hourly BSL
monitoring.
- Extra monitoring may be required eg IAL
Postoperative if LUSCS:
- admission to HDU for closer monitoring +/- CPAP
- DVT prophylaxis: - early mobilization, TEDS (right size), hydration,
heparin or LMWH (dose calculated for the weight eg: 60mg s/c clexane
daily)
- Chest physiotherapy for atelectasis
- Analgesia regime: epidural is an option however in most cases,
multimodal analgesia (paracetamol, NSAIDS if normal renal function and
no bleeding post op, tramadol, oral or PCA opioid)
- Ongoing monitoring of BSL for 6-8 weeks post.
October 2009 Question 14

A patient has smoked 20 cigarettes a day for over 25 years.

1. What are the expected physiological changes that would occur in the first 3 months following
cessation of smoking? Include a time frame for the changes you describe. (60%)

2. What are the clinical benefits, with regard to anaesthesia, of smoking cessation in this patient?
(40%)
1) Respiratory changes:
- reduced carboxyhaemoglobin levels (normal at 2-3 days) — hence reversal
of negative inotropic effect and arrythmogenic effect of CoHb
- reduced nicotine levels at 12-24 hrs — reversal of nicotine related
myocardial effects
- improved ciliary function at 2-3days
- reduced sputum production at 1-2 weeks
- improved respiratory function tests at 4-6 weeks
- improved small airway function at 4-6 weeks (laryngeal and bronchial
reactivity settles)
Cardiovascular changes
- reversal of polycythemia and increased viscosity
Miscellaneous
- immune function recovers at 6-8weeks
- normal metabolism at 6-8 weeks
- analgesic requirements normalize at 6-8 weeks
2) Clinical benefits
- Improved oxygen carrying capacity of blood
- reduced risk of post operative myocardial morbidity
- reduced risk of chest infection
- reduced risk of atelectasis
- reduced cough
- reduced mucus production and airway reactivity — decreased
broncho/laryngospasm
- reduced risk of desaturation/ hypoxia perioperatively
- improved wound healing and reduced infections overall
- normal analgesic requirements post operatively
- reduced perioperative anxiety
- return to normal hepatic metabolism of drugs such as aminosteroid
neuromuscular blocking agents
- reduced rate of unplanned ICU admissions
- reduced incidence of prolonged hospital stay, hence reduced costs
October 2009 Question 15

Describe the principles of cerebral protection in a patient with an isolated closed head injury.
Question 15
Describe the principles of cerebral protection in a patient with an
isolated head injury.
The main aim is to prevent secondary injury ie, minimize adverse
pathophysiological changes that can occur as a result of the primary
insult.
A) Secondary injury
Minimise ischemia, oedema, inflammation and hypoperfusion
1) Maintain oxygenation
- avoid anemia — maintain Hb>10g/dl
- avoid hypoxia — intubate of GCS<9
2) Avoid hypo [hyperglycemia
- hyperglycemia can aggravate brain injury by causing cerebral
lactic acidosis. Treat BSL>11mmol/L
3) Anti Thromboembolism
- TEDS
- avoid heparin in trauma
4-) Avoid cerebral bleeds — no NSAIDS, anticoagulants, treat DIC
B) Optimise cerebral perfusion pressure and reduce ICP
Brain is a closed vault, therefore decrease in any of its components
will reduce ICP and cerebral swelling and hence reduce cerebral
ischemia. Adequate monitoring of ICP with ICP monitor is vital.
1) Blood volume
CBF = CPP/CVR
CPP = MAP — ICP/CVP
Cerebral perfusion pressure can be compromised due to increased ICP leading
to cerebral ischemia. However increase in CBF can increase blood volume
causing increase in ICP.
Aim to achieve a balance between optimizing CPP whilst decreasing CBF.
To maintain CPP:
- maintain adequate MAP. Aim for CPP=70mmHg. Therefore if ICP
=20mmHg, then MAP=90mmHg. Use vasopressors (metaraminol,
noradrenaline ) or inotropes.
To reduce CBF:
a) Avoid excessive increases in MAP
- Smooth induction with alfentanil (30mcg/kg) and propofol (2-3mg/kg)
and adequate muscle relaxation (suxamethonium 1mg/kg) +/- IV
lignocaine
- Smooth emergence with low dose remifentanil or low dose propofol to
avoid coughing

- Adequate sedation (propofol TIVA), anaesthesia for surgery (TIVA
propofol or iso/sevoflurane), analgesia and muscle relaxation
b) Reduce CVP
- avoid increases in intrathoracic pressure — no PEEP, treat pneumothorax
- head up at 30 degrees
- avoid neck vein compression — ETT ties, C spine collar
- avoid high airway pressures
c) Increase CVR
Autoregulation usually disrupted due to trauma
- Reduce CMRO2 — adequate analgesia, anaesthesia, sedation
- Thiopentone or propofol boluses to achieve isoelectric
EEG — no proven benefit and may decrease MAP compromising CPP
- No N20 or ketamine
- Anti seizure prophylaxis — phenytoin 15mg/kg ( No
RCTs, only case controlled studies to show that it decreases early
seizures)
- Avoid hypoxia (Decrease Pao2 — increases CBF)
- Avoid hypercarbia — aim PaCO2 of 30-35 mmHg — may need
hyperventilation. If excessive increased ICP, may reduce to <30mmHg but
will last only for 24- hours. Also has increased risk of cerebral ischemia
from excessive cerebral vasoconstriction.
- Avoid hyperthermia — induced hypothermia to 35C may be of some
benefit in head injury if lCP>20mmHg
d) Removal of haematoma by surgery
2) Extravascular fluid
To reduce oedema and hence ICP
- avoid fluid overload — aim for neutral or negative fluid balance. May need
fluid restriction to 30ml/kg/day and UO>0.5ml/kg/hr
- avoid hypotonic fluids — use 0.9% normal saline
- diuretics — mannitol 0.5-1g/kg (2-5ml/kg of 20%solution) or frusemide if
LV dysfunction (1mg/kg)
- avoid hyponatremia
- surgical removal of skull flap to relieve pressure
- No evidence of steroids in head injury
3) E
Placement of intraventricular drain to remove excess CSF to desired ICP
4) Brain tissue
Relevant in brain tumours
Patient must be monitored very carefully along with close recording of the ICP.
Staff should be familiar with the management of head injury patients. Regular
observations are vital.
May 2009 Question 1

What do the terms decontamination, disinfection and sterilisation mean? 30%

What measures should be in place to minimise the risk of transmission of infection to the respiratory tract of patients via anaesthetic equipment? 70%
Decontamination is the removal of microorganisms and unwanted matter from contaminated materials or living tissue.

Disinfection is the inactivation of non-sporing micro-organisms using either thermal or chemical means.

Sterilisation is the complete destruction of all micro-organisms including spores.

Measures which are required to minimise the risk of transmission of infection to the respiratory tract of patients via anaesthetic equipment include: (a summary of PS28 College guidelines)

1) Staff : Hand washing (anaesthetist and anaesthetic technicians/assistants), vaccination programs, sick leave and use of masks if potentially infectious
2) Equipment : Ensuring disposable equipment only used once and that appropriate levels of sterility, disinfection and decontamination maintained to all reusable equipment. Flexible laryngoscopes and bronchoscopes : high level disinfection is the minimum level of processing required and ideally bronchoscopes should be sterilised between use.
3) Airway : mainly disposable once only use eg ETT / LMA / bougie and should be kept sterile / covered until use. Used laryngoscope blades should be isolated and disinfected before reuse (not sterilised). Laryngoscope handles should be decontaminated between use. Reusable LMAs should be sterilised as per manufacturer advice. Reusable face masks should be decontaminated and then disinfected prior to reuse. Ensure used/soiled and unused items are separated.
4) Breathing circuit : protected by an appropriate protective filter, ensure disposable items between patient and filter are discarded after each case. Change mask for each case. Circuits that are not filter protected should be discarded or decontaminated and sterilised for each new patient. Ensure anaesthetic machine / bag etc wiped down in between patients.
5) Sampling lines : need to be proximal to filter. Should have one way flow and should not be returned to circuit unless first passed via a filter.
6) CO2 absorbers and ventilators : should be cleaned and disinfected regularly.
7) Issues with special patients – eg TB or variant CJD, VRE (needs special disinfection and 1 hour wait in between cases or as per institution protocol).
8) Ensure regular audit and education of staff about infection control protocols with involvement of infection control officer and microbiologist.
May 2009 Question 2

What are the essential safety requirements for delivery of gases via anaesthetic machines and their associated breathing circuits in use in Australian and New Zealand? (Do not include ventilators or scavenging in your answers)
PS18, T3 and PS31

Safe anaesthetic machines and delivery of gases are essential to the provision of safe patient care.

Gas supply to the machine –
1) Non-interchangeable gas hose connectors (eg Diameter Index Safety system or Sleeve index system connectors) must be present on any gas inlet socket to prevent incorrect gas hose connections. Diameter index safety system in place from pipeline to anaesthesia machine. Filter helps trap debris from wall supply and one way check valve prevents retrograde flow of gas.
2) Connections for medical gas cylinders must be pin indexed. A reserve supply of oxygen (from an attached oxygen cylinder) must be functionally attached to the anaesthetic machine in such a manner that it is easily activated should the oxygen supply failure warning alarm sounds. (Cylinders attach to the machine utilize a pin index system.)
3) Level one check must be performed by a qualified person / engineer at specified times and include minimizing leaks, excluding crossed pipelines, determining the integrity of oxygen failure prevention and warning devices, checking the composition of delivered gases and their flow rates.
4) Level 2 check – check O2 cylinder and contents, check that piped gas supplies are at specified pressures, confirm using a multi-gas analyzer to check gas composition. (performed at the start of every list)
5) Level 3 check – performed at the start of every case.

Anaesthetic Machine and Breathing Circuits
6) Means to display gas supply line and cylinder pressures shall be provided. Display needs to be visible from the front of the machine.
7) Oxygen supply failure alarm must be present. This must activate automatically when the oxygen supply pressure falls below a predetermined critical level with an auditory alarm, prevent the delivery of hypoxic gas from the fresh gas outlet.
8) Emergency oxygen flush button must be present.
9) If a mechanical oxygen flow knob is provided, it must be fluted and larger than the other flow knobs so that tactile identification of the oxygen knob is possible. Check flow controls operational.
10) If the machine incorporates a gas flowmeter bank, oxygen must be the last gas to enter the common gas manifold at the top of the flowmeter tubes.
11) If the anaesthetic machine is capable of delivering N2O, means must be provided to prevent unintentional selection of a hypoxic gas mixture. Check anti-hypoxic device functioning.
12) A fresh gas outlet must be 22mm outer diameter and 15mm inner diameter, visible to the operator and should be connected to the breathing system in such a way as to prevent accidental disconnection.
13) A vaporizer interlock system must be in place, ie only one vaporizer can be ‘on’ at a time.
14) A breathing system high pressure relief valve or other means of automatically preventing dangerously high pressures in the breathing system must be provided as well as an alarm when high pressures or disconnection/low pressures are achieved.
15) Check for circuit leaks prior to the start of case. Ensure movement of unidirectional valves and appropriate function of Adjustable pressure limiting (APL) valve.
16) Adequate maintenance of the machine must be performed regularly.
17) There must be a secondary facility to maintain oxygenation and ventilation of the patient should failure of the primary system occur.

Monitoring
1) When monitoring is in use the alarms (visual and audible) must be enabled and appropriate.
2) Ventilation must be monitored continuously by both direct and indirect means.
3) Oxygen analyzer – a device incorporating an audible signal to warn of low oxygen concentrations must be in continuous operation for every patient.
4) Breathing system disconnection or ventilator failure alarm must be in continuous operation.
5) Pulse oximetry must be used in conjunction with clinical observation of the patient (with variable pulse tone and low threshold alarm must be on).
6) Carbon dioxide monitor / capnography – a monitor of the carbon dioxide level in inhaled and exhaled gases must be in use for every patient undergoing GA.
7) Volatile anaesthetic agent concentration monitor – must be in use for every patient undergoing GA.
May 2009 Question 3

A previously well 80kg 19-year old male is anaesthetised for an ORIF of a fractured tibia and fibula. He has had rapid sequence induction including suxemethonium and is intubated and ventilated via a circle system at 12 breaths per minute and a tidal volume of 700mls with an FiO2 of 0.5. He has had a 500mcg fentanyl and anaesthesia is mainted with 1.5MAC sevolfurane. He develops an increasing sinus tachycardia to 160/min with frequent VEBs and his ETCO2 rises to 60mmHg despite increasing his ventilation. There is no rebreathing evident on capnography. ABGs show :
pO2 : 105mmHg
pCO2 : 65 mmHg
pH: 7.12
HCO3 : 20.7mmol/L
BE: -10

Outline the steps you would follow to manage this situation.
This is an emergency situation requiring rapid assessment and treatment.

The most likely diagnosis is malignant hyperthermia, characterised by the tachycardia, elevated ETCO2 and severe acidaemia. There is a severe mixed respiratory and metabolic acidosis as shown by the low HCO3, low base excess and high pCO2 on ABG.

Management
1) Call for help, declare emergency. Follow MH protocol/box of equipment. Need adequate staff to mix dantrolene. Team leader to allocate tasks.
2) Alert surgeons – look for rigidity suggestive of MH, rapid closure.
3) Confirm Airway, Breathing, place on 100% O2, increase ventilation, turn off volatile and use TIVA with propofol, support Circulation (HR, BP), give iv fluids. No further doses of suxamethonium.
4) Definitive treatment is to give dantrolene 2.5mg/kg iv repeat as required at 10-15 minute intervals up to 10mg/kg iv, as early as possible.
5) Flush machine with 100% O2 with volatile off. (Given emergency situation, it will be difficult to exchange anaesthetic machines.)
6) Monitoring – obtain arterial line and consider central venous line, central /core temperature probe. Serial blood investigations (ABG, UEC, coagulation status, DIC possible, CK, urine myoglobin). Consider use of BIS monitoring with TIVA.
7) Assess temperature and if high suggestive of MH, institute active cooling (surface cooling, cold iv fluids to core temp 38-39, ice packs in axilla and groin, lavage stomach, bladder, rectum, decrease room temperature)
8) Renal protection – risk of renal failure with myoglobinuria, ensure increased iv fluids. Consider urinary alkalinisation and forced diuresis.
9) Other therapy – monitor for hyperkalaemia and treat with insulin/dextrose if K > 6.5 mmol/L and/or if ECG changes / arrhythmias. Management of metabolic acidosis – consider sodium bicarbonate but beware further increase in pCO2, will need to further increase ventilation.
10) Ongoing care: Transfer to ICU once stable for continued monitoring and treatment. Will need to plan for completion of fracture management (MH trigger-free anaesthetic).
11) Follow-up: Ensure full documentation in history, notification to patient and family. Refer patient to MH centre for further definitive testing and consider genetic testing and muscle biopsy testing of family if deemed appropriate. Staff debriefing.
May 2009 Question 4

Draw a cross section of the arm at the level of the axilla illustrating the anatomy relevant to performing a brachial plexus block for surgery on the forearm. (50%)


List the advantages and disadvantages of a block at this level compared to a supraclavicular block. (50%)
• Axillary sheath contains – axillary artery, radial / median / ulnar nerves, but NOT musculocutaenous nerve


Advantages of Axillary Block:
• No risk of Pneumothorax
• Excellent operating conditions for forearm and hand surgery
• Block easy to learn and master
• Able to easily apply pressure if axillary artery puncture
• No risk of Horners Syndrome
• No risk of Phrenic nerve palsy

Disadvantages of Axillary Block:
• May be difficult to anaesthetize Musculocutaneous nerve (located out of sheath)
• Arm must be positioned in abduction for performance of block – may be difficult in patients with arthritis / fractures
• Higher risk of intravascular injection
• Higher risk of LA toxicity, and larger volume of LA required
• Difficult to place catheter for continuous infusions
• Tourniquet pain may persist
May 2009 Question 5.

70 year old man with a 10 year history of Parkinsons Disease presents for a total knee replacement. He is on Levodopa/carbidopa five times a day.

Outline the main issues to consider in relation to his Parkinsons disease in planning the perioperative management of this patient?
Parkinsons Disease:
• Parkinsonism is a neurodegenerative syndrome characterized by resting tremor, bradykinesia, rigidity and postural instability.
• Results from a loss of dopaminergic neurons of the pars compacta in the Substantia Nigra.
• Balance between doperminergic and cholingeric activity. Therapy is aimed at increasing dopa or reducing cholingeric. Upto 20% of patients are unresponsive.

Drug Therapy
1) Levodopa / Carbidopa:
• Drug therapy is aimed at restoring the dopamine within the brain:
o Levodopa (prodrug) is converted to dopamine in the brain
o Administered in combination with a peripheral DOPA carboxylase inhibitor, eg Carbidopa, to minimize peripheral dopamine side-effects
• Very useful in early Parkinsons Disease, but the effect is not sustained over years, as may be the case in this patient
• Levodopa has a short half-life (1-3 hours)

Anaesthetic Issues relate to the
1) Head and neck
-pharyngeal muscle dysfunction : post op airway obstruction
-dysphagia : difficult oral drug administration
-sialorrhea : antisialagogue premed, intubation

2) Respiratory
-rigidity, bradykinesia, uncoodinated involuntary movement
-difficult pre-oxygenation
-difficult ventilation

3) Cardiovascular
-postural hypotension
-cardiac arrythmia
-hypertension
-hypovolaemia
-autonomic dysfunction : blood pressure instability

4) GIT
-GORD
-gastric stasis
-weight loss
-malnutrition

5) URO
-urinary retension

6) Musculoskeletal
-rigidty : positioning

7) CNS
-confusion/dementia : consent, regional
-depression
-speech impairment

8) Drug reactions
Pethidine : hypertension with MOA-B
Synthetic opiods : increased muscle rigidty
Inhalation agents : potentiate L-dopa induced arrythmias
Antiemetics : metocholpramide, droperidol, prochlorperazine : worsen Parkinson's symptoms, use domperidone and ondancetron
Antipsychotics : phenothiazines, butrophenons, piperzine : worsen Parkinson's symptoms, use clozapine or respiridone
May 2009 Question 6.

Describe the physiological effects of pneumoperitoneum with CO2 for laparoscopic surgery.
Physiological changes associated with pneumoperitoneum result from:
1. Increased intraabdominal pressure (IAP)
2. Increased CO2 load

Effects of Increased IAP:
• Normal IAP is < 5mmHG
• Alterations in hemodynamics occur with peritoneal insufflations to > 10mmHg
• IAP of 15mmHg is adequate for most procedures, as long as muscle relaxation and/or deep plane of anaesthesia are present
• CVS:
o Decreased ventricular preload
• Initial and transient increased preload due to autotransfusion of pooled blood from splanchnic circulation
• IVC compression reducing venous return
• Increased intrathoracic pressures reduce atrial and ventricular compliance
o Increased SVR / PVR:
• Due to vasopressin release and circulating catecholamines
• Due directly to increased IAP and intrathoracic pressures


o Decreased cardiac output:
• Due to a combination of increased SVR and decreased preload
• Proportional to the increase in IAP
o Increased MAP:
• MAP = CO x SVR
• SVR is increased to a greater extent than CO is decreased
• At IAP > 20 mmHg MAP is reduced, due to reduced venous return and severely reduced cardiac output
o Heart Rate:
• May be unchanged or increased slightly due to sympathetic stimulation
• Vagal stimulation due to peritoneal insufflations may cause bradycardia, junctional rhythm or asystole
o Increased myocardial workload:
• Due to increased SVR, MAP and tachycardia
• Respiratory:
o Decreased respiratory compliance by 30-50%
o V/Q mismatch:
• Elevation of diaphragm and resultant lung compression causing decreased FRC and basal atelectasis
• Potential hypoxemia and hypercarbia
o Increased airway resistance
• Increases risk of barotraumas with positive pressure ventilation
• CNS:
o Increased ICP – independent of changes in PaCO2
o Raised IOP not clinically significant
• GI:
o Increased intragastric pressure may lead to increased risk of regurgitation (ETT preferred to LMA)
o Compression of intestinal capillaries can theoretically lead to bowel ischaemia with excessive pressures
• Rarity of case reports suggest that this is not clinically significant
• Renal effects:
o Renal blood flow, GFR and urine output have all been documented to decrease by >50%
o At pressures > 15mmHg oliguria may occur, and >40mmHg anuria may occur

Effects of Increased CO2 Load:
• Respiratory:
o Increased PaCO2:
• A moderate increase in minute ventilation 20-30% may be required to maintain normocapnia
• Plateau level within 15-30 mins
• CVS:
o Hypertension
o Increased Heart rate due to sympathetic stimulation
o Increased cerebral and coronary blood flows
o Increased cardiac output
o Vasodilatation
o Reduced contractility
o Predisposition to arrhythmias
• CNS:
o Increased ICP
o Sympathetic stimulation
o CNS depression at PaCO2 > 100mmHg
• Haematology:
o Right shift of oxygen dissociation curve
• Metabolic:
o Acidosis is buffering capacity exceeded
o Hyperkalaemia
o Hypercalcaemia
May 2009 Question 7.

Outline the coagulation changes you would expect in a patient with a ruptured liver from a blunt abdominal trauma requiring massive transfusion (50%) and describe how you would minimise them (50%)
• Massive transfusion definition-
1. transfusion of over 10 units of PRCs in a 24 hr period
2. transfusion of over one bloood volume in a 24 hr period
3. blood loss of >150ml/min

• Coagulation abnormalities may be due to primary liver injury or as a result of the massive transfusion

• Causes of coagulopathy
1. Tissue trauma- massive activation of coagulation cascade with consumption of factors/fibrinogen
2. Shock- organ hypoperfusion with unfavourable microvascular milieu for clotting
3. Haemodilution- inadequate concentration of clotting factors
4. Hypothermia- decreased enzymatic activity at < 35 deg. Celcius
5. Acidaemia- also unfavourable to clotting enzymes
6. Inflammation- disrupted normal balance of pro/anti coagulants by cytokines

• Extent of coagulopathy will depend on time from injury and resuscitation magnitude

Expected changes:
1. Tissue damage initiating coagulation- TF activates proteases forming thrombin and ulitimately fibrin from fibrinogen
2. Hyperfibrinolysis- direct consequence of tissue injury and shock. Localised fibrinolysis to the site of injury may be lost and instead be widespread
3. Blood test changes- anaemia/thrombocytopaenia (from both blood loss and dilutional from transfusion). Elevated INR, APTT and D-dimer. Decreased fibrinogen (< 1.0 severe).
4. Point of care tests (eg. TEG) may be abnormal.
5. Electrolyte abnormalities- decreased Ca++, increased K+, other 'storage lesions' from massive transfusion
6. Hypothermia from both injury/exposure and cold blood transfusion

Minimising changes:
1. Transport to hospital- keep warm, avoid over-zealous crystalloid
2. Contact key personnel to expedite blood products (eg. "trauma pack" or "Code White" to mobilise resources)
3. Stop further active surgical bleeding (surgery, radiological embolisation, conservative eg. pressure bandages/tourniquets)
4. Cell salvage
5. Early use of blood products to avoid haemodilution
• Target Platelet count > 75, fibrinogen > 1.0 g/L (cryo/FFP), normal INR/APTT(FFP)
• Recent literature gives evidence for early 1/1/1 blood/FFP/platelet transfusion (mimics whole blood)
6. Targetting specific complications of blood transfusion
• Acidosis- consider buffers (eg. bicarb) esp. for older RBCs with storage lesion
• Hypocalcaemia- IV calcium to target ionised Ca++ level > 0.9 mmol/L
• Haemocrit- aim > 30%. RBC concentration important for platelet radial transport and adhesion to damaged endothelium
• Hypothermia- active patient warming (Bair-Hugger, avoid exposure, warm theatre) and mandatory use of fluid warmer for blood and all other fluids
May 2009 Question 8.

Outline the features and clinical management of amniotic fluid embolism (AFE)
• Rare complication of pregnancy (1:8000- 1:80000 pregnancies) but carries a very high mortality (up to 80%). Responsible for up to 10% of direct maternal deaths in Australia.
• Major morbidity in survivors- 85% of survivors have neurological impairment
• Pathophysiology- amniotic fluid enters maternal circulation. Fetal antigens activate a cascade of maternal immune mediators, with response similar to an anaphylactoid reaction
• May occur at any stage of pregnancy (including post-partum). 70% happen in labour. May even occur in first trimester with trauma/procedures (eg. amniocentesis)

• Risk factors
1. Advanced maternal age
2. Multiparity
3. Maternal history of atopy/allergy
4. Intrauterine death
5. Microsomia
6. Meconium stained liquor
7. Chorioamnionitis
8. Polyhydramnios
9. Strong or tetatnic uterine contractions, uterine rupture
10. Augmented labour
11. Placenta accreta

Features
• Maternal collapse (other causes: DVT/PE, arrhythmia, massive haemorrhage, eclampsia, peri-partum cardiomyopathy, MI, sepsis)
• Respiratory- dyspnoea, cough, bronchospasm, wheeze
• Cardiovascular- hypotension, pulmonary oedema, tachycardia, cyanosis, chest pain, transient hypertension, LV/RV failure
• Neurological- seizures, headache
• Other- foetal distress (may be first sign), uterine atony, DIC and coagulopathy (amniotic fluid contains factors II, VII, X and tissue factor)

Investigations
• No pathognomic test
• FBE, Coags
• CXR, ABG, ECG, V-Q scan
• Pulmonary blood sample (from PAC)- fetal squames
• Zinc coproporphyrin may be elevated

Management
• Recognise anaesthetic and obstetric crisis
• Supportive therapy. No definitive treatment. Prompt resuscitation required
• Multidisciplinary approach (obstetrics, anaesthetics, intensive care)
• Deliver foetus if not already done
• If collapse- resusctiation as per ALS guidelines (CPR with compression 100/minutes, breaths 30:2 (8-10/min with advanced airway), drug therapy directed at cardiac rhythm. Important caveats:
1. Left lateral tilt
2. Peri-mortem Caesarean if no ROSC within 4 minutes and > 20 weeks gestation
3. Recognise challenges to resuscitation of pregnant patient (CPR with gravid uterus, potential for difficult airway)
4. Call for help early
5. Maintain uterine tone (oxytocics) to prevent further bleeding
• Early haematology involvement for coagulopathy. Blood tests including coags, may require FFP/platelets/cryoprecipitate/Recombinant Factor VII
• Large volume fluid resuscitation and adrenaline are needed given anaphylactoid pathophysiology
• Transfer for further stabilisation to critical care environment
• Debriefing for team/with colleagues
• Notification of patient's family
May 2009 Question 9.

A 65-year-old man presents in your pre-admisison clinic. He is scheduled for femoral-popliteal bypass surgery for peripheral vascular disease in 4 days time. He has ischaemic rest pain in his leg. Evaluate the usefulness of initiating therapy with beta-blockers to reduce the incidence of peri-operative myocardial infarction in this man.
Guidance on peri-operative beta blockade is controversial and relies on expert guidelines applied to this particular patient. Recommendations below are from AHA/ACC 2009

• Vascular surgery- associated with high incidence of peri-operative cardiovascular complications
-Without knowing more, this is a high risk patient unergoing high risk surgery- risk of major cardiovascular complications is at least 5%
- This would be reduced if revascularised (eg. CABG) in last five years and
free of ischaemia in this time (POISE exclusion). No need for beta blockade
• If he was already on beta blockers- he should continue them (Class 1 recommendation)
• If there is ischaemia on pre-operative testing, he should be beta blocked (Class 1)
• If > 1 clinical risk factor of if coronary artery disease, he should probably be beta blocked (Class IIa recommendation)
• If only 1 clinical risk factor in the absence of coronary artery disease, uncertain whether beta blockade is useful (Class IIb)
• If contraindications to beta blockade, should not be given (Class III) (eg. asthma, chronic airways disease with bronchospasm, Class 4 heart failure, heart block, significant bradycardia, known adverse reactions, concomitant verapamil use (POISE trial exclusion). Severe PVD as in this man is a relative CI.

• "Clinical risk factors" are:
1. Ischaemic heart disease
2. Compensated or prior heart failure
3. Cerebrovascular disease
4. Diabetes mellitus
5. Renal insufficiency (pre-op serum creatinine > 2mg/dL (> 180 micromol/L))

Contradicting evidence from trials:
• MAVS 2006 (Metoprolol after Vascular Surgery, Am Heart J 2006)- vascular surgical cohort, NO difference in mortality/events but increased bradycardia and hypotension requiring Rx in beta blocked group
• POBBLE 2005 (J Vasc Surg 2005)- vascular surgical cohort. Also no difference in cardiovascular event but increased bradycardia/hypotension
• POISE (Lancet 2008). > 8000 patients RCT multicentre. Reduced hazard ratio for composite endpoint (HR 0.83) of cardiovascular events and death BUT more patients died (HR 1.3) More strokes (HR 2.2), more sepsis deaths (HR 2.0).
• For every 1000 patients treated in POISE: 15 MIs prevented, 7 cases of AF prevented, 3 post-op CABG avoided BUT 8 excess deaths, 5 strokes and 53 significant hypotension.
• One criticism of POISE was large dose used (100mg metoprolol daily), unclear whether a smaller dose in this patient titrated to heart rate would be better. Hard to do this in clinic.
• Timing of commencement important. Most trials (incl. POISE) started on day of surgery. Unclear whether commencing 4 days earlier would confer more benefit.
• Principally, mortality in POISE was by increased stroke rate. If this man is at increased stroke risk (eg. carotid stenoses), should probably be avoided.
May 2009 Question 10

Draw a diagram illustrating the bronchial anatomy to the level of the
lobar bronchi (50%) and describe how you would use a fibreoptic bronchoscope to correctly position a right sided DLT
Bronchopulmonary segments of right lung
*A PALM Seed Makes Another Little Palm
* superior lobe
* apical
* posterior
* anterior
* middle lobe
* lateral
* medial
* inferior lobe
* superior
* medial-basal
* anterior-basal
* lateral-basal
* posterior-basal

Bronchopulmonary segments of left lung
*ASIA ALPS
* superior lobe
* apico-posterior (merger of “apical” and “posterior”)
* anterior
* lingula of superior lobe
* inferior lingular
* superior lingular
* inferior lobe
* superior
* anteromedial basal (merger of “anterior basal” and “medial basal”)
* posterior basal
* lateral basal

Diaphragm apertures:
Aortic hiatus= 12 letters = T12
Oesophagus= 10 letters = T10
Vena cava= 8 letters = T8


Right sided DLT use uncommon - indications
• Tumour in left bronchus
• Left pneumonectomy
• Left lung transplantation
• Left tracheobronchial disruption
• Left bronchial stent in situ
• Distorted left bronchial anatomy
Using appropriate equipment (that has been checked), assistant, environment, monitoring
and induction of anaesthesia

Sizing
- 35F/37F females - require 3.1 mm bronchoscope (esp 35)
-39F/41F males - standard 4.2 mm bronchoscope ok
-26F 8-10 yo child

Sequence of events
- Induce anaesthesia as appropriate
-Laryngoscopy - bronchial curvature facing 12 oʼclock DLT (tube at 3 oʼclock) advance
through cords and then rotate tube anti-clockwise 90° advanced, make note of depth
-Inflate tracheal balloon, Attach both lumen to circuit and hand ventilate (bilateral axillae)
-1-1.5 mL air endobronchial balloon, Disconnect tracheal lumen, clamp circuit / open tube
to air and hand ventilate & auscultate (R lung should be isolated)
-Confirmation:
-FOB down tracheal lumen - bronchial cuff should be visible but not obstructing carina
-FOB down bronchial (R) lumen - ensure port & RUL orifice aligned with balloon up
-Reconnect & Ventilate both lungs until surgery ready to begin
-repeat this sequence with change of position to ensure DLT stays endobronchial
May 2009 Questions 11

A 40-year old otherwise healthy male presents following a sub-arachnoid haemorrhage. He is scheduled for clipping of the middle cerebral artery anaeurysm. Outline the major issues in providing anaesthesia for this patients and decribe how you would address them
Pre-operative
*Grade of SAH I-V based on presence & degree of ACS & neurology
-Pre-op 3+ grade = intubation post-op & ICU bed booked
-Pre-op - HD stable? blood pressure particularly important (during clipping)
*Nimodipine
*Early complications of SAH
*Cardiac - Coronary Ischaemia (ECG)
*Neurological - Neurogenic APO (CXR)
*SIADH, CSWS (electrolytes, urine monitoring)

Peri-operative
*Investigations -FBE, UEC, COAG, CXM as soon as practicable
*Monitoring
-Depth of anaesthesia - Clinical, ETAG, BIS,
-Arterial BP prior to induction, 2 x IV lines (1 x 16g or better) (dedicated
line for TIVA) keep pressors/volume on separate line to anaesthesia.
*Reduce risk of re-bleeding
*blunt hypertensive response during stimulation:
-intubation
-mayfield pin application
- incision
- extubation
*Remifentanil + Volatile/TIVA anaesthetic facilitates
*Risk of rupture
-communicate with surgeon - permissive hypotension, then blood products
*Reduce cerebral blood volume
-avoid hypercarbia, hypoxia, acidosisMAP
-avoid restricting venous drainage : Position - ↑ venous drainage Head up, no tapes
- TIVA propofol or volatile < 1 MAC, X N2O
-0.5-1g / kg Mannitol (10%) slowly may be needed to reduce ICP rapidly
-hypertonic saline
*Defend the MAP and cerebral perfusion
*Fluid management - crystalloid 0.9% judiciously.
-Temporary clipping
-100% O2, normocapnoea, normothermia
-Neuroprotection
-Hypothermia increases bacteraemia for no neurological benefit

-Post-operative
-Avoid coughing (leave remifentanil on, all others off for extubation)
-ICU/HDU, ventilated as guided by pre-op grade
-Vasospasm post clipping
-triple H therapy (hypertension, hypervolaemia, haemodilution) No RCT
-CVP useful for titration of the above
-ionotropes to acheive BPs that result in neurology reversal
-Nimodipine reduce clinical vasospasm(but not radiological) if given early
-dose 60 mg o qid has evidence (no evidence for 1-2 mg/hr IV)
-Communication with surgeon re: BP, CVP goals
-Early neurological assessment, neurology/delayed emergence = urgent CT
-Care with opioid analgesics following operation
May 2009 Question 12

Describe the management of a patient who has had a total thyroidectomy who developed respiratory distress int he recovery room.
This is an emergency - Immediate attendance as a matter of priority
-Get someone to call the surgeon +/- the anaesthestist who did the case
-Call for help - get senior assistance ASAP

Differential Diagnoses
-Surgical
-Haematoma / Vocal cord paralysis (recurrent laryngeal nerve injury) / Tracheomalacia (Esp malignant thyroid) / Pneumothorax (due to mediastinal dissection) / Airway oedema (from prolonged operation) / Hypocalcaemia (unlikely in recovery - develops later on)

-Anaesthesia : Neuromuscular blockade / Recurarisation / Inadequate reversal / NMB error (eg unflushed IVC) / Oversedation / excessive opioid

-Patient
-Anaphylaxis /Respiratory disease / Pre-existing - Asthma / COPD / Acute event - Collapse / aspiration / obstructed airway /Cardiac disease / Pre-existing CCF / Acute event - ischaemia / Hypoglycaemia

Immediate management
-Assess and support Airway, Breathing & Circulation simultaneously
-Establish GCS
-Get anaesthetic chart, history and collateral history from recovery staff
-Monitoring - SpO2, HR, NIBP

Airway assessment
-Patent? Stridor? Breathing efficacy? Respiratory effort?
-Suction / exclude supraglottic obstruction
-Consider intubation

Breathing
-Air entry / Chest movement / Tracheal tug or deviation
-Drain tubes if present
-Apply O2 high flow via non rebreather or support with BMV or Mapleson
-Consider intubation

Circulation
-BP compromised / Mottled / arrhythmia
-May direct a) diagnosis b) therapy (ie respiratory distress causing
cardiovascular embarrassment)
-Consider intubation

Residual NMB
-Consider neostigmine / sugammadex if rocuronium / vecuronium

Surgical swelling - venous/lymphatic oedema
-Assess for airway compromise & optimise: sit up 45°
-Consider reopening wound
-In extremis - at bedside, otherwise urgent return to theatre
-Anticipate difficult airway (distortion due to oedema)
-Plan A - Inhalational induction & direct laryngoscopy
-Plan B - Asleep FOI / indirect device
-Plan C (ILMA) & D (cricothyroid / surgeon) available

Recurrent laryngeal nerve palsy
-Bilateral - apposition of cords, inability to vocalise, stridor, obstruction
-Unilateral - hoarseness
-Nasendoscopy to quantify / diagnose
-Urgent intubation & tracheostomy may be required
-Induction as above

Tracheomalacia
-Urgent bronchoscopy with SV -Induction as above

Pre-existing conditions treated with definitive therapy when diagnosed
-Asthma/ COPD - steroid / bronchodilators
-CCF - diuretics / PEEP
-Pneumothorax - ICC insertion

Ongoing management
-Definitive surgical airway possible
-ICU / HDU arranged
May 2009 Question 13
Outline the steps you would take to ensure the safe introduction of elective paediatric surgery at your local private hospital.
Elective private hospital: issues:
1) Elective can be day cases or admission to wards
2) Private implies lack of 24 hours on site medical personnel/possible lack
of ICU /HDU
3) Safe - comply with ANZCA guidelines for all areas

Steps include:
1) Form a committee:
- hospital representative (chairman)
- financial advisor (for the budget)
- Theatre /PACU coordinator
- Ward nurse coordinator
- Head of surgery
- Head of anaesthesia (You!)
2) Discuss requirements
- Requirement of expertise - paediatric surgeon and anaesthetist.
Depending on area of expertise, select range of paediatric surgeries to
be performed and number of lists per week.
- Requirement in theatre - anaesthetic equipment - paediatric
airways/masks, lV lines, T piece, circuit, monitoring equipment,
resuscitation trolley including diffiicult intubation trolley etc (as per
college guidelines). ?set up anaesthetic room so as to induce in the
anaesthetic room. Similarly paediatric surgical equipment as advised
by surgeon.
- Requirement in PACU - staff expertise in recovery/monitoring
equipment] discharge criteria for day surgery and to the ward
- Requirement in ward - staff expertise] resuscitation equipment
- Follow up organized with the surgeon in outpatients
3) Discuss selection criteria
- Depends on availability of HDU and on site medical personnel.
Unlikely in small private hospital
- Selection criteria
- Age>3 years old (just an eg)
- Fit patient with no major medical co-morbidity eg OSA/severe
asthma
- Nil history of syndromes or congenital disease
- Nil history of major reactons to previous anaesthesia
- Requiring surgery which has minimal blood loss, with varying
degrees of invasiveness. Eg: tonsillectomy, grommets, hernias
- Nearest HDU/ICU if there is an emergency and patient needs to be
transferred

4) Discuss discharge criteria from day surgery and from ward
As per college guidelines - social and medical criteria for day surgery
Social criteria:
- responsible person to transport patient to home
- responsible adult to stay with patient
- responsible adult must understand requirements for postanaesthetic
care
- patient should be within one hour of medical help until next moming
- parents should have access to telephone
- parents should be given advice about when child can resume normal
activity
Medical criteria:
- suitable vital signs for at least one hour
- orientated
- adequate pain control
- minimal nausea, vomiting and dizziness
- adequate hydration
- minimal bleeding and wound drainage
- patient at risk of urinary retention should have passed urine
- discharge authorized by appropriate staff member
- written and verbal instructions given to parents
- suitable analgesia for at least one day of discharge with written
instructions on how to use it.
- Telephone inquiry on patient's wellbeing the next day should be made by dedicated staff member
May 2009 Question 14
A previously healthy 20 year old male has persistent pain 12 weeks alter compound fracture to his lower leg. l-le is on Sr oxycodone 80mg bd and immediate release oxycodone 20mg 4 hourly. Discuss the advantages and disadvantages of switching his opioid to methadone in this situation and how may this be achieved safely?
Methadone:
Synthetic opioid — binds mu, kappa and delta receptors
Used in opioid addiction and chronic pain states.
This patient has high opioid requirements possibly due to tolerance/dependence
and due to onset of chronic pain (often neuropathic in nature).

Advantages of methadone:
O Good oral bioavailability (70-80%)
O Available in liquid and oral form
O High potency - highly lipid soluble hence rapid absorption with
extensive tissue distribution
O Rapid onset of analgesia — 30-60mins
O Long duration of action -24 hrs
O N 0 active metabolites; safe in renal failure and stable liver disease
O Low cost
O Legal
O Very effective for pain management including neuropathic pain.
Relatively safe opioid to use in chronic pain
O Helps reduce ‘cravings’ and wean off high doses of oxycodone as
in this patient. Patient also probably taking excess doses due to
toleranm - conversion to equianalgesic doses can decrease opioid use and decrease side effects (ceiling effect taken into account
during conversion)
O Weak NMDA receptor antagonist — reduced propensity to develop
opioid tolerance and increase efficacy for neuropathic pain
O Less constipation than other opioids
O Regular use of methadone will mean regular reviews and
monitoring by health professional

Disadvantages:
O Whilst on methadone, patient still physically dependent on
opiates
O Patient needs to follow dosage regime exactly to prevent
withdrawal. Due to prolonged duration of action, withdrawal can
take longer
O ‘stigma’ socially attached to methadone by both patient and
physician
O Initially require adequate dosing - need motivation, discipline and
time
O Side effects - unpredictable half time (3-Sdays), so accumulation
risk present. high dose a/w prolonged QT interval; lethargy;
weight gain
O Risk of opiate overdose exists

*(Drug interactions: metabolized by cytochrome pi-50; methadone
levels increased by 550.1 and decreased by phe nytoin and
carbamazepine
O Dose conversion is complex - depends on absolute doses of other
opioids and duration of treatment and individual variation is wide
O Due to long half life difiicult to titrate quickly
O Inadequate promotion of its use by drug companies clue to low
cost. Therefore lack of experience amongst clinicians.


Switching to methadone:
O Must involve pain specialist
O Due to risk of overdose . start low and go slow
O Opioid tolerant patient - rescue medication (immediate acting
oxycodone to prevent withdrawal
O Takes 5-7 days for plasma levels to reach maximum therefore
adjust every 5-7 days. Warn patients that it may take several days
for optimal pain control
O If patient on high dose of opioids, increased toxicity risk
Therefore conversion ratios are less. For eg: if morphine
equivalent is >600mg, calculate dose at 8% not 10%.
0 Titrate cautiously and slowly since patient is an outpatient
0 Educate patient about side effects and toxicity
0 Select patient who is responsible. comprehends, motivated and
compliant Rule out patient related pharmacokinetic changes and
disease progression
O Maximise use of nonopioid analgesics eg gabapentin for
neuropathic pain
O Patient should have written information and phone number to
answer queries

Calculating close: See attached article
1] Cease all opioids
2] Daily dose of current opioid use
= 80x2 + 20x6
=280 mg oxycodone
3] Equianalgesic dose units = 280/20 = 14 EDU [equianalgesic dose for oxycodone =20mg]
4] Morphine equivalent dose = 14 EDUx30mg morphine = 420mg
5] Morphine conversion to methadone
10% of morphine equivalent = 10/ 100 x 420 = 42mg
Total daily dose of methadone is 42mg
6] Give methadone tds - 42/3 = 14mg
Approximately 15mg every 8hrs for 5-7 days, then titrate depending on patient response
May 2009 Question 15
How is appropriate sample size for a clinical trial determined? What are
the ethical implications of using an inapprop riate sample size in a clinical
trial?
Sample size determinants :

1] Chosen alpha error - Type 1 error
Probability of incorrectly rejecting the null hypothesis
Usually set at 0.05
Smaller the alpha error - larger the sample size required

2] Chosen beta error
Probability of incorrectly accepting the null hypothesis
Set at 0.05-2.'D
Smaller the beta error. larger the sample size needed

3] Effect size
Smaller effe ct size will require larger sample size

4] Variance
Larger variance within a population requires larger sample size. This
cannot be chosen by estimated From pilot studies. Variance can be
minimized by maximizing measurement precision ie, use more precisetools for measurements.

5] Number of endpoints being studied Rarer events re quire larger sample size

6] Distribution of data
Sample size estimations assume normal distributio n, so if the study data is nonparametric then increase sample size by at least 10%.
‘Power analysis‘ used to determine sample size required for a chosen
alpha error, beta error and hence power. Done by computer programs.
importance of appropriate sample size:

If sample size is too small:
1] The difference between a sample mean and population mean is the
sampling error. Smaller sample size means larger sampling error.
2] Smaller sample size decreased power of the study. It is important to
have adequate power so that the study is accurately and reliably able
to answer the question being asked If power is too small. there may
not be a stafistically significant difference found from the study even
if a difference exists between groups.
Issues with inadequate power:
- waste of resources and time if adequate sample size was able to
be calculated properly beforehand.

- Subjecting patients to useless experimentation and side effects of
treatment with no benefit to scientific knowledge
- A good treatment not accessible for the population due to inability
to find statistically significant difference due to low sample size
- Lack of power means lack of applicability to patient population.
3] Had the sample size been calculated adequately, then could have lengthened inclusion period, included another centre or aborted the
study if ‘n’ could not be reached feasibly.

lf sample size too Jorge:
1] Time and resources wasted for minimal gain
2] Larger number of patients exposed to potential side effects of treatment - this increases rik/ benefit ratio.
October 2008 Question 1.

Outline the operating principles and safety features of a modern variable bypass out of circuit vaporiser.
Volatile anaesthetics must be vaporized before being delivered to the patient. Vaporisers have concentration-calibrated dials that precisely add volatile anaesthetic agents to the combined gas flow. They must be located between the flowmeters and the common gas outlet.

Operating features
1) Controlled splitting of FGF into two streams, one of which goes through a high resistance vaporizer chamber
2) Full saturation of gas in the vaporization chamber by large surface area/ wicks
3) Output stabilized for high/low gas flows
4) Temperature compensated (for latent heat of vaporization by bimetallic strip/heat sink)

Modern vaporizers are agent specific, capable of delivering a constant concentration of agent regardless of temperature changes or flow through the vaporizer. Turning a single calibrated control knob counterclockwise to the desired percentage divides the total gas flow into the carrier gas which flows over the liquid anaesthetic in a vaporizing chamber, and the balance, which exits the vaporizer unchanged. Because some of the gas is never exposed to anaesthetic liquid, this type of agent specific vaporizer is called a variable bypass vaporizer. Variable bypass vaporizers are used to deliver sevoflurane and isoflurane but not desflurane.

At a given temperature the molecules of a volatile agent in a closed container are distributed between the liquid and gaseous phases. The higher the temperature the greater the tendency for the liquid molecules to escape into the gaseous phase and the higher the vapour pressure. Vaporisation requires energy which is supplied as loss of heat from the liquid. Vaporisers contain a chamber in which a carrier gas becomes saturated with the volatile agent. (See page 65 of Clinical Anaes Morgan/Mikhail)

Temperature compensation is achieved by a metal strip of two different metals. The strips expand and contract differently to changes in temperature allowing the strip to bend allowing more/less gas to pass through the bypass chamber of the vaporizer (eg warmer temperature = more gas through bypass chamber, less gas through vaporizing chamber).



Generic variable bypass vaporiser



Ohmeda Tec 4 vaporiser

Safety features
1) Agent specific (calibrated, colour coded, key indexed filling devices).
2) Interlock device (to prevent turning on 2 vaporisers) Unless the machine accepts only one vaporizer at a time (eg Datex-Ohmeda Aladin cassette vaporisers) all anaesthesia machines should have an interlocking or exclusion device that prevents the concurrent use of more than one vaporizer. Modern vaporizers offer agent-specific keyed filling ports to prevent filling with an incorrect agent and causing overdose.
3) Pressure resistance / anti-pumping
4) Locking spindle (and o-rings to prevent gas leakage from back-bar)
5) Anti-tilt measures, visible agent level
6) Vaporizers automatically compensate for changes in ambient pressures (eg altitude changes)
7) A one-way check valve between vaporizer and the oxygen flush valve (in Datex-Ohmeda) minimises any reversal of flow through the vaporizer leading to unpredictable agent delivery.

NB With desflurane, because of its high vapour pressure, it requires an electronic vaporizer. A reservoir containing desflurane is electrically heated to 39 degrees Celsius. Unlike a variable gas vaporizer no fresh gas flows through the desflurane reservoir, rather pure desflurane vapour joins the fresh gas mixture before exiting the vaporizer. The amount of desflurane vapour released from the reservoir depends on the concentration selected by turning the dial.
October 2008 Question 1.

Outline the operating principles and safety features of a modern variable bypass out of circuit vaporiser.
Volatile anaesthetics must be vaporized before being delivered to the patient. Vaporisers have concentration-calibrated dials that precisely add volatile anaesthetic agents to the combined gas flow. They must be located between the flowmeters and the common gas outlet.

Operating features
1) Controlled splitting of FGF into two streams, one of which goes through a high resistance vaporizer chamber
2) Full saturation of gas in the vaporization chamber by large surface area/ wicks
3) Output stabilized for high/low gas flows
4) Temperature compensated (for latent heat of vaporization by bimetallic strip/heat sink)

Modern vaporizers are agent specific, capable of delivering a constant concentration of agent regardless of temperature changes or flow through the vaporizer. Turning a single calibrated control knob counterclockwise to the desired percentage divides the total gas flow into the carrier gas which flows over the liquid anaesthetic in a vaporizing chamber, and the balance, which exits the vaporizer unchanged. Because some of the gas is never exposed to anaesthetic liquid, this type of agent specific vaporizer is called a variable bypass vaporizer. Variable bypass vaporizers are used to deliver sevoflurane and isoflurane but not desflurane.

At a given temperature the molecules of a volatile agent in a closed container are distributed between the liquid and gaseous phases. The higher the temperature the greater the tendency for the liquid molecules to escape into the gaseous phase and the higher the vapour pressure. Vaporisation requires energy which is supplied as loss of heat from the liquid. Vaporisers contain a chamber in which a carrier gas becomes saturated with the volatile agent. (See page 65 of Clinical Anaes Morgan/Mikhail)

Temperature compensation is achieved by a metal strip of two different metals. The strips expand and contract differently to changes in temperature allowing the strip to bend allowing more/less gas to pass through the bypass chamber of the vaporizer (eg warmer temperature = more gas through bypass chamber, less gas through vaporizing chamber).



Generic variable bypass vaporiser



Ohmeda Tec 4 vaporiser

Safety features
1) Agent specific (calibrated, colour coded, key indexed filling devices).
2) Interlock device (to prevent turning on 2 vaporisers) Unless the machine accepts only one vaporizer at a time (eg Datex-Ohmeda Aladin cassette vaporisers) all anaesthesia machines should have an interlocking or exclusion device that prevents the concurrent use of more than one vaporizer. Modern vaporizers offer agent-specific keyed filling ports to prevent filling with an incorrect agent and causing overdose.
3) Pressure resistance / anti-pumping
4) Locking spindle (and o-rings to prevent gas leakage from back-bar)
5) Anti-tilt measures, visible agent level
6) Vaporizers automatically compensate for changes in ambient pressures (eg altitude changes)
7) A one-way check valve between vaporizer and the oxygen flush valve (in Datex-Ohmeda) minimises any reversal of flow through the vaporizer leading to unpredictable agent delivery.

NB With desflurane, because of its high vapour pressure, it requires an electronic vaporizer. A reservoir containing desflurane is electrically heated to 39 degrees Celsius. Unlike a variable gas vaporizer no fresh gas flows through the desflurane reservoir, rather pure desflurane vapour joins the fresh gas mixture before exiting the vaporizer. The amount of desflurane vapour released from the reservoir depends on the concentration selected by turning the dial.
October 2008 Question 2.

Describe the innervation of the lower anterior abdominal wall from the umbilicus to the pubic symphysis. Describe a technique of peripheral nerve block (not wound infiltration) to provide post- operative analgesia for a low transverse abdominal incision.
Nine segments from the anterior rami of the T6-L1 spinal nerves innervate the abdominal wall. The lower anterior abdominal wall from umbilicus to pubic symphysis is supplied by T10 to T12 (sensory) (T12 cutaneous branches of the subcostal nerve).

The anterior primary rami of T10-T12 pass inferoanteriorly from the intercostal spaces and run in a neurovascular plane between the internal oblique and transversus abdominis muscles. This plane is known as the transversus abdominis plane (TAP). The branch of the lateral cutaneous nerve arises from the mid-axillary line and innervates the abdominal wall to the edge of the rectus abdominis muscle. The anterior cutaneous branch travels forward to supply the skin of the anterior abdomen.




Figure 1. Transverse section of the abdominal wall showing the path of nerves T7-T12 (left) and L1 (right)within the transversus plane

Lower most fibres of internal oblique and transversus abdominis supplied by iliohypogastic and ilioinguinal.
The ilioinguinal and iliohypogastric nerves arise from anterior rami of L1 and emerge from the upper part of the lateral border of the psoas major muscle. The ilioinguinal nerve is a smaller nerve and courses caudad to the iliohypogastric nerve. Both nerves cross obliquely anterior to the quadratus lumborum and iliacus muscles and perforate the transverse abdominis muscle near the anterior part of the iliac crest. In the anterior abdominal trunk, the nerves travel between the transverse abdominis and the internal oblique muscles above ASIS. After travelling a short distance inferomedially, their ventral rami pierce the internal oblique to lie between the internal and external oblique muscles before giving off branches, which pierce the external oblique to provide cutaneous sensation. The iliohypogastric nerve supplies the skin over the inguinal region. The ilioinguinal nerve runs anteroinferiorly to the superficial inguinal ring, where it emerges to supply the skin on the superomedial aspect of the thigh.




A transversus abdominis plane (TAP) block is an example of a peripheral nerve block technique which provides post-op analgesia for low transverse abdominal incisions.

Ultrasound guided technique provides greater success rate, lower risk of complications eg bowel injury.
Pt supine, sterile technique with appropriate monitoring of pt
Place ultrasound probe at the lateral abdominal wall at the level of the umbilicus (or slightly above).
Locate 3 layers on ultrasound. Insert needle into anterior abdominal wall approx 5cm from probe using in-plane approach. Start with needle perpendicular to skin, keeping needle tip in view continuously. Aim for needle tip in TAP, may need to angle to approx 45 degrees.
Test dose to confirm needle position in transversus abdominal plane, following negative aspiration.
Inject 20mls (in 5 ml increments) of LA eg ropivicaine 0.5% into TAP (between internal oblique and transversus abdominis).
Repeat on opposite side for bilateral block.
October 2008 Question 3

What would make you suspect venous gas embolism occurring during a surgical procedure?

Briefly outline the principles of management of venous gas embolism causing haemodynamic compromise.
Venous gas embolism is a complication that may occur whenever the operative site is higher than the right atrium.

High risk surgery includes sitting position craniotomy/posterior fossa surgery/ spinal surgery, procedures where open veins may be at low pressure in relation to the heart, orthopaedic arthroplasty, caesarean section, termination of pregnancy, head and neck laser procedures, laparoscopy or procedures involving gas insufflation and endoscopy. Other causes include unprimed intravenous lines such as during rapid intravenous infusions. Intracardiac septal defects are an absolute contraindication for sitting position surgery.

Venous gas embolism causes pulmonary microvascular occlusion resulting in increased physiological dead space. Air in the right atrium → RV outflow tract obstruction and reduced cardiac output. Bronchospasm may also occur.

Signs and symptoms of venous gas embolism include
1) Increased heart rate +/- dysrhythmias (RV strain)
2) Sudden reduction in ETCO2 (useful monitor, sensitive)
3) Hypotension
4) increased PA pressures, CVP, neck vein distension
5) hypoxia / decreased saturations / cardiovascular collapse may be late sign

In awake patients – the presentation may include SOB, chest pain, decreased conscious state.

Detection of venous air embolism (VAE)
1) TOE – most sensitive and specific but invasive and may be difficult to place
2) Doppler ultrasound (most sensitive non-invasive monitor, but not quantitative and does not differentiate between massive and insignificant air embolism.
3) ETCO2 – sensitive but not specific, less invasive, affected by cardiac output.
4) Increased end tidal nitrogen – detects sooner than ETCO2
5) PA catheters (sensitive but invasive and not specific for air embolism)
6) Millwheel murmur on praecordial stethoscope (least sensitive and only apparent after massive embolism)

Prevention
1) avoid sitting position unless essential
2) elevate head only as much as necessary (reduce height between operative site and the right atrium)
3) ensure adequate blood volume to maintain a positive CVP, jugular venous compression during periods of high risk
4) small amounts of PEEP (5-10cmH2O) may reduce the risk of air entrainment (controversial and should be avoided in sitting neurosurgery)

Management
1) Inform surgeon of a sudden change in ETCO2 > 5mmHg, stop further embolism / entrainment eg by flooding the area with fluid. Discontinue abdominal insufflation.
2) Call for help
3) Operation site below heart eg Trendelenburg if possible / Debatable Head down left lateral positioning if possible to keep air in R atrium. Surgeon may request bilateral jugular venous pressure.
4) Secure Airway / Breathing: 100% O2, STOP N2O if being used (N2O does not increase the risk of VAE but may worsen its outcome)
5) Circulation: IV fluid bolus if hypotensive / vasopressors or inotropes / IV Lignocaine (may be beneficial but unproven)
6) CPR if necessary
7) Aspirate air from RA/RV if CVC present (tip should be placed close to junction of SVC and RA.
8) PEEP controversial, increasing intrathoracic pressure with a Valsalva (reduces venous return preventing further air entry)
9) Hyperbaric Oxygen if available for paradoxical embolism
10) Management Post Event / ICU HDU if significant VAE / Patient information / Debriefing and case review
October 2008 Question 4.

In what circumstances is it permissible to permanently handover responsibility for an anaesthetic to a colleague and how would you ensure that this handover occurs safely?
PS 53 : Statement on the Handover Responsibilities of an Anaesthetist

Permanent handover of responsibility for an anaesthetic to a colleague may be permissible in the following circumstances:
• Anaesthetist fatigue
• Illness
• Suitable and willing colleague
• Any other legitimate commitment

Handover may be advantageous to patient care by preventing undue fatigue of primary anaesthetist.

Safe Handover:
• Will not compromise patient safety provided that appropriate procedures are followed, including:
o The primary anaesthetist is satisfied as to the competence of the relieving anaesthetist
o Handover must occur at a time of clinical stability, and no potential adverse events are likely to occur
o Notification of handover to operating surgeon
• The following information must be imparted:
o Patient:
• Medical / Surgical / Anaesthetic history, Medications, Allergies, examination findings and investigation results
o Surgical:
• Nature of surgery
• Stage of surgery
o Anaesthetic:
• Description of anaesthetic – drugs, IV lines, airway security, fluid management, untoward events and forseeable problems
• Plan for intraoperative and postoperative management – Analgesia, IV fluids, Anti-emetics, Disposition, and supplementary oxygen requirements.
o Other:
• Documentation must be uptodate
• Relieving anaesthetist satisfied and all queries addressed
• Leave contact details
October 2008 Question 5.

An otherwise well 60-year old man is having a radical prostectomy. List and briefly evaluate strategies to prevent peri-operative thromboembolism.
General measures:
Both these measures are easy to implement and low cost
• Adequate hydration:
o To prevent blood hyperviscosity (Virchows Triad), which predisposes to venous thromboembolism (VTE)
• Early ambulation:
o Significantly lowers risk of VTE in THR, as well as reduced hospital length of stay, complications and mortality at 6 months
o Should be a routine part of post-op care, and may be considered as sole prophylaxis in low risk surgical patients

Mechanical methods:
No mechanical prophylaxis has been demonstrated to reduce the risk of death or pulmonary embolism (PE)
Generally less efficacious than pharmacological methods
• Thromboembolic Deterrent (TED) Stockings:
o Improve venous flow
o Efficacy: 50-60% reduction in risk of VTE
o Lower efficacy than anticoagulant based prophylaxis, especially for preventing proximal DVTs
o When combined with unfractionated heparin, they have been shown to produce a FURTHER 75% reduction in risk of DVT as compared to heparin alone
o Must be used with caution in patients with arterial insufficiency, and fitted properly
• Intermittent Pneumatic Compression Devices:
o Improved venous flow by a direct effect of pumping venous blood
o They may also promote clearance of prothrombotic clotting factors and increase local plasminogen activators (enhanced fibrinolysis)
o As effective as LMWH or warfarin, but only among patients with BMI < 25
o Potential for skin breakdown and patient discomfort
o They are only effective if used continuously while patients are non-ambulatory (~15 hours per day)
o Method of choice for solo VTE prophylaxis when patients are at increased risk for bleeding with anticoagulants
• IVC filters:
o Not recommended for prophylaxis in absence of DVT
o Reduce the incidence of CLINICAL PE, but no demonstrated decrease in incidence of FATAL PE
• Neuraxial Blockade:
o Decrease DVT risk possibly from decreased venous stasis, and decrease in post-operative hypercoagulable tendency due to modulation of neuroendocrine response to tissue injury
o Have not demonstrated a decrease in incidence of PE
o Protective effect of neuraxial anaesthesia is less than that of pharmacological prophylaxis and is less apparent when pharmacological prophylaxis is used
o Inadequate for solo thromboprophylaxis
o Timing and dosing of pharmacological prophylaxis must be borne in mind when used in conjunction with neuraxial blockade (ASRA guidelines)

Pharmacological methods:
• Unfractionated Heparin:
o Very effective prophylactic agent that clearly decreases the incidence of symptomatic and asymptomatic VTE by at least 60-70%
o Equivalent efficacy to LMWH in moderate or high risk surgical patients for SYMPTOMATIC VTE, but less effective than LMWH for ASYMPTOMATIC DVT.
o Less effective than LMWH for any VTE in very high risk surgery
o There is an increased risk of minor bleeding and HITTS, as compared to LMWH
o Dose: 5000 units SC BD (moderate risk) and 5000 units SC TDS (high risk)
• LMWH: (Enoxaparin, Dalteparin)
o At least as effective as unfractionated heparin
o Agent of choice for very high risk surgical patients
• Warfarin:
o Higher incidence of asymptomatic DVT, presumably due to delay in anticoagulant effect
o Incidence of symptomatic DVT within a 3 month post-operative period is comparable between warfarin and LMWH
o Useful especially for high risk surgical patients and those who require extended thromboprophylaxis
• Fondaparinux:
o Antithrombin mediated Xa inhibitor
o Associated with a lower incidence of asymptomatic DVT but no difference in symptomatic VTE as compared to standard LMWH
• Direct Thrombin Inhibitors:
o Desirudin – similar safety profile to LMWH with greater efficacy for VTE prevention
o Rivaroxaban – RECORD trial indicate that its possibly superior to LMWH in prevention of VTE in knee and hip replacements, but with an increased risk of bleeding. Safety with regards to nerve blockade is not established.

The ultimate choice of prescription for VTE thromboprophylaxis will depend on:
• Type of surgery
o Prostate surgery probably moderate risk
• Presence of patient risk factors for VTE:
o Eg age, medical illness, trauma, previous DVT, malignancy, obesity, smoking and period of immobilization
• Likelihood of bleeding
• Consequences of bleeding
• Timing of commencement and duration of prophylaxis
October 2008 Question 6.

You are covering ICU in your local district hospital when a 14 year old boy presents to your emergency department obtunded and hypotensive with a rash suggestive of meningococcal sepsis.
Describe your resuscitation.
Meningococcal sepsis is an acute life-threatening emergency requiring immediate attendance and management, treating specifically the features of shock and increase intracranial pressure (ICP)

Immediate management
• Airway:
o Assessment of airway patency
o Intubation would be indicated if:
• Unable to protect airway
• GCS equal to or < 8
• Hypoxic despite supplementary oxygen
• Refractory shock
• Breathing:
o Assessment of breathing – look for chest movement, auscultate lung fields
o High flow oxygen to aim for Sats > 95%
o If intubated will require controlled ventilation in ICU (hyperventilation if signs of raised ICP, otherwise aim for normocapnia)
• Circulation:
o Assessment for signs of shock:
• Tachycardia, hypotension, cool peripheries, pallor, prolonged capillary refill time (>3s), tachypnoea, hypoxia, confusion, drowsiness, low urine output
o Establishment of 2 x large bore IV access
o Boluses of IV Normal Saline 20ml/kg, and review hemodynamic parameters
o 2 further boluses of 20ml/kg crystalloid if unimproved hemodynamics
o If refractory shock, start noradrenalin infusion (warm shock) or adrenalin infusion (cold shock)
• Disability:
o Assess conscious state – GCS score, AVPU scale
o Pupils – size and reactivity
o Blood glucose:
• If BSL < 3.5, treat with 10% dextrose 5ml/kg
o Look for signs of raised ICP:
• Headache
• Nausea and vomiting
• Drowsiness, confusion, or fluctuating level of consciousness
• Papilloedema
• Unilateral Pupillary dilatation
• Abnormal posturing
• Focal neurological signs
• Seizures
o If ICP raised, treat with:
• Mannitol 0.5g-1.0/kg IV
• Frusemide 0.5-1mg/kg IV
• 3% Hypertonic saline (via CVC ideally)


Specific management – Immediate antibiotics are essential
• Bloods – Blood cultures, Meningococal PCR, FBE, Urea/Electrolytes, Ca/Mg, Coagulation studies, X-match, BSL, Lactate, ABG
• Lumbar puncture NOT to be performed due to signs of raised ICP
• Empirical antibiotics once blood cultures obtained – Ceftriaxone 50mg/kg IV bd, or Cefotaxime 50mg/kg IV QID
• Referral to Infectious Diseases Team

Further management:
• Correct electrolyte abnormalities – Mg, K, Na, Ca
• Consider sodium bicarbonate 8.4% if pH < 7.1
• Treat seizures – Clonazepam, and consider Phenytoin for prophylaxis
• Consider Hydrocortisone 4mg/kg if shock poorly responsive to inotropes / vasopressors
• CXR – to verify ETT and central venous line positions, evidence of pulmonary oedema
• CVC access – for infusion of vasoirritant drugs, CVP monitoring
• Intra-Arterial Line – for ABG sampling and real time BP monitoring
• Nasogastric tube
• IDC – for monitoring adequacy of fluid resuscitation, aim for urine output >0.5mg/kg/hr
• Arrange for transfer to PICU

Other
• Prophylaxis of household contacts – Ciprofloxacin, ceftriaxone
o Contact tracing
• Inform Department of Health of possible meningococcal infection
• Standard and Additional Precautions (advice from ID team)
• Nursing in Isolation room (with negative ambient pressure)
October 2008 Question 7.

A 6-year-old girl with severe spastic cerebral palsy presents for major orthopaedic surgery to correct lower limb deformities.

Outline the implications of cerebral palsy for anaesthesia management for this operation.
Cerebral palsy is a clinical entity of motor, sensory and/or intellectual impairment of varying severity caused by congenital or acquired insults.

Anaesthetic implications:

1. General paediatric anaesthetic considerations:
• Altered drug doses- rule of thumb for weight (age + 4) x 2 may not apply in child with cerebral palsy, malnutrition
• Paediatric anatomy- intubation/positioning implications
• Parents- expectations regarding induction, management of parents perioperatively
• Difficult IV access- local anaesthetic cream prior, gaseous induction (see below re aspiration risk)
• Communication difficulties- understanding of operation at age six, intellectual impairment with cerebral palsy
• Association with other syndromes with anaesthetic implications

2. Respiratory/airway considerations
• Aspiration- oesophageal dysmotility/abnormal LOS tone, predisposes to GORD. Rapid sequence induction may be indicated. Antacid prophylaxis prior if time.
• Weak cough, respiratory muscle hypotonia- predisposes to chest infections
• Chronic lung disease- often ex-premature with neonatal respiratory distress
• Restrictive lung disease- secondary to severe scoliosis if truncal muscles involved
• Sensitivity to opiates
• Poor dentition- damage to teeth during airway management.
• Temporo-mandibular joint dislocation joint common

3. Muskuloskeletal
• Positioning- fixed flexion deformities, muscle contractures. Require much padding, attention to pressure areas
• Vascular access difficult due to above
• Heat loss- very little subcutaneous fat- require active warming prophylactically (Bair hugger, warmed fluids) with minimisation of operative loss
• Continue anti-spasmodics peri-operatively.

4. Neurological
• Epilepsy- increased risk. Continue anticonvulsants peri-operatively. Care with pro-convulsant drugs (eg. ketamine, tramadol)
• Intellectual impairment. Communication difficult. Enlist parents, anxiolysis pre-operatively may be indicated.

5. Particular operative issues
• SPICA application at end- requires moving patient down table. Possible dislodging airway, new pressure areas, must not be too light prior to application
• Prolonged operation. Potential for large volume blood loss- regular Hb monitoring, may require blood available

6. Post-operative
• Analgesia. Painful operation. Regional useful (eg. lumbar epidural placed after induction. Combination of local anaesthetic with clonidine may reduce painful post-op spasms. PCA likely unfeasible given age/intellect. Nurse initiated opiate boluses/infusion morphine helpful. Avoid ketamine if seizures predominate.
• Physiotherapy
• Respiratory monitoring. Sensitivity to opiates may warrant continuous pulse oximetry/line of sight nursing
• Higher nursing requirements given above issues
October 2008 Question 8.

A 25-year-old primigravida patient presents to the delivery suite at 38 weeks gestation complaining of a headache and difficulty with her vision. Her BP is 180/115 and she has clonus. Cardiotocograph monitoring shows no indication of foetal distress.

Outline your initial management of her preeclampsia.
• Preeclampsia- triad of:
1. Protenuria (> 0.3g/day or spot protein:creatinine ratio of > 0.03)
2. Hypertension (DBP > 90 mmHg, SBP > 140 mmHg or an increase in DBP of > 15 mmHg or SBP > 30 mmHg)
3. Oedema (peripheral)
• Aetiology poorly understood- multisystem disorder manifesting with endothelial dysfunction with likely immunological basis, triggered by the feto-placental unit.
• Severe pre-eclampsia if:
1. BP > 160/110
2. Protenuria > 5g/day
3. UO < 400mL/day
4. Pulmonary oedema
5. Hepatic rupture/RUQ pain
6. Platelet count < 100 x 109/L
7. Cerebral complications
• Pre-eclampsia- 6-8% of pregnancies, severe preeclampsia- 0.25% - 0.5%
• This woman has severe pre-eclampsia
• If untreated- may progress to eclampsia with seizures and marked increase in morbidity (pulmonary oedema, right heart failure, renal failure, seizures) and mortality.
• Management must be in a multidisciplinary team involving obstetricians, midwifery and paediatricians.
• Clinical assessment
1. History- general obstetric (prior pregnancies/problems/pre-eclampsia), diabetes, medications, other causes of hypertension (essential/secondary causes) and treatment, estimated date of delivery
2. Examination- vitals (SaO2, RR, PR, BP), cardiorespiratory (pulmonary oedema, right heart failure- tricuspid regurg, elevated JVP), airway (pharyngeal oedema/stridor), neurological- (clonus, brisk reflexes, muscle spasm, reduced GCS, raised ICP- papilloedema), abdominal- (RUQ pain, fundal height)
3. Investigations- blood- FBE (pl count), coags, LFTs (HELLP syndrome), uric acid, UEC (renal failure), haemolytic screen (HELLP)

Management principles
• Urgent blood pressure control, stabilisation of woman prior to delivery (no foetal distress so urgent delivery not mandated) and prevention of progression to eclampsia)
1. Magnesium- MAGPIE trial (LANCET 2002) > 10 000 patients shows halved risk of eclampsia, probably reduced maternal death and reduced risk of placental abruption. 100 women with pre-eclampsia or 70 women with severe pre-eclampsia need to be treated to prevent one seizure. Loading dose 4 g, maintenance dose 1g/hr.
2. Antihypertensives- hydralazine 5mg IV inital dose. Takes 15 mins to work, wait before repeat dose. Labetalol- 200mg orally or 20 – 40mg IV every 10 mins. Nifedipine 10mg orally. Latter two drugs may be preferable (hydralazine- more tachycardia, more fetal complications). Avoid too rapid reduction- fetal compromise. Aim 10-20 mmHg reduction each 10 mins.
3. Fluid restriction- total fluids < 1 mL/kg/hr. If oliguria (0.5 mL/kg/hr), cautious fluid bolus 250 mL colloid/crystalloid may be considered. Pulmonary oedema major cause of death. CVP monitoring if refractory oliguria/signs pulmonary oedema
4. Regional anaesthesia desirable (exaggerated hypertensive repsonse at laryngoscopy, potential for ICH, increased incidence of failed intubation). Platelet count > 80 for spinal, > 100 for epidural (considering speed of decline, other risks for bleeding, comparative risks of GA LUSCS. Based on TEG studies in late 90s).
5. Third stage management- avoid ergometrine
• Disease way worsen post-partum. Up to 44% of eclampsia occurs post-partum. Continue anti-hyptertensive therapy (may be up to 3 months). Admission to HDU. Continue magnesium until diuresis established.
October 2008 Question 9.

Describe a technique of peribulbar block for cataract surgery.

Describe how you would minimise complications of this block.
Technique
• Check consent, side of surgery (marked by surgeon), patient ID and understanding of operation
• Establish IV access and monitoring
• Minimal sedation (eg. alfentanil 200 mcg or midazolam 0.5-1 mg)
• Topicalise- Amethocaine 0.5% drops
• Patient supine. Ask to look straight ahead
• ETOH/Chlorhex swab skin for transcutaneous entry
• Junction of medial 2/3 and lateral 1/3 of inferior orbital rim (where maxilla joins zygoma). Insertion point is just lateral to this and 1 mm above rim).
• Needle- long (25mm) 25G orange needle. 10mL syringe. Pass slowly backwards perpendicular to all planes until tip level with posterior pole (hub in plane of iris).
• Watch carefully for globe movement- indicates engagement with needle
• 6-8 mLs injected after aspiration (mix 1500IU of hyaluronidase in 10 mLs NSaline, titrate out 2 mLs added to 4 mLs 2% lignocaine and 4 mLs bupivicaine 0.5%. Final mix is 0.8% lignocaine, 0.2% bupivicaine with 15 IU/mL hyaluronidase).
• Following injection, Honan balloon inflated to 25 mmHg for 5 mins.
• If further top up needed, medial canthus injection. Just medial to caruncle, needle passed backward at 10 degree angle to sagittal plane, directed towards medial wall of orbit.

Minimising Complications
• Appropriate selection of patients
1. Axial length < 25 mm (less chance of staphyloma with globe perforation risk)
2. Severely myopic patients- same reason
3. Coagulopathy- caution with coagulopathic patients (risk retrobulbar haemorrhage). Warfarinised- INR > 2.0 contraindiciation. Subtenon's instead.
4. Localised skin/conjunctival infection or systemic sepsis- avoid.
5. Perforated or infected eye
6. Inability to lie still
7. True amide LA allergy- avoid
• Single injection technique (inferolateral) preferable (two injection technique doubles risks)
• Do not 'wiggle' needle to ensure disengaged from globe
• Do not insert more than 15mm
• Aspirate prior to injection
• Stop if pain/loss of vision
• Stop if patient moves eye
• Stop if globe becomes tense/proptosed or if ptosis (likely retrobulbar- smaller volume)
• Use of Honan balloon will reduce the raised IOP caused by block prior to eye opening.
• If suspect retrobulbar haemorrhage- notify surgeon immediately for lateral canthotomy
October 2008 Question 10.

Discuss management of vasospasm following coiling of a cerebral aneurysm.
Cerebral vasospasm is serious : 25% following SAH have cerebral ischaemia/infarction.
-? cause - thought to be due to irritant blood causing vasocontriction, worsening perfusion
-Symptoms & Signs include
-Headache, Altered Conscious state, Focal neurology, Seizures
-Haemodynamic instability, pulmonary oedema, arrhythmia

Discuss utility of World Federation of Neurlogical Surgeons

Discuss utility of Fisher Grading system



General - supportive treatment
-airway maintenance, avoid hypoxia, posture & pressure care. Normothermia.
-Correction of biochemical and haematological abnormalities.

Triple H therapy
-Thought to benefit if commenced prophylactically (no evidence, no RCTs)
-Hypertension & Hypervolaemia
-3 L+ /day 0.9% NaCl - ↑ CBF to ischaemic areas with impaired autoregulation
-Vasopressors (norad) usually required as hypervolaaemia itself not enough to raise BP
-Haemodilution
-Aim 30-33% Hct; ↓ improves flow rheology (CBF α 1/CVR)BUT ↓ Hct will ↓ CaO2
-Colloids vs crystalloids
-controversial - 0.9% NaCl will result in ↑ECF with equilibration and ? cerebral oedema
-disrupted BBB - colloid may leak into brain, causing osmotic gradient into brain further
-Keep [Na+] > 130 (Hypertonic saline of no benefit in SAH)
-MAP - titrate up 10-20 mmHg until reversal of neurology (Max SBP 160 untreated 200 if
treated) maintain for 3-7 days until vasospasm resolves
-CVP - Aim: 8-12 mmHg
Complications of triple H therapy
-CNS - Cerebral oedema, ↑ICP, Rebleeding into infarcted area
-CVS/Overload - APO, hyperdynamic circulation and CCF

Cerebral vasodilators
Nimodipime
- ↓ infarction by 33% if given prophylactically, improves OUTCOME, not mortality
-To benefit must be given between 4th-24th day post haemorrhage
-1-2mg/kr IVI or 60 mg po QID (interestingly the oral version has evidence only..)
-May also be given intra-arterially during coiling
Magnesium
-Neuroprotective in animal studies if given within 24 hours of injury. ? Premature newborns
-BUT no evidence for vasospasm (yet) - same for calcium channel blockers

Radiological Intervention
May be required should vasospasm be severe, refractory to triple H and nimodipine therapy
- Angioplasty & Intra arterial infusion of vasodilators
-Papaverine & Verapamil
-Problems with radiological intervention - failure of therapy / Rupture during intervention
-heralded by ↑ SBP or ↑ ICP +/- HR changes or extravasation of contrast
-Treatment
-Reverse heparin, ↓BP to pre-rupture levels
-↓ ICP - Head elevation, hyperventilation, steroids, mannitol, anti-epileptics
-May need transfer to OT for urgent clipping
October 2008 Question 11.

List the risk factors for the development of chronic pain following a surgical procedure. Outline possible mechanisms for the progression of acute to chronic pain
Chronic post-surgical pain risk factors
-Pre-operative
-Female gender
-↓ Age (younger patients more likely)
-Pain moderate to severe pre-surgery
-Pre-op pain > 1 month duration
-Repeat surgery
-Psychological vulnerability (neuroticism, emotional distress, poor social setting)
-Genetic predisposition
-Workers compensation (secondary gain)
-Intra-operative factors:
-↑ Duration / complexity of surgery
-Surgical approach with risk of nerve damage
-↓ Laparoscopic approach (hernia repair & cholecystectomy)
-High risk surgery
-Amputation
-Thoracotomy
-Breast surgery - especially extensive clearance
-Cholecystectomy
-Inguinal hernia
-Post-operative:
-Pain severity in post-op period (acute, moderate to severe)
-Radiotherapy / chemotherapy

Mechanisms for progression of acute to chronic pain
-Multimodal, complex changes that are not fully understood, each domain (peripheral, spinal
cord & central) results in positive feedback to the others
-Peripheral
-Nociceptor stimulation
-repeated firing causes peripheral sensitisation / conformational change
-Calcium intracellular homeostasis disrupted
-Neuroma formation from direct damage to axons
-ectopic firing on nociceptor neurons
-Sympathetic - CRPS type II
-upregulated α2 receptors
-Spinal cord
-wide dynamic range neurons in dorsal horn & C fibre constant stimulation
-Central sensitisation of NDMA receptors & substance P → Wind up
-C fibres & Aβ fibres in dorsal horn may sprout to lamina I&II
-Particularly in amputation/breast surgery due to loss of afferent imput
- Ectopic activity in spinothalamic tract
- Cross talk from Aδ fibres in substantia gelatinosa (rexed II&III)
-Central
-Decreased descending inhibition - GABA, glycine containing neurons
-Wind up
October 2008 Question 12.

List the indications and contra-indications for the use of an intra-aortic balloon pump. Describe how its performance is optimised.
Indications
- Ischaemic heart disease
- Acute cardiogenic shock
- Emergency angioplasty
- Acute MR & VSD
- Refractory unstable angina
- Refractory ventricular ectopy
- Cardiomyopathy
- eg Acute viral
- Cardiac surgery
- Pre-op - high risk patient
- Cardiac: Severe LMCA, LV < 30%, acute MR, CCF
- Non-cardiac: CRF, CVA
- Post-op to facilitate weaning from CPB

Contra-indications
-Aortic disease
-Aortic regurgitation (worsens regurgitant fraction - even mild may become severe)
-Aortic dissection
-Aortic stents
-Abdominal aortic aneursym
-Peripheral vascular disease
-Bilateral femoral-popliteal bypass grafts
-Tachyarrhythmias
-↓ diastolic time
-Other
-Uncontrolled sepsis - Local or generalised
-Bleeding diathesis - relative - patient will be on anticoagulation for IABP
-Futility not a definitive procedure

Optimising Performance
-Fluoroscopic guidance
-Tip 2-3 cm distal to the origin of left subclavian artery (sternal angle)
-Optimal balloon volume
-25 (< 152 cm tall) - 50 mL (>183 cm tall)
-When fully expanded, should not exceed 80-90% diameter of descending thoracic aorta
-Balloon timing
-Inflation @ start diastole and deflation just before systole crucial to good
augmentation
-early/late inflation, early/late deflation

Trigger Inflate: Onset of diastole Deflate: just before systole

ECG Mid T wave Peak of R wave
Arterial waveform Dichrotic notch Just prior to upstroke
- Regular rhythm
-Allows IABP to correctly time inflation and deflation, if not regular sub-optimal
augmentation will occur, and may worsen myocardial performance

Complications (outside scope of question)
*Local
-Infection
-Haematoma
-False aneurysm
-Bleeding

*Distal
-Distant thromboembolism
-Loss of pulse <-> Limb ischaemia

*Proximal
-Aortic dissection
-Balloon rupture and gas embolus (helium is pretty friendly)
-Balloon entrapment (require vascular surgeon)
-Cardiac tamponade
-Malposition
-Cerebral / renal blood flow compromise
-Haematological
-Thrombocytopaenia
-Anaemia
October 2008 Question 14

What are the signs that may make you suspect opioid abuse in a colleague? If you had suspicions of opioid abuse in a colleague outline the principles that should guide intervention.
Signs of opioid abuse:
Major signs:
- colleague signing out increasing quantities of drugs, or quantities of drugs which are inappropriately high for the use specified.
- Increasingly inaccurate, illegible or unusual record keeping
- A consistent pattern of complaints regarding excessive pain in patients of a particular anaesthetist. Pain is out of proportion to the amount of pain relief recorded.
- Reports of change in attitude or behaviour – decreased socialising, withdrawn, erratic outbursts, low mood or very high mood
- Observation of intoxicated behaviour
- Observation of withdrawal symptoms
- Observation of pills, syringes, ampoules in any non work place environment
- Observation of injection marks on the body
- Observation of syringe in vein of anaesthetist
Circumstantial signs:
- Significant changes in behaviour, personality or emotions
- Intoxicated behaviour, pin-point pupils, weight loss, pale skin
- Increased sick leave or other absenteeism.

Principles of intervention:

Professional responsibilities:

• Principle of non-maleficence - patient safety and well being
• The care of the patient takes precedence
• Duty of care to ensure that my colleague practices in a manner that does not put their patients at risk – at the initial stage when there is no evidence, ensure the colleague is not working alone.
• He/she should be relieved of clinical duties if a judgement is made by the head of department that there is danger to patients
• Confidentiality and welfare of colleague
• Important to maintain strict confidentiality
• Recognise it is a sensitive issue and there is potential for career destruction and immediate risk of the colleague committing suicide once confronted about the issue
• Others e.g. department of anaesthesia, college of anaesthetists, medical board, registration etc.

Strategy:
• This matter needs to be taken seriously and warrants immediate review
• I would inform the head of department or senior department figure, and establish if there is department policy to help deal with this issue (there may be predetermined people to guide the situation)
• Collect objective evidence of chronic impairment - discreet investigation
• Own observations or those of reliable staff (e.g. anaesthetic nurses, recovery nurses) – see above
• Pre-planned confrontation
• Best to avoid 1:1 confrontations unless I know this colleague on a personal level
• A formal meeting should be planned as regard to venue, timing and strategy, where the evidence is described in detail without personal judgement – environment should be controlled; may have drug and alcohol specialist
• The colleague should be reassured of confidentiality and ongoing support
• The colleague may attend with his advocate/support.
• Plan for colleague post-confrontation
• As there is a high risk of suicide, professional help must be enlisted to provide support after the meeting
• A list of resources are provided depending on the type of impairment e.g. hospital substance abuse committee, Doctor’s Help Advisory Service, Psychiatrist/Psychologist, ANZCA, Welfare of Anaesthetists, SIG etc.
• Support and reassurance re: re-entry into the workforce
• Notification of appropriate authorities
• The hospital administration and Medical Board should be informed of all proceedings and undertakings.
October 2008 Question 15

What symptoms and signs suggest the presence of OSA in a patient presenting for preoperative assessment? How does the presence of OSA alter your anaesthetic plan?
Obstructive sleep apnoea defintion:
Disordered breathing and airway obstruction syndrome – cessation of airflow at the mouth and nostrils for more than 10 seconds occurring more than 5 times per hour. Usually associated with a decrease in arterial oxygenation saturation of at least 4%. OSA is independent risk factor for hypertension, CVS morbidity, mortality and sudden death secondary to arrythmias.

Symptoms and signs of OSA:
• Presence of snoring – majority of patients with OSA snore. Increased likelihood of OSA if also obese.
(Obesity – increased fat deposits adjacent to pharyngeal airway (cf neck circumference) and waist circumference (cf neck circumference) are important predictors of OSA severity.)
• History of apnoea during sleep by partner
• Daytime somnolence
• Lethargy during the day despite adequate number of hours sleep
• Enuresis
• May have appearance of difficult airway -enlarged waist circumference
• enlarged neck circumference (>42cm in men, >40cm in women)
• enlarged tonsils, adenoids, nasal turbinates
• prominent tonsillar pillars
• narrowed maxilla, mandible or retrognathia
• macroglossia
• mallampati score of 3 or 4
• Evidence of complications:
• pulmonary hypertension – loud P2
• systemic hypertension
• ischaemic heart disease
• right heart failure

Patient may have results of formal sleep study – Apnoea-Hypopnea Index of >40/hr = severe OSA

Anaesthetic considerations: OSA patients are high risk of pharyngeal obstruction.
Preoperative:
• OSA undiagnosed in large number of patients presenting for surgery. Have high index of suspicion in obese patients.
• If surgery not urgent, then can send patient for sleep study to assess severity of OSA especially if symptoms and signs suggestive. Helps in determining if preop nasal CPAP to be commenced in severe OSA and also post op care location. Discuss with surgeon and patient re; benefit of delaying surgery. No need to delay if patient already using cpap at home.
• OSA is risk factor for difficult mask ventilation and airway intubation (fat deposition in the neck, narrowed airway – pathology causing OSA). Hence adequate assessment and preparation for intubation (difficult intubation trolley, Intubating LMA etc)
• Explanation of risks to patient and plan for anaesthesia and analgesia.

Intraoperative:
• Patients with OSA are often obese. Issues with obesity:
• Decreased FRC – more likely to desaturate quickly
• Problem with spontaneous ventilation due to restrictive defect : more likely to be intubated
• Induction
• Difficult mask ventilation /intubation – optimal positioning, 3mins preoxygenation with 100%O2 and tight fitting mask.
• Increased risk of negative pressure APO if failure to maintain airway during induction. Recommend RSI with sux (be aware of rapid desaturation) or modified rapid sequence with ROC (ventilatewith 02 post NDMB and have sugammedex available) or awake FOB with light/no sedation. Use guedel airway early.
• Increased risk of GORD in OSA - RSI if symptoms of GORD.
• Analgesia and anaesthesia
• avoid long acting sedatives –eg: clonidine, midazolam
• adequate analgesia vital esp if there are CVS comorbidities
– narcotic analgesia can depress ventilation and cause upper airway obstruction secondary to sedation. Minimise use of narcotics. Use multimodal analgesia – paracetamol, NSAIDS, use of regional anaesthesia, avoid opioids in neuraxial anaesthesia. Decrease bolus dose in narcotic PCAs.
• Muscle relaxation – avoid muscle relaxation if possible
• Extubation
• adequate muscle relaxant reversal (use peripheral nerve monitoring)
• ensure patient alert and awake
• upright or lateral position
• suctioning/nasal airway insertion can cause coughing/hemo
dynamic instability – perform before or after emergence
- avoid emergence agitation - dangerous
Postoperative:
• Residual effects of analgesia and anaesthesia present in immediate post operative period. Monitor in HDU if moderate to severe OSA – sitting or lateral position.
• Minimise use of sedatives and narcotics.
• Pulse oximetry and respiration monitoring for 24 hours post op.
• Use CPAP machine with O2 if patient has one at home. No use commencing CPAP treatment in perioperative period unless required. Issues with settings, mask fittings and adjusting to the machine. Preop acclimisation to device important for post op use.
• Difficulty with airway management under anaesthesia is a strong predictor of OSA. If patient undiagnosed, the refer to appropriate specialists and organise sleep study.
May 2008 Question 1.

Outline how oxygen is stored at the hospital and delivered to operating theatres up to and including the wall outlet. In your answer include features that ensure the safety of the system.
VIE (Vacuum insulated evaporator)- main supply of oxygen. Stored in liquid oxygen form below its critical temperature around -160°C at a pressure of 1000 kPa (which is the vapour pressure of oxygen at that temperature.
Safety features at the vacuum insulated evaporator:
• Pressure relief valves to exhaust O2 gas if pressure > 1500 kPa
• If demand is excessive causing the pressure to drop, then a control valve opens to allow liquid oxygen to evaporate through copper coil superheaters
• Differential pressure transducers and situating the VIE on a scale to weigh it. Both of these will determine the amount of liquid oxygen within the vessel, and alarm when refilling is required
• Gas specific valves and connections that only allow VIE to be filled with liquid oxygen
• Pressure regulators that reduce the VIE pressure of 1000 kPa to pipeline pressure of 400 kPa. These are multistage and are designed to handle peak oxygen demands without compromising outlet pressure and are in parallel to allow for servicing
• Often large hospitals will have a secondary, smaller VIE in case of primary failure`

A reserve bank of cylinders with manifolds connecting them to hospital pipelines exist if the VIE fails.
Safety features:
• Enough reserve to supply hospital for two days in case of VIE failure
• Stored in secure, dry environment, free from extremes of temperature
• Full and empty cylinders stored in separate areas
• Black cylinders, white shoulders. Pin index system governs gas-specific connections.

Pipelines- deliver oxygen to operating theatres. High quality copper.
Safety features:
• Non-interchangeable coupling (Schrader-type valves)
• High quality copper, preventing rupture/rusting
• Appropriate size to ensure adequate gas supply delivery without pressure drop
• Cleaned with medical air, constructed without contamination
• Checked prior to use with gas analyser at wall to ensure no cross-links to non-oxygen lines

Wall outlet- governed
Safety features:
• Sleeve-index system- gas specific colour and shape coded (white for oxygen)
• Sealing allows for removal of hoses from outlet without injury to staff
• Gas supplied at 4 bar (surgical instrument driving gas = 7 bar).
• Free of oil/grease

The whole system must be commissioned prior to use, conform to relevant Australian and NZ standards and have an oxygen failure alarm within the operating theatre.
May 2008 Question 2.

Why is the radial artery a common site for arterial cannulation? What complications may occur from radial artery cannulation and how may they be minimised?
Reasons for choice of radial artery:
• Convenient access. Usually palpable even in thin/obese patients
• Anatomically consistent- predictable course, straight
• Comparatively clean site, skin more easily disinfected (compared with femoral)
• Superficial, easily palpable
• Not end artery- collateral circulation minimises distal ischaemic risk
• Compressible
• Located discrete from nerves, minimising neurological injury. Nerve blood supply not dependent on radial artery. Thrombosis will not compromise neurological function.
• Correlates well with BP

Complications/ways to minimise
• Infection. Hand wash prior/sterile gloves. Adequate skin preparation (chlorhex/ETOH). Ensure tip/cannula remain sterile. ANZCA PS 28.
• Thrombosis. Continual slow infusion heparinised saline. Monitoring for hand ischaemia. Prompt removal if concerns.
• Hand ischaemia. Rare (Miller = 0.1%). Allen's test prior (ensures adequate ulnar artery collateral flow). Exclude all air bubbles from fluid column. Avoid manual flushing. Clear labelling of arterial line to avoid inadvertant drug injection. Continual monitoring for adequate hand perfusion.
• Drug injection. Clear labelling of arterial line. Red cap. Avoid manual flushing of line. Dedicated tubing without injection ports.
• Line disconnection. Potential hemorrhage risk. Clear line of sight view of line and attachments at all times. Secure, tight Luer lock connections.
• Pseudoaneurysm. More likely with multiple attempts. Avoid.
• Misinterpretation of data (eg. subclavian stenosis). Correlate with NIBP/other arm. Avoid catheter kinking at skin.
• Overlying skin pressure necrosis. Care with padding/taping too tightly to skin.
May 2008 Question 3

Describe the anatomy of the brachial plexus relevant to performing an interscalene block under ultrasound guidance. Include a drawing illustrating the real or sono anatomy you would expect to see in a transverse view of the brachial plexus at the point of needle insertion.
Interscalene block aims to block superior, middle and inferior trunks of the brachial plexus between scalenus anterior and scalenus medius, in the neck

Anatomy
• Trunks lie between scalenus anterior and scalenus medius in the 'interscalene groove'
• Landmarks: cricoid cartilage (C6) and posterior border of sternocleidomastoid muscle (often where external jugular vein crosses)
• Lateral to this- interscalene groove palpable

Sonoanatomical landmarks
• Carotid artery
• Internal jugular vein
• Place ultrasound probe lateral to larynx and visualize thyroid gland
• IJV is compressible, carotid artery not. Colour flow may differentiate
• Move probe sideways to lateral border of sternocleidomastoid muscle, direct tip slilghtly caudad
• In short axis view, brachial plexus will become visible
• Superior, middle and inferior trunks are seen as distinct hypoechoic structures
• Overlap each other
• Plexus is quite superficial
o Complications are caused by advancing needle tip too deeply
o Contraction of levator scapulae if needle is posterior (dorsal scapular nerve)
o Contraction of diaphragm (phrenic nerve) if needle is too anterior
o Never aim needle cephalad as this may cause subarachnoid spread
Anatomical variations
• Sometimes a portion of the brachial plexus may penetrate and lie within or anterior to the anterior scalene muscle

Various complications
• Inadvertent subarachnoid injection
• Inadvertent phrenic nerve palsy
• Inadvertent injection into artery or vein
May 2008 Question 4

Describe the clinical features and treatment of Fat Embolism Syndrome.
Fat embolism syndrome
A multi-system disorder that occurs when fat emboli in the circulation produce serious clinical manifestations. Two main pathogenic theories:
1. Mechanical theory. Obstructive bone marrow fat enters the venous system as a result of high intramedullary pressures after reaming/cement application/prosthesis insertion. Reaches systemic circulation either through pulmonary shunts or a PFO.
2. Chemical theory. Chemical mediators released at time of trauma result in free fatty acids released from endothelium, that act on pneumocytes and result in acute lung injury.

Main symptoms and signs are respiratory, neurological and cutaneous.
Clinical features
Onset
• Either fulminating or gradual in onset
• May occur intraoperatively when reaming/cementing long bones
• May also occur following long bone fracture in trauma
• Gradual onset usually 12-36 hours following injury is typical

Natural history
• Pulmonary dysfunction typically resolves within 3-7 days
• 10-44% require mechanical ventilation
• mortality of 1-20%
• early surgery following traumatic bone fracture reduces mortality
• avoidance of intramedullary fixation of fracture reduces mortality

Incidence
• Reported <1%
• Depends on sensitivity of testing
• Highest incidence with lower limb fractures
• Can also occur in bone marrow necrosis, acute sickle cell crisis, acute pancreatitis
• Severe neurological symptoms of FES frequently resolve

Major clinical signs
Respiratory
• Tachypnoea
• Dyspnoea
• Raised PA pressures
• Respiratory failure
• hypoxaemia
Neurological
• Drowsiness or confusion
• Coma
• Seizures
• Global dysfunction, hemiparesis reported
Petechial rash
• Over upper half of body/mucous membranes of mouth/conjunctivae
• Develops in 50-60% of cases
• Retinal exudates/haemorrhages

Minor clinical signs
Hypertension
Fever
Tachycardia
Renal- oliguria or anuria
Jaundice
Retinal changes- fat or petechiae

Laboratory indices
Thrombocytopaenia- platelet count < 150x10^9/L
Unexplained anaemia
DIC
Increased lipase
Increased phospholipase A2
CXR- patchy areas of consolidation typically in middle and upper zones
BAL- macrophages with fat globules
Urine- fat globules
Blood- fat macroglobulaemia in peripheral film
Raised ESR

Management of FES
Mainly supportive treatment
Early fluid resuscitation and haemodynamic stabilization vital
Maintain saturations- 100% oxygen in acute intraoperative setting.

Respiratory support- most deaths occur secondary to respiratory complications
• Initially increase inspired oxygen fraction
• Progress to CPAP with additional PEEP if required
• Intubation and ventilation if continued hypoxia

Early fracture fixation reduces incidence. Surgical reduction of intramedullary pressures imperative (vacuum venting, avoidance of cement if possible).

No specific prophylactic/therapeutic measures.

Steroids
• Controversial
• Stabilizes pulmonary capillary membrane
• Blunts inflammatory response
• Stabilizes complement system activation
• Retards platelet aggregation
• Problem however with overwhelming sepsis in high dose steroids
• Methylprednisolone does not modify pulmonary hypertension

Heparin
• Controversial
• Low dose heparin increases risk of bleeding in trauma patient
• Theory for heparin is clearing lipaemic serum by stimulating lipase activity

Aspirin
• Prevents gas exchange abnormalities
• Blocks production of thromboxane
• Reduces platelet adhesiveness

No good quality evidence for these latter three therapies.
May 2008 Question 5.

A 65 year-old female who ways 85 kg and is 165 cm tall (BMI 31) is scheduled for total knee replacement surgery. She has no other health problems. Discuss the pros and cons of intra-thecal morphine for post-operative analgesia in this patient.
Analgesia crucial following TKR due to early mobilisation and passive ROM devices.
Nature of pain with TKJR is severe in acute setting especially with passive ROM devices

Benefits of intrathecal morphine
• Good quality analgesia
• High incidence patient satisfaction
• Simple regime of single injection intrathecal dose
• Technical ease of administration
• Usual dose 100-300mcg single dose
• No further opioids required for further 24 hours
• No motor blockade (unlike local anaesthetic techniques)
• Earlier return of oral intake postoperatively
• Earlier onset of mobilization
• Cost-effective
• Equal efficacy to peripheral nerve blockade with femoral nerve catheter techniques
o No difference in side effects

Disadvantages to technique of intrathecal morphine
• Errors in titration
• Requires diluted form of morphine from pharmacy
• If 10mg ampoule morphine only available
o Potential for titration errors
o Small titration errors result in less than 24 hours in duration
o Large titration errors result in respiratory depression
• Requires vigilance from ward and nurses 24 hours postoperatively
o Window for error with parenteral administration of morphine
• Sedation in BMI 31 patient with obstructive sleep apnoea may cause hypoxia
• Side effects are commonly reported
o Nausea and vomiting
o Need to avoid sedating antiemetics ie droperidol
o Pruritus – common and difficult to treat
o Treatment regimes for pruritus may be sedating ie anti-histamines
• Respiratory depression may be late
• Requires appropriate monitoring in HDU setting
• Continuous pulse oximetry if available
• Requires pain service input
• Masks acute compartment syndrome
• Difficult to manage if breakthrough pain in 1st 24 hours
o Potential causes include
o Incorrect titration
o Incomplete delivery of full dose (some of morphine not deposited in intrathecal space)
o Patient may still be at risk of delayed respiratory depression
o Further doses of parenteral morphine may be sedating
o Intra-thecal risks as well : PDPH, Nerve damage etc
May 2008 Question Q6.

These electrolyte results were taken from a 38 year old woman found obtunded 30 hours after abdominal hysterectomy. She had no concurrent illnesses prior to surgery.

Na 110
K 3
Cl 80
HCO3 25
Glucose 5
Urea 3
Creatinine 0.06
Measured osmolality 225

Explain how these electrolyte abnormalities are most likely to have arisen & describe how you would correct them.
This patient has severe Hyponatraemia by definition - [Na+] < 135 mmol/L and obtunded
-Critically unwell, needs prompt diagnosis & management in ICU setting
-15% inpatients have ↓[Na+], usually 2-7 days post insult, self limiting
-accounts for massive increase in mortality (60% above baseline risk)


Differential Diagnosis
Differential Diagnosis
Hypovolaemic
• Source from the kidney
o Diuretic
o Polycystic kidney disease
o Addision’s disease
o ATN
• Source elsewhere
o Third space losses : small bowl obstruction
o Pancreatitis
o Fistula
o Burns
o Endurance sports

Normovolaemic
• Loss of Na and water
o Glucocorticoid deficiency
o Severe hypothyroidism
• SIADH
• Excess water
o Excess 5% dextrose
o Water overload

Hypervolaemic
• Retention of Na and water : Cirrhosis
• CCF
• Nephrotic syndrome
• Renal failure
• Primary polydipsia

Assessment
-Neurological
-Obtundation → airway / respiratory support, others: seizures, hyporeflexia
-Volume status crucial
-euvolaemia in SIADH, hypovolaemia in CSWS
-Review of history, charts
-new drugs, fluid balance, urine output
-Urine osmolality/[Na+]/Ketones (SIADH: ↑ urine osmolality, Urine [Na+] > 18)
-Check TFTs, cortisol (+/- synacthen), UEC

Treatment (assuming SIADH)
- STOP any iatrogenic cause - fluid, anti-convulsant, operation etc
-Gradual correction of [Na+] @ 0.5 mmol/l/hr or 8-10 mmol/L/day
-If faster - my get central pontine myelinolysis
-Fluid restriction < 1L/day (conservative therapy) will not be effective here
-Hypertonic saline 3%
-Seizure/coma: begin at 2-4 mL/kg/hr then 0.5-1 mL/kg/hr for 12-24 hours -
regular ABG to guide infusion. Stop infusion [Na+] > 120 mmol/L & fluid restrict
-Drugs
-Frusemide (must replace with isotonic fluids)
-Demeclocycline (↓ renal sensitivity to ADH)
-ADH receptor antagonists - ? licenced yet
May 2008 Question 7

A 34 year old woman present at 36 weeks gestation with an anterior placenta praevia and Caesarean section is scheduled. She has no intercurrent health problems. She has a history of 2 previous Caesarean sections under regional anaesthesia. Describe & justify the changes this history would make to your routine pre-operative & intra-operative management plan for Caesarean section
Anterior placenta praevia on background 2 x previous LUSCS
-vaginal delivery not possible if placenta within 2 cm of the os
-anterior lower segment placenta = obstetrician will cut into placenta = massive blood loss
-Uterine scar from previous pregnancy = ↑ risk of accreta
-2 x LUSCS = 50% accreta this time, if Accreta = 66% risk Caesar-hysterectomy
-(percreta = 95% Caesar-hysterectomy, 7% mortality)

Pre-operative changes
-Team consultation and planning in advance
-High risk of major haemorrhage, availability of key staff, perfusionist etc
-Schedule operation on elective list in business hours
-Resource allocation allows obstetric, anaesthetic and nursing staffing appropriate
-Experienced obstetrician +/- Gynae oncology surgeon
-likelihood of difficult dissection, caesar-hysterectomy, internal iliac ligation
-Haematology support
-Blood cross matched and in theatre (usually G&H will suffice)
-Novoseven - consider use before hysterectomy
-Blood conservation techniques/perfusionist
-Acute normovalaemic haemodilution (up to 1L) OR Cell salvage
-Anaesthetic plan - GA Vs Regional
-GA ʻconservativeʼ
-attention not diverted from reassuring parturient/partner
-comparable blood loss in placenta praevia (cf: regular population)
-no stress of ʻconvert to GAʼ when unstable, spinal wearing off
-airway secured, ventilation controlled
-no problems with neuraxial needle/catheter and coagulopathy
-Regional
-allows parturient/partner to be involved in birth
-may make more hypotensive if haemorrhage
-only attempt if experienced obstetric anaesthetist, help available
-Pros and cons if overwhelming indication for regional
-Discussion with patient & partner re: plan for peri-operative, post-op management
-Post-op care
-HDU or ICU (may not be appropriate for general maternity if complicated)
Intra-operative changes
-Fluid balance management & monitoring
-additional large cannula (14-16g) required, consider arterial line +/- CVP
-Major haemorrhage risk requires early recognition and prompt treatment
-Warming
-Fluid warmer, forced air warmer
-Cold worsens coagulopathy, causes left shift of ODC, neurologic effects as
well as potentiating arrhythmia, poor wound healing and wound infection
-2nd line oxytocic agents
-Ergotamine (250 mcg IV and IM)
-PGF2α (250 mcg intramyometrial injection)
-alternatively carbaprost IM (NB: severe bronchospasm)
-Second anaesthetist allows designated roles in evolving crisis
-ie ʻproceduralistʼ, ʻblood bank liason/transfusionistʼ, ʻanaesthestistʼ
May 2008 Question 8

You are asked to provide assistance to resuscitate a baby. One minute after birth the baby is apnoeic, grey/blue all over, floppy and unresponsive to stimulation, with a pulse felt in the umbilical cord stump 60/min. What is this babyʼs APGAR score? Describe resuscitation of the baby.
APGAR SCORE - out of 10, 0-10 possible, done at 1 minute & 5 minutes after birth:



Management
-General measures
-Warm and dry neonate - has this been done sufficiently?
-Check glucose
-Airway & Breathing
-Commence PPV / Consider ETT
-Correction of hypoxia will increase HR
-ETT - 3.5 mm internal diameter, 8.5-9.0 cm @ the lips (Based on a term
neonate weighing 3.5kg)
-should have an audible leak around it
-size 3 & 4 available if necessary
-IPPV rate @ 30 /min via neopuff device
-initially may require very high inspiratory pressures to expand the lung for the first
time (up to 70 cm H2O) - within minutes, FRC is established
-May require suctioning of the airway under direct vision if meconium liquor
-Circulation
-If not responding to above - Chest compression - rate 90/min (3:1 resp)
-Encircling technique with thumbs on sternum
-IV access - Peripheral or Umbilical vein, intraosseus line
-Resuscitation
-IV Adrenaline 10-30 mcg/kg (ETT adrenaline 30-100 mcg/kg)
-0.4-0.8 mL of 1:10,000 then flush 1 - 4 mL of 1:10:000
-IV Atropine 10 mcg/kg (Tracheal atropine 30 mcg/kg)
-IV crystalloid bolus - 10 mL/kg (35 mL x 2)

If at any point there is spontaneous breathing or HR > 100, and pink, cease APLS and
continue supportive therapy
May 2008 Question 9.

A 25 year old man is to have laser surgery for a vocal cord papilloma. What are the hazards associated with the use of a laser in this situation and how can they be minimized?
LASER:
• “Light Amplification by Stimulated Emission of Radiation”
• Laser is a beam of radiation which is monochromatic, non-divergent and coherent (in phase)
• It is focused onto a small area of very high power density, which may be hazardous

Hazards associated with intra-op laser:
• Risk of airway fire – Due to the presence of fuel (ETT / airway device), combustible gas (Oxygen) and igntion (laser itself)
• Risk of eye damage – patient and theater personnel
• Tissue burns:
o Cutaneous tissue of patient or staff
o Patients normal tissue adjacent to operative field
• Laser Plume:
o Refers to vapour and cellular debris produced during laser use
o Possible detrimental effects on PVR, gas exchange and mucociliary function
o Possible aerosolisation of bacteria and viruses (HIV, HBV, HPV)
• Hypoxia:
o Inadequate ventilation (Jet Ventilation)
o Distal collection of secretions, blood, debris and smoke is a major cause of morbidity and mortality during and after laser surgery
• Gas Embolism - By gas coolant used in Nd:YAG contact probes

Risk Minimisation:
• Signs outside Theater
• All doors should be locked and windows covered
• Protective Eyewear:
o Staff – Safety goggles, which are appropriate for laser in use
o Patient – Eyes taped closed and covered with wet gauze or metal shields
• Special masks – to prevent inhalational of potentially aerosolized viral particles
• Wet gauze / Surgical sponge – applied to patients skin, teeth and normal tissue adjacent to surgical field
• Laser Compatible Airway:
o Eliminating ETT – Jet ventilation or intermittent ventilation
o Modifying standard ETT:
• Wrapping reflective metal tape in a spiral manner around ETT
• Fill cuff with methylene blue tinted water (dye allows early visualization of cuff injury)
o Laser-resistant ETT:
• Made of non-combustible substance (but may still catch fire)
• Eg Laserflex, and LaserSheild tubes
• Minimise concentration of gases supporting combustion:
o Use oxygen/air mixture rather than oxygen/N2O
o Minimum FiO2 compatible with adequate saturation
o Avoid FiO2 > 0.4
• Flame resistant surgical drapes
• Surgical instruments – matte finished to prevent reflection and inadvertent misdirection of laser beam
• Surgical Technique – minimum power of laser, smallest spot size, and shortest exposure time
• Scavenge laser plume as much as possible
• Availability of non-water based fire extinguisher
• Development of Hospital Policy in accordance with Department of Health guidelines on Safe Use of Lasers
• Appointment of Laser Protection Supervisor to ensure compliance with Hospital Policy
May 2008 Question 10.

A patient with an automatic implanted cardiac defibrillator (AICD) with biventricular pacing presents for elective surgery.

Describe how the presence of this device influences your perioperative management of this patient.
BiVentricular Pacemaker:
• Is an essential component of Cardiac Resynchronisation Therapy (CRT), in patients with Atrioventricular Asynchrony, Interventricular Asynchrony and Intraventricular Asynchrony
• These asynchronous conduction lead to hemodynamic compromise, and biventricular pacing restores cardiac cycle efficiency
• Patients requiring CRT suffer from moderate to severe chronic heart failure, and are at significantly increased risk of perioperative morbidity and mortality
o Only surgery that is absolutely necessary should be performed
• Biventricular pacemakers deliver a therapy with each beat – Patients are pacing dependent

Pre-operative:
• History and Examination:
o Signs/symptoms of heart failure – exertional dyspnoea, ankle oedema, PND, orthopnoea, functional status
o Other - chest pain, palpitations
o Medications for heart failure and compliance
o Effect on symptoms post-pacemaker insertion and medical therapy
o When was device last checked?
• Pacemaker device check and reprogramming:
o Identification of pacemaker presence
o Indication for pacemaker
o When was device implanted
o When was device last checked – battery status, reset mode information, and confirmation of satisfactory thresholds, arrhythmia history and defibrillator discharges
o Effect of magnet?
o Disable rate-responsiveness, anti-tachycardia function and defibrillator component
o Maintain biventricular pacing, do NOT change to asynchronous mode
• Investigations:
o 12 Lead ECG – Check underlying rhythm, signs of pacemaker activity, evidence of electrical capture
o CXR – Check anatomical position of generator, lead position and integrity, evidence of heart failure
o U&E – correction of electrolyte abnormalities which may cause loss of capture (hypokalaemia, hypomagnesemia)
o Echo – Assess LV / RV function, valvular abnormalities, regional wall motion abnormalities, pulmonary arterial pressure estimations

Intra-operative:
• No change in anaesthetic technique
• Monitoring:
o ECG – Disable filtering of pacemaker spikes
o Arterial Line – almost mandatory to detect mechanical asystole
• Electrocautery:
o Avoid if possible
o Bipolar preferable
o If monopolar is necessary:
• Safer if vector is perpendicular to pacemaker current
• Keep pacemaker generator out of path of cautery current
• Use in short bursts
• Use lowest possible amplitude current
• Pacemaker Failure:
o External defibrillation pads should be applied prior to induction and defibrillator must be readily available in operating theater
o Chemical pacing – isoprenaline infusion
o External pacing - transthoracic
o Temporary pacing with transvenous wires (Cardiology referral)

Post-operative:
• Full telemetric check
• Re-enabling of anti-tachycardia, rate-responsiveness and defibrillator components
• Telemetry monitoring, perhaps in HDU setting
• Adequate post-op analgesia
May 2008 Question 11.

You are the anaesthetist at a childrens hospital. A three-year-old child scheduled for dental restoration and extractions is found to have a systolic murmur during your preoperative assessment on the day of surgery. They have been on a waiting list for 6 months and have had a dental abscess that settled with antibiotics. Describe how you would evaluate the significance of this murmur and how this evaluation would affect your decision to proceed or not with surgery.
Childhood murmurs are common and can be classified into:
• Innocent murmurs:
o Commonest age is 2-4 years
o Physiological / functional murmurs with no underlying structural heart disease. These are of little consequence and surgery may proceed.
o Eg. Stills murmur (inferior LLSB, systolic ejection), Pulmonary Flow Murmur, Peripheral Pulmonary Stenosis Murmur (ULSB, Supraclavicular murmur, Continuous venous hum (infraclavicular through cardiac cycle, diminishes with JVP compression/head turning), Aortic systolic murmur
• Pathological murmurs:
o Have underlying structural heart disease

Evaluation:
History:
• Antenatal history:
o Maternal Diabetes, Drug / Alcohol exposure, medication exposure are associated with structural heart disease
• Birth history:
o Gestational age at birth, mode of delivery, emergency delivery?
o Cyanosis at birth, fetal distress – structural heart disease more likely
• Developmental history:
o Birth weight
o Failure to thrive? (underlying cardiac failure)
o Feeding history – volume per feeding, length of time per feeding, associated symptoms such as diaphoresis, tachypnoea (likened to exercise tolerance)
o Delayed developmental milestones
• Past History:
o Known cardiac disease?
o Syndromic?
o Other congenital diseases?
o Exercise and play tolerance
o Cyanotic spells

Examination
• General:
o Active? Or irritable, diaphoretic and breathless at rest?
o Dysmorphic features?
o Respiratory distress?
o Height and weight centiles
• Vital signs, esp SaO2
• CVS: Looking for signs of heart failure
o Capillary refill – normal <3s
o Peripheral cyanosis
o Pulse:
• Rate, rhythm, character, radioradial inequality or radiofemoral delay
o Precordium:
• Palpation:
• Presence of heave or thrill suggest pathological murmur
• Auscultation:
• Heart sounds:
o Abnormalities in S2 are associated with pathological murmur
• Systolic clicks / snaps – always of pathological origin
• Murmurs:
o Majority of innocent murmurs are:
• Early systolic
• Grade < 4, no associated thrill
• With normal heart sounds
• Absence of clicks / snaps
o Otherwise high suspicion for pathological murmur

Investigation:
• ECG – if abnormal, likely to be pathological murmur
• CXR – must be normal for a patient to be diagnosed with a functional murmur
• Echo – if results are available, or referral if suspicion of pathological murmur

Cardiology Referral:
• If clinical assessment is not consistent with innocent murmur
• If not confident about clinical assessment
• This patient has presented to a paediatric hospital, so review would be easily and promptly organized
• Cardiology would organize echo

Innocent murmur:
• Child > 1 year
• Past history normal
• Unrestricted exercise tolerance
• Normal growth and weight gain
• Acyanotic appearance with normal SaO2
• Normal pulses
• Murmur – early systolic, Grade < 4, no associated thrill, normal second heart sound and no systolic clicks / snaps
• Normal ECG

If murmur is innocent, surgery may proceed
Surgery:
• The procedure required is elective, but high priority due to a complication already suffered (dental abscess)
• Dental procedures have a high incidence of bacteremia (with subsequent seeding of focus into possible structurally abnormal valve), and therefore an accurate assessment of murmur is required prior to proceeding
• If there is any suggestion of a pathological murmur, an expert cardiology and echo should be sought at earliest available time, and surgery deferred.
May 2008 Question 12.

Outline the issues in the pre-operative assessment specific to a patient presenting for trans-sphenoidal hypophysectomy for acromegaly.
Acromegaly:
• A result of excess Growth Hormone (GH) production by a pituitary adenoma
• Clinical features are a result of mass effect (produced by parasellar extension of anterior pituitary) or peripheral endocrine effects (of excess GH)

Pre-operative assessment:
o A thorough history and examination is essential with particular attention to the following potential issues:

Treatment to date and response:
o Previous surgery
o Radiotherapy
o Bromocriptine therapy
o Therapy to treat associated conditions or complications

Tumour mass effect, producing:
o Visual disturbance:
o Tumour compressing on optic nerve (unilateral visual disturbance or loss) or optic chiasma (bitemporal hemianopia)
o Lateral expansion of tumour may compress abducens nerve (CN VI), resulting in a lateral rectus palsy
o Measurement of visual acuity and fields essential pre-operatively
o Raised ICP:
o Due to expansion of anterior pituitary adenoma
o History of headaches, visual disturbance, vomiting, seizures and reduced conscious state
o A review of MRI Brain to determine extent of spread and potential surgical difficulty (also discuss with surgeon)

Difficult airway:
• Difficult airway is more likely and thorough airway assessment is mandatory, especially review of available previous anaesthetic chart
o Difficult Mask Ventilation – due to distorted facial anatomy, nasal turbinate enlargement, increased force required to maintain mask seal
o Difficult laryngoscopy:
• Osteoarthritis of TMJ and spine
• Increased mandible length may mean long blade insufficient to reach larynx
• Large tongue, lips, epiglottis and hypertrophy of larynx, trachea and airway structures
• Possibility of thyroid enlargement
o Difficult intubation:
• Vocal cord thickening, glottic/subglottic stenosis, narrow cricoids ring
• Large ETTs may be required, rarely upto 10mm

Presence of Associated conditions:
o Obstructive sleep apnoea:
o As a result of factors which create a difficult airway
o History of snoring, daytime hypersomnolence, tiredness and fatiguability
o May require further assessment, HDU/ICU admission
o Cardiovascular disease:
o Ischaemic Heart Disease
o Hypertension
o Cardiomyopathy
o Arrhythmmias
• ECG, CXR, Echocardiogram and other investigations as indicated
o Endocrine disease:
o Impaired Glucose Tolerance or Diabetes Mellitus is common due to excess GH secretion.
o Hypothyroidism and Goitre – Secondary to expanding tumour resulting in reduced production of TSH
o Hypoadrenalism – Secondary to expanding tumour resulting in reduced production of ACTH
• Measurement of TSH/T3/T4, ACTH/Cortisol, Prolactin, FSH/LH, GH levels
• Measurement of Urea and Electrolytes
o Compression syndromes:
o Due to skeletal and connective tissue overgrowth
o May result in peripheral neuropathy or recurrent laryngeal nerve paralysis, further compromising airway patency.
o May result in compression of ulnar artery, producing inadequate flow. This possibility must be considered when placing intra-arterial line
o Myopathy – may occur, and potentially involve respiratory muscles

Referrals:
o Endocrinology consultation and opinion regarding medical optimization pre-operatively

Other:
• Difficulty with monitoring – large hands and feet may present a problem for placing SaO2 probe
• Positioning – Long operating table and beds may be needed
May 2008 Question 13

Evaluate the role of gabapentin in acute and chronic post surgical pain management.
Gabapentin is an acetic acid derivative (gabapentinoid) which is a structural analogue of GABA.

Used in the treatment of
1) Neuropathic pain
2) Painful diabetic neuropathy
3) Acute and chronic post surgical pain
4) post herpetic neuralgia
5) seizures

Mechanism of action – interacts with a unique binding site at voltage-dependent calcium channels and may increase the synaptic release of GABA as well as modulate neuronal transmission at NMDA receptors.

Pharmacokinetics
- only available orally but well absorbed with bioavailability of 60%
- 3% protein bound, Vd 0.85L/kg
- Not metabolised but excreted unchanged in urine, elimination half life 5-7 hours with clearance dependent on creatinine clearance. Need dose reduction in renal impairment.
Pharmacodynamics
- CNS → analgesic and anticonvulsant effects
- SEs → dizziness, ataxia, nystagmus, headache, tremor, diplopia, nausea and vomiting.

Role in acute post-surgical pain
• Most trials involve single preoperative dose
• A number of meta-analyses have shown that perioperative gabapentinoids (gabapentin/pregabalin) reduce post-operative pain and opioid requirements and reduce the incidence of vomiting, pruritus and urinary retention, but increase the incidence of sedation (level 1).
• Improved analgesia and opioid sparing efficacy after various surgical procedures similar to NSAIDs.
• Anti-allodynia and anti-hyperalgesia effects
• Gabapentin reduces movement-evoked pain and this can lead to enhanced functional postoperative recovery.
• Reduced pain and opioid consumption following acute burns.
• Limitations include the fact that few trials have shown reduced opioid related adverse effect, also increased levels of sedation and only available as an oral preparation.
• Unproven whether its use will decrease the incidence of chronic pain following surgery though it is used in at risk groups.
• Incidence and intensity of post-amputation pain was not reduced by gabapentin administered in 1st 30 days post-amputation.


Chronic post surgical pain
• Used in chronic post surgical pain where there is a neuropathic component and in patients at who are at risk of chronic pain.
• In the treatment of chronic neuropathic pain NNT 4.3.
• The safety and efficacy of gabapentin has been demonstrated in neuropathic pain particularly diabetic neuropathy, pain following spinal cord injury and phantom limb pain.
• While it may be used in other chronic post surgical pain states, specific evidence in these areas are lacking.
• Equal efficacy to tricyclics in neuropathic pain but a safer side effect profile.
May 2008 Question 14

Describe the advantages and disadvantages of multi-centre clinical trials in anaesthesia research.
Advantages
• Recruitment of large numbers of patients
• Can undertake studies that are not feasible at a single institution
• Greater statistical power (eg for assessing rare events like death)
• Quicker patient recruitment
• Wider range of patients and clinical settings and better generalizability of results than a single centre study
• New researchers develop skills and beneficial relationships with experienced investigators
• Development of relationships for future trials and research collaborations
• Patients have benefit of closer supervision than in usual standard of care (especially in small centres)

Disadvantages
• Logistics of managing many centres and staff at remote locations (eg main investigator, site investigator, patients)
• Supervision, reliability, honesty and protocol compliance of remote investigators / sites difficult to ensure
• Expensive (need to source large funds usually from competitive grants or industry sponsorship that may bias protocol and outcome)
• Protocol must be applicable and approved at all centres
• Protocol must conform with local standards and practices and ethics
• Logistics of getting protocol through different ethics committees; variable review standards but this is being streamlined through single ethical review (eg NEAF)
• Little ability for an individual site to change or influence protocol
• Studies of procedures depend on level of skill and facilities / equipment at each centre (eg epidural analgesia, HDU availability)
• Some sites may not recruit adequately and not offset set-up costs
• Additional cost of central administration, burden of ensuring data quality, data queries and data cleanup.
• Need to develop method of remote data submission.
• Need 24 hour support for troubleshooting (especially for centres in different time zones)
• Control of data analysis, writing of paper and publication relinquished to another body.
• Data analysis more complicated than single centre study; as it needs to adjust for effect of centres (ie cannot pool all patients as if similar)
• Approval of international studies has possible problems of variable national standards, practices, consent, ethics.
• Local review of multi-centre studies is a burden for local ethics committees
May 2008 Question 15

Outline the problems in providing general anaesthesia for an adult in the MRI suite.
Providing general anaesthesia for an adult in the MRI suite has certain specific problems.

• Remote location of MRI suites with implications such as -
• Foreign and hostile environment, personnel
• Difficulty with assessment of patients, transfer and recovery

• Anaesthesia specific issues with regard to MRI suite including –
• decreased access to patient during scan, decreased ability to assess pt
• decreased access to airway, iv access / accidental disconnections
(MADE)
• Monitoring – Must have in use oxygen/ventilator disconnect alarm, pulse oximetry, capnography, O2 analyser, gas analysis. Must have available ECG, NIBP, NMT, temperature monitor. MRI compatible monitoring eg special ECG and pulse oximetry to avoid burns to patient.
• Assistance - Lack of personnel and need for skilled assistants (need dedicated anaesthetic nurse)
• Drugs – routine drugs and emergency drugs need to be available
• Equipment – anaesthesia machine, suction, gas supply, electrical safety, defibrillator. Need for special non-ferromagnetic equipment (airway equipment and special laryngoscopes, beware batteries), ear protection for patient against noise.
• Plan for management of emergencies

• Magnet specific environment – absolute contraindications include PPM/AICD, intracranial clips, cochlear implants, intraocular metal. Relative contraindications include prosthetic heart valves, non-ferromagnetic implants, tattoos.
• Patient and procedure specific issues – need for GA vs sedation in adult (eg claustrophobia / confused / impaired / unstable). Avoid MRI in the haemodynamically unstable patient.
• Management of positioning and emergency protocols eg quenching the magnet.
• Appropriate recovery area with skilled staff
July 2007 Question 1

Explain the features of the electrical power supply to operating theatres that protects patients from macroshock
• Macroshock –" simultaneous contact between skin and 2 points of unequal electrical potential, and the physiological consequences of this contact"
• Differs from microshock, which is:
1. electrical surrent directly applied to myocardium
2. small contact area, thus higher current density despite low actual current.
3. smaller actual current:
- Microshock Ventricular Fibrillation 0.1 mA
- Threshold of sensation through skin 0.5 mA
- Painful Sensation 1 mA
- Muscle Spasm 10-20 mA
- Macroshock: Ventricular Fibrillation >100 mA
- Muscle Burns >1000 mA
• Patient vulnerability where:
- Alternating current (50 Hz in Australia thus dangerous, most risk 30-100 Hz)
- Active wire shorts to case of equipment
- High current leak to case of equipment
- Very high current drain through several outlets heats the wires starting fire
- Active wire shorts to part of equipment which is not earthed
- areas of electrical protection designated by classes by Australasian standard AS/NZS 3003
Class Z (B)
- unlimited maximum current through patient
- earthed sockets with neutral return wire
- fuse boxes which limit maximum current
- 8A fusewire for lighting circuits, 10A for power
- the earth wire allows a pathway for current to flow should the equipment be shorted to the case → current flows to earth, and if the current is very high the fuse will blow
- even if fuse does not blow the earth will drain most of the current as the resistance is lower than the person touching it, UNLESS, the patient is in contact with water or other grounding equipment

Class B (BF) – 5mA max leak (body protected)
- as for Class A plus earth leakage detection devices e.g. earth leakage core balance (ELCB) or isolating transformers with line isolation monitors (LIM)

- ELCB
- works on principle of differential current in active and neutral wire
- Current through active and neutral wire should always be equal
- Unequal current implies fault (drain), the ELCB is tripped and power is cut; alarm sounds. Power supply will however be interrupted.
- A current difference of 5-10 mA is detected (therefore microshock still possible)
- Will not detect problems where electricity flows through patient and back to neutral wire → very rare

- Isolating transformers and LIM
- Isolating transformer converts mains supply current to a floating supply
- Floating supply is not earth referenced, therefore presence of a circuit through the patient does not allow current to flow
- LIM checks that floating supply is not earth referenced, and indicates on a dial how much current would flow if there as an earth connection (tests with small amount of current)
- If alarms, indicates that a first fault exists, ie. one power line is grounded. If a second ground fault occurs, current may flow.
- Will not cut off current however, therefore is continual supply.
- Only alarms if current >5mA, therefore microshock can still occur




(from Miller 7th ed pg 3045)

Class A (CF) – 0.05 mA max leak (cardiac protected)
- as for Class B but equipotential earthing
- all potential sources of leak current is equipotentially earthed by special low-impedance green cables, including anaesthetic machine, IV poles, IMEDS, etc
- suitable for cardiac surgery
- no exposed leads on intracardiac equipment
- no mains leads adjacent to pacemaker wires (capacitative coupling)

Other generalised measures
- do not leave moisture on floor and bed
- high resistance shoes – rubber
- regular check and maintenance of equipment/sockets
- no exposed leads on intracardiac devices
- no extension cords in theatre
July 2007 Question 2

A 60 year olf man develops a large haemo/pneumothorax following attempted insertion of a haemodialysis catheter via the left subclavian route. Describe you technique of chest tube insertion to drain this and the features of the pleural drainage system you would connect to it.
- Appropriate prior rescuscitation, involving ABC approach with particular attention to circulatory instability in setting of large cavity bleed. Large bore IV access obtained, crystalloid/colloid infusion, bloods including crossmatch
- Adequate discussion of procedure with patient, including informed consent
- Appropriate analgesia as below
- Environment- adequate staff, lighting, drugs, monitoring, resuscitation equipment
- Consider surgical consultation/assistance
- Position- supine, with shoulder abducted, give supplemental O2
- Landmarks – 5th ICS anterior to midaxillary line on correct (ipsilateral) side. Alternatively 2nd ICS midclavicular line, but not as effective for haemopneumothoraces. Correlate with CXR.
- Sterile conditions – gown, glove, mask
- Chlorexidine prep
- 1% lignocaine with 25G needle – infiltrate in the inferior aspect of space (i.e. above 6th rib)
- 2-3 cm horizontal incision
- Blunt dissection with forceps/fingers
- Insert chest tube over superior aspect of rib to avoid neurovascular bundle, with trocar removed with clamp on external end to prevent further pneumothorax. Aim superiorly. Use size 34 F. Fogging inside chest tube is a sign of correct placement.
- Suture chest tube into place at skin. Apply clear occlusive dressings.
- Attach chest tube to UWSD with a minimum of one chamber, preferably three or four chamber. Advantages over one chamber include:
- separating drainage chamber so water height (and hence resistance to drainage) does not change
- allowance for quantification of drainage loss
- ability to apply suction
- atmospheric vent to prevent accumulation of pneumothorax
- Confirm position with CXR
- If central access still required, try for L IJV or either femoral veins
- Renal failure may complicate procedure by:
- presence of coagulopathy
- difficult IV access given fistulae/thrombosed central veins
- haemodynamic instability with fluid shifts
- anaemia secondary to renal failure


Chest drain
- acts as a one-way valve allowing air to exit the pleural cavity but not to enter
- one bottle system
o drain tube extends into collection chamber
o extends 2 cm into depth of fluid
o exit vent allows escape of pleural air from chamber into atmosphere
o keep chamber at least 100 cm below patient à risk of stuff going back into pleural cavity if held above patient
- two bottle system
o 1st chamber is collection receptacle and second chamber is underwater seal
o Better than the one chamber system as in that system, as fluid drained into chamber the height of the fluid overlying the drain would increase, and the resistance would increase
- 3 bottle system
o Allows for suction; a 3rd chamber connected to the 2nd chamber
o Height of fluid in the 3rd chamber determines amount of suction pressure, not the actual suction pressure applied from wall unit
o If suction fails, there is no vent to atmosphere, and further pneumothorax may result


Most commonly use a commercial 3 bottle system (Pneumovac)
July 2007 Question 3

Outline guidelines you think should be in place for reducing both the incidence and the morbidity of epidural space infections as a complication of epidural analgesia.
Reducing Incidence of Epidural Space Infections

- Justify use of epidural anaesthesia given intervention with significant risks (for example MASTER trial would suggest restricting use to particular patient populations eg. respiratory cripple having laparotomy)
- Avoid those in high risk
- Immunocompromised
o Disease process e.g. immunodeficiencies
o Drugs e.g. chemotherapy, immunomodulators, long term steroid use
- Septic patients, localised skin infection
- Poor communication/dementia- cannot report back pain/neurological symptoms
- Sterile insertion
- sterile gown/gloves/mask
- clean environment e.g OR
- sterile equipment and drugs – single use
- chlorhexidine/ETOH prep solution
- Trained personnel managing catheter on the ward, with listing of signs/symptoms of epidural space infection
- sterile techniques when changing solutions
- water proof dressings – clear ones would allow visualisation of insertion site
- regular neuro obs including temperature, muscle power, dermatomes and site
- Specialised pain unit twice daily review of patient when epidural in situ
- Limit catheter duration to 3 days (exponential increase in infection rates after this)- remove earlier if signs of infection/fever

Reducing Morbidity of Epidural Space Infections

- Written and verbal instructions for discharge with contact details for anaesthetic department to patient, including instructions to present immediately to emergency department
- Emergency department protocols for immediate referral of such patients to anaesthesia
- Urgent investigation by MRI when any clinical suspicion.
- The triad of fever, back pain, muscle weakness is rare
- Urgent neurosurgical referral for decompression/drainage
- Empirical IVABs if strong index of suspicion- flucloxacillin/gentamicin. Coverage for MRSA would include vancomycin
July 2007 Question 4

A 40-year old woman presents having been trampled on by a horse. She has a compound fracture of her arm requiring surgery and bruising over the centre of the chest with a fractured sternum.

List the injuries to the heart that may be caused by this blunt trauma.
If she had no signs or symptoms of cardiac injury list and justify any screening investigations for cardiac injury you would perform prior to anaesthesia.
Possible Injuries
Acute:
- myocardial contusion
- cardiac chamber rupture causing septal defects
- valvular damage causing acute incompetence
- pericardial rupture
- coronary artery damage with ischaemia
- cardiac tamponade (either from cardiac rupture or coronary artery damage)
Chronic:
- intracardiac shunts/fistulae
- aneurysm
- pericarditis
- coronary artery thrombosis

Screening Investigations
1. ECG is the first initial test. Quick, non invasive, immediate results. Recommended in asymptomatic patients to guide admission/further investigation. Injury patterns suggestive:
- tachyarrhythmias
- conduction disturbances (1st degree block, new bundle branch blocks)
2. Serum cardiac specific troponins- justifiable in patient going to theatre. Sensitive indicator of significant blunt injuries but may not pick up myocardial contusion.
3. Echocardiography- TOE better than TTE (better view of RV free wall, most commonly injured site). Invasive however. Only if haemodynamic instability or abnormal screening test (ECG/troponins). Also gives functional assessment of wall motion/valves/pericardial abnormalities.
4. CXR- will have been done in trauma patient. Poor test for cardiac problems but may show other injuries or sequelae eg. pulmonary oedema.
July 2007 Question 5

A 50 year old, 110kg builder is on your list for an arthroscopic acromioplasty which is to be performed in the beach chair position.

List the problems associated with this position and describe how you could minimise them.
Positioning - requires co-operation and communication between surgeon, anaesthetist, non-medical staff.

cerebral hypoperfusion
• intra-arterial bp monitoring may be indicated
• position transducer at level of circle of Willis (mastoid)
• avoid hypotension
• maintain CPP to normal awake levels
• Initial volume loading (eg. 500ml gelofusine, 1000ml CSL) and then vasopressor (eg. Metaraminol infusion)
• Gentle induction with gradual achievement of head up position
• if chronic hypertensive then should maintain blood pressure at circle of Willis to within 10% of baseline
• ensure positioning of patient does not compromise carotid flow or obstruct IJV drainage
• positioning as little head up as possible; have legs positioned so there is some hip and knee flexion, with legs as high as feasible; TEDS can help; this will all reduce the reduction in venous return

air embolism
• risk is low - orthopods use lots of irrigation
• can be minimized by surgical technique (avoid opening veins) and keeping minimal height b/w the shoulder and the heart (Does not warrant TOE or PAC), IPPV
• Maintain high central venous filling pressures with the initial volume loading

hypotensive bradycardic events
• atropine is the best treatment (not glycopyrrolate) in the event of significant bradycardia
• treat heart rate < 55 with ephedrine 10mg
• HR < 40 with atropine 600mcg
• more common if regional without GA as anaesthetic technique

lack of access to the head and airway
• intubate patient and secure the ETT well with tape; IPPV will reduce entrainment of gas
• reduce risk of disconnections by checking all connectors and firmly in
• have everything well secured and optimally positioned with ventilator tubing taped to the patient
• have usual disconnect alarms
• 2nd IV readily accessible

nerve injuries
• risk is from pressure or stretch - have patient optimally positioned / padded under pressure areas
• Elbow - protect ulnar nerve at the elbow
• Knee - protect sciatic nerve by having some knee flexion (pillows underknee)
• avoid brachial plexus stretch
• small risk of cervical spinal cord hypoperfusion via positioning of neck
- occipital/auricular nerve compression have been reported - ensure neck not excessively flexed/rotated
- sacral area should be attended to as is an area of little padding

post-op visual loss
• has been described probably via hypoperfusion - ensure eyes padded

ETT/IPPV is helpful
• Guarantee ETCO2 - cerebrovascular effects
• Minimize entrainment in head up position
• 110 kg man
• Remote airway
July 2007 Question 6

Define circulatory shock. Categorise the causes of circulatory shock and give an example in each category.
Definition of circulatory shock
• A fall in tissue perfusion or oxygen delivery so severe that perfusion and oxygen delivery to vital organs is inadequate to meet the body’s metabolic needs
• results in hypoperfusion, lactic acidosis, tissue hypoxia, mult-organ failure, death etc
• typically a downward spiral due to positive feedback / gain

Causes of circulatory shock
Types of shock
- classically divided according to aetiology; share common features
- Cardiogenic shock
o Failure of myocardium as a pump
o Commonly due to MI → death/stunning of myocardium causes decrease in contractility


- Hypovolaemic shock
o Inadequate venous return and preload → decreases stroke volume as per Frank-Starling curve
o Most common cause is haemorrhage

- Distributive shock – relative hypovolaemia
o Septic shock
• Microbial components release toxins and inflammatory mediators (TNF, IL1, IL6) which cause increase capillary permeability and vasodilatation and reduced intravascular volume
• Cardiac output often increases to supply more O2 to tissues, but as O2 demand increases> supply, lactate acidosis ensues
• May have deficiency in micro-perfusion

o Anaphylactic shock
o Neurogenic shock
• Decrease/absent sympathetic output to smooth muscles in vessels

- Obstructive shock
o Obstruction to blood flow
o Large PE, AS, air embolus, fat embolus

- Endocrine/metabolic shock
o Addisonian crisis
July 2007 Question 7

A 25 year old, 65kg woman with acute severe asthma requires intubation and ventilation. Explain the problems associated with initiating ventilatory support in this patient and describe how you would overcome them.
1) - Distressed patient, difficult to consent

Enlist help of family members, calm manner, full explanation with much reassurance to patient.

2) - Emergency life threatening situation in unfamilar environment

Summon help, declare emergency, prepare enviroment with staff and equipment
- 100% O2 with BMV, sucker, intubating equipment and monitoring
- Drugs
- Salbutamol 250 mcg IV/2.5 mg neb PRN (MDI x 10 via ETT attachment)
- aminophylline 250 mg
- Magnesium 2 g
- Hydrocortisone 200 mg IV
- Preparing the environment well
- appropriate staff & equipment


3) Potentially full stomach
- Pros and Cons of RSI / gas induction
- Attenuating intubation
- Needs to be deep prior to intubation, alfentanil opioid useful (esp with ++ beta ag)

4) Respiratory failure
- Hypoxic, hypercarbic, acidotic


How to set up ventilator
- TV 6 -8 ml/kg
- Low RR 6-8 /min
- I:E ratio 1:3
- Keep Pplat <30 cmH20 or auto PEEP <15 cmH20 (ideally <10 cmH20)

Ventilation goals:
- Pa02 > 60mmHg OR Sp02 92-95%
- Minimize DHI
- Permissive hypercapnia
o deep sedation and NMB
o pH > 7.1-7.2 acceptable
- May need to disconnect tube and squeeze chest to resusicitate

5) Associated side effects of
- b agonists - hypokalaemia
- long term steroid use - cushingoid, diabetes, HTN etc

- Appropriate fluid & inotropic management of cardiovascular problems
- Arterial line
- K+ supplementation required
- ECG monitoring

6) Bronchospasm worsening on induction

7) Ventilation difficulty
- high airway pressure, barotrauma, autoPEEP, peumothoraces
- Dynamic airway compression and associated cardiac arrest with initiation of IPPV

7) Induction drugs

- Appropriate on-going treatment of asthma
- Preventer, Specialist referral, further mx
- Appropriate selection of anaesthetic drugs.
- Avoid atracurium
- Sevo and ketamine are good
- Propofol > STP
July 2007 Question 8

You are asked to provide epidural pain relief for a woman in labour. She is having primigravida, and is 3cm dilated.
Describe and justify both you choice of drugs for and the mode of administration of epidural analgesia in this situation.
Focused history and examination
- Parity / Gestation
- Health in pregnancy
- Allergies / Medications
- Previous anaesthetic experience
- Airway assessment
- Consent / discussion of what epidural analgesia means
- Perform epidural insertion with local anaesthetic to skin/subcut / full sterile precautions
- 18 g tuohy loss of resistance to saline technique and feed an appropriately sized catheter
(decreased risk of PDPH cf 16g)
- Chlorehexidine impregnated dressings (?prep) better at preventing colonisation cf: placebo or poviodine (Level 1)

Indications
- Maternal request
- Obstetric indication
- CVA / PET / CVS disease / Obesity
- Better outcomes are associated with epidural analgesia in these groups

Drugs
- Test dose of local anaesthetic 3-4 mL 0.2% ropivicaine with fentanyl 0.5-1 mcg/kg
• Loading dose of 5ml + 5ml + 5 ml + 5 mL 0.2% ropivicaine
• NIBP needed after each dose of epidural

CSE
o faster onset of analgesia compared with epidural analgesia.
o did NOT improve satisfaction or mobilisation, and increased the risk of pruritus.

(1) Justification based on:
• Effect:
i. onset, duration of ropivicaine favourable
ii. new evidence that PCEA + background results in better maternal satisfaction and reduced clinician interventions than PCEA alone (Level 1 - p194 pain book)
iii. clonidine in epidural prolongs block BUT does not improve it
iv. opioid reduces overall LA dose requirement
• Toxicity
i. Ie high concentration lignocaine causing local toxicity
ii. CC:CNS ratios - worse with bupivicaine in partuents
iii. High dose LA associated with more urinary retention cf: CSE
• Patient acceptance
i. Motor block found to be less with ropivicaine in one study
ii. levobupivicaine associated with less satisifaction and worse pain scores
iii. PCEA - cultural, language limitations
• Staff acceptance
i. PCEA / Bolus / Infusion

(2) Monitoring
• Motor block / Sensory block to ice - ? evidence for monitoring.
July 2007 Question 9

"It is no longer justifiable to use aprotinin during cardiac surgical procedures." Disucss
Aprotinin is a serine protease inhibitor which was previously used during cardiac surgical procedures to limit blood loss as it functions as an anti-fibrinolytic agent. It inhibits kallikrein leading to the inhibition of the formation of factor XIIa (inhibiting the intrinsic pathway and fibrinolysis) and it also inhibits plasmin (slowing fibrinolysis).

The advantages of aprotinin includes its proven efficacy in reducing blood loss and need for blood transfusion in patients undergoing cardiac surgery and cardiopulmonary bypass. The Cochrane Review by Henry DA et al in 2001 (211 RCTs) suggested an advantage of aprotinin over transexamic acid and aminocaproic acid in terms of operative blood loss but the differences were small and may be explained by publication bias. Aprotinin did not increase the risk of myocardial infarction, stroke, renal dysfunction or overall mortality.


In the majority of these studies there was no evidence of significant side effects, in some a suggestion of reduced incidence of stroke.

Aprotinin was withdrawn primary due to Mangano’s study in 2006 ‘The risk associated with aprotinin in cardiac surgery’ NEJM.

Mangano et al (NEJM 2006)
- observational study in 4374 patients undergoind revascularization
- aprotinin vs aminocaproic acid vs tranexamic acid vs no agent
- all equally efficacious in reducing blood loss and transfusion
- aprotinin a/w 2x risk of renal failure requiring dialysis, 55% increase in risk of MI/CCF (p<0.001), 181% increase risk of CVA
- Neither ACA or TXA was associated with an increased risk of renal, cardiac or cerebral events
- however, this was an observational study, with its inherent flaws in methodology
- might have had a lot of confounders
Further follow-up studies have also found similar adverse effects of aprotinin.

BART study (NEJM 2008)
- 3000 patients RCT in Canada, multicentre blinded trial
- Aprotinin vs aminocaproic acid vs tranexamic acid
- Stopped prematurely → 30 day mortality in aprotinin group almost reached statistical significance at interim analysis
- Modest reduction in risk of massive bleeding but strong and consistent negative mortality trend in those receiving aprotinin compared to ACA and TXA precludes its use in high-risk cardiac surgery.
- In Australia, aprotinin withdrawn from market permanently

It is currently no longer justifiable to use aprotinin in high risk cardiac surgery.
July 2007 Question 10

Describe the blood supply of the spinal cord.

Explain the determinants of spinal cord perfusion.
Blood supply to the spinal cord

(1) Anterior spinal artery
• Formed by union of vertebral arteries at foramen magnum
• Runs on anterior median fissure, midline
• Supplies larger part of anterior spinal cord (supplies whole cord anterior to the posterior grey columns)
(2) Posterior spinal artery
• One or two on each side
• Formed from posterior cerebellar arteries
• Smaller in size
• Scanty connexions with the anterior spinal artery
• Reinforced by spinal branches from number of nearby vessels
• Supplies posterior portion of spinal cord (supplies the grey and white posterior columns of its own side)
(3) Radicular arteries
• Provide blood supply to BOTH anterior and posterior spinal arteries
• Arise from regions in
i. Cervical
ii. Thoracic
iii. Lumbar
• Usually numbers between 3 and 6 larger vessels
• One vessel is particularly large - Arteria radicularis magna “Artery of Adamkiewicz” around T10/T11 roots on left
• May provide dominant supply to lower 2/3 spinal cord

Arterial blood supply of the spinal cord is vulnerable
Does not have direct anastomoses and cord infarction is possible after thrombosis, hypotension, surgical occlusion, trauma and vasoconstriction

Veins anastomose freely, spinal veins form loose-knit plexuses anteriorly and posteriorly.

Determinants of spinal cord perfusion
Cord Perfusion pressure = MAP(at spinal arteries) – venous pressure in spinal veins/CSF pressure whichever is greater

Determinants of perfusion pressure
• MAP = CO X SVR
• Autoregulation
o Between MAP 50-150mmHg, may be abolished in pathological conditions eg cord odema or compression.
• Spinal venous pressure (obstruction of venous return)
o Raised intra-abdominal pressure in prone position
• Raised CSF pressures (factors influencing absorption and production, mannitol and frusemide will decrease CSF pressure) Aortic cross clamp may increase CSF pressure (drain may be inserted to improve perfusion)
• Vasodilates in presence of
o Hypoxaemia
o Hypercapnea
• Extrinsic compression of aorta
o Tumour
o Cross clamping of aorta (compromised flow to radicular arteries)
• Hypothermia
o Reduces O2 consumption and spinal cord metabolism, improves outcomes during cross clamp.
• Anatomical variation
o Lack of true anaestomoses between anterior and posterior spinal arterial systems
• Thiopentone
o Reduces O2 consumption, preserves spinal cord
• Vasoconstrictors
o Increases SVR
o Localised vasoconstrictors eg epidural adrenaline
• Vasodilators
• Localised embolus eg ant spinal artery syndrome
July 2007 Question 11
An 18 year old otherwise healthy female is to have 2 impacted wisdom teeth surgically removed as a day stay patient.

Describe and justify features of your anaesthetic technique that may help prevent the common post operative problems you would anticipate in this patient.
The common post-operative problems that may be encountered in this patient are pain, post-operative nausea and vomiting and sedation. Less common problems include dehydration, hypovolaemia and bleeding.

Features of my anaesthetic technique which would aim to prevent these problems include:
Midazolam premed in preop bay if very anxious (helps in the prevention of PONV but may delay discharge due to sedation)
IV induction technique with propofol and short acting opioids (alfentanil)
- minimise post-op sedation, respiratory depression
- propofol has antiemetic effect as well
IV fluids (replacement of fasting losses and minimises PONV)
Prophylactic antiemetics – dual agents as she is a high risk patient having high risk surgery – dexamethasone and a 5HT3 antagonist eg. granisetron

Adjuvant analgesia
- minimise opioids which will cause sedation and nausea
- NSAIDs/parecoxib, paracetamol, consider tramadol
- LA infiltration improves post-operative pain

LMA, spontaneous ventilation techniques
- avoids muscle relaxants
- avoids residual paralysis as case is likely to be short
- appropriate in fasted patients with no contraindications eg reflux
- throat pack to reduce ingested blood which can exacerbate nausea (need to ensure throat pack removed and accounted for at end of case)
- appropriate with a skilled surgeon as likely to be minimal blood loss
- the use of a reinforced LMA allows greater surgical access
Maintenance with sevoflurane – shorter acting than isoflurane and more titratable

If patient is agreeable, can consider LA with sedation – combination of propofol and alfentanil; supplementary O2 via prongs. This would minimise the risk of sedation with a GA delaying discharge.

Need to ensure discharge criteria are met in the Day Surgery Centre according to the ANZCA College guidelines prior to discharge home. This includes the patient being appropriate, alert, stable vital signs, adequate pain control and minimal nausea and vomiting, able to tolerate oral intake. There must also be suitable analgesia given and minimal bleeding. The patient must be accompanied home by a responsible adult with overnight care as well as instructions on when to return.
July 2007 Question 12

A 3 week old male infant who was born by uncomplicated vaginal delivery at term presents with projectile vomiting for 2 weeks. His weight is now 2.8kg from a birth weight of 3.1kg. His presumed diagnosis is pyloric stenosis. His blood chemistry results are :

Na 129
K 3
Cl 84
HCO3 36
Creatinine 69
Glucose 3

Explain how thse abnormal results come about.

Describe an appropriate fluid resuscitation regime for this infant.

List the laboratory criteria by which you would consider him sufficiently resusciated for surgery.
Pyloric stenosis is characterised by a hypochloraemic metabolic alkalosis due to persisting vomiting.

Pathophysiology of abnormal results
• Loss of water and ions
• Loss of H+ and Cl- but also Na and some K from stomach
• Loss of H+ and decrease in pancreatic HCO3 secretion results in rise in serum HCO3
• Metabolic alkalosis results
• Increased HCO3 load to distal tubule of kidney results in an alkaline urine initially and loss of Na and water in the urine until the kidney adjusts to conserve HCO3 more efficiently
• Dehydration and stress activate the rennin-angiotensin-aldosterone system resulting in attempted Na conservation but significant K loss from urine
• MAIN source of K loss FROM BODY IS FROM THE KIDNEY → resulting hypokalaemia
• ADH secretion and water retention along with Na loss from stomach and Na loss in kidney linked to HCO3 contributes to hyponatraemia
• Extreme K losses, H+ ions exchanged for K in kidney → worsening alkalosis and paradoxical acidotic urine

Fluid resuscitation for infant
• Maintenance fluids 1st 48 hours life = 75mL/kg/day = 3mLs/kg/hr
• 2 days to 1 month = 150mLs/kg/day = 6mLs/kg/hr
• 1 month onward = 4mLS/kg/day (up to 10kg)

Na requirements
• 3-5mEq/kg/day
K requirements
• 2-3mEq/kg/day
Chloride
• 1-3mEq/kg/day

Resuscitation is based on weight loss and clinical signs
• skin turgor
• dry mucous membranes
• sunken fontanelle
• decreased consciousness (signs of severe dehydration)
• urine output
• pulse
• BP

In this case, patient is 300mLs deficit (10% of his weight)
• This is moderate fluid loss (severe is 15-20% fluid loss)
• Blood glucose of 3.0 indicative of hypoglycaemia
• Give 10mLs/kg x 2 bolus of NaCl (or 20mls/kg) or use colloid/albumin
• Give over several hours, repeat if necessary until improved capillary return, skin turgor
• Thereafter the deficit 300mLs needs to be replaced
• Maintenance of 12mLs/hr in addition, monitor and account for nasogastric losses
• 300mLs deficit can be replaced over 24 hours
o 150mLs in first 8 hours
o then 150mLs in next 16 hours
o this gives 19mLs/hr first 8 hours
o then 9mLs/hr for 2nd 16 hours
• appropriate fluid is 0.45% NaCl and +5% dextrose (Na 77mEq, Cl 77mEq, osmolarity 406 mOsmol/L in 1000mls, hypertonic)
• add 20mmol of K to each 1000mL bag once renal function is established (ie 6mmol K per day if 300mls fluid given per day)
• repeat bloods to monitor K and Cl and HCO3


Lab criteria for adequate resuscitation
• Cl > 105mmol/L (serum Cl needs to be >105mmol/L for the vast majority of infants to have no residual alkalosis)
• K > 3.5mmol/L
• HCO3 in normal range 18-24mmol/L
• Glucose 3.5-6.0mmol/L
• Na will tend to be normalized anyway
• Important to correct metabolic alkalosis or there is a risk of postoperative respiratory depression due to compensatory respiratory acidosis
• Use weight as a marker also
• Ideally all lab markers should be normal before proceeding but important to normalise Cl and HCO3 and correct alkalosis
• Urine Cl > 20mmol/L
July 2007 Question 13

Describe a technique of neural blockade in the popliteal fossa for surgery on the foot and ankle including a description of the relevant anatomy.
Anatomy:
• The Sciatic Nerve:
o Formed from anterior and posterior divisions of ventral nerve roots of L4-S3
o Courses down the posterior aspect of thigh along posteromedial aspect of femur
o At apex of popliteal fossa it divides into Tibial and Common Peroneal nerves
o Supplies all parts of lower leg except supply over medial aspect of tibia / medial malleolus / Great toe (Saphenous nerve)
o Effective nerve blockade for foot / ankle surgery requires blockade of Saphenous nerve also
• Popliteal fossa:
o The sciatic nerve is superficial (~2cm from skin) at popliteal fossa
o Sciatic nerve is:
• Superficial to Popliteal Artery and Veins
• Medial to Biceps Femoris
• Lateral to Semimembranosus and Semitendonosus
Nerve Block Technique: Ultrasound Guided
• Preparation - Assessment, consent, monitoring, IV access
• The patient is placed in the lateral position with the limb to be blocked uppermost.
• Premedication and Oxygen (if required)
• Aseptic technique (Chlorhex/ETOH wash, Sterile gloves, Drapes, Sterile Ultrasound probe cover)
• Ultrasound:
o Linear probe
o 5 MHz
o Depth setting - 4cm
• Probe is placed with beam transverse to limb at popliteal fossa
• The popliteal artery and vein (hypoechoic) are identified in short axis and confirmed with Doppler
• The sciatic nerve (hyperechoic) is found superficial to popliteal vein and towards the lateral side
• The nerve is tracked proximally and distally to identify the point of division into Tibial and Common Peroneal components
• At the level where the nerves are together:
o Use 10cm 22-Gauge Stimiplex needle
o Perpendicular approach by introducing needle from the lateral side 4cm from the ultrasound probe and in line with beam (local anaesthetic skin infiltration prior to insertion of Stimiplex needle)
o The needle is advanced to position tip both superficial and deep to nerve to surround it with a total of 25-30ml of 0.75% Ropivacaine (Max dose 3mg/kg)
o Safety - Aspiration, incremental injection, observe ECG for changes
o The local anaesthetic will appear hypoechoic
• Onset - 30 mins, Duration ~ 18 hours
• Complications:
o Haematoma
o Local infection
o Local anaesthetic toxicity
o Failure
o Intravascular injection
o Intraneural injection
July 2007 Question 14

A Clinical Trial is planned to evaluate a new analgesic. Discuss the ethical considerations in having a placebo group in the trial.
• Placebo - "A treatment thought not to have a specific effect"
• In clinical trials, the 'placebo group' is allocated the 'non treatment' arm of the study.
• It may involve the use of inert substances, subtherapeutic doses of active substances or sham procedures.
o Relief, as well as adverse effects, have been reported in patients with objective evidence of tissue damage (post-op) to placebo.
o The results of a placebo group may be invalid in light of above findings.
• To evaluate a new analgesic, the sample groups of the study must be suffering from pain.
• The inadequate treatment of pain using an inferior therapeutic modality, such as placebos, is unethical as analgesia is a basic patient right.
• The Declaration of Helsinki by WHO in 2000, states that patients enrolled in trials should not be subject to placebo, if proven effective treatments already exist.
o The sole use of a placebo in a trial group may expose the subjects to undue suffering, however the use of placebo in a multimodal regimen may be somewhat ethically justifiable.
• The central ethical tenets in health care include autonomy, beneficence, non-maleficence, and justice
• Autonomy:
o Patients have a right to consent to or refuse a treatment offered to them.
o Deception of patients about clinical treatments, by using placebo, violates this right
o Patients must be fully informed about the trial treatments and must consent to participate.
• Beneficence:
o Is the act of doing good
o Subjecting a trial group to placebo and not providing a beneficial treatment undermines this principle
o Clinicians have an obligation to offer effective treatment for pain
• Non-maleficence:
o Means to do no harm
o The placebo group, by not being able to access analgesia, suffer from ongoing harm
o Clinicians should refrain from providing ineffective treatments
• Justice:
o Refers to fair and equal treatment for all with similar conditions
o In a clinical trial where two treatments are being compared in similar patients groups, this is not possible
o If the beneficial effects of a new treatment are thought to be similar to a proven treatment, then it may be considered
• The use of a placebo does not constitute justice between treatment groups
July 2007 Question 15

Draw Flow-Volume Loops associated with:
• Fixed Upper Airway Obstruction
• Variable Extrathoracic Airway Obstruction
• Variable Intrathoracic Airway Obstruction

Explain briefly the physiological reasons for the shape of these loops.
Normal Flow-volume Loop and Basic Principles:

• Airways beyond Generation 11 have no structural rigidity and rely on traction on their walls from elastic recoil of lung tissue to keep them open
o Therefore, intrathoracic airways have a tendency to collapse on expiration
• Larger airways are supported with cartilage in their walls to keep them open
o Therefore, extrathoracic airways have a tendency to collapse on inspiration
• During a maximal forced expiration, intrathoracic pressure rises may rise above the pressure within the airway (equal pressure point)
o Downstream of the equal pressure point, airway closure results.
o The equal pressure point is also influenced by lung volume at start of expiration
• To an extent, the greater the effort, the greater the peak expiratory flow rate (Effort-dependent portion)
• However, all expiratory curves terminate in a final common pathway
o This flow rate is limited by airway collapse, and the maximal air flow rate is governed by lung volume (Effort-independent portion)
o Flow rates are therefore more accurately quantified at a particular lung volume:
• FEF50 = Forced Expiratory Flow rate at 50% of FVC
• FIF50 = Forced Inspiratory Flow rate at 50% of FVC
• FEF50/FIF50 = The ratio of Forced Expiratory to Forced Inspiratory Flow at 50% of FVC
• Airway obstruction can be diagnosed by either inspecting flow-volume loops or by calculation of various indices, incl FEF50/FIF50
o FEF50/FIF50 = 1, in Fixed airway obstruction
o FEF50/FIF50 > 1, in Variable Extrathoracic airway obstruction
o FEF50/FIF50 <1, in Variable Intrathoracic airway obstruction
Fixed Upper Airway Obstruction (Diagram Below)
• The airway diameter is fixed and reduced by the pathology, which then limits maximal inspiratory and expiratory flow rates (rather than effort, airway closure or lung volume)
• This is shown by a plateau during inspiration and expiration
• Therefore, FEF50/FIF50 = 1
Variable Extrathoracic Airway obstruction (Diagram Below)
• Generation of a negative intrathoracic pressure during forced inspiration leads to airway closure with increasing effort (Structural rigidity vs negative intraluminal pressure), therefore reducing inspiratory flow
• The inspiratory phase on flow-volume loop is flattened
• During expiration the positive pressure within the airways tends to decrease the obstruction and expiratory flow is less reduced, and may even be normal
• Therefore, FEF50/FIF50 > 1

Variable Intrathoracic Airway obstruction (Diagram Below)
• Generation of a positive intrathoracic pressure during forced expiration leads to external pressure on airways, hence decreasing airway diameter.
• The equal pressure point moves proximally and results in airway closure, limiting expiratory flow rates
• The expiration phase on flow-volume loop is flattened
• Therefore, FEF50/FIF50 < 1
May 2007 Question 1

What are the principles of ventilatory management of patients with ARDS?
ARDS
- “syndrome of increased pulmonary capillary permeability and inflammation”
- Diagnostic criteria
o PaO2/FiO2 ratio < 200
o bilateral infiltrates on CXR
o PCWP <18 mmHg or no evidence of increased LAP

Principles/priority of ventilation
- Maintain adequate oxygenation
o >= SaO2 90% or PaO2 60 mmHg

- Minimise ventilator trauma
o Volutrauma
• areas of collapsed alveoli interspersed with normal ones – high volume will overdistend the normal ones
o Atelecto-trauma
• repeated closing and opening of airspaces by tidal recruitment results in diffuse alveolar damage
o Barotrauma
• high plateau pressures will cause pneumothoracies
o Biotrauma
• alveolar inflammation with high levels of alveolar cytokines spilling into systemic circulation


ARDS Network study
- 6 ml/kg better survival c/f 12 ml/kg tidal volume
- Plateau insp pressure <30 cmH20
- Oxygenation maintain SpO2 >88%
o FiO2 0.3 - 1.0
o PEEP 5 - 24 mmHg
- I:E ratios between 1:1 and 1:3
- RR up to 35 BPM to minimise hypercapnia
- Bicarbonate to maintain pH >7.3

Non invasive ventilation
- theoretically would decrease VILI, but not proven clinically
- can be considered, but most patients would progress to IV anyway

PEEP & Open lung approach PCV
- pressure-targeted ventilation strategies with PEEP to maintain
- maintain above lower inflection point on pressure-volume curve
- 15 cmH20 is reasonable target
- decreases atelectrauma from repeated opening and closing

Permissive hypercapnia
- any attempts at reducing hypercapnia will increase VILI
- if occurs slowly, intracellular acidosis will compensate anyway
- may actually augment CO and BP; will worsen pulmonary hypertension though
- will require deep sedation to inhibit spont resp

FiO2
- start with 1.0, but decrease as able <0.6 as O2 toxicity may cause diffuse alveolar damage

Recruitment manoeuvres
- high level CPAP 30 cmH2O applied for 30-40s in an apnoeic patient followed by return for baseline PEEP
- unclear whether there is additional benefit
- may result in transient improvements in oxygenation

Other methods
- prone
o 70% will result in significant increase in PaO2 from recruitment of dorsal lung and better V/Q matching
o No improvement in mortality
o Temporising only
o Risk of extubation, decannulation of lines, haemodynamic instability

- NO/prostacyclin
o Improved oxygenation but not survival

- HFV
o Small tidal volumes (3 ml/kg) with rapid RR 100 bpm
o Theoretically will decrease VILI, but again survival figures do not reflect this

- ECMO
o Only when the cause is reversible
o Significant cost and risks to patient
May 2007 Question 2

Describe the relevant anatomy and technique for field block for inguinal hernia repair.
Nerves which need to be blocked:

Iliohypogastric
- L1
- Lies between transverus abdominus (TA) and internal oblique (IO) but pierces IO just medial to ASIS
- May be accompanied by subcostal (T12) nerve
- Supplies suprapubic skin


Iliohypogastric
- L1
- Lies between transverus abdominus (TA) and internal oblique (IO) but pierces IO just medial to ASIS
- May be accompanied by subcostal (T12) nerve
- Supplies suprapubic skin

Ilioinguinal
- L1
- Lies between TA and IO but pierces IO more medial to ASIS; traverses inguinal canal in front of spermatic cord
- Supply scrotum/upper thigh

Genitofemoral nerve (genital branch)
- L1/L2
- lateral to pubic tubercle below the inguinal ligament; better blocked by surgeons at the ext inguinal ring

Technique
- Need up to 40 ml of solution, therefore calculate maximum dose of LA and dilute to 40 ml (0.5% ropivacaine is alright)
- 22G short bevel needle
- Asepsis
- 2 cm medial and inferior to ASIS (same position as for a LFCN of thigh block)
- Aim cephalolateral
o 1st pop (EO) inject 7 ml solution to block iliohypogastric
o 2nd pop (IO) inject 7 ml solution to block ilioinguinal
o Subcutaneous infiltration fan-wise to block subcostal nerve 3-4 mL (cephalad)
- Contralateral overlap innervation at midline (3-4 ml)
- Another 10 ml LA for genitofemoral nerve
- Allow another 10 ml of LA for top up infiltration during surgery

Complications
- femoral nerve block – leg weakness
- failure
- bowel perforation
- local haematoma – interfere with surgical view
- Peripheral nerve damage
- Systemic local anaesthetic toxicity
May 2007 Question 3

How does soda lime work? List the hazards associated with its use.
Primary function
- remove CO2 in circle systems.
- Placed AFTER APL valve.
- Small granules size to increase surface area for reactions while keeping resistance low

Soda lime consists of
- NaOH, Ca(OH)2, ethyl violet indicator held in canister
- Ethyl violet indicator turns purple colour when exhausted

Equations:
CO2 + H2O → H2CO3

H2CO3 + 2NaOH → Na2CO3 + H2O + heat

Na2CO3 + Ca(OH)2 → CaCO3 + 2NaOH (slow reaction)

Hazards of use:

- Compound A & Sevoflurane
o nephrotoxic in rats with concentrations of 50 – 100 ppm
o compound A metabolized via beta-lyase pathway to form reactive thiols
o during closed circuit anaesthesia with sevoflurane in humans > 5 hours, the average concentration of Compound A was found to be <20 ppm and there was no evidence of renal failure based on serum urea and creatinine
o reaction is temperature dependent
o worsened by:
• Baralyme (higher temperature), low flows (recommended at least 2L/min) & increased minute ventilation and CO2 production (higher temperature)
o drying out of soda lime less Compound A produced (but more in Baralyme)

- Carbon Monoxide & other fluorinated ethers
o CHF2 group (iso, des) degraded by strong bases (NaOH/KOH) to CO
o as high as 30% carboxyhaemoglobin attributed to this effect
o Exacerbated by
• dryness (prolonged high flows - ie 1st case monday if left on all weekend)high temperatures & low flows, high production of CO2.
• baralyme > soda lime
o desflurane > enflurane > isoflurane
o anaemic, paediatric patients more vulnerable
o Diagnosed by moderately dec oximeter reading & wrong gas analyzer reading
• trifluoromethane also released during reaction with absorber

- others:
o absorption of volatile agent
• reduce delivery of agent delaying induction
• may be retained and released to subsequent patients (*MH susceptible)
o sevoflurane can react with dry Baralyme to produce CO → fire
o trichloroethylene in the presence of soda lime and heat
• phosgene (pulmonary toxin) & dichloroacetylene (neurotoxin)
o Halothane
• difluorovinyl compounds from degradation – less toxic c/f Compound A


- CO2 absorber increases resistance of circuit
- Exhaustion/circuit disconnection causes increase in FiCO2
o Channelling may escape detection by clinician (ie hidden path of least resistance)
- Heat production --> fire risk
- Dust inhalation
- skin irritation to staff
May 2007 Question 4

A 2 year old child has burns to lower body from immersion into a hot bath.

Describe your assessment and management of pain and fluid requirements in the first 2 hours following injury.
Estimate depth and area of burn
- each leg is 14%, front 18%, back 18%
o 30-55% TSA from information given - (i’ll use 40% for this answer)

Fluid Management
- Parkland formula: Bolus 4 ml/kg/% TBA – Hartmann’s solution
o ½ in first 8 hours after burn
o ½ in the following 16 hours
o 4 x 13 x 40 = ~ 2000 mL (1 L in 8/24 then 1L over 16/24)
- Maintenance concurrently - 4-2-1 formula – 5% Dextrose & 0.9% NS
o Expected weight – (2X age) + 9 = 13 kg
o 40 + 6 = 46 ml/hour
- Adjust maintenance with resultant urine output. if UO < 1 ml/kg/hr
o bolus 10 ml/kg of Hartmann’s
• 130 mL
o maintenance increased to 150% of current rate
• 69 mL/hr
- If there is myoglobin and haemoglobin in urine
o aim for 2 ml/kg/hr urine output
o alkanlinise urine by adding 25 mmol bicarbonate to each litre of Hartmann’s
- Check Hb → may need transfusion of packed RBC to maintain Hb> 80g/L


Analgesia
- burns are exquisitely painful
- Assess/estimate pain
o if preverbal use scale e.g. FLACC score – face, legs, activity, cry, consolability
- multimodal analgesia
o paracetamol
• 15-30 mg/kg then 15 mg/kg QID
o X NSAIDs
• potential renal failure
o opioids with caution
• as child may be haemodynamically unstable/shocked
• bolus morphine 50-100 mcg/kg, then IVI 10-20 mcg/kg/hr
• ie 0.6 - 1.2 mg bolus followed by 0.13-0.26 mg/hr
• use fentanyl if renal impairment
o ketamine
• if severe burns, ketamine would be useful
• less resp depression
• more haemodynamically stablity

o cautious sedation
• clonidine – 1 mcg/kg will also aid in analgesia

o BURNS REQUIRE LOTS OF BLOOD IN THEATRE
May 2007 Question 5

Discuss the usefulness of the continuous measurement of mixed venous oxygen saturation in the intensive care patient.
Mixed venous oxygen saturation (SvO2) is the saturation of Hb in the blood found in the proximal pulmonary artery just before it reaches the alveoli. It represents the average of deoxygenated blood, after mixing the blood from SVC and IVC.
Rearrangement of the Fick equation illustrates that venous oxygen content is determined by
- arterial oxygen content
- cardiac output
- Hb concentration
- Oxygen consumption (O2 extraction/tissue metabolism)

CO = V02
CaO2 –CvO2

Mixed venous oxygen saturations reflect the difference between global oxygen delivery and global oxygen consumption.

Measurement of venous saturation may be performed either intermittently by blood sampling and cooximetry or continuously through the use of a spectrophotometric catheter (specialised fibreoptic pulmonary artery catheter). Normal values are between 60-80% (75%). (NB. Central venous saturations are usually 2-5% less than mixed venous saturations in healthy individuals.)

Advantages
- continuous reading – able to detect changes early, easier to see changes with therapy
- Derangements in Svo2 occur before any changes in mean arterial pressure or heart rate are observed and they appear to correlate well with changes in cardiac index (hence an advantage to have continuous SvO2 monitoring in ICU patients).
- usually used with other forms of monitoring
o able to detect which component is causing deterioration
o for example, if SvO2 decreases, measure cardiac output to determine if it’s the cause

- can guide therapy
o PEEP levels, respiratory support, FiO2, blood transfusions, inotrope/vasopressors
o Sedation, analgesia, anaesthesia
Disadvantages
- falsely elevated in
o admixture with arterial blood e.g. blood shunted from certain organs like the kidney therefore falsely raises the SVO2
o histotoxic hypoxia e.g. cyanide poisoning in SNP
o leads to falsely lowering cardiac output/oxygenation/failing to transfuse
o sepsis
- requires insertion of a specialised PAC with its inherent risks
- very expensive
- variability / sampling errors
o movement of tip of catheter and sampling from an incorrect site
o wedging the balloon will raise the SvO2
- interpretation errors may arise due to intracardiac shunts, TR and catheter misplacement
- signal drift (needs recalibration)
- reflects only global changes
o i.e. normal SvO2 does not mean that an individual organ is well perfused
May 2007 Question 6

List the patterns of peripheral nerve stimulation that may be used to monitor non-depolarising neuromuscular blockade during anaesthesia and describe how each is used in clinical practice.
Peripheral nerve stimulation that may be used to monitor non-depolarising neuromuscular blockade during anaesthesia include
- single twitch
- Train of Four (TOF) count
- TOF ratio
- Double burst stimulation
- Post-tetanic count
In general peripheral nerve stimulation involves electrical stimulation with supramaximal square wave pulses.
Single twitch
- Single pulse 0.2ms in duration, repeated every 10s
- requires control values for valid comparisons, may appear normal with considerable weakness
- poor indicator of deep paralysis

TOF count is used in clinical practice to:
- assess intensity of blockade (eg TOF 1-2 is adequate for abdominal surgery)
- able to give reversal if TOF 2 with medium length acting NDMRs
- consists of 4 successive twitches in 2s (2Hz frequency) each 0.2ms long
- twitches fade as relaxation increases
- disappearance of the 4th twitch = 75% block, 3rd twitch = 80% block, 2nd twitch = 90% block
TOF ratio:
- able to assess residual blockade
- need T4/T1 ratio (measured with accelerography), need greater than 75%-90% to have adequate reversal. Pts fully reversed if TOF ratio = 1.
- able to assess fade
Double burst stimulation
- assess residual blockade
- more sensitive than TOF for the clinical (visual) evaluation of fade, an absence of fade is good evidence of reversal
- 3 short 0.2ms 50 Hz high frequency bursts separated by 20ms intervals followed 750ms later by another 3 bursts

Post tetanic count
- assesses deep blockade when TOF count is zero (eg may be required in neurosurgery)
- This consists of counting 1 Hz twitches 3 seconds after 5 seconds of 50Hz tetany and can give an approximate time to return of response to single twitches and hence permits assessment of block too deep for any other technique. A Post-Tetanic Count (PTC) of 2 by palpation suggests no twitch response for about 20-30 minutes, PTC of 5 about 10-15 minutes. PTC 10-15 suggests TOF is about to reappear when using medium duration NDMRs.
May 2007 Question 7

A 65 year old man with a 40 pack year history of smoking is scheduled for right pneumonectomy for carcinoma.

Describe you preoperative evaluation of his respiratory system to decide his capacity to undergoes this operation.
A right pneumonectomy (resection of entire R lung) is a significant operation on a patient with likely significant pre-existing respiratory compromise. A pneumonectomy has a 6-8% mortality (2-4% for lobectomy). Acute lung injury occurs in 4-5% of resections and is 3 times more likely after a pneumonectomy.

Assessment and optimisation of clinical condition
1) Hx
– presenting complaint eg weight loss, tumour
- paraneoplastic syndromes (SIADH, Eaton-Lambert)
- current level of stability and optimisation of respiratory capacity (assessment of COPD/asthma severity, control, treatment → bronchodilators/steroids, hospital admissions, reversibility). Refer to Respiratory physician for optimisation if required.
- exercise tolerance / current functional status
- smoking hx (significant if current smoker), pack years, attempts at stopping, advise to stop ASAP
- cough, sputum
- medications (inhalers, steroids)
- nodal involvement and any possible symptoms of tracheal compression / SVC obstruction / Horner’s syndrome
- assess for comorbiddities (IHD, CCF)
- Chronic etoh abuse (risk factor for perioperative mortality in pneumonectomy)


2) Examination
- Cardiorespiratory assessment
- Airway assessment with respect to placement of double lumen tube (more difficult than normal intubation) Mallampati, TMD, mouth opening
- double lumen size – looking at CXR estimate tracheal diameter (actual diameter will be 90% that of CXR)
3) Investigations
- ECG – tall R wave in V1 with strain, R axis deviation and RBBB.
- FBE (anaemia), LFTs, Coags (may preclude regional analgesia)
- Albumin <30
- ABG (assess pO2, pC02) type 1 (hypoxic) resp failure, type 2 (hypercarbic and hypoxic) resp failure
- CXR – to assess tumour size and tracheal displacement, to size DLT
- CT chest and neck – review of imaging to ensure no tracheal compression or SVC obstruction, assess bronchial size and anatomy
- RFTs (see below for details)
- TTE to assess for pulmonary hypertension and cor pulmonale which would most likely preclude pneumonectomy
- Cardiopulmonary stress testing (see below for details)

Assesment of operability and resectability
Resectability
- determined by staging of tumour, i.e. whether it is possible to remove the tumour anatomically
- usually decided by surgeon, oncologist and pathologist

Operability
- amount of tissue safe to be resected
- i.e. can the patient survive with remaining tissue

RFTs
Pre-op testing
- FEV1 use % of predicted normal value, FEV1 > 2L for pneumonectomy and > 1.5L for lobectomy
- DLCO >60% (DLCO to assess alveolar and capillary function, <80% predicted postop is associated with worse outcomes)
- Hypercapnia on ABG reflects higher risk
Predicted post-op
- Raw value >1L post-op necessary
- Multiply preop value by the calculated number of postop segments remaining allows estimation of postop FEV1
- If <40% predicted postop = poor outcome
- crude method: pre-op FEV1 x (19-no of segments resected)/19 – should be >10ml/kg
- quantitative V/Q scan – measure amount of radioactive gas taken up by each lung
- mimic post op conditions – temporary occlusion of pulmonary artery with balloon

CPX testing → dynamic measure of ventilatory volumes, O2 consumption and CO2 production. Anaerobic threshold is point where metabolic requirements for O2 exceed transport capability.
Low risk AT >11ml/kg/min = VO2 max >20mls/kg/min
Intermediate AT range 8-11
High risk AT <8mls/kg/min = VO2 max <10ml/kg/min

Plan post-operative analgesia – discussions about epidural / paravertebral / PCA, assess for any contraindications to regional.
Obtain appropriate consent
Plan and book postop ICU bed
May 2007 Question 8

Describe how the ECG should be used to monitor for intraoperative myocardial ischemia in a patient with ischemic heart disease.
The ECG should be used in conjunction with other monitors and interpreted in the clinical context (e.g. ST depression in child is rarely ischaemia). It is also vital to ensure that the equipment is functioning correctly and in good condition.
Correct lead placement and selection (shave skin)
- lead II best for arrhythmias; when used alone can only pick up 30% of ischaemia (when compared to 12 lead)
- when combined with lead V5 (in 5 lead system) and V4, 96% of ischaemia may be identified. (Suggest 5 lead ECG in patients with IHD)
- modified 3 lead systems
o CM5: Right arm electrode placed on the manubrium and left arm electrode placed at the surface marking of the V5 position (just above the 5th interspace in the anterior axillary line). The left leg lead acts as a neutral and may be placed anywhere - the C refers to 'clavicle' where it is often placed. To select the CM5 lead on the monitor, turn the selector dial to 'lead I'
Mode
- In monitor mode, the low frequency filter (also called the high-pass filter because signals above the threshold are allowed to pass) is set at either 0.5 Hz or 1 Hz and the high frequency filter (also called the low-pass filter because signals below the threshold are allowed to pass) is set at 40 Hz. This limits artifact for routine cardiac rhythm monitoring. The high-pass filter helps reduce wandering baseline and the low pass filter helps reduce 50 or 60 Hz power line noise
- In diagnostic mode, the high pass filter is set at 0.05 Hz, which allows accurate ST segments to be recorded. The low pass filter is set to 40, 100, or 150 Hz

Display should allow interrogation of trends as well as real-time viewing.
Highest sensitivity when a combination of leads is used. Ideally an inferior lead (III) and 2 praecordial leads (V3 and V5 or V4 and V5). When only one praecordial lead can be utilised the most isoelectic lead of V3, V4 or V5 should be used.

Changes suggestive of ischaemia
- ST segment changes most sensitive and specific
o depression
• Upsloping ST segment 2 mm depression 80 msec after J point
• Horizontal ST segment 1mm depression at 60-80 msec after J point
• Downsloping ST segment >1mm from top of curve to PQ junction

o elevation
• > 1mm at 60-80 msec after J point

- T wave inversion
- Bradycardia (arrhythmias and conduction defects may be indicative of ischaemia)
- Heart blocks
May 2007 Question 9

The T-Piece is obsolete in modern anaesthesia practice. Discuss.
Almost true with the introduction of Circle System (closed and semi-closed).

However T-peice is still used in recovery, ECT and Paeadtrics

● The T-Piece is a type of breathing circuit, shaped like a ‘T’
○ The exhaled gases are passed into a reservoir tube (Stem of T), which may itself be attached to a bag
○ Fresh Gas Flow is attached to one Branch of T
○ Patient mask is attached to other Branch of T
○ There are no valves
● The T-Piece group of circuits is composed of:
○ Mapleson D - ‘Bain’ circuit (Coaxial modification)
○ Mapleson E - ‘Ayres’ T-piece
○ Mapleson F - Jackson-Rees modification


Advantages:
● Light - easy to use in paediatrics. Coaxial tubing reduces weight on mask.
● Low Dead Space - Because FGF is near patient connection, dead space is minimised.
● Low Resistance - FGF near patient connection minimises resistance to inhalation, and large (open) reservoir minimises resistance to expiration
● Fast Washin - FGF near patient connection
● Low compliance of system - low distensibility of tubing
● Bain / Jackson-Rees modification - Bag allows ability to ‘feel’ compliance of respiratory system
● Portability - Able to attach to any Oxygen outlet, and can be taken to Recovery easily
Disadvantages:
● Low Efficiency - To prevent rebreathing of expiratory gases, need high FGF of ~2.5x minute ventilation (IPPV or SV)
● Complexity - If unfamiliar with setup
● Rebreathing - Complex interaction between FGF, MV and CO2 production
● Low humidity and no HME filter
● Coaxial system can fracture, which is difficult to detect
● No scavenging - Theater pollution by expired gases
Alternatives:
● The circle rebreathing circuits have the following advantages:
○ More efficient, hence cheaper
○ Have CO2 absorber
○ Heat and Moisture exchanger
○ Low volume tubing for paediatrics
○ Scavenging available
○ Spirometry can be performed
○ More accurate End Tidal monitoring
● Due to the above advantages of the Circle system, the T-piece is becoming obsolete in modern anaesthesia practice
May 2008 Question 10

A 56 year old diabetic is scheduled for laparoscopy nephrectomy. This is his pre-operative 12 lead ECG.
Ten minutes into the procedure his BP is 70/30 and his ECG lead II monitor looks like this.
What does ECG 1 Show? What does ECG 2 Show? Outline your management of the situation associated with ECG 2.
ECG 1 shows - L Axis Deviation, RBBB, 1st Degree Heart Block. Rate = ??bpm
= Trifascicular block
ECG 2 shows - AV dissociation, with a ventricular rate of 30bpm
= 3rd Degree Heart Block

Considerations:
● Patient:
○ Diabetes: Associated potential microvascular (retinopathy, nephropathy, neuropathy) and macrovascular complications (Cerebrovascular, ischaemic heart, and peripheral vascular diseases), which are likely to predispose to intraoperative cardiac events.
● Surgical:
○ Laparoscopy /Pneumoperitoneum
○ Hemorrhage
● Anaesthetic:
○ Fasting and intravascular depletion:
■ ?omission of drugs
■ Electrolyte abnormalities
○ Positioning - Trendelenburg
○ Iatrogenic - anaesthetic overdose, drug error, hypersensitivity reaction
Management:
● This is a crisis situation with severe hemodynamic compromise
● Underlying abnormality = absolute bradycardia (<40bpm)
● Call for help / assistance
● Immediate management:
○ Stop case and notify surgeon
○ Relieve pneumoperitoneum
○ Flatten from Trendelenburg
○ Re-assess Airway / Breathing / Circulation
■ Correct hypoxia / hypercarbia
■ Exclude Pneumothorax
■ Exclude hemorrhage
○ If CVS collapse - Adrenalin indicated, CPR
● Specific Management:
○ IV Fluids - crystalloid bolus 10ml/kg x 2
○ Chronotropes:
■ Atropine 0.6mg IV increments
■ Ephedrine 6mg IV increments
○ Chemical pacing:
■ Adrenalin infusion 1-20 mcg/min
■ Isoprenaline infusion 1-10 mcg/min
○ Mechanical pacing:
■ Transcutaneous
■ Transvenous
■ Permanent Pacemaker
● Further investigation:
○ Exclude AMI - Serial troponins, ECGs, Echocardiogram
○ Correct Electrolyte / Acid-base disturbance
○ Mast cell tryptase re: ?Anaphylaxis
○ Cardiology referral - Electrophysiology studies
● Disposition:
○ Transfer to ICU / CCU for ongoing care
○ Inform relatives
May 2007 Question 11

Why is consent for a medical procedure necessary?

What makes consent for a medical procedure valid?
Consent should be obtained for all medical treatment for ethical and medicolegal reasons
● It is a basic tenet of our society that everyone has a right to determine what is done to his/her own body, and is entitled to know the implications of any treatment before it is administered, and to seek clarification of any issues that may be of concern.
○ Consent shows respect for patient autonomy
● Valid consent protects against charges of battery / assault. When detailing risks and benefits, it may provide protection against claims for negligence should a discussed complication arise despite the procedure being carried out competently.

Consent is only valid if:
○ It is given voluntarily and without coercion.
■ Refusal or withdrawal of consent must be an option
○ It is given by a person capable of doing so:
■ Should be able to understand the nature, purpose and possible consequences of proposed treatment, as well as consequences of non-treatment,
■ All persons are presumed to be competent to give consent, unless there are reasonable grounds for believing otherwise
■ Consent can be obtained from patient, legal guardian, parent, or next-of-kin.
○ It is informed:
■ Provision of information that a ‘reasonable’ patient in the position of the patient might wish to know, and to which he/she might attach significance
● Any information significant enough to cause the patient to change their mind about the procedure.
■ Information should be presented in a form that patient is likely to understand
■ Nature of treatment explained
■ Risks and benefits of proposed treatment
■ Risks and benefits of alternative treatments
■ Risks and benefits of no treatment at all
■ Opportunity must be given to discuss concerns and to answer questions in an honest and accurate manner
■ Financial implications, if any, of treatment should also be discussed
○ It is obtained by a person who understands and is able to discuss the risks and benefits of the proposed / alternative / no treatment, themself.

● Documentation as part of patients history, including reference to discussion of material risks and agreement by patient to undergo treatment
○ Not a legal requirement, but highly recommended to defend against any claims
● A qualified interpreter (not family member) should be used whenever necessary
● Consent should be obtained ideally by anaesthetist conducting the treatment
May 2007 Question 12.

How do you assess an otherwise well patient with regard to difficulty of intubation at the bedside?
How accurate is the assessment?
● Difficult Intubation (ASA, 1993) =
○ “The proper insertion of an ETT with conventional laryngoscopy that requires more than 3 attempts and / or more than 10 minutes”
● Difficult Laryngoscopy = Cormack and Lehane Grade 3 or 4 view
Anaesthetic Assessment:
● History:
○ Surgical procedure:
■ Indication for intubation
■ Alternative to intubation - supraglottic airway, regional anaesthesia
○ Past history of difficult / failed intubation:
■ Review previous anaesthetic chart
■ Letter from previous anaesthetist
■ Awake fibreoptic intubation?
○ Associations:
■ Obstructive Sleep Apnoea / Obesity
■ Syndromes:
● Downs
● Golden Haar
● Klippel Feil
● Pierre Robin
■ Neck pathology:
● Actual or potential for C-spine fracture
● Rheumatoid arthritis / Ankylosing spondylitis
● Post radiation / burns
■ Late pregnancy


● Examination: (MMMINT-B)
○ Mallampati Score (1 to 4)
■ Assessment of visibility of soft palate and uvula in relation to tongue size
■ Mallampati 1 and 2 - Easy intubation likely (small false negative rate)
■ Mallampati 3 and 4 - associated with difficult intubation (High false positive rate)
○ Mouth opening (Interincisor Gap)
■ <2.5cm - difficult insertion of laryngoscope / rescue LMA
○ Mandibular movement:
■ Presence of trismus
■ Mandibular protrusion
● The ability to prognathe is associated with more laxity when lifting soft tissues with laryngoscope
○ Incisors / Oral Cavity:
■ Teeth:
● Nil - easier
● Loose or poor - risk of dislodgment into airway
● Precarious single tooth and prominent incisors - difficulty in placing laryngoscope
■ High arched palate / cleft palate
■ Oral tumours / abscess
○ Neck and Thyromental distance:
■ Range of movement - especially neck extension
■ Reduced ROM associated with difficult intubation
○ Body Habitus:
■ BMI alone has poor correlation with difficult intubation
■ Neck circumference > 40cm associated with difficult intubation
■ Large breast may obstruct laryngoscope handle
■ No neck visible - may hinder external laryngeal manipulation
■ Pregnancy - Difficult intubation more common 1:250

Accuracy of Assessment:
● Overall:
○ Difficult intubation is rare - all tests have low positive predictive value
○ No test has high sensitivity - therefore true difficult intubations are easily missed
○ All bedside assessments in isolation are not great indicators of difficult intubation, but collectively give much better correlation
● Correlation with difficult intubation:
○ Mallampati Score > Thyromental Distance > Mouth Opening > Neck Circumference > Neck ROM > History > BMI



MP : good with high false positive
TM : good
MO : good
Neck ROM : moderate
BMI : poor
History : moderate
Neck circumfence : good
May 2007 Question 13

Ambulance officer performing CPR with bag mask ventilation. She has been rescued from a swimming pool.

1. Describe how basic life support should be provided in the emergency department

She has no pulse and her ECG shows ventricular fibrillation

2) Outline the advanced life support algorism you would now follow
1. Basic life support:
- Danger (ie wet patient. Awareness during DCR)
- A- patent airway. BMV
- B- assess breathing. Look, listen, feel.
- C- palpable pulses?
- CPR- 30:2 compressions to breaths. 100 chest compressions per minute. Depress 4-5 cm in centre of chest.
- Monitor ECG, pulse, attach monitor/defibrillator without interruption of compressions.

2. Advanced life support:
Follow 'shockable' algorithm:
- Advanced airway (attempt intubation). IV access.
- DCR- 150 J biphasic or 360 J monophasic. Immediately resume CPR.
- After 2 mins- assess rhythm. Repeat shock if still VF (150-360 J biphasic). Repeat cycle
- Adrenaline 1 mg IV prior to further shock
- Amiodarone 300 mg IV
- Correct reversible causes (5Hs, 5Ts)- in this case esp. hypothermia, hypoxia, electrolyte disturbances
- Role for hypothermia post resuscitation
- Transfer to intensive care setting for further post-resuscitation management.
May 2007 Question 14

An otherwise fit 30 year old man is having microvascular re-implantation of his forearm. Describe the methods available to optimize the perfusion of the re-implanted limb in the post-operative period.
1. Preparation of limb- wrap in moistened gauze, place in crushed ice and water covered in plastic. Flush main artery with 'cold storage solution' if available.
2. Monitoring limb to detect microvascular thrombosis- regular limb obs, monitor skin temperature, capillary return.
3. Optimise factors controlling perfusion/O2 delivery
- Blood flow:
- cardiac output- avoid hypovolaemia, may require inotrope therapy
- BP- aim for MAP > 70 mmHg. May require vasopressors/inotropes in HDU
- blood volume- optimise fluid loading/monitor urine output
- blood rheology - optimal Hb (10 g/dL) to minimise viscosity. Avoid hypovolaemia
- consider dextran 40 (microcirculation)
- anticoagulation
- temperature- keep warm, reduces vasoconstriction

- Oxygenation carrying capacity
- Optimise Hb (10 g/dL- balance between O2 carrying capacity/viscosity)
- O2 saturation- may require supplemental O2 for SpO2 > 94%
- Temperature- keep warm (assists offloading of O2), don't overheat (increases O2 consumption)
- normal pH (influences position of Hb-O2 dissociation curve)

- Vascular resistance
- brachial plexus block – also provides analgesia which avoids rise in SNS, with concomitant vasoconstriction
- drugs- consideration to vasodilators if signs of decreased perfusion

- Venous pressure
- slightly elevate limb to encourage venous and lymphatic drainage
- avoid too tight dressings
- leeches to promote venous drainage possible if venous anastomoses insufficient
May 2007 Question 15

Outline the steps necessary to diagnosis brain death in a 38 year old woman who is comatose following a subarachnoid haemorrhage.
Known cause of irreversible loss of brain function

Acute brain pathology consistent with the irreversible loss of brain function must be present.

Period of continuous observation of apparent loss of neurological function

At least four hours of observation (24 hours for hypoxic-ischaemic encephalopathy) and mechanical ventilation during which there has been unresposive coma, non-reactive pupils, absent cough/tracheal reflex and no spontaneous breathing efforts.

Preconditions (must be met)
1. Hypothermia not present (temp > 35 degrees Celsius)
2. Blood pressure adequate (MAP > 60 mmHg)
3. Sedative drugs excluded (sedative free period/antagonists administered)
4. No severe metabolic/endocrine/electrolyte disturbance
5. Neuromuscular function intact
6. It is possible to examine brain stem reflexes (ie one eye/ear available)
7. It is possible to perform apnoea testing

Clinical testing (require two doctors in two full and separate examinations with expertise to do so- medical practitioner for at least five years, not be the remover of organs, one should be a specialist)
1. No motor response in the cranial nerve distribution to noxious stimulation of the face,
trunk and four limbs and there is no response in the trunk or limbs to noxious stimulation within the cranial nerve distribution
2. There are no pupillary responses to light
3. There are no corneal reflexes
4. There is no gag (pharyngeal) reflex
5. There is no cough (tracheal) reflex
6. There are no vestibulo-ocular reflexes on ice-cold caloric testing
7. Breathing is absent (despite arterial PCO2 > 60mmHg (8 kPa) and arterial pH < 7.30)

Determination of brain death when clinical examination cannot be done:
There is no intracranial blood flow, demonstrated by either intra-arterial angiography
or other suitably reliable method (radionuclide imaging/CT angiography as per ANZICS guidelines).

Brain death (ie. time of death) is the time at the end of assessment by the second doctor.
October 2006 Question 1

Discuss the risks and benefits associated with intermittent positive pressure ventilation through proseal laryngeal mask airways for a patient undergoing laparoscopic cholecystectomy.
The proseal is a variant of the Laryngeal Mask Airway (LMA), a supraglottic airway device designed to provide and maintain a seal around the laryngeal inlet for spontaneous ventilation and allow controlled ventilation with intermittent positive pressure ventilation at modest levels of positive pressure (<30 cm H2O).

In a patient undergoing a laparoscopic cholecystectomy, the effects of a pneumoperitoneum also needs to be considered as this will likely require increased airway pressures in order to achieve adequate ventilation and prevent hypercarbia.

Risks:
- The proseal does not offer the same level of airway protection as an ETT. It is not a definitive airway – there is a risk of aspiration. However, studies in carefully selected patients (fasted non-obese patients with no GORD) show that the incidence of aspiration is similar to an ETT
- If the proseal is not positioned properly, or if the size is incorrect, there can be leaks around it. This could lead to an inability to achieve adequate ventilation leading to hypoxaemia, hypercapnia, and inadequate delivery of inhalational agents.
- A limit of 30 cmH20 should be adequate for most patients, but in some e.g. obese patients it may be difficult to ventilate with a pneumoperitoneum.
- Not appropriate in a known or suspected difficult intubation as loss of the airway mid-case will be difficult to manage. In these patients a definitive airway needs to be established at the start of the case.

Benefits:
- Provides a more effective seal compared to a classic LMA especially when on IPPV, and allows higher inspiratory pressures compared to a classic LMA, able to tolerate pressures up to 30cmH2O before leak.
- Proseal causes less airway manipulation compared to ETT. Less risk of dental trauma.
- The Proseal has an oesophageal lumen which allows greater protection from aspiration if it is correctly positioned compared to a LMA. The oesophageal lumen allows the passage of an orogastric tube to drain any residual gastric contents; whether this reduces aspiration is unclear. The presence of a tube through the oesophageal sphincter may actually increase reflux.
- The above would also allow drainage of gas from the stomach from the positive pressure ventilation.
- The presence of an in-built bite block is useful in maintaining a patent airway on emergence and decreases the risk of negative pressure pulmonary oedema.
- It may possible to avoid the use of muscle relaxants by suppressing ventilation using drugs and/or hyperventilation, which can be useful in patients where muscle relaxation or reversal would have added risk (eg. Muscular dystrophies). However in most patients muscle relaxation will still be required by the surgeons for the procedure.
- Intermittent positive pressure ventilation is beneficial over spontaneous ventilation to control the arterial levels of carbon dioxide. Otherwise the absorption of carbon dioxide from the pneumoperitoneum would lead to hypercapnia and acidosis.
- The reverse Trendelenburg position is often required during a laparoscopic cholecystectomy which will aid ventilation.
- Possibly quicker turnover time – the Proseal LMA can be left in the patient during transfer to recovery, if the patient is spontaneously ventilating and their ventilation/oxygenation is satisfactory.
October 2006 Question 2

Describe your technique for performing a continuous paravertebral block in a 50 year old man with fractured 5th-10th left ribs. Include possible complications and relevant anatomy.
A paravertebral catheter and infusion provides good analgesia in patients with unilateral fractured ribs.
The thoracic paravertebral space is a wedge-shaped area that lies on either side of the vertebral column. Its walls are formed by the parietal pleura anterolaterally; vertebral body, the intervertebral disk, and intervertebral foramen medially; and the superior costotransverse process posteriorly (runs from neck of rib to transverse process above). Its contents include – intercostal nerve (dorsal ramus), sympathetic chain anteriorly and intercostal vessels.

Preparation:
- explanation to the patient and obtain informed consent
- equipment
- monitoring +/- sedation/analgesia
- Position the patient- can be done in the lateral position or sitting, but in this case probably best to have lateral with right side down to avoid discomfort.

Performing the procedure:
- Draw a line in the midline along the spinous processes.
- Draw a second line parallel to this 2.5 cm lateral to the left of the previous line.
- Full barrier aseptic technique with alcoholic chlorhexidine skin prep, mask, gloves and drapes
- Infiltrate the skin at a point corresponding to T6 or 7 (1 level below the highest dermatome) on the second line. (NB For continuous paravertebral blockade the catheter is ideally inserted 1-2 segmental levels below the thoracotomy incision line.) Insert the needle perpendicular to the skin 2.5cm lateral from the midline at the level of the spinous process requiring anaesthesia.
- Using the Tuohy 18G 8cm needle advance perpendicular to the skin until the transverse process is reached (2-4 cm between T5-T10). Then withdraw the needle to the skin and this time advance 10-15 degrees cephalad 1cm past the transverse process. Sometimes a subtle LOR is felt once the paravertebral space is reached but this is not reliable and it is best to use 1cm past the transverse process for a more accurate catheter placement.
- Check for air, blood and CSF. Once these have been excluded inject initial bolus 10mls of 0.5% bupivicaine. Advance catheter 3-5cm beyond the needle tip and secure.
- Commence infusion (eg. 10mls/hr of 0.2% ropivacaine).

Complications:
- Infection
- Haematoma (esp if multiple attempts in an anticoagulated patient)
- Vascular puncture
- Pneumothorax- rare, if lateral needle angulation
- Inadvertent placement of the catheter in the epidural or intrathecal space. This is rare and can be minimised by avoiding any medial angulation of the needle.
- Block failure
- Local anaesthetic toxicity
- Total spinal anaesthesia – do not angle needle medially
- Nerve injury, cord injury
- Horner’s syndrome
- Paravertebral muscle pain (mainly in muscular young men)
- Quadriceps weakness (if the block is too low- below L1)
October 2006 Question 3

Describe the cardiovascular changes which occur during clamping and unclamping of the supra-renal aorta during repair of an abdominal aortic aneurysm in a patient with normal ventricular function and outline your strategies to maintain critical organ perfusion during these times.
Clamping:
- increased arterial blood pressure
- increased afterload
- increased catecholamines and vasoconstrictors (sympathoadrenal response)
- passive recoil distant to clamps → some redistribution to splanchnic vessels (depends if supracoeliac or infracoeliac cross-clamp), shift of blood proximally to clamp and increased venous return
- increased preload and CVP
- increased segmental wall motion abnormalities
- increased left ventricular wall tension
- if normal heart and ventricular function, contractility and cardiac output can increase (which in turn increases coronary flow)
- if abnormal heart, contractility and cardiac output will decrease – potential for ischaemia because of high ventricular wall tension
- reduced renal blood flow
- increased pulmonary occlusion pressure unlikely with normal pre-op ventricular function.
- increased coronary blood flow
- increased cerebral blood flow
- With prolonged cross-clamping → distal tissue ischaemia → mediator release → distal vasodilatation and increased capillary permeability → distal shift of blood and loss of intravascular fluid → central hypovolaemia.

Unclamping:
- reduced arterial blood pressure
- reduced venous return and CVP – contribution by vasoactive mediators from 'ischaemic' organs
- reduced cardiac output
- reduced myocardial contractility (due to release of mediators)
- increased pulmonary artery pressure unlikely if myocardial function is normal

Maintaining critical organ perfusion:
- most important of all --> minimise clamp time!!!! (usually 30-60 minutes)
- myocardial perfusion and protection: coronary vasodilators, preload and afterload reduction during clamping (with GTN or beta blockers) GTN maintains oxygen extraction ratio and SVO2. Aim for controlled unclamping, replacement of volume prior to / during unclamping, may need vasoconstrictor and augmentation of contractility. If severe haemodynamic instability → reapply cross clamp.
- renal perfusion and protection: during clamping consider mannitol (osmotic diuretic and also free radical scavenger), drugs (eg. Diuretics). Fluid administration just prior to unclamping, distal aortic perfusion techniques
- Neurological – ensure adequate brain perfusion (adequate MAP) and spinal cord perfusion – since this is AAA not TAAA unlikely to clamp above artery of Adamkiewicsz, otherwise consider CSF drain)
- lower limb ischaemia – high risk as they are vasculopaths. During clamping warming blankets should not be placed on lower limbs as this may worsen limb ischaemia.
- adequate Hb, oxygenation
- Ventilate adequately during cross clamp to minimise effect of acidosis with unclamping. May need bicarbonate with unclamping.
- avoid hyperthermia
- bowels – as above
October 2006 Question 4

Describe the features and management of phantom limb pain.
(see ANZCA Pain book 3rd edition page 238)
Phantom limb pain is defined as any noxious sensory phenomenon in the missing limb or organ. The incidence of PLP is 30-85% after limb amputation and occurs usually in the distal portion of the missing limb. It can be immediate – 75% of patients report pain within the first few days after surgery or be delayed in onset.
Factors that may be predictive of PLP – severity of preamputation pain, the degree of postop stump pain and chemotherapy. Acute post-operative pain and passive coping strategies/catastrophising are also strong predictors for phantom limb pain.
If preamputation pain was present, phantom pain may resemble that pain in character and localisation.
Phantom limb pain is usually intermittent and diminishes with time after amputation. Triggers may be – prosthesis, pressure, weather changes, fatigue. Phantom pain is described as shooting, stabbing, boring, squeezing, throbbing, and burning.
It should be differentiated from stump pain (pain localised to the site of amputation) and phantom sensations (defined as any sensory perception of the missing body part with the exclusion of pain), though sometimes they may coexist.

Prevention
1) Evidence for benefit of epidural analgesia in preventing phantom limb pain is inconclusive but perioperative epidural reduces the incidence of severe phantom limb pain (NNT 5.8).
2) Ketamine reduces the incidence of severe phantom limb pain (bolus and infusion started prior to skin incision and continued for 72 hours) though not the overall incidence of long-term phantom limb pain.
3) Perioperative gabapentin ineffective in reducing incidence and severity of phantom limb pain. As are peripheral nerve sheath catheters though they provide excellent acute post-op analgesia.

Treatment
Multiple treatment modalities but convincing evidence is lacking for most of them. The more established treatments are:

1. Calcitonin – is effective in the treatment of acute phantom limb pain (level II). Can be given IV, nasal or S/C. (eg 100 units s/c daily for 3 days). Mechanism of action uncertain ?works via serotonin receptors
2. Ketamine provides short term relief of phantom limb and stump pain. (level II)
3. Opioids – oral controlled release morphine and iv infusions of morphine reduced phantom limb pain (level II).
4. Gabapentin effective in reducing phantom limb pain (level II)
5. Amitryptiline and tramadol provided good control of phantom limb pain and stump pain in amputees. (level II)
6. Injections of LA into painful myofascial areas of the contralateral limb reduced phantom limb pain and sensations.
7. Non-pharmacological treatments that are effective include – sensory discrimination training and motor imagery (imagined movements and mirror movements) for 2 weeks. (level II)

It is also important to ensure multi-modal analgesia for post-operative pain relieve (as severe post-op pain is a risk factor for development of phantom limb pain) – include paracetamol and continuous regional blockade via nerve sheaths (no preventive effect on phantom limb pain but provides excellent postop analgesia). Consider NSAIDs – lacking good evidence.

IV lignocaine reduced stump pain but had no effect on phantom limb pain.

Surgical treatments including stump revision, cordotomy, thaladotomy, etc. largely not done because lack of efficacy, esp in long term.
October 2006 Question 5

Describe and justify an appropriate strategy for the use of Low Molecular Weight Heparin in a patient undergoing knee replacement surgery with an epidural block.
• Knee replacement surgery places a patient at high risk for developing DVT (40-60%), and subsequent PE.
• Thromboembolism (TE) prophylaxis with LMWH significantly reduces the risk of developing DVT / PE (by ~75%)
• Patients receiving haemostasis-altering drugs, such as LMWH, as well as epidural blockade are at risk of developing an epidural haematoma
• An appropriate strategy may significantly reduce the risk of epidural haematoma but not eliminate it
• Enoxaparin is a LMWH used commonly as a 40mg single daily subcutaneous dosing (unless patient light weight, or renal impairment)
• My anaesthetic approach would be:
o Catheter insertion:
• Should occur at least 10-12 hours after last LMWH dose, if on TE prophylaxis dose (40mg d)
• Should occur at least 24 hours after last LMWH dose, if on higher treatment dose (1mg/kg/bd, or 1.5mg/kg)
o Post-operative LMWH:
• Is safe to administer in the presence of epidural catheters provided guidelines are followed
• Management is based on dosing schedule and timing of first post-operative dose
• The first LMWH dose should be administered 6 to 8 hours post-operatively
• Subsequent dosing should be every 24 hours
o Follow-up:
• Anaesthetics:
• Twice daily patient reviews to specifically detect signs and symptoms of epidural haematoma (midline back pain, sensory and motor deficits, bowel / bladder dysfunction)
• Use of low concentration and low potency local anaesthetic solutions (eg 0.2% Ropivacaine with 2mcg/ml Fentanyl) to minimise motor blockade yet providing adequate analgesia and early detection of epidural haematoma
• Ward nursing staff and patient:
• Educated in regards to signs and symptoms of epidural haematoma and importance of prompt reporting
o Catheter removal:
• Should occur at least 10-12 hours after the last dose of LMWH
• Subsequent LMWH dose should occur after a minimum of 2 hours have elapsed
• Other information:
o If catheter insertion was traumatic, with presence of blood in catheter, the administration of LMWH should be delayed until 24 hours after catheter placement
o Anti-Xa level is not predictive of the risk of bleeding, and has no place in routine monitoring
o Other haemostasis-altering drugs, such as oral anticoagulants and antiplatelet agents, increase the risk of haematoma formation and should be avoided.


Ref: ASRA 2010 guidelines
Reg Anes Pain Med, Vol 35(1), Jan-Feb 2010, pp102-105
October 2006 Question 6

List and Explain the typical electrolyte abnormalities of Chronic Renal Failure.
Chronic renal failure is the irreversible progressive loss of both tubular and glomerular function, resulting in altered electrolyte homeostasis, amongst other derangements. These electrolyte abnormalities include:

• Hyperkalaemia:
o Due to decreased potassium secretion by distal renal tubules
o Hyperkalaemia does not develop until GFR ~10% of normal
• Hypocalcemia:
o Diminished activity of enzyme cholecalciferol 1a-hydroxylase
o Reduced activation of Vitamin D
o Reduced calcium absorption from the intestine
o Also results in secondary hyperparathyroidism
• Hypermagnesemia:
o Due to reduced GFR and thus reduced magnesium excretion
• Hyperphosphatemia:
o Due to reduced GFR and thus reduced phosphate excretion
• Hyponatraemia:
o Due to water retention
o Capacity to dilute urine becomes limited only at very low GFR
• Acidosis:
o Reduction in H+ secretion
o Reduction in ability of body to produce NH4+ (ammonia)
o Reduced ability to reabsorb HCO3-
October 2006 Question 7

While performing an epidural for labour analgesia in an otherwise healthy primigravida in first stage, you inadvertently cause a dural puncture with the Tuohy needle.
Describe and Justify your management of this complication.
Management of inadvertent dural puncture can be divided into immediate and late.

Immediate Management:
• The initial aim is to achieve effective analgesia without causing further complication, this can be achieved by:
1. Passing ‘epidural’ catheter into intrathecal space
• Clearly labelling catheter as ‘intrathecal’, and only allowing anaesthetist to perform any further intermittent top-ups
• Top-up: 1ml of 0.25% Bupivacaine + 5 mcg Fentanyl
• Advantages:
• Excellent analgesia likely
• No possibility of another dural puncture on re-insertion of epidural
• Unpredictable spread of epidural solution or catheter through dural tear is eliminated
• Incidence of post-dural puncture headache is reduced (but only if catheter is left in situ for more than 24 hours)
• Disadvantages:
• Early blood patch cannot be performed
• Theoretical risk of introducing infection
• Catheter mistaken for epidural catheter
• Labour intensive
2. Remove Epidural catheter
• Reinsert the epidural at a different space (one space higher), and ensure that needle not inserted past depth causing dural puncture
• All top-ups to be performed by anaesthetist
• Advantages:
• Early blood patch may be instituted
• Risks associated with intrathecal catheter not present
• Disadvantages:
• Unpredictable spread of epidural solution through dural tear
• Labour intensive
• Risk of another dural puncture
• The patient should be informed at earliest opportunity of dural puncture and likely sequelae.
• Labour allowed to continue normally
• Daily post-natal followup

Late Management:
• Following dural puncture with 16-18G Tuohy needle, incidence of post-dural puncture headache (PDPH) is ~70%
• Headaches in postnatal period are common. The differentiating features of PDPH are:
o Usually bifronto-occipital, with associated neck stiffness
o Onset 24-48 hours post dural puncture, and lasting 7-10 days untreated
o Worse on standing, and relieved with bed rest
o Photophobia, diplopia and difficulty in accommodation are common
• Prophylaxis:
o Bed rest does not prevent PDPH
o Epidural blood patches have no role in prophylaxis
o No evidence that IV hydration reduces incidence
• Treatment:
o Simple analgesia – Paracetamol, NSAIDs
o Epidural blood patch
• Follow-up:
o Daily post-natal follow up whilst inpatient
o Written information and anaesthetic contact details if concerns / worsening
October 2006 Question 8

Describe the principles of cerebral protection in a patient with an isolated closed head injury.
The main aim is prevention of secondary injury, by maintaining cerebral oxygenation
Cerebral ischaemia may develop due to oxygen supply-demand imbalance:

Reduce oxygen demand by:
• Avoiding hyperthermia
• Prompt treatment of seizures and consider prophylaxis
• Sedation and mechanical ventilation
• Analgesia for pain (avoid ketamine)
Increase oxygen supply by:
• Normalising oxygen carrying capacity of blood:
• Avoid hypoxia / desaturation
• Treat Anaemia
• Maintenance of Cerebral Perfusion Pressure (CPP):
• CPP = MAP – ICP / JVP
• Aim CPP 70-80mmHg
• Reduce Intracranial Pressure (ICP)
• Oedema:
• Avoid hyponatraemia – use Normal Saline as IV fluid, and avoid hypotonic solutions
• Diuretics – Frusemide, Mannitol
• Surgical cranioplasty to relieve pressure
• Blood:
• PaCO2 –
o Cerebral blood volume is directly proportional to PaCO2
o Hyperventilation and reduction in PaCO2 to no less than 25mmHg (risk of ischaemia)
o Non-sustained effect lasting 6-18 hours
• Burr Hole / Surgical decompression of hematoma
• CSF – External ventricular drainage to reduce ICP
• Reduce JVP
• Extrathoracic vascular occlusion:
• Avoid circumferential neck pressure (tube ties)
• Neutral head position
• Intrathoracic vascular occlusion:
• Treat pneumothorax
• Avoid bronchospasm
• Avoid coughing / straining (paralysis if sedated / mechanically ventilated)
• Increase Mean Arterial Pressure (MAP):
• Maintain normal to high MAP
• Use of vasopressors or inotropes

Other considerations:
• Maintain normoglycemia
o Hyperglycemia may worsen cerebral swelling (osmotic gradient)
o Hypoglycemia causes cognitive dysfunction, as glucose is primary source of energy
• Hypothermia
o Reduces cerebral metabolism and oxygen demand in animals
o Benefit unproven in humans
October 2006 Question 9

Nitrous oxide should not be used routinely as a component of general anaesthesia. Discuss.
Historically and in modern times, N2O has been used as an adjunct to GA due to its speed of onset and ability to spare doses of other (usually volatile) anaesthetic agents. Usual practice is to administer 70% N2O in oxygen with a volatile agent (eg. sevoflurane) or propofol. In recent years this practice has been questioned as a result of increasing awareness of problems:
cons:
- inhibits methionine synthase –usually converts homocysteine to methionine used to form tetrahydrofolate and thymidine in DNA synthesis
o megaloblastic anemia (impaired Vit B12 metabolism)
o reduced leucocyte and immune function
o hypercoagulability
o myocardial ischemia
o nerve and spinal cord toxicity
o carcinogenicity
o teratogenicity (esp. with occupational exposure)
- increased PONV – NNT when removing N2O is about 5; level I evidence
- increased PA pressures if pre-existing pulmonary ht
- increased risk of hypoxia – diffusion hypoxia, need to reduce the FiO2
- expansion of air-containing spaces – bowel, pneumothorax etc
- increased ICP
- supports combustion
- theatre pollution / greenhouse gas (used in high concentrations so environmental pollution more relevant for N2O than for volatiles)
o contribution to greenhouse gas is actually quite significant
- high risk of awareness if sole anaesthetic
- obviates the benefits of high FiO2 (eg 80%) – eg. Reduced wound infection in colorectal surgery, reduced PONV

A study in 2000 (A&A) in 90 patients undergoing carotid EA showed increased levels of plasma homocysteine and significantly increased episodes of myocardial ischaemia with nitrous oxide.

The ENIGMA trial (Anesthesiology 2007: 2050 patients, multicentre RCT) showed nitrous oxide (FiO2 0.3 with nitrous versus FiO2 0.8-1.0 with nitrogen) increased postoperative wound infections, severe vomiting, pneumothoraces, atelectasis and pneumonia, with a greater length of stay in ICU patients only (the primary endpoint). Non-significant increases in AMI, awareness and mortality were also observed. It is unclear how much the necessary reduction in Fi02 contributed. ENIGMA II aims to address this.
Balanced against this are its pros:
- long hx of safe use, most anaesthetists very familiar with its use
- cheap
- insoluble, therefore rapid onset and offset (only desflurane is more insoluble)
o eg. More rapid awakening than volatile agent alone
- 2nd gas effect – of questionable clinical relevance
- significant analgesic effect
- reduction of amount of volatile agent –with reduction in side effects (eg cardiovascular depression)

In conclusion
- In general I agree with this statement – no drug should be used routinely in anaesthesia
- Good evidence based anaesthesia involves careful consideration to choosing the best anaesthetic for each individual patient taking in to account patient, surgical, anaesthetic factors, and cost
- situations exists where it makes sense to eliminate N2O, or where it is absolutely or relatively CI
o high risk of PONV – patient or surgical factors
o where expansion of gas-containing spaces is an issue – eg. Laparascopic colorectal surgery, patient with pneumothorax, risk of VAE, craniotomy
o patient with pulmonary hypertension, or where increase in PA pressures needs to be avoided – eg. Potential R to L shunt
o ICU/malnourished patient having multiple/repeated/long operations and where effects on VitB12 metabolism might be relevant
o Where high FiO2 required – hypoxic patient with resp failure etc
o Laser surgery or other surgery where airway fire is a risk
- Situations exist where it remains reasonable to use it
o eg GA caesarean section where it allows you to reduce concentration of volatile and minimize the uterine relaxation after delivery
o paediatric practice where speed of inhalational induction/odourless nature is desirable
October 2006 Question 10

Discuss in detail the technique of rapid sequence induction with cricoid pressure in a child. Include the reasons for your choice of relaxant.
Background
• The goals of RSI are to secure and control the airway rapidly in order to reduce the risk of pulmonary aspiration, and to use short acting agents that allow as swift offset and return of spontaneous ventilation as possible in the event of failed intubation.
• The paediatric airway differs from the adult airway in several ways. There is a higher incidence of airway obstruction due to a large tongue and occiput, floppy U shaped epiglottis, small mouth and short neck. The trachea is also shorter increasing the risk of right bronchial intubation

Basic steps involved in a RSI
1. Preparation
• Relevant anaesthetic history from the parent(s) and focused examination of the airway
• Prepare equipment: masks, guedel airways, 2 functioning laryngoscopes, appropriate size ETTs, air-viva, LMAs. A fully working and checked anaesthetic machine and skilled assistance is required
• An uncuffed ETT has traditionally been used for children below 12 yrs old as the paediatric airway is narrowest in the subglottic area. Newer cuffed tubes exist that may lessen this risk and offer greater protection against aspiration.
• Suction working and under the pillow
• Apply routine monitoring to ANZCA standards: pulse oximeter, ECG, NIBP, ETCO2
• Draw up predefined doses of drugs

2. Preoxygenation
• Administer 100% oxygen via a tight-fitting mask for 3 minutes until ETO2 > 90% or, in extreme emergency, 4 vital capacity breaths
• Any entrainment of air reverses the process and preoxygenation should start again
• Do not remove the mask until laryngoscopy
• Preschool children are unlikely to co-operate with this and an inhalational induction may be necessary (usually possible in infants and older children)
• Desaturation during apnoea occurs more quickly due to a higher oxygen consumption. RSI may therefore be modified with gentle mask ventilation after induction to prevent hypoxaemia before or during laryngoscopy, taking care to not insufflate the stomach with high inspiratory pressures (cricoid may need to be relaxed).

3. Rapid sedation
• IV thiopentone 3-5 mg/kg or IV propofol 2-3 mg/kg
• Thiopentone is relatively contraindicated in asthmatic patients because of histamine release

4. Cricoid pressure
• An assistant applies cricoid pressure as consciousness is lost and maintained until the ETT is confirmed to be in the trachea
• Cricoid pressure serves a dual function: 1) occludes the oesophagus and prevents passive reflux of gastric contents into the oropharynx, and 2) shifts the larynx posteriorly making visualization of the cords and tube placement easier
• If the child vomits after application of cricoid but before induction, it should be released
• The force required is 10N whilst awake increasing to 30 N with lost of consciousness (i.e. just enough to indent a tennis ball with your finger). In children, the forces required are not established and they may only tolerate cricoid pressure after induction
• Furthermore, it is difficult to accurately locate the cricoid in neonates or infants and is often disregarded in this age group
5. Paralysis
• Sux 1-1.5 mg/kg if >10kg, 1.5-2 mg/kg if <10kg
• Sux should be preceded by atropine to prevent bradycardia in children under 5 yrs old. Neonates and infants have increased vagal tone and are prone to bradycardia, which may be caused by hypoxia, vagal stimulation during laryngoscopy or pharmacologic agents
• Give atropine 20 mcg/kg (min 0.1 mg, max 1 mg) at least 1-2 mins before intubation. Also helps dry secretions allowing easier visualization of the larynx
• Despite its well known side-effects, sux has a rapid onset and offset, making it an ideal relaxant in RSI
• Rocuronium is an alternative. Doses of 0.6 mg/kg and 0.8 mg/kg produce paralysis in infants and children in 60 and 28 secs respectively. But the recovery time is significantly longer in infants < 10 mths and slightly longer in children up to 5 yrs old

6. Intubation
• Intubate after fasciculations cease or 45-60 secs after administration of sux
• Inflate cuff, hand ventilate and confirm correct ETT placement by capnography and bilateral auscultation of the chest and stomach
• Ensure there is an audible leak around the ETT
• If the tube is correctly placed, ask the assistant to remove cricoid pressure
• Difficult laryngoscopy and intubation is much less common in children
• Extubate awake in the left lateral position
October 2006 Question 11

Critically evaluate the use of Beta blockers in the perioperative period to prevent myocardial infarction.
Rationale for use
• Perioperative MI is a major cause of morbidity and mortality- 1-2% risk of MI in patients without IHD, compared with 4% in patients with.
• Perioperative MI is frequently 'silent' (only 50% have any symptom/sign)
• Beta blockers are postulated to be of use from a pathophysiological perspective:
1. reduced myocardial O2 demand/consumption- reduced HR, contractility
2. reduced risk of plaque fissuring due to decreased shear stress
3. anti-arrhythmic potential
4. anti-inflammatory effects promoting plaque stability
5. decreased stress state (reduced plasma catecholamines)
6. anti-renin angiotensin properties

Problems with use
1. Withdrawal of beta blocker therapy shortly before surgery, or in the immediate postoperative period produces a ‘rebound’ effect with hypertension, tachycardia which may contribute to cardiac events
2. Other side effects of beta blockers – hypoglycaemia, confusion, intraop hypotension/bradycardia, pre-op hypotension, inability to gauge effect hypnotic effect of anaesthetic agents or effects of ‘analgesics’

Early landmark studies (encouraging)
1. Mangano et al. (NEJM 1996): 200 patients with or at risk for CAD undergoing non-cardiac surgery. RCT, double blind, placebo controlled. Reduction in incidence of myocardial ischaemia with a short perioperative course of atenolol (immediately before induction and up to 7 days after surgery).
2. Poldermans et al. (NEJM 1999): 112 patients undergoing vascular surgery with positive dobutamine stress echo. Stopped early as bisoprolol started at least 7 days preoperatively reduced the perioperative incidence of non-fatal MI and death from cardiac causes.

Later studies (less encouraging)
1. MAVS 2006- metoprolol vs placebo in vascular surgery.
2. POBBLE 2005- infrarenal vascular surgery.
3. DIPOM 2006 (BMJ)- diabetic patients undergoing major surgery
All these studies demonstrated no benefit of beta blockade, rather increased bradycardia/hypotension in beta blocked group requiring treatment.
Criticism of these studies surrounds less than optimal HR control and short interval between initiation and surgery.

Before POISE, a meta-analysis estimated the pooled OR of cardiac events in favour of beta blockade (OR = 0.89, 055-1.43)

Most recent evidence
1. POISE (Lancet 2008)- 8351 patients, multicentre, blinded RCT, metoprolol vs placebo for 30 days, with or at risk for IHD. Lowered hazard ratio with metoprolol (0.83) in primary outcome (non-fatal MI, cardiac death, arrest) but this was due primarily to less MIs. Importantly, MORE patients died (HR = 1.33), and MORE strokes (HR = 2.17), of whom one half were permanently incapacitated or dependent on care.
For every 1000 patients treated as per POISE, there will be 15 less MIs, 7 less cases of new AF, and 3 less post-op CABGs, BUT 8 excess deaths, 5 strokes and 53 cases of significant hypotension requiring treatment.
Criticism regards the high dosing used (100mg pre-op, 200mg daily thereafter). This was based on dosing from the COMMIT trial (Lancet 2005), which appeared safe in interim analysis but was later found to have an increased risk of cardiogenic shock.

AHA/ACC 2009 guidelines
Class 1 (benefit > > > risk)
- beta blockers should be continued in patients undergoing surgery who are receiving them already

Class 2a (benefit > > risk)
- beta blockers titrated to HR and BP are reasonable for vascular surgery in patients:
- with known CAD or ischaemia on pre-op testing
- at high cardiac risk (>1 clinical risk factor)
- beta blockers titrated to HR and BP are reasonable for intermediate-risk surgery in patients with CAD or high cardiac risk (>1 clinical risk factor)

Class 2b (benefit > or = risk)
- beta blockers uncertain in either vascular or intermediate risk surgery in patients without CAD or with only 0-1 clinical risk factors

Class 3 (risk > benefit)
- routine administration of high dose beta blockers may be harmful
- do not give if absolute contraidications to beta blockade.
October 2006 Question 12

Discuss the role of ketamine in current anaesthesia practice.
Background
• Derivative of phencyclidine (‘angle dust’)
• Induces a state of dissociative anaesthesia with open eyes and laryngeal tone maintained
• Non-competitive antagonist of NMDA receptors
• Modulates activity opioid and muscarinic receptors, descending inhibitory monoaminergic pathways
• Increases sympathetic tone by central stimulation and inhibits uptake of noradrenaline peripherally, thus maintaining BP and CO
• Racemic mixture of R (-) and S (+) isomers (latter is more potent and less psychoactive)

Roles of ketamine
Anaesthesia
1. Induction and maintenance of anaesthesia in shocked patients e.g. hypovolaemia, cardiac tamponade, restrictive pericarditis
2. Induction of anaesthesia in acute, severe asthmatics (potent dilator)
3. Anaesthesia for mass casualties in the field
4. Remote anaesthesia (ie third world setting. Cheap, reliable, reduced requirement for advanced airway management).

IV induction 1-2 mg/kg. Onset 30-60 secs, duration 5-15 mins
IV maintenance with intermittent boluses of 0.25-0.5 mg/kg or infusion 0.5-3 mg/kg/hr
IM induction 5-10 mg/kg. Onset 3-5 mins (slower with smaller doses), duration 20-30 mins

Analgesia
1. For positioning of patients with painful limbs prior to a regional block (10 mg IV increments or 2 mg/kg IM)
2. Adjunct to acute postoperative pain management (0.3 – 0.5mg/kg loading dose intraop followed by infusion 0.1 mg/kg/hr).
3. Procedural analgesia eg. change of burns dressings
4. Patients with chronic pain – CRPS, neuropathic pain
5. Patients who are opioid dependent – action independent to opioid receptors
6. Caudal/epidural analgesia. Analgesic effects are mediated by spinal cord NMDA receptor antagonism and opioid mu receptor agonism. Concerns re neurotoxicity however (preservative free available)
7. Analgesia for mass casualties in the field (0.25-0.5 mg/kg IV, infusion 0.3-0.6 mg/kg/hr)
8. Preventive analgesia eg. reduction in phantom limb pain if started intraoperatively

Sedation
1. Premedication in combative and uncooperative children e.g. to facilitate IV cannulation (2-3 mg/kg IM)
2. For short procedures in children e.g. plaster change, burns dressings, laceration repair (0.25-0.5 mg IV)
3. ICU sedation in severe, unresponsive asthma (0.5-3 mg/kg/hr infusion)

Co-administering an anti-sialogogue may be required in children (e.g. glycopyrrolate 2-5 mcg/kg IM or IV, atropine 5-20 mcg/kg IM or IV)

Side effects
- dysphoria, unpleasant hallucinations (dose-dependent, can reduce with midazolam)
- increase in bp, hr
- effect on BIS is unpredictable; not validated
- increases CBF and metabolism hence contraindicated in patients with raised ICP and open eye injury
October 2006 Question 13

List the risks associated with the placement of a central venous catheter? Discuss the ways in which these risks may be modified.
Early risks

1. Mechanical e.g. arterial puncture, haematoma, pneumothorax/haemothorax/chylothorax
• Pneumothorax is more commonly associated with subclavian or low anterior IJV approaches. Haemothorax is associated primarily with the subclavian approach and occurs secondary to subclavian artery laceration. Chylothorax is associated with left IJV approaches and occurs secondary to laceration or puncture of the thoracic duct (can reach as high as 3-4 cm above the sternal end of the clavicle as it wraps around the IJV)
• Use of ultrasound guidance to identify the vein reduces the risks of mechanical complications compared with the standard landmark technique. This method is not applicable to the subclavian approach as the clavicle is in the way
• Apply direct pressure where arterial puncture has occurred
• If in doubt whether artery is punctured, take a blood gas sample and compare with ABG results from a peripheral artery. Alternatively, insert a single lumen tube without dilating the vessel and connect to pressure transducer
• Avoid the subclavian approach in coagulopathic or hypoxaemic patients
• Avoid the IJV approach in markedly obese patients or else use ultrasound guidance

2. Arrhythmias
• ECG monitoring is required during insertion, if ectopics, then withdraw wire
• After insertion, catheter position must be checked by X-Ray. The catheter tip should ideally lie in the SVC above the pericardial reflection. Radiologically, this correlates with the right tracheobronchial angle and carina

3. Cardiac perforation/tamponade
• Occurs due to erosion of the catheter tip through the SVC and into the pericardial sac
• Check catheter position by X-Ray

4. Air embolism
• Patients should be in the head-down position for jugular and subclavian cannulation until the catheter is inserted and the hub of the catheter is occluded
• All lumens should be flushed with saline prior to insertion, or alternatively after insertion, aspirate on each lumen first before use

5. Shearing and embolisation of the guidewire
• If the guidewire cannot be advanced through an introducer needle, remove the needle and wire in unison

6. Needle-stick injury to the operator
• Dispose of sharps after use

7. Nerve injury e.g. phrenic, recurrent laryngeal, Horner’s syndrome
• Use ultrasound guidance for IJV approaches

8. Anaphylaxis with antibiotic or chlorhexidine impregnated lines
• Shown to reduce catheter-related sepsis from 7.6 / 1000 catheter days to 1.6 / 1000 catheter days
• Avoid if patient has antibiotic allergy


Late risks

1. Thrombosis
• Higher rates of catheter-related thrombosis with femoral lines (20%) cf. IJV (8%) and subclavian (2%). But high catheter tip placements in the SVC may increase the risk of thrombosis with IJV and subclavian lines
• Avoid femoral approach if anything more than short term requirement for CVC is necessary

2. Catheter-related sepsis
• Insert under aseptic conditions i.e. adequate hand-washing, skin disinfection (Alcohol chlorhexidine is superior to povidone iodine)
• Use antibiotic impregnated lines
• Daily inspection of catheter site
• Remove CVC as soon as it is not required
• Replace after 5-7 days as the risk of catheter colonization increases after this period
• Avoid guidewire exchange as this increases the risk of infection
• Avoid femoral approach if there is gross contamination of the femoral region

3. Disconnection leading to bleeding or air embolism
• Hubs should be occluded at all times
• During removal of IJV or subclavian lines, the patient should be in a head down position and in expiration to raise the venous pressure at the point of removal, thereby preventing air aspirating into the vein through the evacuate catheter tract. Then apply external pressure until clot formation has sealed the vessel

4. Arteriovenous fistula

5. Superior vena caval erosion leading to hydrothorax, cardiac tamponade
• Check catheter position daily
October 2006 Question 14

Critically evaluate the role of recombinant factor VIIa in blood loss requiring massive transfusion in the trauma patient.
Background
• Recombinant factor VIIa (rFVIIa) is currently licensed for use in haemophiliacs with antibodies to factor VIII or factor IX
• At present rFVIIa is not licensed in any country for use in trauma and haemorrhage, and its use is purely on a ‘compassionate’ basis
• Given its high cost, it should be restricted to those situations where it is likely to be of maximum benefit
• It works by increasing formation of FX (TF-VIIa complex converts X to Xa), which initiates conversion of prothrombin into thrombin leading to the formation of a haemostatic plug by converting fibrinogen into fibrin. There are 2 mechanisms of action: a) binding to tissue factor (TF) at the site of endothelial damage; this TF:VIIa complex activates factors IX and X; and b) ‘thrombin burst’ reaction where FIX and FX on platelets are directly activated; FIX also activates FX amplifying this pathway dramatically
• Theoretically, both of these mechanisms localize the action of FVIIa to the site of injury, and hence avoid the complication of thromboses in other vascular beds

Pro arguments
1. A good theoretical basis for its usage
2. Encouraging results from case reports and case series
3. A recent multicentre randomized doubled-blinded placebo-controlled study by Boffard et al. showed a reduction in RBC transfusion requirements and reduced incidence of ARDS in all patients with blunt trauma (but no significant difference was seen in penetrating trauma)
4. In studies where no benefit was demonstrated, rFVIIa may have been given too late as salvage therapy, or haemorrhage was controlled at the time of drug dosing
5. It is quick and easy to use, with no storage problems
6. It may avoid many of the problems of continuing transfusion e.g. hypothermia, acid/base disturbance, volume overload etc.
7. It does not carry a risk of infectious diseases transmission
8. It may be of use in Jehovah Witness patients

Con arguments
1. No evidence for its use
• Although blood transfusion requirements are reduced in certain indications, there is currently no evidence that this effect translates to improvement in morbidity and mortality
2. Possibly a publication bias, as there is a tendency not to report bad outcomes
3. Off-license product, thus problems of consent, paediatric use? etc.
4. Cost
• 1.2 mg vial $1195
• 2.4 mg vial $2391
• 4.8 mg vial $4783
5. Availability
6. Agreement on usage between interested parties such as haematology, anaesthesia
7. Side Effects?
• Current data shows it is generally well tolerated with a good safety profile
• But rFVIIa leads to a hypercoagulable state which theoretically increases the risk of DVT/PE, AMI, stroke
• It can also lead to microvascular coagulation with increased risk of ALI/ARDS, acute renal failure and multiple organ dysfunction syndrome (MODS)
8. No real agreed protocol for its use
• There are no clear guidance as to which patients are suitable for treatment, the appropriate timing or dosage

Practical issues with the use of rFVIIa
1. Each hospital should have a set of guidelines in place for the use of rFVIIa, and its use should be closely monitored and submitted to a national or international registry
2. In general, it should only be considered in massively transfused patients with life threatening bleeding despite:
• Appropriate blood component transfusion (FFP, platelets, cryoprecipitate). All these blood components need to be present for full effect
• Pharmacological measures (DDAVP, anti-fibrinolytic agents e.g. aprotinin)
• Correction of hypothermia and hypocalcaemia
3. rFVIIa will not stop surgical haemorrhage
4. rFVIIa should not be given instead of other blood product administration
5. rFVIIa should not be used too early, but neither should it be used only after ‘super-massive’ transfusions of 40 to 60 units. Therapy at between 8 and 20 units is probably appropriate (opinion only)

Summary opinion on the role of rFVIIa in trauma
1. In patients with blunt trauma, an initial dose of 0.1 mg/kg rFVIIa, repeated at 1-2 hourly intervals if necessary 1 mg/kg, may reduce RBC transfusion requirements, the need for massive transfusion and the incidence of ARDS
2. In patients with penetrating trauma, the effects of rFVIIa are uncertain, and no recommendations can be made for this indication
October 2006 Question 15

Discuss the usefulness of the ASA grading as a measure of perioperative risk.
Background
• The ASA classification is used world-wide by anesthesia providers to assess the preoperative physical state of patients
• This score is used in many areas of anesthesia including administrative policy-making, performance evaluations, resource allocation, research and reimbursement of anesthesia services
• The authors of the system clearly stated that it was not an estimate of perioperative risk, but rather its intention was to give practitioners a common language in referring to the severity of systemic disease in various patients
• Despite this, it has long been used to assess perioperative risk and several studies have demonstrated a correlation between ASA classification and postperative adverse outcomes e.g. the 30-day risk of mortality almost doubles with each higher ASA status level (MJA article below)

The ASA Physical Status classification system

Class I: A normal healthy patient without organic, physiologic or psychiatric disturbance
Mortality 0.05%
Class II: A patient with mild systemic disease that results in no functional limitations
e.g. controlled hypertension or diabetes without systemic effects, cigarette smoking without COAD, mild obesity, extremes of age (< 1 or > 70 yrs old), pregnancy
Mortality 0.4%
Class III: A patient with severe systemic disease which results in functional limitations, but is not incapacitating
e.g. stable angina, previous AMI, controlled CCF, morbid obesity, pulmonary disease that limits activity, CRF
Mortality 4.5%
Class IV: A patient with severe systemic disease that is a constant threat to life
e.g. unstable angina or CCF, advanced pulmonary/hepatic/renal dysfunction
Mortality 25%
Class V: A moribund patient who is not expected to survive 24 hrs with or without operation
e.g. multiorgan failure, sepsis syndrome with haemodynamic instability, ruptured AAA, PE, head injury with increased ICP
Mortality 50%
Class VI: A declared brain-dead patient undergoing anaesthesia for the purposes of organ donation

Class E: Added to any patient for emergency operation

Positive aspects
1. Most widely used preoperative risk assessment scheme. No other schemes have achieved the same widespread use
2. Simple to use (compared with eg. APACHE II)
3. Can be applied to every patient
4. A measure of the overall severity of systemic disease and represents an estimate of the patient’s physiological status without the need for clinical resources
5. Takes into account emergency surgery which is an important predictor of perioperative mortality
6. In USA, anaesthetists’ renumeration depends on ASA coding – widely used and applied


Negative aspects
1. Assignment to ASA classes is subjective and may be inconsistent between anaesthetists. Thus, the same patient may be assigned to different categories depending on who performs the assessment
2. Does not take into account the complexity of the surgery (eg. 1E multitrauma)
3. There is no differentiation between a systemic disease that leads to operation and one that is an incidental chronic finding
4. Does not take into account multiple coexisting diseases of varying severity
5. Unclear definition of emergency (not all non-elective surgery is emergency surgery)
6. Does not take into account extremes of age (eg neonates/elderly) nor pregnancy.
May 2006 Question 5

A 50 year old man taking corticosteriods and pyridostigmine for myasthenia gravis is to have a n elective right hemicolectomy under general anaesthesia.

Discuss your management of his myasthenia pre and post operatively.
Myasthenia Gravis- an autoimmune, progressive neuromuscular disease characterised by circulating antibodies that either destroy or block transmission at cholinergic receptors on the post-synaptic neuromuscular junction. Clinical picture is of fluctuating muscular weakness and fatiguability.

Major anaesthetic considerations:
1. Risk of regurgitation/aspiration
2. Reduced need for muscle relaxants
3. Respiratory dysfunction
4. Timing/location of extubation
5. Ongoing need to ensure adequate muscle function

Pre-operative

Assessment of myasthenia:
- dosage of pyridostigmine – including what happens if he misses a dose
- previous plasmapheresis (if recent, can decrease esterase levels thus prolonging effect of mivacurium, sux, ester-linked LAs)
- degree of weakness; ? bulbar palsy (aspiration risk)
- recent RFTs (particularly FVC < 2.9 L)
- Previous GAs → ?post-op ventilation required, if so, duration
- Other autoimmune disease ? thyrotoxicosis/SLE/rheumatoid arthritis
- recent exacerbation? (pregnancy/surgery/stress/infection)

Leventhal criteria – predicts requirement for post-op ventilation
- duration of disease > 6 years
- co-existing respiratory disease
- FVC <2.9L
- Dose requirements >750 mg/day less than 48 hrs before surgery
- Recent history of dysphagia

This is major body cavity surgery, thus post-op ventilation will be highly likely required. As this is an elective case, an ICU bed should be booked and case should not proceed unless the bed is available
Management
- corticosteroid and pyridostigmine should be continued until and including day of surgery
- consider optimising prior (plasmapheresis)
- minimal/no sedation → risk of aspiration higher in bulbar palsy
- RSI or consider awake technique. If sux is used – need higher dose (1.5 mg/kg) as decreased sensitivity of receptors. However, higher risk of Phase 2 block as high dose and low esterase levels if plasmapheresed
- patients have increased sensitivity for NDMRs – use smaller doses (around 1/10th) and titrate according to nerve stimulator. Choose drugs which are short acting and not dependent on enzyme metabolism e.g. cisatracurium is good choice
- regional analgesia could be considered – may be helpful in maintaining respiratory function but immunosuppressed
- it may be better to be slightly myasthenic than cholinergic, as being cholinergic may cause increased vagal reflexes, bowel anastomosis disruption, and prolonged sux duration (due to excess anticholinesterase)

Post-operative
- would most probably remain ventilated in ICU
- ideally avoid muscle relaxant reversal if possible (increased risk of cholinergic crisis). Max reversal dose depends on daily pyridostigmine conversion)
- (may consider extubation if:
o short case (less than 1 hour)
o no muscle relaxants used
o pre-op mild disease only
o no fade on nerve stimulator, good TV, sustained head lift >5 s
o analgesia controlled
o oral meds not likely to be missed
o can extubate in OR, but I will still monitor in HDU for at least 24-48 hours)
o consider ABG prior to extubation ?Type 2 resp failure
- since patient would most likely be kept NBM – change pyridostigmine dose to neostigmine IV/IM/SC– 30 mg pyridostigmine = 1 mg neostigmine
- steroids would need to be replaced intra- and post-op (HPA axis would be suppressed)
- as soon as bowel function returns → oral pyridostigmine. NGT may be required
- post-extubation requires monitoring – HDU setting appropriate
- neurologist input useful
- monitor for cholinergic crisis (excess anticholinesterase- bradycardia, sweating, salivation, miosis, bronchoconstriction, muscle weakness. Rx with atropine)
May 2006 Question 6

Describe your immediate assessment and management of the airway in a patient with smoke inhalation injury.
History
• Location of burn – involving face / trunk / peripheries
• Current airway – may already be intubated/guedel airway by paramedics
• Burn factors
o Explosion = index of severity of smoke inhalation injury
o Enclosed space = more likely potential for smoke inhalation
o What was burning = chemical inhalation, irritants
• Smoke inhalation injury – toxic effects of products, chemical pneumonitis
• Superheated air or steam – airway oedema
• Indirect injury – may develop SIRS, ARDS
• Patient factors
o GCS at the time
o Concurrent alcohol or drug use
o Relevant co-morbidities
• Evidence of carbon monoxide poisoning
o Neurologic impairment
o Chest pain
o Inability to assess adequately
o Loss of consciousness at scene

Examination
• Conscious state
o Awake or unconscious
o If unconscious, beware carbon monoxide poisoning
o Neurologic impairment (potential head injury or carbon monoxide poisoning)
• Assessment of airway burns
o Facial involvement
o Singed nasal hairs
o Hoarse voice
o Stridor
o Carbonaceous sputum
o Productive cough
o Obstructive respiratory pattern – concern about airway obstruction
o Evidene of respiratory distress, work of breathing
• Nasal endoscopy
o Degree of oedema of airway
o Carbonaceous sputum
• Development of acute respiratory distress
o Pulse oximetry may be normal
o Pulse oximetry overestimates true PaO2 in carbon monoxide poisoning
• Markers of severe burns
o Facial and circumferential neck burns
o Full thickness burns

Investigations
(1) ECG
(2) Arterial blood gas
a. Co-oximetry to determine % oxyhaemoglobin
b. PaO2 will be normal
c. Need carbon monoxide co-oximetry
d. But no correlation between COHb level and severity of symptoms
(3) Nasoendoscopy as above
(4) CXR – if respiratory failure and distress, evolving ARDS

Management
(1) Depends on severity of smoke inhalation
(2) All should receive 100% O2
• 100% O2 reduces t1/2 COHb from 4 hours to 45 minutes
(3) Criteria for intubation
• Needing transfer
• Facial and neck circumferential burns
• Respiratory distress
• Reduced conscious state
• Requiring early surgery
• Consider in major burns with associated trauma
• If any doubt- conservative early approach is to intubate
(4) Technique- either RSI or fibreoptic awake intubation
• Presence of ENT surgeon for tracheostomy may be advisable
• Topicalise airway with nebulised lignocaine/adrenaline
• Remifentanil sedation to decrease coughing
• Pass largest ETT possible
• Remember C spine protection if not cleared
(5) If decision to not intubate- monitor in HDU/ICU environment
(6) No evidence that hyperbaric O2 reduces morbidity
• Difficult to monitor patient
• No significant benefit
• Delays neurologic sequelae from carbon monoxide poisoning
• Indicated if neurologic signs already
May 2006 Question 7

A 75 year old man having a TURP under spinal anaesthesia which has been uneventfu, become restless 70 minutes into the procedure. He has 2 milligrams of midazalam at the start of the case and no further sedation.

Describe your assessment and management of this problem
Assessment:
− Can this case proceed? - is he trashing around? Needs early decision on whether 1. to stop case or 2. GA and continue 3. Sedation and continue
− 70 minutes is a long time for TURP (usually <60 minutes). Other risk factors for TURP syndrome- large prostate, open prostatic veins, excessive height of irrigation fluids
− systematic ruling out of causes
− COVER ABCD A SWIFT CHECK
− C – circulation, colour
− check SPO2, BP, ECG and pulse -change to 5 lead if suspect myocardial ischaemia
− O – Oxygen
− Give supplementary O2 via Hudson mask
− V – ventilation
− quick auscultation of chest. ?oedema
− E – ETT, elimination of machine, endocrine (check sugars)
− R- review monitors, equipment
− ABC
− D- drugs
− wrong drugs infused
− irrigation solution --> is this TURP syndrome

Possible causes:
- TURP syndrome
− hypoxia → Give oxygen
− pulmonary oedema
− PE
− hypoventilation
− profound anaemia
− hypotension → Vasopressors
− infarct – myocardial, CVA → ECG, increase peripheral oxygen delivery
− drugs -2 mg midazolam is unlikely to cause confusion 70 mins after
− glycine toxicity possible if used in irrigation → causes blindness, confusion
− endocrine
− hypoglycaemia
− hypercalcaemia
− hyponatraemia/hypoosmolality (it is the osmolality which causes confusion from cerebral oedema)
- receding block- pain
- patient discomfort/boredom/cold

Management- directed at cause

(1) Immediate management
• Call for help – 2nd anaesthetic assistant
• Apply O2 by nasal prongs or facemask
• Treat hypotension with boluses of vasopressors
• If pulmonary oedema – GA, intubate and PPV with O2
• Draw sample of blood for urgent serum Na analysis, ABG
(2) Monitoring
• Consider insertion of arterial line
• Insert urinary catheter to monitor urine output
• Estimate amount of fluid irrigated and absorbed
Volume absorbed= (preoperative [Na]/postoperative [Na] x ECF)-ECF
(3) Investigations
• Urgent Na analysis
• ABG
• ECG monitoring – for evidence of LV strain, ischaemia
• May consider CVC insertion when more stable to guide fluid administration
(4) Correction of hyponatraemia and hypoosmolality
• Frusemide 10-20mg IV
• Slow correctIon- not more than 1 mmol/L/hr (may correct more aggresively till symptom resolution then slow down
• If NA < 120mmol/L, consider hypertonic saline 3%
• Hypertonic saline can be considered but SHOULD NOT exceed 100mLs/hr. May worsen fluid overload by drawing fluid into intravascular space
• Maximum rise of Na should not exceed 12mmol/L over 24 hours
(5) Correction of seizures from cerebral oedema
• Midazolam 1-2mg IV immediately
• Probably require intubation
(6) Correct hypoglycaemia if evident
(7) Postoperative destination- likely HDU/ICU. If intubated, extubate when awake and cooperative
(8) Hyperammoniaemia- from glycine toxicity. Can treat with L-arginine, however the significance of ammonia in TURP syndrome is unknown

Preventing TURP syndrome
• Limit resection time to <60 mins
• Irrigation pressure < 40cmH2O
• Experienced operator
• Appropriate irrigation fluid (1.5% glycine)
• Limit volumes of irrigant used 1-1.5L
• Never place irrigation fluid bag >100cm high
• Anticipate if prostate size > 50g
May 2006 Question 8

The first patient on your orthopedic list tomorrow is schedules for left total hip replacement. He has an implanted (permanent) cardiac pacemaker.

Discuss the relevant factors in your pre-anaesthetic assessment of this patient.
This is a patient with cardiac pacemaker having elective orthopaedic surgery where there is moderate risk of haemodynamic stability.

History
- reason for cardiac pacemaker (eg sick sinus, CHB, biventricular failure)
- other relevant history (eg. symptoms of/known IHD, CCF)
- type of pacemaker – brand, type, defibrillator as well?
- Dependency on pacemaker- previous admissions, symptoms of pacemaker failure (eg. syncope, palpitations)
- Prior GAs- any pacemaker complications
- Date of last pacemaker check (check pacemaker booklet)
- Consider calling pacemaker company to get perioperative recommendations, default mode if magnet placed on pacemaker, behaviour with diathermy (many of the newer pacemakers will filter diathermy current)
Examination
- murmurs/cardiac enlargement
- determine location of pacing box (usually subclavicular)
Investigations
- ECG- underlying rhythm, pacing capture? (ie. pacing spike followed by a QRS complex)
- possibly CXR- number of leads, lead fracture if doubt about operation

If possible get technicians in to interrogate pacemaker, and reprogram to non-sensing rate (VVO). Patient may have to be moved further down the list to facilitate this. If pacemaker has not been checked for last 6 months and technicians not available for that day, I would cancel the patient – this is an elective procedure

Diathermy will be required – if and where possible use bipolar diathermy
If unipolar limit to short bursts only on lowest energy level
Ensure plate is placed such that current flow does not go near pacemaker
Ensure that defibrillator with external pacing capabilities is available
Patient should have continuous ECG monitoring and also strongly consider an arterial line – long case, potential for blood loss, ability to monitor output at all times (pacemaker may not capture). Also if pacemaker is at fixed rate, reductions in preload cannot be compensated by increase in heart rate → continuous BP monitoring is even more crucial
Post-op the pacemaker will need to be interrogated by technician again, and settings restored in recovery if they were changed.
May 2006 Question 9.

Describe the clinical features and management of bupivicaine toxicity
Bupivicaine toxicity is more common than other agents due to:
-High potency
-Lower CC:CNS ratio (pronounced in partuents)

Local Anaesthetic toxicity is predisposed to by:
-Error in dosing resulting in overdose
-Direct intravascular injection or into a vascular area
-Aspiration to confirm placement before each injection, inject slowly
-Cumulative doses administered or continuous infusion
-Concurrent acidosis, hypercarbia and hypoxia (all potentiate)
Bupivicaine maximum recommended dose 2mg/kg with or without vasoconstrictor

Mild toxicity
• Perioral paraesthesia
• Metallic taste
• Tinnitus
• Visual disturbance
• Slurred speech

Signs of severe toxicity:
Firstly CNS
• Sudden loss of consciousness
• +/- tonic-clonic convulsion
then Cardiovascular collapse
• sinus bradycardia & conduction blocks
• asystole
• ventricular tachyarrhythmias

NB: Local anaesthetic (LA) toxicity may occur some time after the initial injection
Immediate management:
1. Remove precipitant
• If you are still injecting local... STOP NOW!
2.Call for help
• Youʼll need intralipid - where is it kept? ICU? Regional trolley? Crash Cart?
3. Airway
• Maintain the airway +/- ETT
4. Breathing
• Give 100% oxygen and ensure adequate lung ventilation (hyperventilation may help
by increasing pH in the presence of metabolic acidosis)
5. Circulation
• Confirm or establish intravenous access
• Start CPR if cardiac arrest
6. Seizure termination
• administer benzodiazepine, thiopental or propofol in small incremental doses
• remember: ETT, O2, CPR and seizure treatment will reduce the toxicity of LA.
" " " (by reducing acidosis, hypercarbia and hypoxia)
Management of cardiac arrest associated with LA injection:
• Start CPR & treat arrhythmias with amiodarone (Likely to be refractory)
• Prolonged resuscitation may be necessary; consider other options:
• Consider the use of cardiopulmonary bypass if available
• Consider treatment with lipid emulsion (red brackets 70 kg patient dose)
• IV bolus Intralipid® 20% 1.5 ml.kg-1 over 1 min (Give a bolus of 100 ml)
• then IV infusion at 0.25 ml.kg-1.min-1(Give at a rate of 400 ml over 20 min)
• Repeat bolus twice at 5 min intervals if circulation has not been restored
(Give two further boluses of 100 ml at 5 min intervals)
• After another 5 min, increase the rate to 0.5 ml.kg-1.min-1 if circulation
has not been restored (Give at a rate of 400 ml over 10 min)
• Continue infusion until a stable and adequate circulation has been
restored (a further 10 minutes at least)
• Recommended max dose is 10 ml/kg over 30 minutes
Remember:
• Continue CPR throughout treatment with lipid emulsion
• Recovery from LA-induced cardiac arrest may take >1 h
• Propofol is not a suitable substitute for Intralipid (sig cardiovascular depression) in
small doses it may have beneficial effects on seizure termination but the intralipid
contained is small compared with what is required
• Replace your supply of Intralipid® 20% after use
• Intralipid® 20% has been shown to reverse LA-induced cardiac arrest in animal
models and in human case reports, and has been reported in the treatment of life-
threatening toxicity without cardiac arrest.
• Avoid vasopressin, calcium channel blockers, β-blockers, or LA
May 2006 Question 10

Describe the anatomy of the orbit relevant to a peribulbar eye block.
The orbit is a cavity
- truncated square pyramid shape, apex posterior, and its base the anterior aperture.
- 40-50 mm deep with Volume 30 mL (7 mL globe)
40-50MM DEEP. 30ML VOLUME. 7ML GLOBE.
- mainly filled by adipose tissue, and the globe is suspended in its anterior part.
- only enough room ~10 mL max of local anaesthetic solution before inc IOL
- 5 mL 0.5% bupivicaine, 2 mL 2% lignocaine, 2 mL dilute hyalase (10-30 units/mL)

The globe
- sits high and lateral in the orbit
- hence a low lateral approach for peribulbar block to avoid globe perforation
- medial approaches (including medial canthus peribulbar top-up) safe too.
- usually 25 mm diameter
- use 25 mm needle so as NOT to pass the equator or enter the cone
- if > 26 mm Axial diameter - increased staphyloma (Peribulbar Xindcated)
- is a fixed structure in a bony orbit - so volume in orbit may increase IOP
- a honan’s balloon useful for dec IOP (reduces blood and aqueous in the eye)

The cone
- The 4 rectus muscles of the eye insert anteriorly near the equator of the globe.
- Posteriorly, they insert together at the apex on the Zinn tendinous annulus, through which
the optic nerve enters the orbit.
- The 4 rectus muscles make up the cone, which is not sealed by any intermuscular
membrane.
- Peribulbar block relies on diffusion between the recti to work
- The cone contains
- optic, sensory nerves (direct connection to CNS ?infection - peribulbar xindicated)
- ciliarly ganglion (sensory, symps from carotid plexus and parasymps CN III)
- retinal artery and vein
- Inadvertant entry to the cone can increase complications
- Retrobulbar haemorrhage (INR > 2, peribulbar X indicated)
- Intravascular injection
- needle insertion perpendicular to avoid intraconal, globe perf
Sensory innervation:
- Globe - ophthalmic nerve (CN V1), which passes through the muscular cone also
supplies Sclera/cornea, upper eyelid and upper conunctiva
- Lower eyelid and inferior conjunctiva - CN V2 (extraconal)
- A single shot peribulbar may not block all of these nerves (medial top-up)
Motor innervation
- Trochlear nerve (CN IV) superior oblique
- abducens nerve (CN VI) lateral rectus
- oculomotor nerve (CN III) to all other extraocular muscles. All but the trochlear nerve
pass through the muscular conus.
- Therefore, injecting local anesthetics inside the cone is logically expected to
provide anesthesia and akinesia of the globe and of the extraocular muscles.
- As a peribulbar block is not intra-conal - akinesia cannot be guaranteed.
- Superior oblique - innervation being extra conal may also be missed.
- Only the motor nerve to the orbicularis muscle of the eyelids has an extraorbital course,
coming from the superior branch of the facial nerve (VII).
- Orbicularis occuli paralysis requires facial nerve block
May 2008 Question 11

Discuss the principles underlying the management of a GA for CEA
Carotid endartrectomy
-Indication
-Reduces stroke / CVA in symptomatic patients with >70% stenosis
-Mortality / Major morbidity
-2% combined death (AMI/CVA)
-CVA: 2% if no past history of stroke, 5% if past history

Pre-operative assessment
-Usually elderly with comorbidities
-HTN needs to be controlled pre-op
-Obtaining baseline BP important for intra-op goals
-PVD, IHD, DM, renal impairment all common
-Neurology
-Pre-existing deficits documented - intra/post op assessment
-Contraindications for LA (therefore GA indicated)
-Can patient sit still in recumbent position?
-Can the patient tolerate ipsilateral phrenic nerve palsy?

Intra-operative
-2 x big IV access & arterial line (contralateral arm) Vasoactives ready (up & down)
-potential for sudden blood loss, hypotension and deterioration
-Clamping can cause BP to swing wildly
-5 lead ECG
-potential for intra-op events
-Monitoring during cross-clamping
-EEG, evoked potentials, stump pressure, MCA doppler, near-IR spectroscopy
-GA
-Gentle induction
-short acting opioids/lignocaine to cords to blunt hypertensive response.
-ETT
-LMA may reduce carotid perfusion
-Remote airway once surgery starts (secure connections)
-isoflurane good side effect profile
-Normocarbia
-keep ICP static, reduce vasodilatory effects and metabolic demands
-Avoid coughing on extubation
-ICP, patency of graft anastomosis
-consider remifentanil
-Blocks
-C2-4 +/- trigeminal nerve (traction) +/- carotid sheath and supplementation
-prepared/motivated patient
-monitor: neurology dictates shunt reqt (LOC, dysphasia, confusion, slow answers)
-15 mL 0.5% bupivicaine at C3 or C2-4 (split evenly)
-remifentanil & propofol TCI can be helpful for instituting the block
-Convert to GA 2.5% (LMA)

Post-operative
-Prolonged recovery/HDU post op - Allows rapid identification & management of
-neurological deficit +/- reoperation
-cervical haematoma (5-10%) - remove stitches asap
-haemodynamic instability
-Clear haemodynamic parameters (ie report systolic > 160 or < 100)
May 2008 Question12

List the causes of acute atrial fibrillation in the perioperative period. Describe your management of acute atrial fibrillation which occurs in the PACU in a patient who has had a total hip replacement.
Causes of Atrial Fibrillation:
-Structural heart diseae
-Hypertension
-Myocardial Ischaemia
-Pericarditis, Mediastinitis
-Thoracic/oesophageal surgery
-Mitral Valve disease
-Electrolyte disturbance
-Hypokalaemia, Hypomagnesaemia
-Sepsis
-Thyrotoxicosis
-Alcohol

Implications:
-Decreased cardiac output due to lower end diastolic volume
-atrial thrombus may occur resulting embolic events (May end up on warfarin)

Management:
-Assess ABC
-Administer oxygen via hudson mask, Confirm IV access
-Rule out AMI, hypotension
-History
-chest tightness / SOB / dizzyness
-previous AF / IHD / EtOH / sepsis / pain
-Examination
-BP / Chest / occult blood loss / diaphoresis / pallor / lines
-Initial management
-Treat electrolytes
-KCl 10-20 mmol/hr to K+ 4-4.5, Mg 10 mmol over 30 minutes
-Serial 12 lead ECG/Troponin
-CXR (?APO)
-Team approach Cardiology / Orthopods (+/- Intensive care) to liase and assess
-Rhythm & Rate control
-Rate control priority
-Beta blockade - may be appropriate if no LV impairment
-X if thyrotoxic or concurrent Ca channel blockers
-Digoxin - wont revert but will slow
-X if hyperkalaemic
-Rhythm control
-Amiodarone if haemodynamicaly unstable or LV impairment
-X if stable
-DC cardioversion
-If haemodynamically unstable
-X if atrial fibrillation > 48 hours
-May require anticoagulation if > 48 hours
-Liase with surgical team & cardiology when and what anti-coagulation would be appropriate, as well as follow up if not reverted.

ARITHMATIC Alcohol, Rh fever, Ischaemia, Thyrotoxicosis, Hypertension/Hypoxia/Hypercarbia, Mitral stenosis / MI / Myxoma (atrial) ASD, Toxins, Idiopathic/Infection (sepsis) Cardiomyopathy/Constrictive pericarditis. Electrolyte disturbance.
May 2008 Question 13

What is the physiological basis of preoxygenation?

Describe your method of preoxygenation including how you assess its adequecy.
Preoxygenation – the filling of the Functional Residual Capacity (FRC) and O2 stores of the lungs with 100% oxygen pre-induction; effectively replacing nitrogen with O2.

The FRC is the volume of gas which remains in the lungs at the end of a normal expiration.
The functions of the FRC are:
1) an O2 store and buffer to maintain a steady arterial pO2
2) prevent atelectasis
3) minimize work of breathing and minimize pulmonary vascular resistance
4) minimize V/Q mismatch
5) keep airways resistance low

Preoxygenation increases the oxygen stores of the lungs helping to prevent hypoxaemia whilst the patient is apnoeic post induction before an airway is secured and effective ventilation commenced. It is most commonly performed prior to a RSI.

FRC is 30 mls/kg in an adult in the supine position (approx 2 litres). If replaced with O2, maximal PO2 of 660 mmHg, which is about 87% O2 in the alveolus. It is the only oxygen store in the body which can be significantly increased rapidly using the technique of preoxygenation with 100% oxygen. Breathing 100% oxygen from a close fitting mask will denitrogenate the lungs and increased the oxygen store to about 1800mls. Normal basal O2 consumption is 250 mls/min, therefore the store can last about 7 mins.
Preoxygenation will also increase O2 stores minimally in dissolved O2.

Factors which will decrease this period of oxygen stores:
- decrease in FRC – supine position, obesity, muscle paralysis, anaesthesia, pregnancy, pulmonary disease causing increased elastic recoil of the lungs
- Increasing age leads to an increase in closing volume and closing capacity. Closing capacity (CC) is the lung volume at which the small airways first start to close, impairing gas exchange. CC is equal to FRC at 66 years (erect position) or 44 years (supine).
- increased O2 consumption – pregnancy, sepsis, hyperthermia

Method of preoxygenation – ensure tight seal of face mask
- tidal volume breathing for 3-5 mins
- 4 deep breaths in 0.5 min
- Eight deep breaths in 1 min

Adequacy
- clinical
o ETO2 about 90% (above 80% needed for RSI)
o ETN2 less than 5%
o Duration of pre-oxygenation
o Tight seal on mask – check by listening for leaks and looking at capnograph / bag movement
- PaO2
o Should be about 600 mmHg
o Assess saturations

- rate of decline of SpO2 post-induction
May 2006 Question 14

Discuss the role of desflurane in current anaesthetic practice.
Desflurane
Pharmacokinetics
- halogenated volatile agent
- low boiling point, extremely volatile, high saturated vapour pressure 88.5kPa at 20 degrees Celsius
o requires specific pressurized and heated vaporizer which heats desflurane to 39 C to maintain a constant vapour pressure
- high MAC (6.6%)
- oil/gas solubility coefficient 18.7
- low blood gas solubility coefficient 0.42
o rapid onset and induction of anaesthesia
o rapid offset, despite prolonged duration of administration
- Minimal hepatic metabolism (0.02%) to trifluoroacetic acid, can trigger autoimmune hepatitis (like halothane but less likely with desflurane as minimal metabolism). Trace quantities of which are excreted renally.
- may react with CO2 absorber to form CO (> sevoflurane/isoflurane) especially with high flows (dry dessicated soda lime) and more with Baralyme.
Pharmacodynamics
- CVS – a decrease in myocardial contractility but sympathetic tone is relatively preserved. Rapid alterations in desflurane concentrations may cause transient rise in catecholamine levels.
- Resp – respiratory depressant, causes dose dependent decrease in TV and increase RR. Irritant to the respiratory tract / pungent smell.
- CNS – causes cerebral vasodilatation like other volatiles, decreases CMRO2.

Advantages
- main advantage of desflurane is its low solubility in blood (approaches N2O) therefore has rapid onset and more importantly rapid offset
o day cases
o affords titratability
o neurosurgery cases where rapid assessment of neurological function required post GA
o prolonged cases, where sevoflurane and isoflurane would have accumulated in the tissues

- little hepatic metabolism
o good for hepatic failure patients

- not epileptogenic
o c/f sevoflurane and enflurane

- less arrhythmic potential / sympathetic tone relatively preserved
o c/f halothane


Disadvantages
- CO production
o Especially in dry Baralyme absorbers (first case in morning), high temperature, high flows

- needs special vaporizer
o which needs power supply
o expense
o not atmospheric pressure regulated
o don’t use during transport

- pungent
o unsuitable for spontaneously ventilating GA → produces bronchospasm, breath holding
o avoid in brittle asthmatics

- as with other volatiles
o avoid in ↑ ICP – uncouples metabolic autoregulation
o may trigger malignant hyperthermia
o potentiates the action of NDMRs

Desflurane is a volatile anaesthetic agent that is characterized by its high insolubility in blood enabling a rapid induction and more importantly emergence from anaesthesia giving it a vital role in current anaesthesia practice.
May 2006 Question 15

Discuss the elements you consider important when obtaining consent for epidrual analgesia in labor.
Epidural Analgesia

It is important to obtain informed consent for epidural analgesia. Informed consent requires a competent patient, provision of adequate information in a way that the patient understands as well as alternative options available. Finally the patient must be allowed to make a voluntary decision. This includes a discussion of the process/positioning and risks and benefits that any reasonable person would want before agreeing to treatment.

Risk of failure (5%)
Hypotension (3-30%)
Risk of dural puncture (0.16-1.3%) and PDPH (1%)
Risk of epidural haematoma 0.0004-0.003% (1:150,000)
Risk of nerve injury 0.016-0.56% (transient neuropraxia 1:6700, permanent neuropraxia 1:15,000, paralysis 1:100,000-250,000)
Neuropathy following obstetric epidurals 1:10995 (Paech IJOA 1998)
Risk of infection (vertebral/paravertebral abscess 1:145,000, epidural abscess 0.01-0.05% (1:5000) and meningitis)
Risk of subdural block (1:1000) or total spinal (1:5000 to 1:50000)
Backache (no evidence that epidurals cause long term backache)
Shivering/shakes
Itch
Accidental intravenous injection and systemic LA toxicity (1:10000)

A discussion about the need for an IDC with the epidural and the possibility of dose dependent leg numbness and weakness during the epidural infusion, is also necessary. Epidurals may prolong the 1st and 2nd stage of labour with a slight increase in instrumental delivery, no increase in rate of LUSCS and no difference in neonatal outcomes.

The patient needs to be cooperative and ideally English-speaking, otherwise an interpreter is required, family members should not be used as interpreters.

Alternative labour analgesia must be discussed and offered. Other options include a fentanyl or remifentanil PCA if the institution has appropriate protocols to support this and if there are contraindications to epidural analgesia. N2O is also an option.

A working epidural provides excellent pain relief during labour. Ideally the patient should be provided with written information about epidural analgesia in the antenatal period as in active labour it can be difficult to have lengthy discussions about epidural risks and for the patient to give informed consent. It is also often useful to have the partner present during the consent process.

Ensure an opportunity is given for the patient / partner to ask questions. Document discussions in the history.
October 2005 Question 1:

What are the indications for tracheal intubation in a 3 year old who presents with “croup”. Describe your technique for intubation.
• Croup- laryngotracheobronchitis, a viral respiratory illness usually caused by parainfluenza virus (also RSV and influenza).
• Ages- 6 months to 3 yrs (peak incidence 2 yrs) characterised by a barking cough which is of gradual onset (24-72 hours) and is self-limiting. Less than 1% require intubation.
• Symptoms- barking cough developing after 48-72 hours into an URTI. Increased resp. effort, low grade fever, inspiratory stridor.

Need for intubation
APLS guidelines predict need:
3E’s- 1) Effort: accessory muscle use, tracheal tug, nasal flaring, grunting
2) Efficacy: chest wall movement, tidal volume, PCO2, SpO2
3) Effect: effects of hypoventilation (altered conscious state, irregular ventilation, irregular cardiac rhythm secondary to acidosis)
The 'Croup Score' also predicts need for intubation (score out of 10, based on five parameters. Breath sounds, stridor, cough, recession/flaring, cyanosis. Higher score 7-10 = intubate). See CEACCP article 2007 ‘Acute stridor in children’.
Markers for intubation include severe respiratory distress, obtunded child, PaO2< 50mmHg, hypercapnia, HR> 170, RR > 55, failure to improve with medical therapy
Impending respiratory arrest: obtundation, slowing respiratory rate, apnoeaic episodes, silent chest, bradycardia.
Technique for intubation
1) Preparation: call for help (skilled paediatric anaesthetist), standard monitoring (O2 sats/ETCO2 at minimum), skilled assistant, variety of ETT’s (start 1 size smaller due to oedema), variety of laryngoscopes (straight and curved, different sizes), paediatric difficult airway trolley in the room, IV access, IVT with burette, drugs (thiopentone, suxamethonium, atropine, atracurium) in correct doses, ENT surgeon available in case tracheostomy is needed. Transport to theatre by anaesthetist if safe.
2) Induction:
- Cautious gas induction with sevoflurane in O2 only (decreased alveolar ventilation means prolonged induction time). Maintain spontaneous ventilation. Gentle CPAP.
- Laryngoscopy usually not difficult (obstruction below cords)
- Oral tube first- change to nasal if straightforward (better fixation/care in ICU)
- Commence morphine/midazolam infusions.
- Leak around ETT usually not evident until oedema resolving.
October 2005 Question 2:

Discuss ways in which you can decrease bias in a clinical trial for a new antihypertensive agent.
Bias is a systematic deviation from the truth.

Preparation for this trial would involve a comprehensive literature review, formulating a research question, then proposing a null hypothesis. This could be that a new drug (Drug X) does not cause any significant drop in systolic, diastolic, and/or mean blood pressure when compared to placebo. Sometimes, instead of comparing drug X to placebo, the researchers might decide to compare it with another antihypertensive drug (eg. Drug A) known to be effective. In this case the null hypothesis may be: Drug X does not cause a significantly different drop in blood pressure compared to drug A.

Bias which can affect the outcome/interpretation of results can enter at various stages of the trial.

Stage 1- literature review
Stage 2- formulation of null hypothesis and methods of testing.
Stage 3- Ethics approval and pilot study
Stage 4- testing of the drug
Phase 1- Administration of the drug to a small number of (usually healthy) volunteers
Phase 2- Administration of the drug to a hundreds of patients suffering from hypertension
Phase 3- Extended clinical trials involving thousands of patients at many sites. Randomised,
double-blind, placebo controlled trials are the best.
Phase 4- Post-marketing monitoring of the drug for safety amongst even larger numbers of
patients with hypertension.

1. Selection bias- this occurs when we are comparing groups with respect to some variable (blood pressure) but do not realise that the groups are different in other ways. Eg. the group receiving active drug may be on another drug which may increase the metabolism of drug X, therefore reducing its activity. Reduced by:
- randomisation
- stringent inclusion and exclusion criteria in place.
- crossover trials from existing treatment or placebo group.
- increasing sample size does not reduce bias

2. Confounding bias- this occurs when a third variable which is associated with both the exposure variable and outcome variable modifies that relationship. For example a patient newly diagnosed with hypertension who is given information about ways to reduce their blood pressure may instil life style changes such as diet and exercise which may have a greater effect on lowering his/her blood pressure than drug X. Reduced by:
- adjusting for confounders
- randomisation

3. Measurement bias- this occurs if the methods used for making measurements when comparing different groups have different scales or sensitivities, or if the observers influence the measurement process by being aware of the subjects' group allocation. Reduced by:
- standardisation of measurement process (same equipment, same sites of BP reading, same patient position, objective criteria)
- blinding of both subjects and observers (double) and of clinicians (triple)

4. Publication bias- this occurs when studies are published or not published depending on whether they did or did not find a statistical difference with their research, respectively. Additionally, 'double counting' in systematic reviews/meta-analyses can unduly amplify treatment effects. Reduced by:
- prospective registration of trial with international trials group
- careful literature review
- report negative trials/missing data
October 2005 Question 3:

What are the symptoms, signs and anaesthetic implications for an autonomic neuropathy associated with diabetes mellitus.
History (symptoms)
• Incidence 1 in every 10 diabetics
• Four to fivefold increased incidence in older population with hypertension
• Duration since diagnosis- increased diabetic time correlates with increased risk
• Extent of glycaemic control – poor control correlates with increased risk
• Absence of sweating
• Cardiovascular symptoms
o Orthostatic hypotension
o Silent myocardial ischaemia
o Decompensation – eg evidence of cardiac failure
o Symptoms of failure - dyspnoea
• Gastrointestinal symptoms
o Early satiety/bloating
o Persistent nausea and vomiting
o Gastroparesis
o Constipation (perhaps alternating with diarrhoea)
o Gastro-oesophageal reflux
• Genitourinary
o Impotence
o Urinary retention
o Dysautonomic bladder- recurrent UTIs
• Other
o Absence of sweating/profuse gustatory sweating
o Decreased autonomic response to hypoglycaemia (reduced recognition of hypos)
o Eyes- poor night vision/adjustment to light changes

Examination (signs)
• Absence of sinus arrhythmia/heart rate variability
• Resting tachycardia
• Orthostatic hypotension (> 30 mmHg drop in BP on standing)
• LVH with displaced apex beat
• Gastric distension
• Palpable bladder

Anaesthetic implications
Preoperatively
• Review glycaemic control- stratify risk of autonomic neuropathy
• Consider endocrinologist referral for optimisation of BSL control perioperatively
• Investigations as appropriate- HbA1c, BSL, fasting lipids, ECG, FBE, UEC.
• Withhold ACEI/angiotensin II blocker to reduce likelihood of haemodynamic lability
• Avoid preoperative sedation (aspiration risk)
• Premedicate with ranitidine at least 2 hours preop/sodium citrate prior/consider metoclopramide to increase gastric emptying
• Care with BSLs during fasting- symptoms/signs of hypoglycaemia may be blunted
• Anticipate a difficult airway (glycosylation of tissues)
Monitoring (in addition to minimum ANZCA requirements)
• Invasive arterial access for beat-to-beat monitoring
• Routine ECG monitoring with 5 lead ECG (II and V5)- better ischaemia detection
• Oropharyngeal temperature probe

Intraoperatively
• Careful induction with particular focus on preventing
(1) haemodynamic response to intubation
(2) severe hypotension on induction prior to intubation (may lack compensatory tachycardia)
• administer vasopressors as required
• anticipate hypotension- fluid load
• rapid sequence induction with cricoid pressure if autonomic neuropathy suspected
• Monitor BSL with hourly ABG/finger prick
• Multimodal analgesia for opioid sparing effect- compound gastroparesis
• Care to avoid hypothermia
o Poor temperature regulation
o Increased shivering on emergence precipitates myocardial ischaemia
• Extubate awake with patient in left lateral position
• Aim for smooth extubation – eg. lignocaine 1mg/kg 3 minutes pre-extubation

Postoperatively
• Monitor glycaemic control as patient will be nil orally initially (again, difficulty with recognition of hypoglycaemia)
• Increased risk of respiratory depression (blunted response to hypoxia)
• Careful fluid management, monitor urine output
• Increased risk of urinary retention- catheterise
• Monitor in HDU setting if concerns about perioperative myocardial ischaemia
• Recognition of increased risk of ‘silent’ myocardial ischaemia- thus post-operative ECG/enzymes if clinical suspicion.
October 2005 Question 4:

Describe the anatomy of the trigeminal nerve relevant to local anaesthesia for dental extraction.
Trigeminal nerve- 5th cranial nerve. Ganglion (three names: 1. Semilunar, or 2. Gasserian, or 3. Trigeminal ganglion) located in 'Meckel's cave', an invaginated pocket of dura lateral to the pons and internal carotid, inferior to attachment of tentorium cerebelli. Three branches:

V1- Opthalmic division (purely sensory). Sensory to orbit, orbital contents, forehead, nose, scalp to vertex, nasal mucosa, frontal/ethmoid sinuses. Not relevant to dental extraction, not further considered.

V2- Maxillary division (purely sensory). Sensory to skin over maxilla and side of nose, maxillary bones, hard palate, upper teeth, gums, upper lip, maxillary sinus.
Leaves trigeminal ganglion, passes through cavernous sinus below V1, exits skull base through foramen rotundum where it gives off 5 branches:
1. Greater palatine nerve
2. Lesser palatine nerve
3. Naso palatine nerve
4. Communicating branches to sphenopalatine ganglion (autonomic to nose,
palate, orbit and nasopharynx)
5. Posterior superior alveolar nerve
• Enters maxillary sinus, sensory to maxillary molars and surrounding gums (exception- first molar)
- Continues as Infraorbital nerve. Within infraorbital canal gives off 4 branches
• 1. Anterior superior alveolar nerve- sensory to cuspids, central/lateral incisors and surrounding gums.
• 2. Middle superior alveolar nerve- sensory to 1st molar, premolars and surrounding gums.
• 3. Zygomatico-facial
• 4. Zygomatico-temporal
• Exits infraorbital foramen, divides into terminal branches (palpebral, nasal, labial)

V3- Mandibular (motor to muscles of mastication, mylohoid, anterior belly digastric, tensors tympani/palati). Sensory to temporal region, tragus of ear, skin over mandible/mandible bone, lower teeth and gums, tongue
Leaves trigeminal ganglion, sensory and motor roots pass through foramen ovale, unite deep to lateral pterygoid, divide into anterior and posterior trunks. Key sensory branches:
1. Buccal nerve (mucosa of mouth and gums)
2. Auriculotemporal nerve
3. Lingual nerve (sensation to anterior 2/3 of tongue)
4. Inferior alveolar nerve- enters mandibular canal (sensory to most lower teeth and surrounding gums) and exits mental foramen as mental nerve (sensory to lower incisors and canines/lip/chin)

Types of blocks:

1. Supraperiosteal
Good for maxillary teeth, not so good for mandibular teeth or when site is infected
Advantages: easy, quick onset (2-5 mins)
Disadvantages: anaesthetises one tooth only, therefore not good for multiple extractions.
How: aim at apex of target tooth and inject approx 2ml LA

Upper Teeth

2. Infraorbital nerve
Anaesthesia of upper teeth from midline to canine, perhaps premolars. Skin and mucosa of cheek/upper lip.
Advantages: quick onset (within 5 mins)
How:
• Transmucosal- insert needle directly through muco-gingival junction above 1st canine. Aim supero-laterally. Inject 1mL LA.
• Transcutaneous- 1.5 cm lateral to nasal bone and 1 cm inferior to orbital rim. Angle needle upwards.

3. Superior alveolar nerve block
Anaesthesia of maxillary molar teeth up to the 1st molar with the exception of its mesiobuccal root in some cases. Risk of intravascular injection and haematoma formation.
Advantages: can do multiple extractions
How: Retract lip. Insert the needle at mucobuccal fold above maxillary 2nd molar at a 45 degree angle directed superiorly, medially, and posteriorly. There should be no resistance felt as the needle is advanced to about 16 mm in adults.

4. Maxillary nerve block (foramen rotundum)
Anaesthesia of hemimaxilla on the side of injection (including teeth, hard and soft palate, buccal and lingual tissue). Ideal for multiple extractions. Difficult to perform. Significant risk of intravascular/IT injection).
How: Needle introduced through coronoid notch below midpoint of zygomatic arch. Angle 45 degrees, aiming for apex of orbital cone. Contact lateral pterygoid plate, then walk anteriorly to enter the pterygopalatine fossa

Lower teeth

5. Inferior alveolar nerve
Anaesthesia of all unilateral mandibular teeth, the anterior 2/3 of tongue, and regions of the mental nerve
Advantages: covers many teeth, therefore useful for multiple extractions
How: insert needle 1cm at above 3rd molar about 2.5 cm deep and inject 1.5-2ml of LA. Failure rate 15-20%



6. Mental nerve
Anaesthesia of mandibular mucosa from midline to second premolar. Skin of lower lip/chin.
Advantages: quick onset (within 5 mins)
How: insert level at the level of the mental foramen (which is at the apex of the second premolar), inject 1-2mls of LA on the side required. Contralateral fibres may cross, therefore bilateral blocks required

7. Mandibular nerve block (foramen ovale)
Will provide anaesthesia of the lower teeth as well as mucous membrane of cheek (buccal nerve) and taste and sensation to ant 2/3 of tongue and floor of mouth (lingual nerve). Significant risk of intravascular/IT injection.
How: Introduce needle through coronoid notch below midpoint of zygoma. Contact lat pterygoid plate. Walk off posteriorly until enter foramen ovale (< 0.5cm deeper)
October 2005 Question 5:

What is the role of a laryngeal mask airway in a failed intubation for laparotomy.
The laryngeal mask airway (LMA)
-supraglottic airway device
-designed to make a seal around the laryngeal inlet
-allows spontaneous ventilation and controlled ventilation at modest PIPs (max 20cm H2O)
Failed intubation management depends on

-Emergency or elective surgery
-Ability to BMV or not “Can’t ventilate”
(NB: a dissertation on ‘plan D’ ie cricothryroid isnt discussed here)

Elective case:
1) can’t ventilate, can’t intubate
Recognise emergency - call for help.
-Cease further attempts at intubation to prevent trauma for subsequent laryngoscopy.
-Appropriate manoeuvres (jaw thrust, 2 handed BMV etc) and adjuncts (guedel,NPA) to attain ventilation - if fails:
-Insert an LMA and confirm ventilation the usual way.
-If using NDMR - IPPV until reversible using PIP max 20cm H2O.
Alternatively
- proseal LMA - allows higher positive pressures (30cm H2O)
Surgery should be delayed until the patient has woken up
2) can ventilate, can’t intubate
This is not an emergency but can become an emergency if, at a later stage, mask
ventilation becomes inadequate.
-BMV (drawbacks ++ esp if NDMR intubating dose) until reversible OR LMA insertion
-Delay surgery until fiberoptic intubation is possible.
-Some anaesthetists would perform fiberoptic intubation prior to waking. This has risks:
-Airway trauma (during/previous) making FOI more difficult via bleed/swelling
-Losing the ability to BMV

Emergency case:
Aspiration risk / cricoid / use of sux likely as RSI for emergency laparotomy
1) can’t ventilate, can’t intubate (worst case scenario.)
-LMA/iLMA and intubate through that (confirming ventilation with each step +/- FOI)
-Size 3/4 LMA & 6.0 ETT
-Size 5 LMA & 7.0 ETT
-Cricoid pressure should be maintained throughout.
-If ventilation can only be achieved via LMA - wake patient - aspiration risk
2) can ventilate, can’t intubate
-Ventilation through cricoid pressure until airway secured.
-Options are iLMA/LMA intubation or to prepare for fiberoptic intubation.
-if FOI - technically difficult, time consuming, and maintain cricoid pressure throughout.
Implications for further management of the patient for subsequent anaesthesia
-plan for indirect laryngoscopy (ie awake fiberoptic intubation)
-patient consent/counselling and appropriate follow up
-letter in file, GP, medic alert or ASA anaesthetic risk card
October 2005 Question 6:

How does anaesthesia alter temperature homeostasis?
The inter-threshold range: temperature range in which thermoregulatory responses are inactive.
In anaesthesia the inter-threshold range (ITR) is increased
-Cold response threshold is markedly reduced cf: warm response mildly elevated
-normal value of the ITR changes from ~ 0.2 C to ~ 2-4 C. a substantial increase
With increasing depth of anaesthesia there is
-a slight linear increase in the sweating threshold (in response to heat)
-marked linear decrease in the vasoconstriction and shivering thresholds (in response to
cold - NB shivering may be lost entirely by neuromuscular blockade).
-The response to cold may not be triggered until the core temperature reaches as low as 33 C.
-Some anaesthetic agents, however (such as isoflurane and desflurane) cause a non-linear
decrease in the cold-response threshold.
Response to heat/cold:
-Receptors:
-Majority in skin but also some in spinal cord, abdominal viscera, and the great veins.
-The anterior hypothalamus has cold and heat sensitive neurons.
-Controller:
-Posterior hypothalamus coordinates increased heat production (non shivering
thermogenesis) and/or decreased heat loss (vasoconstriction) to maintain normothermia.
-Effectors
-Sympathetics to vasocontrict/vasodilate, sweating, shiver (mm), non-shivering
thermogenesis in brown fat. piloerection (not useful in humans)
-Voluntary behavioural responses
-putting on/taking off more clothes, reducing/increasing exposed surface areas)
GA: the body is unable to mount these responses until very low core temperature reached.
-Phase 1: Rapid drop
-1 degree drop in core temp in the first hour (central --> peripheral redistribution)
-Phase 2: more gradual drop until the lower end of the ITR is reached.
-(increased heat losses and decreased heat production)
-Heat loss mainly peripheral vasodilation & increased radiant and evaporation.
-Phase 3: A plateau is then reached when the rate of heat production equals heat loss/gain.
-Shivering is usually not activated until the core temp reaches 30-32 degrees celcius.
-Vasoconstriction usually commences at one degree celcius above shivering.
Regional: there is a vasodilatory effect on the areas blocked.
-causes increased heat loss and the inability to vasoconstrict in response to cold can
accelerate the rate of hypothermia.
-If regional/GA combined (eg. Epidural & GA for laparotomy) hypothermia can rapidly occur.
NB: Other contributing factors to altered cold response include lack of behavioural responses
Response to heat:
SNS activation results in cutaneous vasodilation and sweating. The skin acts as a radiator,
variations in cutaneous vasoactivity altering the amount of heat delivered to it.
Heat loss: radiation (60%), evaporation (25%), convection (12%) & conduction (3%).
Hyperthemia: less frequently seen in anaesthesia and the responses to heat are generally
well preserved (only slight increase in the upper level of the inter-threshold range), it is still very
important under GA (NB: behavioural responses are lost here also)
-long procedures with heating devices being used
-pre-operative hyperthermic patient there is potential for overheating,
-Heat loss under GA occurs primarily by sweating.
October 2005 Question 7:

How would you diagnose a clinically significant latex allergy occurring intraop?
Definition of latex
• Types of reactions which can occur

Intraop
• Clinical signs
o Temporal relationship
• DDx
• Tryptase

Post op
• History
o Risk factors
o Previous reactions
o Seen an allergist
• Examination
• Investigation
o Skin prick testing
o Serology
October 2005 Question 8:

A 35 year old patient develops a small left pneumothorax following removal of a breast lump under local anaesthesia in a day surgery facility. How would you manage this?
Possible causes
1. Spontaneous
- 1°: healthy people without lung disease.
o Often smokers and tall people, due to rupture of subpleural blebs/bullae
- 2°: underlying lung disease
2. Traumatic
- High inflation pressures during IPPV
- Complication of CVC insertion
- Complication of intercostal nerve block for analgesia (…unlikely)
History
• Inquire re: symptoms (e.g. dyspnoea, cough, pleuritic chest pain).
o If compromised, a tension pneumothorax needs to be excluded immediately
• Risk factors:
- Spontaneous pneumothorax: emphysema with bullous disease, asthma, cystic fibrosis,
Marfan’s and Ehler-Danlos syndromes
- Barotrauma: IPPV with high peak inspiratory pressures e.g. in asthmatics
- Procedures: CVC esp. subclavian approach, intercostal blocks
• Social situation (important in context of discharge planning)
• High risk activities – flying, diving
• Other resp disease which would limit resp reserves – e.g. OSA, asthma
Examination
• Cyanosis, hypoxia, tachycardia, hypotension
• Degree of respiratory distress (i.e. respiratory rate, accessory muscle use, tracheal tug)
• Exclude tension pneumothorax. Suspect if hypotensive and significant respiratory
distress. Tracheal deviation and jugular venous distension are late signs
• Clinical signs may be absent in small pneumothoraces, but there may be:
o subcutaneous emphysema, ipsilateral reduction of chest wall movement and breath
sounds, increased resonance to percussion +/- audible wheezing or
‘crunch’ (caused by air in the mediastinum = Hamman’s sign)
Investigations
• CXR to assess the size of the pneumothorax
Lung Margin to chest wall Lung apex to cupola
Small < 2 cm < 3 cm
Large > 2 cm > 3 cm
• ABG may show hypoxaemia
Treatment depends on the SIZE of the PTX and the presence of symptoms/stability
• A small, simple pneumothorax in an asymptomatic, clinically stable patient can be
considered for simple observation and discharge home.
- Observed for 4-6 hrs in recovery and then have a repeat CXR.
- Administer O2 via HM help pneumothorax resorb.
- Discharge provided that she fulfills all the criteria for discharge from day
surgery (e.g. lives within close distance to medical attention, staying with competent
adult overnight, analgesia sorted, intake acceptable etc.). Explicit instructions to
present to A&E if she develops breathlessness, and GP f/u the next day.
• If pneumothorax is large and/or the patient is symptomatic, simple aspiration using a
14/16G cannula (@ 2nd ICS MCL) then chest drain insertion (5th ICS MAL)
- Supplemental high flow oxygen (10L/min) should be given
- Suspect tension pneumothorax if hypotensive or significant respiratory distress.
- Where intervention has been necessary, the patient will need to be transferred
out to a medical unit +/- ICU as this is a free standing day surgery centre
October 2005 Question 9:

Discuss the purpose of a postoperative visit.
The post-operative visit is important as it allows the anaesthetist to potentially diagnose anaesthetic related complications early and effect treatment. The post-operative visit should be completed in recovery and on the ward as well depending on individual patient circumstances.

Patient-related purposes
1. Provide general information e.g. general reassurance, answering questions, further management of pain control / review of analgesia, review of sensory/motor recovery from regional blockade. Listen to any concerns the patient may have about the anaesthetic.
2. Inform patient about any complications (e.g. difficult intubation, anaphylaxis, sux apnoea etc.) and appropriate follow up if necessary
3. Review cardiovascular status e.g. haemodynamic stability, management of perioperative drugs (e.g. beta blockers)
4. Review respiratory status e.g. respiratory functional status, sore throat, dental damage
5. Gastrointestinal status e.g. nausea and vomiting, bowel function
6. Renal status e.g. urine output, U&E
7. Haematological status e.g. Hb, blood loss, coagulation
8. Cutaneous e.g. pressure areas, itching

Anaesthetic-related purposes
1. General communication and feedback from patients for purposes of auditing, education, improved practices
2. Management of postoperative care i.e. IV fluids, pain control, sensory/motor recovery from regional blockade or complications thereof (e.g. epidural haematoma/abscess, PDPH, neuropraxia etc.), timing of removal of catheter infusions
3. Discharge check or day surgery follow-up (eg phone call next day)
4. Staff communication
October 2005 Question 10:

Critically evaluate the role of cardioversion in the management of intraoperative arrhythmias.
Background
• Cardioversion is the restoration of sinus rhythm either by electrical (DC current) or chemical (pharmacological) means
• Electrical cardioversion is delivery of current that is synchronized to occur with the R wave of the ECG, as delivery of current during ventricular repolarisation may produce VF (R on T phenomenon)
• Intraoperative arrhythmias are more likely with:
1. Pre-existing cardiac disease (MV disease, LV aneurysm, congenital heart disease, preexcitation syndromes eg short PR intervals, long QT syndrome)
2. Myocardial ischaemia
3. Hypoxaemia, hypercapnia
4. Hypotension
5. Acid-base disturbances
6. Electrolyte abnormalities (esp. K, Ca, Mg)
7. Mechanical stimulation of heart chambers by CVC or PAC
8. Adrenergic stimulation (e.g. light anaesthesia)
9. Endocrine abnormalities (thyrotoxicosis, phaeochromocytoma)
10. Drugs
• Aetiology should always be considered before instituting any therapy, and reversed or corrected if possible (eg correction of hypoxaemia and hypotension/hypovolaemia will aid in restoration of sinus rhythm with cardioversion)
• Electrical cardioversion is indicated when there is associated haemodynamic instability to prevent life-threatening complications of hypoperfusion i.e. cardiac or CNS ischaemia
• Management should be as per ACLS guidelines, including management of airway, breathing, circulation.
• If the patient is haemodynamically stable and does not have a malignant rhythm eg VF/VT, chemical cardioversion may be considered.
• Different routes – transcutaneous most common
o Via pads/paddles
o Need high energy

Indications for cardioversion
1. VT
2. VF
3. AF
4. Atrial flutter
5. SVT
6. Torsades de pointes

Advantages of electrical cardioversion
• Very effective
• In VF/VT mandated as per ACLS guidelines, early DCR within 1 minute of VF/VT improves outcomes.
• If witnessed VF/VT, 3 stacked shocks are recommended. This is because the success of the first shock for VT/VF is 78%, 2nd shock success 35%, 3rd shock 14% (Interactive CardioVasc Thorac Surg 2007).
• Readily available in all operating complexes
• Relatively easy to use

Disadvantages of electrical cardioversion
• Post-shock arrhythmias e.g. refractory asystole, VF (esp. if on digoxin). Remember, except for ventricular fibrillation, all shocks should be synchronized.
• Electrocution of bystanders or self +/- trigger VF
• Damage to pacemakers (avoid placing paddles/pads directly over PPMs)
• Skin burns
• Thoracic pain
• Transient depression of myocardial function, especially with repeated shocks and high energy
• Trauma from vigorous body movement
• Embolization and stroke – more likely in AF (5% non-anticoagulated vs 0.8% coagulated). It is acceptable to attempt cardioversion if acute AF within 48 hours; otherwise only rate control should be attempted. Current recommendation is to anticoagulate chronic AF for 3-4 weeks, then attempt cardioversion with continued anticoagulation for another 4 weeks (due to atrial stunning). Alternatively perform TOE to rule out LA thrombus, then cardioversion followed by anticoagulation for 4 weeks. In intraoperative rate controlled AF with no haemodynamic compromise – avoid electrical cardioversion.

Advantages of chemical cardioversion
• Simple and convenient
• No sedation necessary

Disadvantages of chemical cardioversion
• Lower efficacy than electrical cardioversion
• Drug-related side effects

NB: Avoid adenosine, beta-blockers, CCB, digoxin in WPW. Treat with procainamide, flecainide or amiodarone.
October 2005 Question 11:

How do you assess the severity of cardiac failure in a 75 year old man presenting for joint replacement surgery? Include any relevant investigations.
Assessment of the severity of cardiac failure in a 75 year old man presenting for joint replacement surgery (Intermediate risk surgery, reported cardiac risk 1-5%), requires a thorough history, examination and investigations.

History
• Symptoms and the amount of effort needed to provoke them correlate with severity of CCF. May be classed using NYHA functional classification:
- Class I: No limitations. Ordinary physical activity does not cause undue fatigue, palpitation or dyspnoea or anginal pain
- Class II: Slight limitation of physical activity. Comfortable at rest, but ordinary physical activity results in fatigue, palpitation or dyspnoea
- Class III: Marked limitation of physical activity. Although comfortable at rest, less than ordinary physical activity causes fatigue, palpitation or dyspnoea
- Class IV: Symptomatic at rest, cardiac disease resulting in inability to carry on any physical activity without discomfort.

• The Goldman specific activity scale is based on the metabolic cost of various activities, and classes correlate to NYHA classes. It is less subjective and more reproducible than NYHA classes.
- Class I: Patients can complete any activity up to 7 METS (e.g. carry 11 kg up 8 steps, shovel snow, jog/walk 8 km/hr, play basketball or squash)
- Class II: Patients can complete any activity up to 5 METS (e.g. sexual intercourse without stopping, walk 7 km/hr on level ground)
- Class III: Patients can complete any activity up to 2 METS (e.g. shower or dress without stopping, strip and make bed, play golf, walk 4 km/hr)
- Class IV: Patients cannot complete activities equal to or more than 2 METS (i.e. none of the above)

• LV failure symptoms: orthopnoea, PND, no. of pillows patient sleeps on, coughing up pink frothy sputum
• RV failure symptoms: peripheral oedema, distended abdomen, abdominal discomfort, nausea
• Aetiology of CCF: cardiac causes e.g. myocardial ischaemia (most common), valvular heart disease, cardiomyopathy, congenital heart disease. Non-cardiac causes e.g. poorly controlled hypertension, pulmonary hypertension, high output states
• Medication history and current therapy
• Previous recent admission(s) for exacerbation of CCF (frequency and number of exacerbations) may give an indication of severity
• Duration of CCF
• Ejection fraction documented in the past

Examination
• General appearance:
- Dyspneic at rest
- Unable to lie flat
- Central cyanosis reflects severe hypoxaemia
- Cachexia is a sign of chronic CCF
- Confusion and agitation may be due to hypoxia and poor cerebral perfusion
• Vitals: hypotension, tachycardia, hypoxia
• Raised JVP
• Diffuse, laterally displaced apex beat
• Gallop rhythm (S3) or S4, heart murmurs
• Inspiratory basal crackles
• Pleural effusion
• Hepatomegaly +/- tender, pulsatile
• Ascites
• Peripheral oedema

Investigations
• Bloods
- FBE: Hb ?anaemia
- U&E: electrolyte derangement, impaired renal function reflecting poor renal perfusion
- LFT: elevated enzymes and bilirubin from hepatic congestion
- BNP (Brain Natriuretic Peptide) levels correlate closely with NYHA classes as well as the Goldman specific activity scale
• ECG: Q wave indicating previous MI, LVH and strain, arrhythmias, LBBB
• CXR: cardiomegaly, prominent upper lobe veins (upper lobe venous diversion), perihilar “bat’s wings”, pleural effusion, Kerley B lines, fluid in the fissures
• Echocardiogram: chamber size and thickness, wall motion abnormalities, valvular function, LVEF (helps distinguish systolic dysfunction EF < 0.4 with diastolic dysfunction EF > 0.4)
• Radionuclide imaging: reliable technique for determining global heart function, LVEF and presence of reversible ischaemia
• Coronary angiogram would be useful if CAD is suspected
October 2005 Question 12:

Describe the symptoms and signs of commonly seen perioperative nerve injuries in the upper limb. List the causes and possible strategies for prevention. Do not include injuries due to neural blockade or direct surgical trauma.
Types of upper limb nerve injuries
1. Ulnar nerve (most common, 1/3 of all nerve injuries in the ASA Closed Claims Study)
2. Brachial plexus (20%)
3. Radial nerve
4. Median nerve
5. Musculocutaneous nerve
6. Circumflex nerve
Clinical features
• Nerve injury may be apparent either immediately after recovery from anaesthesia or may occur several days later
• Usually unilateral but there are reports of bilateral and symmetrical injuries
• Symptoms and signs vary with severity of injury from numbness and mild paraesthesia to persistent painful paraesthesia, hyperaesthesia, pain in the distribution of the affected nerve, sensory loss, paresis or paralysis of the affected muscles.
• Progression to CRPS type 2 is also possible.
• In chronic injury, disabling autonomic dysfunction may sometimes occur, and in extreme cases, there is muscle wasting, joint stiffening and bone demineralization
• Specific types of nerve injury:
1. Ulnar nerve
- Numbness and paraesthesia over medial aspect of hand and over little finger is most common presentation
- Weakness of abduction and/or adduction of the fingers, adduction of thumb, loss of hypothenar muscles
- Weakness of flexion at the DIP joints of the little and ring fingers (if the lesion is at the elbow)
- Shouldn’t get numbness of medial forearm as medial cutaneous nerve of forearm is separate entity that is given off BEFORE axillary plexus
- Compression on the table when forearm pronated and extended
- Stretched over medial epicondyle or compression against medial epicondyle and when extreme flexion of elbow across chest
2. Brachial plexus
- Various combinations of lesions within the median, ulnar, radial, musculocutaneous and circumflex nerve territories
- Most commonly involve upper nerve roots
- Stretching of the cord – extreme abduction, ext rotation, post shoulder displacement, lateral decubitus with humeral head compression, extension and lateral flexion of head away from abducted arm
- Sternal retraction, median sternotomy can compress the plexus esp if extra rib on C7 (lower brachial plexus nerve lesions associated with median sternotomy)
- C5,6 lesion
i. Loss of abductors and lateral rotator of shoulder
ii. Loss of supination
iii. Arm hangs medially rotated, extended elbow, pronated
iv. Sensory loss of lateral arm and forearm
- C8, T1 lesion
i. Loss of finger extension
ii. Claw hand
iii. Involves intrinsic muscles of hand
iv. Sensory loss of medial hand and forearm, arm
3. Radial nerve
- Weakness of extension of the DIP joint of the thumb, and of the wrist and finger MCP extensors – wrist drop
- Interphalageal joints can still be straightened by action of interossei and lumbricals
- Numbness over the dorsum of arm and hand, esp over 1st dorsal interossei
- Compression over edge of table by humerus
- compression of nerve in lateral decubitus with uppermost arm abducted and suspended from vertical support
4. Median nerve
- Numbness over the index finger palmar aspect
- Weakness of thumb abduction
- Damage during cannulation of artery/vein in cubital fossa
5. Musculocutaneous nerve
- Weakness of elbow flexion
- BP cuff
6. Circumflex nerve
- Weakness of shoulder abduction

Causes of perioperative nerve injury
Aetiology unclear but possibilities include:
1. Compression and/or stretching of the nerve causing localised ischaemia from malpositioning, tourniquet, NIBP cuff, tight-fitting casts (superficial nerves are especially vulnerable to compression eg ulnar nerve with ischaemic injury likely related to compression of vasa vasorum)
2. Direct trauma by suturing needles, other needles e.g. IJV cannulation, instruments, surgical retractors
3. Thermal injury from diathermy
4. Compression from local infection and abscess or haematoma
Predisposing factors to nerve injury
(1) Patient factors
• Pre-existing generalized peripheral neuropathy
• Local compression neuropathy
• Diabetes
• Patients with perioperative ulnar nerve palsy often have abnormal ulnar nerve conduction on contralateral side
• Congenital abnormalities of thoracic outlet eg extra rib
• Arthritis and instability of joints
• Thin (BMI <25)
• Males – more likely to develop ulnar nerve injury (3:1 due to less fat and narrow cubital tunnel)
• Elderly – with atherosclerotic disease
• Smokers
(2) Anaesthetic factors
• Hypovolaemia/dehydration
• Intraoperative hypotension
• Hypoxia
• Electrolyte abnormalities
• Hypothermia
• Poor patient positioning intraoperatively
(3) Surgical factors
• Arterial tourniquet
• Sternotomy
o High risk injury to brachial plexus (C8, T1)

Preventative strategies
1. Awareness of the potential problems associated with various surgical positions and careful positioning of the patient with appropriate padding
- Supine: avoid arm pronation and extreme elbow flexion (ulnar nerve). Avoid elbow hyperextension. Avoid arm abduction > 90°, external rotation and posterior displacement of the shoulder (brachial plexus), maintain upper limb in neutral position. Avoid rotation and lateral flexion of neck towards the opposite side (brachial plexus); maintain head in neutral position centrally.
- Prone: Avoid arm abduction > 90° (brachial plexus)
- Lateral: dependent arm should be anterior to thorax (brachial plexus), appropriate use of axillary roll
- Transfer with care
2. Identify and correct, if possible, any predisposing risk factors e.g. correct hypovolaemia, treat hypotension, avoid hypothermia.
3. Minimise tourniquet times and always use a pneumatic tourniquet
October 2005 Question 13

The hospital pharmacist notifies you as Director of Anaesthesia that Thiopentone is to be withdrawn from the hospital formulary due to minimal usage. Outline and justify your response.
Initial Letter:
• Concern that the issue had not been previously raised with the Anaesthetic Department
• Request an extension to the planned withdrawal, so that a Response may be produced

Forming a Response:
• Collect statistics on Thiopentone usage:
o Collation of data from Anaesthesia Record
o Identify types of cases in which Thiopentone used commonly & why this may be so. Eg:
• Trauma
• Obstetric Emergencies
• Paediatrics
• Seizures in Theater Complex
• Obtain opinions from Colleagues:
o May include Anaesthetists, Intensivists, and Emergency Physicians
o Thiopentone usage in certain cases – Ideal? Essential? Historical?

Formal Response to Pharmacist:
• Outline the issues raised and trends identified above:
o Justify keeping a limited amount of Thiopentone in Inventory
o Petitions may not be necessary, but may prove useful
• Factors which allow minimising stock
o Long shelf-life
o Limiting access to certain cases
• Request further discussion
o A formal personal meeting to discuss concerns
o To identify other peripheral factors leading to its discontinuation
• Impending cessation of distribution by supplier
• Are the pressures on pharmacy purely monetary

Justification for Response:
• An open relationship and clear communication between Pharmacy and Anaesthetic Departments is beneficial
• There is a shared responsibility for rational drug use
• Discussion is important to appreciate differing viewpoints


INITIAL LETTER = MORE TIME
FORMAL RESPONSE = COLLEAGUES/SPECIFIC SITUATIONS (CHILDREN/PREGNANCY/TRAUMA/SEIZURES- ANTICONVULSANT)
WHY WITHDRAWN? MONETARY VS SUPPLY ISSUE
JUSTIFY = BETTER RELATIONSHIP/SHARED RESPONSIBILITY/ALL STAKEHOLDERS
October 2005 Question 14

You see a patient in the Pre-anaesthetic Clinic who asks you to administer an alternative medicine as part of their anaesthetic for total hip replacement. How would you respond to this?
Alternative medicine:
• Any substance which may be used by an individual for its effects on health or disease process, that is not recognised by the regulatory authorities of that country for these properties
• These may be:
o Harmless substances with no known effects
o Biological substances with physiological effects
o Prescription drug from another country, not locally recognised
• Good patient care revolves around a patient-centred model whereby both the physician and the patient has input into the patients care, but neither can force or withhold a treatment without the others consent.
Response:
• Discussion with the patient in regards to:
o The substance itself:
• What is it? Effects? Side effects?
• Important to know basics of substance in case patient wishes to self-administer it
• May effect surgical outcome
• May alter anaesthesia efficacy / safety (drug interactions, drug reactions)
• Why the patient would like it to be given at that time?
o Patients Rights:
• Respect for Autonomy and patients right to make informed decisions in regards to their own health care
• This may guide our care, but not dictate it
o Ethical and Professional Responsibilities:
• Administration of an unregulated / unapproved substance by the anaesthetist may go against principles of ‘primum non nocere’
• The prescription of approved medicines for ‘off-label’ uses is significantly different as it may be supported by high grade research
Decision:
• There is a requirement for medical practitioners to provide treatment of an appropriate standard, and patient autonomy must be respected as long as it is within these standards
• Notify surgeon of request
• I would not administer the alternative medicine for the patient

NON-MALEFICENCE- UNREGULATED/UNLICENCED DRUG MAY GO AGAINST
DRUG EFFECTS- NON-ACTIVE, ACTIVE, PRESCRIPTION FROM OTHER COUNTRIES
NOTIFY SURGEON
October 2005 Question 15


List the physiological effects of ECT and how they may be modified.
Physiologic effects of ECT are biphasic, and may be due to ECT itself or the induced seizure.

Effects of ECT:
• Autonomic:
o Initial parasympathetic outflow:
• Hypotension, bradycardia, and rarely asystolic arrest
o Followed by sympathetic discharge
• Hypertension and tachycardia (Risk of ischaemia)
• Opioids and Clonidine may blunt these responses
• Beta-blockers, eg Esmolol, may protect against ischaemia
• CNS:
o Seizure activity leads to:
• Increased Cerebral activity
• Increased CMRO2 and production of CO2
• Intracerebral vasodilation
• Increased CBF with resultant rise in ICP
• Adequate pre-oxygenation provides greater oxygen stores for this short-lived increase in cerebral and overall metabolism
o Amnesia is due to seizure itself, but is supplemented by low dose of induction agents, eg Propofol

Effects of Seizure:
• Initial tonic phase may last up to 15 seconds, followed by a clonic phase, rarely lasting more than 1 minute
• Physiological effects:
o CVS:
• Tonic seizures increase venous return and central compartment volume
• Ongoing seizures result in increased muscular metabolism, resulting in vasodilatation
o Resp:
• Airway obstruction with tetanic closure of jaw
• Apnoea secondary to contractions of chest wall musculature and diaphragm (also anaesthetic agents)
• Hypercapnoea and respiratory acidosis
o CNS:
• Increased intra-thoracic and intra-abdominal pressures are transmitted to CSF via epidural and paraspinal venous plexuses
• Increased CSF pressure, and increased blood compartment in brain results in increased ICP
• Post-ictal state
o Musculoskeletal:
• Muscular pain
• Potential for soft tissue injury, fractures, thrashing injuries and tongue-biting
• Motor activity may be diminished by use of muscle relaxants – Suxamethonium used commonly – to reduce the chance of seizure related injuries
• A mouth guard may be inserted to prevent tongue biting incidents
o Gastrointestinal:
• Increased intra-abdominal pressures with increased risk of aspiration
• Fasting - due to nature of psychiatric illness, must be witnessed fasting
• Antacid Prophylaxis if reflux risk
o Genitourinary:
• Incontinence
May 2005 Question 1

Discuss the perioperative use of nimodipine for a patient undergoing clipping of a cerebral aneurysm.
The predominant role of perioperative nimodipine for a patient undergoing clipping of a cerebral aneurysm is to reduce the risk of secondary ischaemia and poor outcomes after a subarachnoid haemorrhage following aneurysm rupture.

SAH – 85% caused by ruptured saccular aneurysm, 10% non-aneurysmal haemorrhage, 5% other causes.
After the initial insult, secondary insults can increase mortality and morbidity:
- early rebleeding
- hydrocephalus
- cerebral haematoma
- cerebral ischaemia from vasospasm as a reaction to blood in the subarachnoid space

Vasospasm
- cause remains obscure but is related to vasoconstrictor substances in the CSF as a result of the haemorrhage in particular the release of oxyhaemoglobin, risk is highest with larger amount of blood in basal cisterns
- After SAH, both vasospasm and secondary cerebral ischemia typically begin at day 3, peak between days 7 and 10, and may continue through until day 21.
- treatment options
o Prevention - early surgery and evacuation of initial haemorrhage
o HHH (hypertension, hypervolaemia, haemodilution) – If the aneurysm has been surgically occluded from the circulation then triple H therapy may overcome the hypoperfusion but there are few randomized studies. Existing data do not support prophylactic use of HHH therapy in high-risk patients although some centres do practise this. BP targets of around 160-200mmHg and volume expansion with colloids are recommended.
o Ca channel blockers – nimodipine most widely studied
o Endovascular techniques and intraarterial vasodilators eg papaverine
o Cerebral balloon angioplasty – for refractory vasospasm
o Antiplatelet agents – minimal evidence
o Clot-lytic agents

Nimodipine
- reduces incidence and severity of neurological deficits from vasospasm
- oral/NGT 60mg Q4H starting within 96 hours, continue for 21 days
- IV 0.5mg/kg/min infusion, start oral ASAP, continue for 21 days
- has been shown to improve neurological outcome in Hunt and Hess SAHs Grade I-V (in particular Grade I-III)
- relative risk reduction of 35% for cerebral infaract
- relative risk reduction of 40% overall mortality
-no difference found between IV and oral (oral is cheaper)
- mechanism of action is unknown (possible vasodilating effects as well as cerebral protective effects)
- highly lipophilic, crossing BBB easily
- low SE profile – hypotension 4.4%, 1 % nausea, bradycardia, headache
- does not interfere with anaesthesia for clipping of aneurysm
May 2005 Question 2

Discuss ways in which the risk of deep venous thrombosis can be minimised in adult patients having intra-abdominal surgery.
The risk of DVT in adult general surgical patients is 15-40%. Chemical prophylaxis decreases this risk by about 60%.
Individual’s risks can be stratified according to patient factors and surgical factors. Risk factors include smoking, obesity, trauma, prolonged immobility, cancer, previous DVT/PE, congenital procoagulant state



For patients undergoing major general surgery (ie moderate risk), thromboprophylaxis with a low-molecular-weight heparin (LMWH), low-dose unfractionated heparin (LDUH), or fondaparinux (each Grade 1A) is recommended (Prevention of Venous Thromboembolism ACCP guidelines 8th edition 2008).

There is evidence that LMWH decreases rate of asymptomatic DVTs compared to LDUH but with no difference in significant thromboembolic events or post-op bleeding. LMWH has a lower risk of developing HITs (but it is still possible).

Mechanical devices such as pneumatic calf compressors or TED stockings have been shown to reduce DVTs but not PEs/death. However, since they are a simple and cheap form of prophylaxis with minimal SEs, they should be used together with chemical prophylaxis in all moderate to high risk patients.

LDUH/LMWH should either start 12 hours before surgery (heparin 5000 units or dalteparin 2500 u SC) or at the start of surgery. The dose of heparin and dalteparin should be delayed for 2 hours if a neuraxial technique has been employed and longer if there was a bloody tap.

In general, prophylaxis should be continued for 3-5 days or until the patient is fully mobile. In the highest risk group, therapy is recommended for 12 days.

If there is bleeding intra-op/post-op or coagulopathies contraindicating the use of LDUH or LMWH, mechanical prophylaxis should be used.

If HITs is a problem, use alternative forms of chemical prophylaxis e.g. fondaparinux (Factor Xa inhibitor), lepirudin (direct thrombin inhibitor), warfarin etc. Advice should be sought from the haematology unit in these patients.

Other general methods:
- stop smoking
- lose weight
- avoid hypovolaemia, haemoconcentration
- early mobilisation
- neuraxial techniques – shown to reduce DVTs in high risk orthopaedic procedures (hip surgery)

(taken from Antithrombotic and Thrombolytic therapy 8th Ed ACCP guidelines)
May 2005 Question 3


Outline the anatomy of the right internal jugular vein as it is relevant to your preferred method of percutaneous cannulation.
R IJV – suitable site for cannulation
- superficial, relatively straight path to R atrium
Not as comfortable c/f subclavian

Runs from its origin in the jugular foramen in the skull, to its termination behind the sternoclavicular joint, where it joins the subclavian vein to form the brachiocephalic vein. The braciocephalic vein then becomes the SVC which empties into the R atrium.
The vein is initially posterior to, then lateral and then anterolateral to the carotid artery during its descent in the neck. Because of the posterior location in the upper neck, the vein is more commonly cannulated in the middle and lower neck. The vein lies most superficially in the upper part of the neck, after which it descends deep to the sternocleidomastoid muscle.
The artery, vein and vagus lie in the carotid sheath. Sympathetic chain directly posterior – these 2 vessels and 2 nerves should be considered as a single structure in the neck
Anterior
Internal carotid artery and vagus nerve.
Posterior
C1, sympathetic chain, dome of the pleura. On the left side, the thoracic duct.
Medial
Carotid arteries, cranial nerves IX-XII
4 major tributaries:
- pharyngeal venous plexus
- common facial vein
- lingual vein
- sup and middle thyroid veins

Head down 15% tilt to avoid venous air embolus and engorge the vein.
Head gently turned to the left.
Midpoint of SCM. Feel for carotid artery pulse medial to the medial border of muscle and aim needle lateral to this, aiming towards ipsilateral nipple. Another technique for entry is to use the junction of the 2 heads of SCM – at this position the IJV is superficial and lateral to the carotid artery
Increasingly US device used for localisation – considerable variation in the relationship between IJV and ICA
May 2005 Question 4


Discuss the requirements for and limitation of the use of patient-controlled analgesia (PCA) as a technique.
Patient controlled analgesia is a method where the patient can initiate their own pain relief without requesting assistance form nursing or medical staff. It generally refers to intravenous analgesia, but can also be used for epidural and other forms of regional blockade.

Requirements:
• Patient factors
o Able to understand the system and comprehend appropriate usage:
• Children > 7
• Intellectually intact
• No dementia
• Staffing/Nursing factors
o Adequate documentation of specialised PCA form
• Pain scores
• Demands vs deliveries: hourly vs total
• Flask refills
o Nursing staff trained in their management
• Equipment factors
o Locked programmable pumps with regular servicing
o Specialised giving sets without a port for injection of other drugs into the PCA line. Must have anti-syphon valves. If using a non-dedicated line, must have anti-reflux valve as well.
o Programmable: dose in mg/ml, lockout, 4 hourly limit, additional background infusion
o Air filter and alarm
o An IV cannula with a continuous slow crystalloid infusion to ensure delivery of the drug
• Medical factors
o Adequate analgesia (loading) needs to be attained before starting PCA (maintenance)
o A pain service for bd plus prn reviews for trouble shooting
o Appropriate knowledge for the prescription. Generally only anaesthetists prescribe them.
o No other opioids should be prescribed
o Avoid other sedative drugs (BZDs) except in special circumstances
o Correct use of multi-modal analgesia


Limitation
• May not provide enough analgesia in certain circumstances due to a set program. This can be overcome with frequent review and adjustment.
• Usually requires IV access although other routes have been described (subcut, intranasal, oral)
• Still have a risk of respiratory depression and other opioid side effects
o Nausea
o Itch
o Drowsiness
• Possible abuse with opioid seeking/dependent patients
• Higher cost
• Careful consideration with the prescription with comorbidity: liver and renal disease.
May 2005 Question 5
Compare the relative merits of gelatin-based intravenous solutions and dextran intravenous solutions.
Gelatin-based (e.g. gelofusine/haemaccel) and dextran – both colloid-type IV fluid

Gelatin-based
- MW 30000, made from bovine gelatine (polypeptide)
- No human products therefore no risk of disease transmission from other humans. Theoretical risk of CJD from cows
- Low risk of anaphylaxis 0.066%–0.146%, skin reactions <0.5%. Haemeccel higher anaphylactic rate c/f dextrans
- Calcium-free – compatible with blood products. Haemeccel contains Ca though
- Need more volume c/f dextran
- Negatively charged
o Repel one another and cause larger volume expansion
o Delays glomerular filtration → stays in circulation longer (effective for 2-4 hours)
- Cl- conc 120 mmol/l → lowers risk of hyperchloraemic acidosis
- Balanced Na+ 154 mmol/l
- No effect on coagulation, except from dilution in massive amounts
- Does not cause rouleaux
- No renal failure
- Stable, long shelf lives


Dextran
- as plasma expanders the usual solution is 6% Dextran 70 (MW 70000)
- manufactured from sucrose by bacteria Leuconostoc mesenteroides
- stable, long shelf-lives
- lasts longer in plasma – about 6 hours
- high chloride content 154 mmol/l
- higher risk of anaphylaxis, as antiDextran antibodies due to synthesis of dextrans by gut bacteria. Risk can be decreased 10 fold by pre-treatment with monovalent heptan-dextran
- anticoagulant effects, especially with higher volumes
o haemodilution
o reduced Factor VIII activity
o plasminogen activator increased
o fibrinolysis increased
o impairment platelet function
- sometimes this anticoagulation is used intra-operatively e.g. vascular surgery (usually with dextran-40) or as prophylaxis against thromboembolism
- causes rouleaux → interferes with blood cross-matching
- Can cause microaggregation in kidneys → renal failure
May 2005 Question 6

Compare the use of a pulmonary artery catheter and transoesophageal echocardiography in evaluating cardiac function intraoperatively.
May 2005 Question 7

List the possible causes of failure to emerge from general anaesthesia and describe how you would differentiate them.
Failure to emerge from anaesthesia can be loosely defined as the patient not opening their eyes to voice and obeying command at a point after the cessation of surgery where the anaesthetist would otherwise expect the patient would be awake. The patient may or may not be breathing


Residual anaesthesia
• Most modern volatile anaesthetic agents wash out very rapidly, but with prolonged surgery there can still be large amount of uptake into adipose tissue which will delay emergence. This can be assessed by the end tidal anaesthetic concentration. The MAC awake is approximately 0.3 – 0.4x the MAC of the drug. This can be used to assess if this is the cause. However, one must also consider that other factors may reduce the MAC of the drug. Particularly concomitant opioids and sedatives
• Propofol during high dose prolonged infusion can accumulate in the body. The context sensitive half time for prolonged infusion is greater than that of sevoflurane. There is no way to ascertain this directly, but will be a diagnosis of exclusion. Use of a diprifusor will give an approximate time to eye opening, but this is often inaccurate


Residual neuromuscular blockade
• This is a less common cause with modern agents, but if not reversed can be a cause. Especially with pancuronium. Assessment with a peripheral nerve stimulator to assess double burst stimulation and train of four ratio are the most sensitive methods of detecting residual block.
• Suxamethonium apnoea may need to be considered


Too much opioid or other sedating analgesic (clonidine, ketamine, tramadol)
• The patient may have pin point pupils (although many other drugs we give can mask this sign by causing dilation: atropine, beta agonists, ketamine, tramadol) and may be apnoeic or hypopnoeic. Using a titrated doses of naloxone and assessing the response to this may help differentiate. Start with 50mcg in adults, because we don’t want to completely reverse all opioid effect unless the patient is in respiratory failure. However, this patient should not have been extubated, or have had their LMA removed, at this stage.
• Central anticholinergic syndrome


Physiological/Pathology/Medical: resulting in a decreased conscious state or in reduced metabolism and clearance of drugs
• Cardiac: failure and hypotension
• Renal failure with profound uraemia
• CNS: raised ICP/cerebral oedema, stroke, intracerebral bleed, non-convulsive seizures
• Metabolic: hyponatraemia (TURP syndrome), hypoglycaemia, hypothyroidism, hypo-osmolality, acidosis
• Type II respiratory failure causing CO2 narcosis, PE, hypoxia
May 2005 Question 8

Draw a circle breathing system & give reasons for the location of the components
The circle system is so named because its components are arranged in a circular manner. It prevents rebreathing of carbon dioxide by use of carbon dioxide absorber but allows partial rebreathing of the other exhaled gases.
The circle system consists of seven components
1. a fresh gas inlet
• is placed between the absorber & the inspiratory valve.
• Positioning it downstream from the inspiratory valve would allow fresh gas to bypass the patient during exhalation & be wasted.
• Fresh gas introduced between the expiration valve & the absorber would be diluted by recirculating gas. Inhalational anaesthetics may be absorbed or released by soda lime granules, thus slowing induction & emergence.
2. inspiratory & expiratory unidirectional valves
• should be close to the patient to prevent backflow into the inspiratory limb if a circuit leak develops.
• Should not be placed in the breathing tube Y-piece, since that makes it difficult to confirm proper orientation & intraoperative function.
3. inspiratory & expiratory corrugated tubes
4. a Y-piece connector
• is used to connect the inspiratory & expiratory sides of the system to the patient.
5. an overflow valve (adjustable pressure limiting valve)
• should be placed immediately before the absorber to conserve absorption capacity & to minimize venting of fresh gas.
6. a reservoir bag
• resistance to exhalation is decreased by locating the reservoir bag in the expiratory limb.
• Bag compression during controlled ventilation will vent alveolar gas through the APL valve, conserving absorbent.
7. a canister containing a carbon dioxide absorbent.
• Located between APL valve & FGF
• Conserve absorbent by minimizing the passing exhaled gases, and prevents rebreathing.
8. Heat & moisture exchanger
• Attaches between the endotracheal tube & the expiratory and inspiratory corrugated tubings.
• Located as close to patient as possible to minimize water & heat loss from the patient, and to protect the breathing circuit & anaesthesia machine from bacterial or viral cross-contamination.
9. Vaporizer outside the circle (between the flowmeters and the common gas outlet)
• To lessen the likelihood of concentration surges during use of the oxygen flush valve.
• Because of its high internal resistance, which is required to limit only a small amount of FGF to go though the vapour.
• With low flows, the volatile delivered is diluted by the existing gases in the circuit
10. gas-sampling ports – usually side-stream sample to decrease resistance
11. manometers
12. spirometers
May 2005 Question 9

What significant side effects are associated with anti-emetic agents?
• Side effect – “a non-therapeutic, undesired and usually adverse effect of a drug, that is often dose-dependent”
• Allergies and anaphylaxis are rare, but possible
• Anti-emetics belong to a number of classes:

1. Dopamine-2 Antagonists:
a. Metoclopramide:
i. Extrapyramidal side effects: Acute Dystonia, Tardive Dyskinesia, Akathisia, and Parkinsonism
ii. Drowsiness
iii. Neuroleptic Malignant Syndrome
iv. Hyperprolactinemia – Amenorrhoea, lactation
v. Sodium and Water Retention
b. Droperidol:
i. Extrapyramidal side effects less common than with Metoclopramide
ii. Drowsiness
iii. QT prolongation – may precipitate arhythmmias
iv. Neuroleptic Malignant Syndrome
c. Prochlorperazine:
i. Extrapyramidal side effects more common than Metoclopramide, and dose-related
ii. Anticholinergic side effects: dry mouth, blurred vision, constipation, urinary retention, delirium, tachycardia
2. 5HT-3 Antagonists (Ondansetron, Granisetron):
a. Headache
b. Flushing
c. Epigastric warmth
d. Lightheadedness
e. Constipation / Diarrhoea
f. Elevated liver transaminases
3. Dexamethasone:
a. Acute hyperglycemia
b. Perianal pain on rapid injection
4. Antihistamines (Promethazine):
a. Sedation
b. QT prolongation
5. Anticholinergics (Hyoscine, Atropine):
a. Dry mouth
b. Constipation
c. Urinary retention
d. Tachycardia
e. Blurred vision
f. Delirium
6. Benzodiazepines (Midazolam):
a. Sedation
7. Ephedrine:
a. Tachycardia
b. Hypertension
8. Propofol:
a. Potential for inadvertent induction of anaesthesia
b. Sedation
c. Hypotension
May 2005 Question 10

What are the problems with the prone position for surgery?
Positioning:
• Patient must be turned 180 degrees under GA from supine into prone position
o Requires many team members and coordination
• Risk of dislodgment of:
o Endotracheal Tube
o IV cannula
o Intra-arterial line
• Risk of hypoxia:
o Unventilated patient due to deliberate circuit disconnection
• Temporary loss of monitoring
• Risk of spinal injury, especially if pre-existing conditions eg vertebral fractures

Access:
• Difficult access to airway, and intravascular devices
• Difficulty in re-establishing lost monitoring, eg ECG leads
• In emergency situation – CPR impossible and DCR difficult

Pressure Injury:
• Face:
o Eyes, nose, ears and lips
• Arms, breasts and genitalia
• Eyes:
o Intraocular pressures increased
o Risk of Visual Loss by:
• Traction on optic nerve by weight of eyeball
• Direct compression
• Hypotension

Nerve Injury:
• Increased risk of peripheral nerve injury
o Brachial plexus injury especially if shoulders excessively abducted
o Ulnar nerve injury
• Risk of shoulder dislocation / injury during positioning
• Excessive head rotation, extension and flexion may result in neurological injury, as well as decreasing cerebral blood flow and venous drainage

Physiological Derangements:
• CVS:
o Vena caval compression, leading to decreased venous return, cardiac output and blood pressure
o Increased venous pressures especially in epidural venous plexus, may lead to increased venous ooze / blood loss
• Respiratory:
o Increased FRC
o Improved V/Q mismatch, unless incorrect positioning
o Decrease in total lung compliance, and increased work of breathing
• Oedema of dependent areas:
o Facial and airway oedema
• Implications for re-instrumentation of airway
• Raised Intra-cranial and intra-ocular pressures
May 2005 Question 11

Discuss the management options for an epidural abscess.
Epidural abscess:
• Localised infection within epidural space
• Rare incidence
• May be spinal or cerebral
• May occur spontaneously or as a complication of an intervention, eg post neuraxial block

Immediate Management:
• Early diagnosis essential
• Full neurological examination
• Regular neurological observation to detect deterioration
• Bloods – FBE, CRP
• Blood cultures – to guide antibiotic therapy
• Urgent MRI – 92% sensitivity
• Neurosurgical referral

Subsequent Management:
• Conservative / Medical:
o Only when no or minimal neurological dysfunction, or when patient too unstable for surgery
o Infectious Diseases Referral early
o Antibiotic cover:
• Staph Aureus and Epidermidis most common pathogen
• Empirical therapy with Flucloxacillin 2g IV 6 hourly + Gentamicin 4-6mg/kg IV daily
• If prolonged hospital stay, indigenous or known MRSA, consider Vancomycin
• If post-bowel surgery, consider cover for Anaerobic pathogens, Metronidazole
• Targeted therapy once culture and sensitivities known
• Monitor WCC, CRP and ESR
• Antibiotics may need to continue for 4 – 6 weeks
o Radiologically guided drainage of abscess percutaneously in conjunction with antibiotics
• Surgery:
o Indications:
• Worsening neurology
• Persistent severe back pain
• Persistent fevers and infection markers
o Surgical decompression, usually posterior laminectomy, is the mainstay of treatment
o IV antibiotics are still prescribed for 4-6 weeks
May 2005 Question 12

Discuss the value of case reports to anaesthetists in the era of evidence based medicine.
Anaesthesia vs Other specialties:
• Anaesthetists are generalists, whereas other areas of medicine show subspecialisation
o This makes it more difficult to stay uptodate with rare diseases
• Anaesthesia does not provide therapeutic benefit
o May not be possible to conduct studies in some cases
• Anaesthetists have limited control over the patients medical condition prior to surgery
o Patients may have diseases or devices that previously had not been seen in patients presenting for surgery

Case Reports:
• Five categories:
o Complications
o Single case studies
o Case series
o Problem solving
o Intensive Care Reports

Advantages:
• Someone else’s accounts of anaesthesia where real lessons can be learnt, without the stress of personal experience
• Rare outcomes can be discussed:
o Unexpected complications:
• Involving patient, equipment or surgery
• May highlight previously unrecognized adverse reactions, which may inspire future research
o Rare concomitant diseases:
• Management and pitfalls
• Integration of references from journals which may not be read by all anaesthetists
o Post-operative issues which may not have been anticipated
• May offer novel approaches to problem solving
• Stimulate discussion in editorials and replies, and be a precursor to formal RCTs
• Can be written up and published quickly, as compared to meta-analyses or RCTs
• Useful in clinical contexts when it may be unethical to conduct a RCT, eg Dantrolene in MH

Disadvantages:
• Not considered to be strong evidence by some
o Classified as lowest level in NHMRCs ‘Levels of Evidence’
o Case report numbers steadily declining in journals
• They may unjustifiably bias future practice:
o This may be offset by reading references and formulating own opinion
o May bias studies which reference them in future publications

In summary, case reports are a valuable resource; they serve a useful role in stimulating further discussion and research. As long as their disadvantages are kept in mind, they have an important place in reputable anaesthesia journals.
May 2005 Question 13

How would you assess a patient’s thyroid function preoperatively at the bedside?
General history
• Previous surgery (e.g. thyroidectomy), other treatments for thyroid disease such as radio-iodine, anti-thyroid drugs or thyroid replacement treatment.

Signs and symptoms of hyper- or hypo- thyroidisms reflect the impact of excessive or deficient amounts of T3 and/or T4 on the speed of biochemical reactions, total body oxygen consumption, & energy (heat) production.

Hyperthyroidism
History
• Heat intolerance, weight loss, increased appetite, palpitation, increased sweating, nervousness, irritability, diarrhea, amenorrhoea, muscle weakness & exertional dyspnoea

Examination
General
• Weight loss, anxious appearance.
Hands
• Fine tremor
• Onycholysis (separation of the distal nail from the nail bed) & thyroid acropachy (clubbing)
• Palmar erythema (a red appearance of the outer parts of the palms)
• Warm/sweaty palms (from sympathetic overactivity)
Pulse
• Sinus tachycardia or atrial fibrillation.
• Collapsing pulse (high cardiac output)
Limbs
• Proximal myopathy (weakness of the muscles at the shoulders & hips)
• Abnormally brisk reflexes.
• Pretibial myxoedema (bilateral firm & elevated nodules & plaques on the shins which may be pink or brown)
Eyes
• Exophthalmos (protrusion of the eyeball out of the orbit)
• Lid retraction
• Lid lag
• Proptosis
Neck
• Thyroid enlargement.
Chest
• Auscultation- systolic flow murmurs
• Signs of cardiac failure (raised JVP, displaced apex beat, pulmonary crepitations)

Hypothyroidism
History
• Cold intolerance, weight gain, lethargy, constipation.

Examination
General
• Mental & physical lethargy.
Skin
• Peripheral cyanosis
• A cool, dry & thickened skin
• Yellow skin discoloration of hypercarotenaemia (a result of reduced metabolism of carotene).
• Alopecia
• Vitiligo
Pulse
• Slow rising pulse, small volume
Eyes
• Periorbital oedema
• Xanthelasma
• Loss or thinning of the outer third of the eyebrows.
Voice
• Coarse, croaking, slow speech.
Reflexes
• Tinel’s sign (tapping over the flexor retinaculum at the wrist causing tingling in the distribution of the median nerve).
• ‘Hung up’ ankle reflex (with the patient kneeling on a chair, the foot plantar flexes briskly when the Achilles tendon is tapped but then dorsiflexes much more slowly).
May 2005 Question 14

A recovery charge nurse approaches you as a Supervisor of Training because she is concerned at the amount of opiates one of your trainees has been signed out for patients. What will be your priorities in addressing the nurses concern?
Main issues:
1. Ensuring patient safety in the context of a potentially impaired doctor
2. Anonymity of complainant
3. Potential for career destruction and self harm by trainee.
Appropriate response will balance these two conflicting aims.

1. Acknowledgement of the nurse’s concern
• ensure confidentiality
• give adequate time and provision for nurse to detail nature of allegation
• reinforce that this is a serious matter that requires immediate review
2. Patient safety/well being
• Has to be addressed immediately
• Ensure that the trainee is not alone for lists, all the patients concerned are appropriately managed both during anaesthesia & in recovery
• Involve rostering support for this, whilst maintaining confidentiality
• Longer term considerations
• The trainee should be relieved of clinical duties if a judgment is made by the head of department that there is danger to patients until such time as the person is deemed safe and ready to re-enter the workforce.
• A departmental policy should be directed towards discreetly investigating a report of drug misuse.
3. Investigating claim
• A departmental policy should exist towards discreetly investigating any report of drug misuse.
• Checking of written records & drug stocks; checking patient files, corroborating with counter-signer for drugs of dependency.
• Markers of substance abuse
• Signing out increasing and inappropriate quantities of drugs.
• inaccurate, illegible or unusual record keeping
• a pattern of complaints regarding excessive pain in patients of a particular anaesthetist (out of proportion with purported analgesia delivered)
• change in attitude or behaviour, pin-point pupils, weight loss
• observation of intoxicated behaviour; withdrawal symptoms; pills, syringes, ampoules in non-work place environment; injection marks on the body etc.
• increased sick leave or other absenteeism.
4. Pre-planned confrontation/”intervention team”
• An intervention team should be appointed including a drug professional
• The meeting should be planned as regard to strategy, venue and timing.
• The trainee should be encouraged to attend with a supporter or advocate.
• The evidence should be described in detail without personal judgment.
• The trainee should be reassured of confidentiality and ongoing support during detoxification and re-entry to the workforce.
• Identification of resources available to assist the trainee
• acceptable to ask for help (Head of Department, Senior colleague, Hospital Substance Abuse Committee, Doctor’s Help Advisory Service, Psychiatrist/Psychologist, ANZCA, Welfare of Anaesthetists, SIG, etc).
• Thereafter the person must not be left alone; immediate specialized help must have been arranged prior to the interview. The potential for suicide is high.
• Pertinent documentation is necessary but must be completely confidential.
5. Notification of appropriate authorities (Head of Department, Hospital Administration, Medical Board, Police etc may be appropriate).
• The Head of Department should be notified from the beginning, and the Hospital Administration pending on the outcome of the investigation.
• The Medical Board should be informed of all proceedings and undertakings. If there is no co-operation the police may need to be informed.

Relevant ANZCA professional statements should be adhered to (PS 49- Guidelines on the health of specialists and trainees. TE 18- Guidelines for assisting trainees with difficulties).
May 2005 Question 15

Discuss the advantages & disadvantages of intra-operative blood salvage.
Intra-operative blood salvage describes the technique of collecting and reinfusing blood lost by a patient during surgery. Broadly, two types of devices:
1. Purely haemofiltration
2. Incorporating washing of blood

The devices consist of suction tubing, filter, reservoir, and a washing bowl that is centrifuge-driven. Anticoagulant (either heparinised saline or citrate) is added to blood sucked from the surgical site to prevent clotting. The anticoagulated blood can then be washed in saline and the final product consists of RBCs suspended in saline, with haematocrit of 50-60%, which is transfused back to the patient.

Advantages

1. Decreased allogeneic blood requirements by 40%
2. Concomitant reduced risk of allogeneic blood (BBV, ABO incompatibility)
3. No change in neurological/CVS/immunological outcomes (Cochrane r/v 2006)
4. The oxygen-transport properties and the survival of recovered RBCs are equivalent or superior to those of stored allogeneic RBCs.
5. Acceptable to some Jehovah’s witness patients by a continuous circuit device: seek specific consent
6. Conservation of scarce blood resources
7. Cost effective (particularly if used frequently)

Disadvantages

1. Contraindications exist
- septic wound contamination (can employ after bowel/wound decontamination)
- malignancy (not recommended by manufacturers, BUT NICE guidelines
approve for urological malignancy. No change in survival/recurrence. Use
a leucodepletion filter)
- obstetrics. Theoretical risk of amniotic fluid embolism. No reports. Use dual
suction/leucodepletion filter. If Rh +ve fetus/-ve mother, will need anti-D after
- sickle cell disease/trait. Manufacturers do not recommend (but case reports of
use exist)
2. Risk of air embolus
3. 'Salvaged blood syndrome'- low grade DIC (microemboli/tissue factor activating microcirculation)
4. Acute lung injury described
5. Acute renal injury due to haemolysis of recovered blood
o Patients exhibit a level of plasma-free haemoglobin that is usually higher than after allogeneic transfusion.
o High concentrations of free haemoglobin may be nephrotoxic to patients with impaired renal function
6. Potential for bacterial contamination
7. Washing causes loss of platelets and clotting factors in the processing, coagulopathy can result
8. Not readily available. High initial cost of machinery and need for trained operators
9. Complex cost-effective argument. To be cost effective, salvage usually requires blood losses greater than 1000-1500ml
September 2004 Question 1

Discuss the advantages and disadvantages of the methods used to avoid hypothermia in the operating theatre
Pre-warming:
+ Easy to achieve with Bair Hugger, cost efficient, may provide some anxiolysis. Minimises hypothermia in theatre post-induction, i.e. decreases heat loss post-induction when patient is exposed
- Bair Huggers may make some patients uncomfortable / feel too warm, requires additional organisation to prewarm patient ½ hour before surgery, may need extra staff

Increasing temperature in OT
+ More comfortable for patients and helps to keep patients warm
- uncomfortable for other staff (esp surgeons in their gowns)

Use of warm fluids/fluid warmers:
+ helps to maintain core temperature and keep patients warm especially if transfusing blood eg massive transfusion in trauma
- not freely available, requires extra equipment and tubing, increased expense
- Fluid bags left for extended periods in warmers may become contaminated or damaged.

Forced air warmers (Bair Huggers):
+ effective, able to actively warm, minimally expensive
- risk of overheating, potential of contaminating surgical field if inflated prior to sterile field not being placed, noisy

Covering the patient:
+ helps to minimise heat loss
- unable to actively warm, inexpensive, no special equipment needed, unable to assess colour
September 2004 Question 2

Outline the possible causes of postoperative loss of vision
Postoperative vision loss after non ocular surgery is a rare complication.
• Incidence 0.1-1%
• More common after spinal surgery in the prone position (67%), cardiac surgery
• Causes
o Most common cause is ischaemic optic neuropathy (81%), followed by retinal artery thrombosis
• Inadequate perfusion pressure
• Severe hypotension intraoperatively eg haemorrhage
• Local vascular disease eg carotid stenosis
• Increased resistance to blood flow
• Local arterial disease eg atherosclerosis
• Increased blood viscosity eg polycythaemia, sickle cell disease
• Increased ↑ IOP eg increased external pressure, increased CVP (head down position), venous engorgement
• Embolism
• Decreased O2 carrying capacity eg severe anaemia
• Hypoxia
o Others
• Retinal artery occlusion - thromboembolic
• Cortical blindness - stroke
• Venous obstruction
• Injury
• Mechanical
• Drying out
• Laser surgery
- eye block complications
- drugs
- TURP syndrome with glycine
Management
1) Adequate consent in high risk patients (prone surgery, glaucoma, HT, diabetes, atherosclerosis, smoking)
2) Careful positioning (10% reverse Trendelenburg in prone position, avoid direct pressure on eyes)
3) Avoid hypotension, higher transfusion trigger
September 2004 Question 3

What are the problems with using the beach chair position for shoulder surgery?
The beach chair position is increasing in popularity in parallel with surgical procedures on the shoulder. The key benefits are that it allows anterior and posterior access to the shoulder, with the upper extremity freely mobile.

Reduced CPP
- The head/brain is at a level higher than the heart.
- The difference in MAP can be 20-30mmHg lower compared to the heart, depending on how upright the patient is.
- There can also be an increase in venous pressure as a result of jugular veins being compressed by immobility masks and tapes.
- CPP depends on MAP as well as venous pressure/intra-cranial pressure. (CPP = MAP – ICP or CVP whichever is higher) Reductions in MAP and increases in venous pressure / ICP can reduce cerebral perfusion pressure and lead to cerebral ischaemia.
- This can result in focal or generalised neurological deficits.

Difficulty in estimating MAP
- Because of the importance of estimating and maintaining CPP many anaesthetists choose to insert arterial lines.
- If the arterial line is not zeroed appropriately at the right level however it can overestimate the MAP.
- The best way to overcome this is to attach an extension to the arterial line such that the transducer can be placed at the level of the heart or at the level of the head to ensure an accurate MAP reading.

Reduced access to airway
- The patient’s head is often covered under drapes and not easily accessible, as such the airway needs to be secured.
- There is a potential for laryngeal mask airways to be dislodged with surgical manipulation of the shoulder and therefore most anaesthetists choose an ETT as the airway of choice.
- Even with an ETT, extreme movements of the head and neck can cause disconnections of the circuit / dislodgements / kinking of the ETT hence careful monitoring is required.

Extreme movements of the head
- Extreme rotation of the head away from the operative side can result in stretching and injury of the brachial plexus.
- If the head is not fixed (using immobility masks and/or tape) it could roll from side to side with surgical manipulation with the potential for injury. Beware surgical pressure on the orbit as well as pressure on other structures of the face (ears, nerves).

Bradycardia/Asystole
- The large reduction in venous return as a result of venous pooling in the legs can cause a reflex bradycardia or even asystole.
- This has been reported in young males.
- Ensure slow tilting / positioning of table.
Hypothermia
- Although this problem is not limited to the beach chair, this is an important problem that can be addressed by the placement of forced air warmers.

Air embolism
- The surgical site is above the level of the heart and so there is a possibility for air embolism if a vein is open to atmospheric pressure.
- This risk can be reduced by continuous irrigation of the surgical site in open procedures.

Loss of vision
- Due to the reduction in MAP to the optic nerve and retina (or rarely visual cortex) ischaemic optic neuropathy can result unless MAP is maintained.

Sacral pressure
- In prolonged cases pressure areas can result as the majority of pressure is exerted on the patient’s sacrum leading to hypoperfusion of the tissues (skin and underlying muscles).

Risk of patient sliding off table
- If the patient is not adequately secured on the table or if the lower limbs are not elevated there is a potential that the patient may fall off the table leading to injury.

Deep venous thrombosis
- Dependent venous pooling in the lower limbs can result in the formation of thrombus.
- The risk can be reduced by the use of non-pharmacological (TED stockings, calf stimulators) as well as pharmacological (heparin/enoxaparin) means.
September 2004 Question 4

Outline the diagnostic criteria for an epidural abscess.
The anatomy of the spinal canal and dura mater determines many features of epidural abscesses. Above the foramen magnum, the dura is adherent to the bone. Below, there is an actual or true epidural space, posterior and lateral to the spinal cord and extending down the length of the spinal canal. This space, which is very small in the cervical region and larger in the sacral region, is filled with fat as well as arteries and a venous plexus.
Anterior to the cord, there is only a potential epidural space because the dura is adherent to the vertebral bodies from the foramen magnum down to L1. As a result, the majority of SEAs are located posteriorly; when anterior SEAs occur, they are usually below L1. SEAcan arise spontaneously but in anaesthesia are to be considered as a complication or neuraxial anaesthesia. The Swedish study published in Anesthesiology - 01-OCT-2004 followed 1,260,000 spinal blockades and 450,000 epidural blockades and found 127 complications found included spinal hematoma (33), cauda equina syndrome (32), meningitis (29), epidural abscess (13), and miscellaneous (20). Permanent neurologic damage was observed in 85 patients.

Risk factors that are reported include: increasing age, diabetes, poor sterility in catheter placement, IVDU, prolonged length of use of epidural catheter

SEA should be suspected in any patient who has had spinal or epidural anaesthesia and presents with fever with severe, localized back pain, especially if the pain is worsened by percussion. Nuchal rigidity is often present. As the condition progress there may be signs and symptoms of cord compression including loss of motor or sensation to the lower limbs and fecal and urinary incontinence.

Once suspected, MRI is the preferred test because it is positive early in the course of the infection and provides the best imaging of the inflammatory changes. CT scanning with gadolinium contrast is an acceptable alternative. Plain radiographs of the spine may reveal changes of osteomyelitis or discitis, but are rarely diagnostic of SEA. Myelography is largely obsolete. FBC and ESR may show changes but these are not diagnostic.
MRI reveals two basic patterns. First, the phlegmonous stage of infection results in homogenous enhancement of the abnormal area. Pathologically, this correlates to granulomatous-thickened tissue with embedded microabscesses without a significant pus collection. The second pattern is that of a liquid abscess surrounded by inflammatory tissue, which shows varying degrees of peripheral enhancement with gadolinium.
Once an intraspinal mass that could be an SEA has been located, it is important to try to isolate the etiologic organism from blood, abscess contents, or cerebrospinal fluid (CSF). The approximate frequency of positive cultures from these sources in SEA is:
Abscess contents — 90 percent
Venous blood — 62 percent
CSF — 19 percent positive (but Gram stain of CSF is almost always negative [6])
Two blood cultures should be drawn. The best specimen for culture is fluid or pus obtained by direct needle aspiration, usually under CT guidance, of the epidural abscess or inflammatory mass. CSF often is not obtained because lumbar puncture is not needed (or is even contraindicated) in most cases.

Treatment: Can be either conservative or surgical.
Conservative Treatment has been reported in some studies to be as effective as surgical intervention however it is generally not considered unless the MRI diagnosis and treatment institution has occurred prior to the onset of neurological signs. Emperical antibiotic therapy includes
Nafcillin (1.5 g every four hours) plus metronidazole (500 mg every six hours) plus either cefotaxime (2 g every six hours) or ceftazidime (2 g every 12 hours)
Vancomycin (1 g every 12 hours) can be substituted for nafcillin if the patient is allergic to penicillin or if infection with methicillin-resistant S. aureus is considered likely
The usual duration of therapy is four to six weeks, or until resolution of the SEA on MRI
In addition to antibiotic therapy, a conservative approach includes the following steps:
A careful monitoring strategy is identified prior to initiating treatment
A follow-up MRI should be performed to confirm diminishing size or resolution of the abscess
Immediate surgery is warranted if neurologic deterioration occurs
September 2004 Question 5

Discuss the problems with Clopidogrel in the perioperative period.
Clopidogrel:
• A thienopyridine derivative
• A prodrug, which after biotransformation in the liver to active metabolites, inhibits ADP-receptor induced platelet aggregation irreversibly

General problems:
• Increased bleeding tendency
• Irreversibility of action, and duration of action is for the life of the platelet (7-10 days)
• Inhibits CC-P450 isoenzymes, and may potentiate action of drugs including aspirin and sedatives
Preoperative:
• Indications for use:
o Prevention of in-stent thrombosis in coronary stents, along with aspirin
• BMS – Only life-saving procedures be performed until 6-12 weeks have lapsed
• DES – Only life-saving procedures be performed until 1 year has elapsed
o Secondary prevention of MI, along with aspirin
o Secondary prevention of ischaemic cerebrovascular events, in patients intolerant of aspirin
• Need for cessation:
o Depends on proposed surgery and anaesthetic
o Should ideally be ceased for 7-10 days for:
• All but minor surgery
• Neuraxial and peripheral nerve blocks
o Number of new functional platelets will be ~30 x 1012 per day
o No accurate tests for platelet function monitoring
• Other Antiplatelets agents:
o Aspirin should be continued wherever possible

Intraoperative:
• Potential to increase bleeding and requirement for blood transfusion
o Depends on site of surgery
o Increased risk with intracranial, cardiac and spinal surgeries
o Ease of surgical access to obtain hemostasis
o Difficult with laparoscopic and endoscopic procedures
• Careful monitoring of intraoperative blood loss and institute measures to deal with unexpected bleeding
• Anaesthetic Technique:
o Increased risk of bleeding with Neuraxial and Peripheral nerve blocks:
o Compression of nerves / roots / spinal cord due to formation of hematoma
o Risk further increased if concomitant use of aspirin or nsNSAIDs (?)
o Careful assessment of risk / benefit ratio for a particular patient
• Therapies to minimise risk of bleeding:
o Antifibrinolytics used in cardiac surgery (tranexamic acid, aprotinin) to reduce bleeding
o DDAVP is effective only in patients with major bleeding syndrome, and poses a thrombotic risk
o Prophylactic platelet transfusions are not recommended, but platelets should be available immediately

Post-operative:
• Restarting clopidogrel:
o Must be in liaison with surgeon and physician
o Once surgical hemostasis achieved
• May increase risk for post-operative bleeding
• All patients must receive thrombo-embolic prophylaxis with LMWH
September 2004 Question 6

What are the considerations in setting the fresh gas flow rate when anaesthetising an adult with sevoflurane and nitrous oxide using a circle absorber system?
Fresh Gas Flow (FGF) Rate:
• The volume of fresh gas (mixture of oxygen, nitrous oxide and sevoflurane) delivered per unit time to the common gas outlet
o Measured in Litres per minute (L/min)
• Considerations when determining FGF rate:
o Sevoflurane delivery
o Nitrous oxide delivery
o Oxygen delivery > oxygen consumption
o Carbon dioxide elimination > production
o Circuit Leaks
o Degradation products
o Costs – financial and environmental

Sevoflurane delivery:
• Higher flow rates are required when changing dosing of sevoflurane:
o During induction and emergence
o When manipulating anaesthetic depth in accordance with surgical stimulation
o The rate of equilibration is proportional to the flow rate, and inversely proportional to the circuit volume (~6L)
• Vaporiser calibration:
o Flow rates set outside a vapourises calibration range may affect the concentration of sevoflurane delivered
o Extra vigilance is required to ensure delivered concentration is similar to ‘dialled’ concentration

Nitrous oxide delivery:
• Higher flow rates are required when changing dosing of nitrous oxide
o Nitrous oxide is an insoluble anaesthetic gas, however, it is delivered in a much higher concentration (upto 70%) as compared to sevoflurane, therefore the dose delivered is much greater
o A higher gas flow is required for nitrous oxide dosing than for sevoflurane dosing, due to this reason

Oxygen consumption:
• Oxygen consumption at rest is 3.5ml/kg/min (perhaps lower in an anaesthetised patient)
• An 80kg patient would require 280ml of oxygen per minute
• FGF must be greater than the oxygen consumption (to prevent delivery of hypoxic mixture)

Carbon dioxide production:
• Carbon dioxide is a by product of patient metabolism, and normally absorbed by CO2 absorber
o If absorber fails, FGF must be increased to minimise FiCO2 and thus prevent rebreathing of expired CO2

Circuit leaks:
• FGF rate must be greater than the rate of circuit volume leak, to continue to allow pressurisation of circuit
• The maximum acceptable leak in the breathing circuit is 150ml/min (ANZCA guidelines)

Degradation products:
• Compound A is produced by interaction of volatile anaesthetic with CO2 absorber
o Low FGF will increase concentration and theoretical toxicity of Compound A (a nephrotoxin in rats)
• Carbon monoxide is produced by interaction of volatile anaesthetic with CO2 absorber
o Not a problem with sevoflurane.

Conserving heat and moisture:
• Heat from the patient is diluted by use of high FGF
• Moisture in the circuit is supplied by patients lungs and CO2 absorber, and is lost by use of high FGF

Financial costs:
• High FGF rates will increase usage of sevoflurane (oxygen and nitrous oxide are cheap) and therefore anaesthetic costs

Environmental costs:
• High FGF increases the amount of sevoflurane release into atmosphere (via scavenging), and the cumulative effects on ozone layer may be detrimental.
• Nitrous oxide is a greenhouse gas!
September 2004 Question 7

Desribe the pathophysiology and diagnosis of diabetes insipidus following head injury.
Diabetes Insipidus (DI):
• May be nephrogenic or neurogenic
o Nephrogenic – Insensitivity of renal tubules to ADH
o Neurogenic – Reduced or absent ADH production or secretion from the posterior pituitary gland
• Net result – impaired water resorption by the kidney
• Neurogenic DI may be due to head injury, metastases, vascular lesions, sarcoidosis, meningitis, pituitary tumours, inherited or idiopathic causes.

Pathophysiology:
• ADH is normally produced in the hypothalamus and stored in the posterior pituitary before being secreted into the blood
• ADH acts at the renal collecting ducts to increase water permeability and water reabsorption
• Absence of ADH reduces the ability of the kidney to concentrate urine, resulting in:
• Large volumes of dilute urine (polyuria / nocturia)
• Plasma hyperosmolality and dehydration
• Polydipsia, as a compensatory mechanism
• There may be three phases:
• Polyuric phase – Reduction in ADH causes increase in urine volume and a fall in urine osmolality, which lasts 4-5 days
• Antidiuretic phase – Release of stored ADH results in rise in urine osmolality, which lasts 5-6 days
• Permanent phase – When stores are exhausted and the cells that produce ADH are absent or unable to further produce ADH

Diagnosis:
• Clinical:
o Polyuria (suspicion if urine output >50ml/kg/day, but usually >90ml/kg/day)
o Low urine osmolality (<200 mOsm/kg)
o Urine specific gravity < 1.005
o High plasma osmolality (>287 mOsm/kg)
o Normoglycemia
o Normal or hypernatraemia
• Tests:
o Water Deprivation Test (Miller-Moses Test)
• If fluid deprivation does not result in concentrated urine (>300 mOsm/kg), then DI suspected
o Response to DDAVP:
• Differentiates neurogenic from nephrogenic DI
• Urine osmolality repeated 1-2 hours after DDAVP 5 units
• An increase of >50% in urine osmolality indicates complete neurogenic DI, whereas a smaller or absent response is seen in nephrogenic DI.
• In normal patients, <9% increase in urine osmolality following DDAVP
o MRI:
• With pituitary and hypothalamus views
September 2004 Question 8

Describe the anatomy relevant to providing an ankle block for surgery on the big toe.
Ankle block for surgery no big toe:
• Requires the blockade of three nerves at the ankle:
o Posterior Tibial nerve
o Deep Peroneal nerve
o Superficial Peroneal nerve
• Due to dermatomal overlap, the Saphenous Nerve may also require blockade

Posterior Tibial Nerve:
• Sensory supply:
o Sole of foot, with exception of extreme lateral and proximal segments
• Blockade:
o The nerve lies dorsal to the posterior tibial artery, dorsal to the medial malleolus
o If artery cannot be palpated, this is one-third between the medial malleolus and the heel
o Insert needle perpendicular to skin, withdraw 1-2 mm when contact with bone, and then inject 5-8 ml of LA solution

Deep Peroneal nerve:
• Sensory supply:
o Skin on the dorsum and between the great and second toes
• Blockade:
o Nerve is in close proximity to the dorsalis pedis artery on the dorsum of the foot, near the ankle joint plane
o Needle inserted between the tendon of extensor hallucis longus and dorsalis pedis artery (LATERAL TO ACTUALLY), on dorsum of foot
o After contact with bone, withdraw 1-2 mm and then inject 5ml of LA solution

Superficial Peroneal nerve:
• Sensory supply:
o Skin on the dorsum of the foot, except skin supplied by deep peroneal nerve
• Blockade:
o Nerve lies subcutaneously across the dorsum of the foot
o Subcutanoeus infiltration of LA solution (7-10ml) in a line joining the medial and lateral malleoli on the dorsum of the foot

Saphenous nerve:
• Sensory supply:
o Skin on medial aspect of foot, from medial malleolus to great toe (variable)
• Blockade:
o Nerve lies close to the great saphenous vein, anterior to medial malleolus
o Subcutaneous infiltration of 7-10 ml of LA solution from the anterior edge of the tibia to the Achilles tendon, approximately a hand-width above the medial malleolus.
September 2004 Question 9

Give reasons for your choice and dose of local anaesthetic agent to provide intravenous regional anaesthesia for a reduction of a Colles' fracture in an 80 year old woman weighing 95kg.
This is an elderly, obese patient with a common fracture amenable to intravenous regional analgesia, or Bier’s block, for reduction.

Pharmocokinetic/dynamic considerations
Obese patient- large Vd for lipid soluble drug
Decreased metabolism and clearance due to aging
Reduced protein binding with aging (if renal/hepatic disease) increasing free fraction of drug
Minimal cardiorespiratory reserve in the event of inadvertent local anaesthetic toxicity

Choice of agent
Prilocaine for the following reasons:
1. Rapid onset of action (within 5-10 mins) due to its low pKa 7.9. This is comparable with lignocaine
2. Safer toxic profile due to its lower protein binding (55%, compared with lignocaine 65% or bupivacaine 95%; and higher hepatic clearance 2.37 L/min, compared with lignocaine 0.95 L/min or bupivacaine 0.47 L/min)
3. Relatively short duration of action due to rapid clearance allows for early neurological assessment in the injured hand
4. Some analgesic effect after reduction
5. Allergic reactions to amide local anaesthetics are rare
6. Inexpensive, stable solution
7. Does not require mixing with HCO3/adrenaline. Less error potential

Disadvantages
1. Potential to cause metHb via its metabolite o-toluidine- usually only in doses in excess of 600 mg, or 6 mg/kg. (treat with 1-2 mg/kg methylene blue)
2. Unfamiliarity with use, particularly in emergency departments

Lignocaine is a reasonable alternative if prilocaine is not available
Bupivacaine and ropivacaine should NOT be used – reports of cardiotoxicity with bupivacaine!

Dose of agent
3 mg/kg of 0.5% prilocaine (= 0.6 ml/kg) without adrenaline
Calculation of dose should be based upon the patient’s ideal body weight, which is likely to be closer to 70-80 kg. Roughly 42-48 mls
Toxic dose of prilocaine is 3-4 mg/kg
Alternatively, lignocaine 0.5% 3 mg/kg (= 0.6 ml/kg) may be used
Or, dosing based on size of arm (with attention paid to maximum dosing as above):
40mLs for small arm = 200mg
50mLs for medium arm = 250mg
60mLS for large arm 300mg
September 2004 Question 10

Justify the use of a laryngeal mask airway in a 25 year old, 80kg man having general anaesthesia for the removal of 4 molar teeth.
The LMA is a supraglottic airway device that is designed to provide and maintain a seal around the laryngeal inlet and facilitate spontaneous ventilation as well as controlled ventilation at modest levels of positive pressure (≤ 20 cmH2O).

Preconditions:
• pt fasted
• no significant GOR
• difficult airway not predicted (controversial- some would advocate use in this setting)
• no associated major cardioresp morbidity (eg. severe OSA)
• dental surgeon is comfortable working around the LMA

Safety
• Patient
o This is a low risk case in a patient who is otherwise likely to be fit and well, without other indications for intubation
• Surgical
o Procedure likely to be of short duration
o If surgeon is comfortable working around the LMA, surgical risk should not be altered. Reinforced LMA though technically more difficult to insert has reduced risk of kinking, can be manipulated out of surgeon's way.
o Airway protection – the LMA when applied correctly provides adequate protection of the airway from loose teeth and blood, especially when used under direct vision of the dental surgeon as in this case. A throat pack can also be used.
• Anaesthetic
o Immediate access to the airway should be achievable if required
o Minimal opioid requirement with local anaesthetic infiltration by dental surgeon will enable a SVGA.
o IPPV if required has been safely employed with the LMA ( Can J Anaesth. 2000 Jul;47(7):622-6.)
o Application of a small amount of PEEP if required can be applied to aid oxygenation.
o Should not require postop ventilation, thus no long term intubation requirements

Advantages of LMA over ETT:
• Insertion
o Easier to insert (90% success first attempt – BJA 2002)
o causes less soft tissue trauma (including laryngopharynx and trachea) or dental trauma and thus less postop sore throat (20-30% vs 40-60%)
o no laryngoscopy, thus less haemodynamic consequences with insertion
o does not require muscle relaxants and thus no risk of adverse drug reaction to muscle relaxants, less risk of an airway emergency, less PONV with avoidand of reversal
• Recovery
o can take pt to PACU with LMA in situ for removal once conscious, improving efficiency/patient turnover
o causes less airway irritation (coughing / straining) than ETT on awakening
o faster recovery time (J Oral Maxillofac Surg. 2002 Jan;60(1):2-4)
• Economy
o reuseable and thus cheaper

Disadvantages
• Not all dentists are happy to work around an LMA, may not provide adequate surgical access
• Does not protect against aspiration
• May need to be moved during surgery, with concomitant risk of dislodgement. The consequences are: a) potential for airway obstruction; and b) reduced integrity of the seal
• May not sit well patients with an abnormal pharynx or larynx anatomy
• Laryngospasm and bronchospasm may still occur if LMA is inserted in light plane of anaesthesia
• If bite block not used, risk of biting on LMA on awakening resulting in negative pressure pulmonary oedema, dental trauma or airway obstruction (case report of patient biting LMA in half).
September 2004 Question 11

What is the role for radionuclide imaging in the assessment of ischaemic heart disease prior to general anaesthesia for non-cardiac surgery?
The presence of IHD increases the risk of perioperative morbidity and mortality for non-cardiac surgery under GA. It is one of six risk factors in the revised cardiac risk index used by the AHA/ACC to stratify perioperative cardiac morbidity (1. Hx of IHD 2. Hx of prior/compensated heart failure 3. Hx of cerebrovascular disease 4. Diabetes 5. Renal insuficiency 6. High risk surgery),

Technicalities
• Cardiac radionuclide imaging is a method of imaging the heart using radioactive isotopes (usually either Thallium-201 or Technetium-99m) to assess cardiac perfusion, structure and function whilst at rest and after physiological or pharmacological stress. This is induced either by:
1. Hyperaemia (with eg. dypyrimadole or adenosine)- potent coronary vasodilator, causes differential flow between normal/stenosed arteries
2. Exercise thallium scintigraphy
• A gamma camera obtains images of myocardium after injection of the tracer
• Decreased uptake of radioisotope after stress indicates an area that is potentially at risk of myocardial ischaemia and/or infarction. The presence of several of these areas indicates the need for coronary revascularisation.
• High negative predictive value (99%), but poor positive predictive value (10-20%).

Information provided
• Perfusion:
- Uptake of radioisotope into perfused myocardium
- Hence radioisotope distribution is proportional to regional blood flow
- Areas of no uptake (i.e. perfusion defects) may be fixed indicating previous infarct, or reversible indicating myocardium ‘at risk’
• Structure:
- Size and thickness of myocardium e.g. LV dilation, post-infarct wall thickness
• Function:
- Ejection fraction

Indications
• ECG not interpretable – eg PPM, LBBB, RBBB
• Unable to exercise to >85% predicted maximal HR
• Exercise stress test gives equivocal results
• Patient has known intermediate lesion (25-75% major vessel stenosis) & functional significance uncertain
• Non-invasive cardiac testing (which may include radionuclide imaging) is recommended by reference to the AHA/ACC guidelines where:
1. 'Active cardiac conditions' prior to noncardiac surgery
• Unstable coronary syndromes (unstable/severe angina/MI within 30 days)
• Decompensated heart failure (NYHA class 4/worsening/new onset)
• Arrhythmias (Mobitz 2/3rd degree/high grade AV block/symptomatic bradycardias or ventricular arrhythmias)
• Valve disease (severe AS/symptomatic MS)
2. 3 or more clinical risk factors, exercise < 4 METS, prior to vascular surgery (Class 2a)
3. 3 or more clinical risk factors, exercise < 4 METS, prior to intermediate risk surgery (Class 2b)- consider if it will change management.
4. 1 or 2 clinical risk factors, exercise < 4 METS, prior to vascular surgery or intermediate risk surgery (Class 2b)

Not indicated
• Emergency surgery
• Coronary revascularisation in last 5 years with no symptoms
• Recent coronary evaluation with favourable result, no symptoms

Benefits
• Identifies patients who may benefit from delayed surgery and revascularisation prior, either by PCI or CABGS
• A reversible myocardial perfusion defect predicts perioperative events, and a fixed defect predicts long-term cardiac events including death
• Cardiac risk significantly increases with a moderate sized defect (20-25% of LV mass)
• Radionuclide imaging has been shown to have a very high NPV (of predicting perioperative ischaemic event) of ~99% while the PPV is much lower at 4-20%. This reflects the relatively low incidence of perioperative ischaemic events even in patients with evidence of IHD (e.g. 1-5% of patients with intermediate clinical predictors)
• For patients with LBBB/LVH with strain/digitalis effect, pharmacological stress scintigraphy superior to exercise stress perfusion imaging due to septal perfusion defects not due to coronary artery disease
• Obese patients in whom echocardiography images of poor quality

Disadvantages/alternatives
• Patients who are ambulatory and able to exercise with a normal resting ECG (most patients) should have an exercise stress ECG. In the presence of an abnormal resting ECG, an exercise stress echo or radionuclide imaging is indicated
• If unable to exercise then a pharmacological stress test is indicated (using either dipyridamole thallium or dobutamine/pacing stress echocardiography). Whether a radionuclide imaging test or echocardiography is superior is controversial
• Dipyramidole/adenosine precluded with significant bronchospasm/critical carotid disease/
• Echocardiography gives additional information about valvular defects
September 2004 Question 12

What are your obligations if you suspect a colleague to be chronically impaired?
Definition
• Inability to safely and competently practice anaesthesia at a standard appropriate for a given environment
• Various potential causes include but not exclusively
o Drugs
o Alcohol
o Mental illness
o Physical illness
o Age
o Fatigue
o Lack of continued medical education
o Unprofessional behaviour
o Personality disorders – impedes integration in a team
o Anger management issues

Primary obligation is to maintain patient safety
• Effective removal of patients from this colleague’s care is urgent
• Ensure that patient is stable, ensure no permanent sequelae
• A diplomatic approach is paramount

Secondary obligation is to colleague who is impaired
• At risk to themselves – if exhibiting lack of insight or dangerous / reckless behaviour eg driving after long 24 hour shift/risk taking behaviour
• At risk of suicide- especially in immediate aftermath of confrontation
• Procedure:
1. Collect objective evidence of chronic impairment from:
- Own observations or those of reliable staff (e.g. anaesthetic nurses, recovery nurses)
- Department appraisals
2. Pre-planned confrontation
- In general, it is best to avoid 1:1 confrontations unless I know this colleague on a personal level
- Otherwise a formal meeting should be planned as regard to venue, timing and strategy, where the evidence is described in detail without personal judgement
- The colleague should be reassured of confidentiality and ongoing support
3. Plan for colleague post-confrontation
- As there is a high risk of suicide, professional help must be enlisted to provide support after the meeting
- A list of resources are provided depending on the type of impairment e.g. hospital substance abuse committee, Doctor’s Help Advisory Service, Psychiatrist/Psychologist, ANZCA, Welfare of Anaesthetists, SIG etc.
• Colleague may also be at risk to their family depending on impairment
• Public may also be at risk – eg if engaging in dangerous practices, eg drink driving, drug-related matters, criminal offences. Police involvement may be necessary

Further obligations are to the department/other staff/college/medical board
• Impaired colleague should be discussed with director of department- ultimate responsibility for course of action
• Medicolegal implications for department/hospital
• ANZCA has policies in place (PS 16- Statement on the standards of practice of a specialist anaesthetist; covers issues such as fatigue/drug dependence).
- Early involvement may be appropriate
- College can appoint mediator/mentors as appropriate
- College will also coordinate return to workforce/counselling etc
• Medical practioner's board have vested interest in standards of practice/public protection
• Other staff- debriefing/forum to air concerns/complaints.
September 2004 Question 13

Describe the technique of applying cricoid pressure to prevent regurgitation of gastric contents.
Introduction
• A manouevre performed as part of RSI to prevent passive regurgitation of gastric contents in patients at high risk for aspiration
• Digital pressure is applied against the cricoid cartilage, pushing it backwards and thus compressing the oesophagus between the posterior aspect of the cricoid and the vertebral bodies behind
• The cricoid is used because it is the only complete ring of cartilage
• Technique originally described by Brian Arthur Sellick in the Lancet 1961, and is known as Sellick’s manoeuvre
• To date, there have been no studies proving that cricoid pressure is truly beneficial


Technique
1. Help assistant confirm the position of cricoid prior to induction
- Found at level of C6, just below the thyroid cartilage (‘Adam’s apple’) which elevates on swallowing, and the cricothyroid membrane, which is a depression
- The cricoid is stabilised between the thumb and middle finger while the index finger pushes it down. The force should be applied directly backwards
- No significant difference between right and left handed applied cricoid. But if assistant is right handed, may be able to apply a more constant pressure for a longer duration of time if using right hand

2. Apply moderate pressure (10N) before loss of consciousness
- The proper amount of pressure is very uncomfortable in the awake patient and may provoke vomiting and obstruct the airway

3. Apply firmer pressure (30-40N) with loss of consciousness
- Roughly equivalent to indenting a tennis ball
- Too little pressure will not prevent regurgitation and too much pressure will distort airway anatomy, may result in cricoid cartilage fracture, and the assistant will tire quickly

4. Maintain cricoid until the trachea is intubated, cuff inflated and position confirmed

5. Caveats
• Improperly applied cricoid may impede view at laryngoscopy by distorting laryngeal anatomy. Cricoid may need to be released or cephalad and dorsal pressure may help in visualisation of the larynx
• Unsuccessful intubation – maintain cricoid if mask ventilation is required as this decreases the flow of gas to the stomach and minimises gastric distension. Release cricoid if mask ventilation is difficult
• NGT – suction to remove air and gastric contents prior to RSI but leave in situ and open to air to act as a release valve
• LMA – cricoid may impede placement of a LMA
• Active vomiting – cricoid should be released immediately to prevent oesophageal rupture
September 2004 Question 14

What are the relative merits of sodium nitroprusside and glyceryl trinitrate in the control of blood pressure in anaesthetised patients?
Both agents are vasodilators used to for the control of hypertension

GTN
• Predominantly a venodilator but at higher doses also dilates arteriolar smooth muscle
• A pro-drug that requires denitration by reaction with sulfhydryl groups (tissue thiols) within vascular smooth muscle, to produce the active metabolite NO
• Causes a decrease in VR (preload) and consequent decrease in CO. Little change in HR
• Venodilatation at doses 0-2 mcg/kg/min (or ~ 10-200 mcg/min). Usually made up as 30 mg in 500 mls 5% dextrose
• Bolus doses of 50-100 mcg can be used to quickly control acutely elevated BP

Merits:
1. Titratable – onset within 2-5 mins, lasts 5-10 mins after stopping infusion
2. Smooth reduction in BP. Excessive hypotension usually not a problem unless > 3 mcg/kg/min
3. Coronary vasodilator without steal, hence useful in treating HT in patients with CAD
4. No problem with cyanide toxicity
5. also decreases spasm in smooth muscle e.g. sphincter of Oddi, uterus (may be disadvantage when used for example in pre-eclampsia)


Disadvantages:
1. Slightly slower onset and offset than SNP
2. Less potent than SNP
3. Adsorbed onto PVC thus special polyethylene administration sets are required
4. Tolerance develops within 48 hrs (due to depletion of sulfhydryl groups)
5. Adverse effects:
- Inhibition of HPV and increased shunting
- Increases CBF and ICP
6. Toxicity:
- MetHb can be produced with metabolism of nitrate
7. Risk of haemolytic anaemia with G6PD deficiency


SNP
• Predominantly an arterial vasodilator but also causes venodilation
• Spontaneous production of NO unlike nitrates
• Reacts with oxyhaemoglobin in RBC to form NO, five CN- ions and metHb
- MetHb combines with CN- to form cyanometHb which is non-toxic
- CN- is converted in liver and kidney by mitochondrial enzyme rhodanase with addition of sulfhydryl group to form thiocyanate (SCN)
- CN- may also combine with vitamin B12 to form non-toxic cyanocobalomin
• Decreases both SVR and VR. Reflex tachycardia
• Initial dose 0.3 mcg/kg/min titrated up to maximum of 10 mcg/kg/min (< 10 mins at max). Doses > 2 mcg/kg/min are in the toxic range but toxicity is rare if kept < 4 mcg/kg/min for 3 hrs

Merits:
1. Titratable – immediate onset, lasts 1-2 mins
2. Potent and very effective
3. decreases afterload as well

Disadvantages:
1. Tachyphylaxis with long term use (sign of CN- toxicity)
2. Rebound HT with cessation of infusion (caused by activation of RAS and increased plasma catecholamine levels)
3. Adverse effects:
- Excessive hypotension leading to hypoperfusion of organs and thus metabolic/lactic acidosis. Reported association with spinal cord ischaemia due to reduced perfusion
- Coronary vasodilation resulting in steal phenomenon
- Inhibition of HPV and increased shunting
- Increased CBF and ICP (together with dec MAP will dec CPP)
- Hypothyroidism – thiocyanate inhibits binding of iodine
4. Toxicity:
- Cyanide (CN-): binds cytochrome oxidase thus inhibiting oxidative phosphorylation/aerobic metabolism leading to histotoxic hypoxia. Onset is characterized by tachyphylaxis
- Thiocyanate (SCN): accumulates with prolonged infusion or in those with renal impairment. Causes drowsiness, N&V progressing to muscle spasms and seizures
- MetHb: impaired oxygenation with false elevation in SpO2
- Phototoxicity: exposure of SNP to sunlight leads to liberation of CN- ions causing discoloration of the solution, thus protect from light
September 2004 Question 15

Describe the function of the anaesthetic circuit shown in the diagram.
The diagram is that of a Mapleson F or Jackson-Rees modification of the Ayre’s T-piece

Description of function
• A semi-open, rebreathing circuit
• Commonly used in paediatrics, especially with children < 20 kg
• The Ayre’s T-piece comprises a 3-way T-tube whose limbs are connected to the fresh gas supply, a length of corrugated reservoir tube and the patient connector
• The Jackson-Ree’s modification of the Ayre’s T-piece connects a 2-ended bag to the expiratory limb of the circuit, gas escaping via the ‘tail’ of the bag
• Occlusion of the ‘tail’ of the bag and squeezing it allows delivery of CPAP, PEEP or IPPV
• As there are no unidirectional valves or CO2 absorption, rebreathing is prevented by using high fresh gas flows to vent exhaled gases
• The volume of the reservoir tubing should be at least 1/3rd of the patient’s tidal volume. If the volume is too large, rebreathing may occur, and if too small, ambient air may be entrained

Advantages of the circuit
1. Compact and lightweight, thus does not place large weight on the patient’s airway
2. Simple
3. Inexpensive
4. No unidirectional valves – low resistance
5. Minimal dead space (i.e. minimal length of tubing between fresh gas inlet and the patient)
6. Minimal resistance to breathing
7. No rebreathing of exhaled gases, provided fresh gas flow is adequate
8. Ability to deliver CPAP, PEEP or IPPV
9. Movement of bag demonstrates breathing during spontaneous ventilation

Disadvantages of the circuit
1. High requirements for fresh gas flow (at least 2x minute volume) to prevent rebreathing of CO2 (alveolar gases consist of 5-6% CO2, thus rebreathing this will cause hypercarbia)
2. In spont vent – performance is waveform dependent
a. High inspiratory flow rates will increase rebreathing (as more of the flow will come from the circuit limb rather than the FGF)
b. High respiratory rates will also increase rebreathing as the expiratory pause allows reservoir of FGF in the limb. A long expiration improves efficiency
3. Environmental pollution due to limited ability to scavenge waste gases
4. The bag may get twisted and impede breathing
5. Lack of conservation of heat and moisture
May 2004 Question 1

A 50 year old patient with a past history of well controlled ischaemic heart disease is anaesthetised for an emergency laparotomy. Thirty minutes into the surgery, you notice new ST depression on the ECG.
ACC/AHA Guidelines
Surgical Risk : Moderate risk (intra abdominal surgery) 1-5% of non fatal MI.

Perioperative MI usually occurs 24-48 hours post. Usually silent.

Cardiac Risk Factors (Goldman Cardiac Risk Index) : high risk surgery / IHD / TIA / CCF / DM / CRI
*this patient has 2 risk factors

ECG critera for ischemia
1. horizontal/downsloping depression >0.1 mV (1 mm) at 60-80 sec after J point
2. upsloping depression >0.20 mV at 80 msec after J point.
3. for elevation it is - >0.1 mV 60-80 msec after J point

Managment
Immediate :
* check leads and ensure accuracy
* inform surgeon : expedite surgery by senior surgeon
* ABC approach
* Vitals : rule quickly reversible causes : hypoxia, hypotension, hypothermia, hypoglyaecemia, anaemia, pain

The principles of treatment are
− increase supply of O2 to myocardium
− decrease demand
− vasodilate, alter coagulation, reperfuse

Increase supply of O2
− increase FiO2 (consider 100%)
− adequate BP (especially DBP)
− use of vasopressors if required; avoiding HT as it wil increase wall tension and O2 demand
− IV fluids
- check haemoglobin, BSL

Decrease demand
− treat tachycardia (aim for HR 50-60)
− treat cause – hypovolaemia, hypercapnia, inadequate analgesia/anaesthesia
− consider beta-blockade – esmolol 0.5 mg/kg bolus with 25-300 mg/kg/min infusion OR metoprolol 1-10 mg titrated over 15 mins
− if beta blockers C.I. - clonidine 75mcg OR verapamil 2.5 mg may be an alternative
− treat hypertension
− increase anaesthetic depth, treat hypercapnia
− GTN (see below)

Special Considerations
* Vasodilate GTN infusion
* Aspirin 300 mg via NGT
* Heparin infusion can be considered in other cases but unlikely to be surgically possible in this laparotomy
* Progressive ST changes ( ST elevation or new LBBB or worsening haemodynamics refractory hypotension) requires urgent TOE intraop ?wall motion defect requiring cardiology input for urgent reperfusion therapy with HDU/ICU support

If patient is stablised and surgery is completed
* patient should be full cardiac monitoring in recovery and referred to HDU/CCU
* history, examination should be completed ASAP in recovery
* patient and family informed of intra operative findings and treatment with a discussion of future management
* Serial ECG and troponin
* Risk stratification (Stress Test) and/or angiography
May 2004 Question 2

Discuss the presence of morbid obesity as a predictor of difficult intubation.
Definitions:
• Morbid obesity is a BMI > 35. Super morbid obesity is a BMI > 55
• Difficult intubation has varying definitions
o Cormack and Lehane grade II or IV
o Multiple laryngoscopies before intubation
o Necessity for different blades or fibreoptic intubation
o ASA (American): intubation takes more than 10 minutes


Morbid obesity has been associated with difficult intubation. The evidence:
• CJA 2003. Study with 200 morbidly obese patients and found that BMI alone did not predict difficult intubation, but rather abnormal teeth, PHx of OSA on sleep study, increased age, TMJ pathology, and MP grades III and IV.
• A&A 2002. Study of 100 patients with BMI > 40. Neither absolute obesity nor body mass index was associated with intubation difficulties. Large neck circumference and high Mallampati score were the only predictors of potential intubation problems. Because in all but one patient the trachea was intubated successfully by direct laryngoscopy, the neck circumference that requires an intervention such as fiberoptic bronchoscopy to establish an airway remains unknown. We conclude that obesity alone is not predictive of tracheal intubation difficulties. With a neck circumference, at the level of the thyroid cartilage) of 40 cm, the probability of a problematic intubation was approximately 5%, and at 60 cm the probability of a problematic intubation was approximately 35%
• Anaesthesia 2003. 50 patients with BMI > 35. They quantified the soft tissue of the neck from the skin to the anterior aspect of the trachea at the vocal cords using ultrasound. Thyromental distance, mouth opening, limited neck mobility, modified Mallampati score, abnormal upper teeth, neck circumference and sleep apnoea were assessed as predictors of difficult laryngoscopy. Of the nine (18%) cases of difficult laryngoscopy, seven (78%) had a history of obstructive sleep apnoea, compared with two of the 41 patients (5%) in whom laryngoscopy was easy (p < 0.001). Patients in whom laryngoscopy was difficult had more pretracheal soft tissue (mean (SD) 28 (2.7) mm vs. 17.5 (1.8) mm; p < 0.001) and a greater neck circumference (50 (3.8) vs. 43.5 (2.2) cm; p < 0.001). None of the other predictors correlated with difficult laryngoscopy. We conclude that an abundance of pretracheal soft tissue at the level of the vocal cords is a good predictor of difficult laryngoscopy in obese patients. However, this is not a readily available test to perform to help us in daily practice

This data was confirmed in a talk on anaesthesia for bariatric surgery at the RMH in late 2005 which a few of us went to, so I think it is reasonable.

Importantly, while intubation may be difficult, ventilation can also often be difficult with potential disastrous consequences. Obesity is associated with OSA which is a predictor of failure to ventilate. Therefore, despite this reassuring data for the patients with most of the adipose on their ass, keeping the patient breathing during induction is still probably a good idea, especially with OSA
May 2004 Question 3

Describe and justify your usual anaesthetic technique for colonoscopy in an otherwise uncompromised patient.
The principle of my technique is provision of safe sedation for a procedure which is usually short with minimal intra-operative and post-operative pain. This is my usual technique; the circumstances in which I would consider using another technique would be:
− long duration expected
− risk of aspiration is high – unfasted, severe GORD, emergency case, haematemesis
− extremes of age
− severe cardiovascular instability expected, haemorrhage
− difficult airway – difficult intubation, morbid obesity
− co-morbidities – unstable IHD, severe resp disease,

Pre-op
− concise history of co-morbidities
− previous colonoscopies- problems with them
− airway, fasting
− pertinent examination and investigations
− consent
− explain to patient that only sedation required
− small chance that patient may 'wake up' during the procedure, however, unlikely that patient will feel pain/paralysis
− usually day case, but patient will be monitored before being sent home. Need for responsible adult to accompany patient and be present for 1st 24 hours
Intra-op
− equipment available
− suction, means of providing 100% O2 and positive pressure ventilation, airway equipment, emergency drugs
− site where there is adequate space, lighting, means to get emergency help
− one assistant (can be shared with proceduralist)
− establish IV access
− monitoring I would apply – pulse oximeter, BP cuff on other arm of IV access, ECG. ECG not mandatory but easier to apply when patient awake and in supine position.
− O2 via Hudson mask 6 L/min
− increases O2 reserve in the event of hypoventilation/apnoea
− attach ETCO2 side stream sampling line to mask – although absolute values of CO2 will not be accurate, still gives an indication of respiratory rate/apnoea
− IV fluids
− routinely give CSL/N saline, 500 ml bolus, another 500 ml over 1 hour
− patients will have had colonic prep – dehydrated
− fasting for at least 8-10 hours
− decreases hypotension, nausea
− total of 1 litre infused, reassess need for more after that
− rate, and volume will be decreased if risk of cardiac failure, pulmonary oedema, renal failure on fluid restrictions
− turn patient to lateral position – helps with the procedure and also avoids aspiration, keeps airway patent
− I use a combination of propofol and alfentanil
− alfentanil
− 0.25-0.5 mg as a bolus, titrated to respiratory rate, HR, BP and patient movement
− rapid onset (peaks 1-2 mins) and duration of action only about 10-15mins
− matches the time line of procedure well
− patients have minimal post-op pain, therefore no need for longer acting opioids
− propofol
− start with 0.25 – 0.5 mg/kg bolus, titrated to patient's verbal response, vital signs
− aim for a spontaneously ventilating patient with no movement with stimulus and BP, HR, SpO2 within normal limits, RR around 10/min
− further boluses 2-5 minutely
− propofol has short context sensitive half time, no hangover effect, antiemetic properties
− alternative is to run propofol infusion, either via TCI or programmed infusion – however equipment not necessarily available, extra time in setting up, extra infusion lines, patients weight must be known
− I do not routinely use midazolam, as the duration of action is around 30 mins and may contribute to post-op sedation and delay in discharge. However the circumstances in which I would consider using it would be – highly anxious patients, in patients where the cardioresp effects of propofol may be siginificantly deleterious (where these patients would be admitted for overnight observations anyway), then midazolam is used as a co-induction agent
− patient may require some airway support – jaw thrust, chin lift during the procedure
− if patient becomes compromised in terms of cardiovascular/respiratory function, priority must be given to attending to the problem, even if it means turning patient supine and stopping colonoscopy

Post-op
− patients left in lateral recovery position
− trained recovery staff
− O2 continued – patient may still be hypoventilating
− Obs continued for at least 30 mins
− if patient is awake, obs stable, no pain/nausea --> step down recovery
− observed for another 1 hour – ensure that patient is mobile, tolerates fluids, no pain
− discharged to care of responsible adult living within 1 hour of hospital, advised not to drive, operate machinery, make legally binding decisions
− if there is some discomfort, I would use IV paracetamol, consider NSAIDS – non-sedating analgesia
− if that is inadequate, surgical review as could be perforated viscus. I would use small doses of fentanyl 10-20 mcg boluses.
May 2004 Question 4

An 85 year old female presents with a three day history of bowel obstruction. The duty surgeon wants to perform a laparotomy as soon as possible.

Discuss the main factors determining the optimal time for anaesthesia and surgery
This is a case of an elderly patient having high risk urgent surgery. The optimal time for surgery would depend on patient, surgical and environment factors.

Daylight hours
Trained staff appropriate (nursing, anaesthetic, surgical)
Site for surgery appropriate (hospital large enough/experienced enough)
Equipment/infrastructure available (eg. ICU on site)
Patient resuscitated/optimized


Patient
− assessment of urgency of laparotomy
− bowel obstruction is not a blanket indication for emergency surgery unless there are signs/investigations suggesting bowel perforation, peritonitis, sepsis, uncontrolled haemorrhage
− assess based on vital signs, response to conservative treatment, relevant investigations (CT looking for perforation, acidosis suggesting tissue ischaemia, etc)
− surgery can be delayed if there are more urgent issues which would compromise the patient during the operation
− hypovolaemia should be corrected
− electrolyte abnormalities – likely with vomiting, overflow diarrhoea
− bleeding – corrrection of anaemia, patient may be coagulopathic from medications, bleeding
− optimisation of patient's co-morbidities e.g.heart failure – diuresis to reduce pulmonary oedema
− adequate investigations including CT scans, bloods, ABGs
− non-surgical treatment can be instituted while patient is being optimised
− nasogastric tube
− IV fluids
− IV antibiotics
− patients/family wishes – surgery may have to be delayed if patient wishes to wait for contact with family

Surgical
− discussion with surgical team with regards to urgency of procedure
− expected length of surgery, blood loss
− need for repeat procedures
− possibility of conservative treatment
− if patient has siginificant comorbidities, poor quality of life, surgery may not be good first option
− possiblity of less invasive treatment e.g. Colonoscopy
− post-op
− if significant co-morbidities or expected surgical resection, patient may need to be suppoorted in ICU post-op
− may have to wait for bed to be available or patient transferred to another hospital

Environment
− availabilty of senior staff – surgeons, anaesthetists especially if long, complicated procedure expected – may mean surgery delayed to daylight hours
− availability of adequate nursing staff
− availability of equipment, blood products
May 2004 Question 5

Discuss the methods you would use to reduce cerebral swelling during craniotomy for removal of a tumour in an adult.
Cerebral Oedema and ICP:
• The cranial vault has the following contents:
1. Brain parenchyma
2. CSF
3. Blood – venous and arterial
4. Extracellular and intracellular fluid – excess is called cerebral oedema
• An increase of any component beyond a critical volume, can lead to a sharp increase in Intracranial Pressure (ICP), as the cranial vault is a non-compliant structure (Monroe-Kellie Doctrine)
• Increased ICP leads to deterioration in neurological function and ultimately death.
• Manipulation of any compartment can normalise / decrease ICP

Pre-operative:
• Steroids (Dexamethasone) – particularly efficacious in peri-neoplastic oedema
o Clinical improvement within 24 hours
o Reduction in ICP requires 48-72 hours

Intra-operative:
• Promote venous drainage:
o Head up
o Neutral head position (prevent vein collapse)
o No circumferential pressure (tube ties, collars) – to prevent extraluminal pressure
o Prevent increased intrathoracic pressures –
• Coughing / straining:
• Use of muscle relaxant intraoperatively
• Remifentanil infusion for smooth induction and emergence
• LA to vocal cords
• Avoid excessive PEEP and minimise airway pressures
• Obstructed ETT / Tension pneumothorax / Bronchospasm
• Anaesthetic drug choice:
o Avoid cerebral vasodilating agents such as volatile anaesthetics, ketamine, nitrous oxide
o Propofol TCI, with BIS monitoring
o Muscle relaxant, infusion or boluses
o Remifentanil infusion
• Other drugs:
o Diuretics – Reduce ECF >> ICF compartments
• Mannitol – osmotic diuretic:
• 0.5 – 1 g / kg at time of skin incision
• Peak effect in 10 mins
• Frusemide – loop diuretic:
• 0.25 – 1 mg/kg
• May also reduce CSF production
o Anticonvulsants - Phenytoin loading 15mg/kg intraoperatively
o IV Fluids:
• Avoid hypo-osmolar and hypotonic solutions, eg 5% Dextrose
• Normal Saline preferred

Smooth induction with appropriate muscle relaxation and analgesia for eg. head pinning.
Smooth emergence with adequate analgesia on board, avoiding if possible coughing/straining on ETT.

• Maintain normocapnia:
o Hypocapnoea may reduce cerebral blood flow and result in ischaemia
o Hypercapnoea may increase cerebral blood flow, and hence ICP
o Aim for PaCO2 of ~30 mmHg
• Other:
o Maintain normal oxygen saturations
o Maintain BP to within 10% of awake levels
o Avoid hyperthermia – may lead to increased CMRO2
o CSF drainage by surgeon – Ventriculostomy, EVD
o Tumour resection itself (reduction in brain parenchyma)

Post-operatively:
• Nurse head-up 15 degrees
• Maintain cerebral perfusion pressure and oxygenation
• Frequent neurological observations
May 2004 Question 6

71 year old man presents with acute herpes zoster involving the ophthalmic division of his left trigeminal nerve. He complains of severe unrelenting facial and eye pain that started 3 days ago.

Discuss the pharmacological treatment options. Include information about relative efficacy of the treatments you describe.
Treatment of Herpes Zoster:
Objectives:
1. Treatment of acute viral infection
2. Treatment of acute pain
3. Prevention and treatment of Post-Herpetic Neuralgia (PHN)

Antiviral drugs:
• Must be commenced within 72 hours of rash
o Reduces duration of rash
o Reduces pain
o Do not reduce the risk the incidence of PHN
• In ophthalmic shingles reduce the risk of developing keratitis and visual loss
• Similar efficacy to acyclovir shown for famcyclovir and valacyclovir, with advantage of three times a day dosing (compared to 5 times daily for acyclovir)

Corticosteroids:
• Use of Prednisolone reduces the pain associated with shingles (possibly by reducing degree of neuritis)
• Conflicting evidence regarding a reduction in PHN
• Must be used in conjunction with antivirals to reduce dissemination of virus

Analgesics:
• NSAIDS:
o Oral formulation not useful, but topical aspirin has been shown to be effective
• Paracetamol:
o Useful as antipyretic or as part of multimodal regimen
• Opioids:
o Needed for excruciating pain
o Good in combination with NSAIDs and Paracetamol

Other ‘Analgesics’
• Calamine containing lotions may be used on open lesions to reduce pain and pruritus
• Capsaicin creams may be applied once the lesions have crusted over
• Topically administered local anaesthetics and nerve blocks have been reported to be effective in reducing pain
• Stellate ganglion block- conflicting evidence on prevention of PHN. Invasive, last resort.

Prevention / Treatment of PHN:
• Immunisation:
o Patients over age 60 with live attenuated vaccine
• Tricyclic Antidepressants:
o First line for treatment of PHN
o NNT = 2.1, for 50% reduction in pain
o Analgesia may be enhanced by giving early in course of shingles infection along with antivirals
• Oxycodone:
o Effective in acute pain and PHN
o NNT = 2.5
• Anticonvulsants:
o Phenytoin, Carbamezapine and Gabapentin:
• In sub-epileptic doses are often used to control neuropathic pain
• Equally effective at NNT = 3.2
May 2004 Question 7

Discuss the advantages and disadvantages of Sub-Tenons eye block compared with other eye block techniques.
• Sub-Tenons eye block is blunt needle regional eye block gaining popularity due to high efficacy, painlessness on administration, and low incidence of catastrophic consequences.
• Other forms of eye blocks are:
o Sharp needle techniques – Retrobulbar and peribulbar blocks
o Topical Local Anaesthetic drops


Advantages:
• Painless for patient:
o Sharp needle blocks involve more discomfort during needle insertion
o The conjunctiva is topically anaesthetised and is the only tissue plane penetrated with Stevens cannula
o Less likely to require sedation
• Reliable anaesthesia compared to other blocks:
o More consistent block with lower reports of intraoperative pain
o Faster onset than sharp needle techniques
o Easily supplemented by surgeon through dissection
o Longer duration of action following single shot
• Lower incidence of catastrophic consequences:
o Rare incidence of retrobulbar hemorrhage, globe perforation, rectus muscle trauma, dural puncture.
o These complications are avoided with topical LA drops
• Other benefits:
o Smaller volumes compared to peribulbar block, therefore less increase in Intraocular pressure
o May be performed in anticoagulated patient, especially if akinesia is desired
o No sharp needle required – reduced risk of needlestick injury
o Better akinesis than topical or peribulbar blocks

Disadvantages:
• Subconjunctival hemorrhage (40%):
o Low incidence with sharp needle block and nil with topical LA
o Minor consequences, but cosmetically poor
o May be minimised by improved skill, avoidance of visible vessels, use of adrenalin and cautery (surgeon)
• Specialised sterile equipment needed:
o Greater cost and labour
o Needle blocks require only standard syringes and needles
• Akinesia not reliable at low doses:
o >7ml required to consistently block superior oblique and levator palpebrae superioris.
o Retrobulbar blocks have more reliable akinesia
• Chemosis – Oedema of the bulbal conjunctiva (40%)
o May interfere with surgical technique
o Decrease with greater experience
o May mean that LA solution not in correct plane
• May be difficult in previously operated eye:
o Contraindicated in previous glaucoma and retinal detachment surgery, where sclera may be scarred
• Does not provide adequate analgesia for enucleation and some retinal surgery (neither do other blocks)

• Technically difficult and takes longer to perform than sharp needle blocks – need greater patient cooperation
• Retained vision in eye during operation- may be distressing for patient
May 2004 Question 8

Discuss the indications for perioperative antibiotic prophylaxis. Include consideration of the appropriate class of antibiotic for each indication.
• Prophylactic antibiotics are administered commonly to prevent surgical wound infection.
• Antibiotic selection targeted toward organisms most likely to cause infection
• Considerations:
o “Type” of wound expected after surgery
o Prophylaxis for Infective endocarditis
o High Risk patients

Wound Type:
• Clean wounds: (account for 40% of wound infections)
o Generally require no prophylaxis, but exceptions include:
• Cardiac or vascular surgery
• Neurosurgery
• Orthopaedics, involving prostheses
• Some ocular surgery
• Abdominal surgery with prosthesis (eg. hernia mesh)
o Require cover for gram positive skin commensals (Staph Aureus, Staph Epidermidis) – Penicillins, First generation cephalosporins, or Vancomycin (if MRSA prevalent, indigenous population, allergy to penicillin / cephalosporin)
o Require cover for gram negative bacilli (cardiac and vascular surgery) if instrumenting urethra - Gentamicin
• Clean-contaminated wounds (resp, GU or alimentary tract penetrated under controlled conditions without obvious soiling)
o Include head and neck, oropharyngeal cavity, GI surgery, hysterectomy, Caesarean section
o Cover for skin commensals as above
• urological surgery
o G-ves
o Gentamicin 2 mg/kg

• gynaecological surgery
o hysterectomy – G+ves and anaerobes – cephalosporin and metronidazole
o LUSCS – skin flora i.e. Staph – cephalosporin after cord is clamped

• Vascular
o amputation of ischaemic leg – anaerobes, Clostridium
o benzylpenicillin + metronidazole

• Thoracic/ Resp tract surgery
o Staph, Strep (aerobic and microaerophilic streptococci)
o Cephalosporin

• In GI surgery also need to cover Clostridium, Gram negative bacilli, and enterococcus
o Metronidazole, Gentamicin, and Penicillin respectively

• Contaminated wounds:
o Ruptured viscous and macroscopic soiling of surgical field
• Use of 3rd Generation Cephalosporin +/- Metronidazole to cover enteric bacteria (above)
o Traumatic wound:
• Use first generation Cephalosporin to cover Staph Aureus and Group A Streptococcus
• Use of Metronidazole to cover Clostridium

Infective Endocarditis Prophylaxis:
• Patients with rheumatic valves or past vegetations from rheumatic fever need cover
• Antibiotic used depends on organisms likely to cause bacteremia during surgery
• For dental and upper airway procedures, Strep species should be covered with Amoxicillin (Erythromycin if allergic)
• For GI surgery, gram negative bacteria also need to be covered – Gentamicin

High risk patients:
• Systemic factors:
o Diabetes
o Corticosteroid use / immunocompromised
o Obesity
o Advanced age
o Malnutrition
o ASA > 3
o Remote infection
• Surgical factors:
o Foreign Body
o Drains
o Injection with adrenalin
• Long operations > 2 hours

In cases of penicillin/ cephalosporin allergy
- Clindamycin 600 mg IV OR Vancomycin 25 mg/kg
Vancomycin not recommended as replacement for cephalosporins unless:
- Preoperative patients infected or colonised with an MRSA strain (hospital-acquired or community-associated) currently or in the past
- patients having major surgery who are at high risk for MRSA colonisation (eg those who have resided for longer than 5 days in a health care facility where MRSA is endemic)
- patients undergoing prosthetic cardiac valve, joint or vascular surgery where the procedure is a re-operation (return to theatre or revision)
- patients hypersensitive to penicillins and/or cephalosporins

(Vancomycin has little G-ve cover and if used instead of cephalosporin, gentamicin should also be added)
May 2004 Question 9

Compare and contrast thiopentone and propofol for use in rapid sequence induction of anaesthesia.
May 2004 Question 11

Discuss the strategies you would consider in order to protect renal function during a laparotomy for an abdominal aortic aneurysm repair?
Development of renal failure following abdominal aortic aneurysm repair is dependent upon

1) Preoperative renal function
2) Supra or infra-renal AAA
3) Duration of clamp time
4) Perfusion pressure before, during and post clamp

The strategy to protect renal function would look at perioperative optimisation of renal function in the elective patient. In the perioperative phase maintaining normal haemodynamics and euvolemia is paramount in the fasting patient. Ensuring adequate postoperative care in a high dependency unit or intensive care unit where close monitoring of fluid balance, and blood pressure can occur is critical prior to proceeding to surgery. This environment also offers early intervention in the form of dialysis for the failing kidney.

Prior to induction two large intravenous lines should be inserted to provide warmed fluids. Invasive and non-invasive blood pressure monitoring and central venous access is important to provide optimal blood pressure observation and central pressure monitoring. A five lead ECG and pulse oximetry completes the minimum required monitoring. Urinary catheter with burette reading can guide treatment intra and postoperatively.

During induction hypotension and hypoperfusion is closely monitored and treated with fluid bolus and alpha agonists such as metaraminol. Gentle induction with anaesthetic agents and avoidance of swings in blood pressure and tachycardia are vital.

Discussion with the surgeon to determine where the clamp is placed either infra or supra renal will alter intraoperative management. A suprarenal clamp will increase the risk of renal injury and bypass perfusion or shunt can be considered in those patients where prolonged clamp time or severe renal impairment exists. If an infrarenal clamp is used renal perfusion pressure needs to be maintained throughout the surgery. Communication with the surgeon for when the clamp is removed allows maintenance of perfusion pressure and avoidance of hypotension or hypovolemia during this period.

A number of pharmacological methods to preserve renal function have been postulated but have not been proven to be of benefit. These include mannitol bolus and/or infusion, dopamine infusion and frusemide bolus or infusion. While these pharmacological agents have suggested renal protection in animal models no randomised placebo control trial has demonstrated their benefit. Avoidance of nephrotoxins such as gentamicin and NSAIDS for example, is a simple pharmacological benefit to preserve renal function. The main aim of pharmacological intervention is to maintain perfusion pressure. Alpha agonists such as noradrenaline, metaraminol, ephedrine and phenylephrine can be used to maintain renal perfusion by preserving mean arterial pressure.

Measurement of electrolytes and blood glucose perioperatively, intra and post operatively can guide treatment. Avoiding hyperkaelemia and hyperglycaemia is important and should be treated if present. If massive blood loss occurs, early transfusion of blood and products with close monitoring of clotting factors is essential. Aiming for normal electrolytes and normal haematinics is important to avoid insult to the kidneys.

The main aim of renal protection in AAA surgery is euvolemia and maintenance of renal perfusion pressure during the operative and postoperative period. Critical times where the kidneys are at risk of ischaemic insult are during the perioperative fasting period, at induction, during clamp time and post clamp time and postoperatively. The main aim of renal protection is to maintain renal perfusion pressure during these at risk periods.

Also hypothermia, generalised or renal selective
May 2004 Question 12

Working in a small obstetric unit you are asked to attend at the birth of a child where there is meconium stained liquor. How will you manage the infant’s resuscitation?
Management of a neonate with meconium stained liquor requires assessment and resuscitation.

Ascertain labour history and foetal distress, arrange preparation of equipment and assistance and ensure a follow up plan.

Assessment of the maternal, foetal, and intrapartum circumstances that may place the newborn infant at risk of needing resuscitation
• Maternal risk factors
o Maternal co-morbidities e.g. hypertension, DM, chronic illness.
o Complicated pregnancy e.g. prolonged rupture of membranes, antepartum bleeding, preeclampsia/eclampsia, chorioamnionitis, infection.
• Foetal risk factors
o E.g. twin pregnancy, preterm or post-term gestation, congenital abnormalities, small for gestational age / IUGR.
• Intrapartum risk factors
o E.g. thick meconium in the amniotic fluid, non-reassuring foetal heart rate pattern on CTG, prolapsed cord, prolonged obstructed labour.
Preparation of equipment and assistance
• Personnel
o A trained assistant (NICU or paediatric nurse ideally)
o If it is anticipated that the infant is at high risk of requiring advanced resuscitation call for help early, more than one experienced person, e.g. a neonatologist / paediatrician should also be present.
• Equipment
o A complete set of resuscitation equipment and drugs should be available and checked.

A graded resuscitation response according to current neonatal ACLS Guidelines.
• If the newborn is breathing or crying, with good muscle tone, then routine care is provided,
o Routine suctioning, either blindly, via direct laryngoscopy or endotracheal tube should not be done if the infant is vigorous and breathing.
o Endotracheal intubation of babies who have meconium stained liquor, and who are vigorous, is not recommended because it does not prevent MAS (meconium aspiration syndrome) and it may cause harm.
• If not,
o Clear airway and position in neutral position. If the fluid contains thick meconium and the infant has absent or depressed respirations and decreased muscle tone, direct laryngoscopy should be carried out immediately after birth with suctioning of residual meconium from the pharynx (under direct vision) and, if needed, intubation and suctioning of the trachea.
o Dry and stimulate
o Provide warmth
• Assess breathing and heart rate, colour and tone
o If heart rate < 100/min or inadequate breathing
• Give positive pressure ventilation with air (supplemental O2 reserved for infants who do not improve) until heart rate > 100 and infant breathing. (NB normal HR around 130)
o If heart rate < 60/min and inadequate breathing
• Ensure effective lung inflation and ventilation
• If heart rate does not increase > 60/min
• Give chest compressions with positive pressure ventilation at 3:1
• If heart rate still does not increase > 60/min
• reassess ventilation technique
• give adrenaline
• may also need to give IV fluids.
A follow-up plan
• Once adequate ventilation and circulation have been established, the infant who has required resuscitation is still at risk and should be cared for in NICU or special care nursery where monitoring, appropriate evaluation and care can be provided.
• Neonates born after meconium aspiration are at risk of developing
o Obstruction of small airways causing ventilation-perfusion mismatching
o Chemical pneumonitis / MAS
o Pulmonary hypertension
o Persistent foetal circulation syndrome
o Pneumothorax
May 2004 Question 13

Describe the characteristics of a ventilator suitable for neonates
Key differences between neonates and adults
• Small tidal volumes
• Increased lung compliance
• Increased risk barotrauma
• Higher respiratory rates
• Closing capacity may exceed or overlap FRC, therefore small airway closure

Common to adults, ventilators need to be able to deliver oxygen & anaesthetic gases to the neonate & be capable of positive pressure ventilation. They should also have:
• ability for inspiratory and expiratory gas analysis
• suitable alarms for circuit disconnection, loss of ET CO2, oxygen loss, power loss.
• Battery back up in the event of failure of primary power supply

Ventilator must be able to achieve particular respiratory parameters for a neonate:
• Rate up to 40-60/min (normal neonate 20-30) due to low compliance of neonatal lungs and increased O2 requirements (O2 consumption 7mLs/kg/min vs adult 3.5mLs/kg/min)
• Tidal volume 7mL/kg (same as adult) but 1/25 of total volumes. Thus volumes as small as 5-10 mLs may be required.
• Alveolar ventilation 100mLs/kg/min (vs adult 60mLs/kg/min)
• FRC 30mLs/kg- approaches closing volume thus AutoPEEP needed
• Dead space 2mLs/kg (same as adult)
• Airway resistance higher 40 mLs H2O/sec (vs adult 2)
• Airway compliance 5mLs/cm H2O (vs adult 100mLs/cmH2O)

Other characteristics
• Minimal apparatus dead space
o Low volume face masks, HME’s, connections distal to Y piece
• Low resistance circuit – especially important with spontaneous ventilation
• Light weight circuit components to minimise risk of kinking ETT
• Low circuit volume – lower volume results in less dilution of anaesthetic gases at induction
• Humidified gases
• Accurate delivery of small tidal volumes- low volume bellows, low circuit compliance, accurate TV measurement
• Inspiratory time adjustment independent of the inspiratory/expiratory ratio
• I/E ratio adjustment
• Leak alarms- especially problematic around uncuffed tubes
• Variable gas flow at least up to 2-3L/kg/min
• Availability of both pressure controlled & volume controlled ventilation
• Able to provide PEEP as intercostal tone & partial glottic adduction that helps maintain FRC is lost with anaesthesia
• Rapid response time for gas flow during spontaneous respiratory effort
• Electronic display of inspiratory pressure wave
• Pressure limited ventilation to protect against inappropriate large tidal volume settings – minimise barotrauma
• Low compliance circuit – circuit expansion during inspiration means less volume delivered to neonate
• Newer neonatal ventilators have ability to both conventionally ventilate and high frequency jet/oscillatory
May 2004 Question 14

Discuss contraindications to spinal anaesthesia for LUSCS
ABSOLUTE CONTRAINDICATIONS
1. Maternal refusal After full discussion of risks/benefits
2. Localised infection at puncture site
3. Severe coagulopathy ASRA guidelines suggest min plt 80 x 109 /L and INR <1.4.
4. Uncorrected hypovolaemia
5. Allergy true amide allergy is rare
6. Bacteraemia/untreated systemic sepsis
7. Intracranial hypertension/raised ICP may occur with pre-eclampsia. Risk of brainstem herniation
8. Severe aortic stenosis Valve area < 0.6 cm2. LV outflow obstruction; valvular aortic, hypertrophic subaortic stenosis.
9. Severe mitral stenosis Valve area < 1.2 cm2.

RELATIVE CONTRAINDICATIONS
1. Expectation of significant haemorrhage Eg. high grade placenta praevia/accreta.
2. Pre-eclampsia with haematological derangement Risk/benefit of spinal haematoma vs risks of GA with pre-eclampsia (cerebral bleed/failed airway)
3. Treated systemic infection/sepsis Eg. chorioamnionitis/UTI. Risk of bacteraemia is minimal where infection being treated with antibiotics.
4. Cardiac disease (eg. valvular disease)
Spinal risks changes in preload / BP / afterload
Redue risk with right sided wedge
5. Bad back /previous back surgery/ back pain Scarring may decrease success. Medicolegal ramifications.
6. Communication difficulties Language barrier/sedation/anxiety. Proper informed consent impossible. Situation ie. emergent LUSCS may still allow
7. Prior back surgery Technically challenging. Increased incidence of failed block requiring potentially risky conversion to GA
8. Multiple pregnancies Protracted operation time/potential for blood loss. Spinal techniques well decribed however.
May 2004 Question 15

Use of a BIS (bispectral index monitor) or other similar monitor should be the standard of care during total intravenous anaesthesia.” Discuss this statement.
TIVA is an anaesthetic technique employing iv agents alone, and avoiding the use of inhalational agents. The disadvantages of TIVA are:
o Requires continued injections or infusion devices, e.g. syringe pumps, etc.
o Prediction of plasma levels of anaesthetic agents is more difficult than with inhalational agents, because of the more complicated pharmacokinetics and lack of measurement of actual plasma level compared with end-tidal agent monitoring.
o Thus potential for awareness or overdosage of anaesthetic is arguably greater

Awareness is the postoperative recollection of events occurring during general anaesthesia. It is a potentially devastating complication of anaesthesia. The incidence of awareness is 0.1-0.2% in the general surgical population, but is greater during high risk procedures: cardiac surgery, caesarean section, trauma surgery, bronchoscopy.

Some studies have shown that excessive dosages of anaesthesia are associated with higher 1-year postoperative death rates in patients 40 years and older undergoing major, noncardiac surgery. Although these studies were not specific to TIVA, one needs to consider the potential risks of excessive anaesthetic level, not to mention the costs of the drugs.

BIS monitor is a continuous highly processed electroencephalography that measures changes in interfrequency coupling. It creates a dimensionless numerical representation of degree of sedation. Lower numbers correspond to greater depth of anaesthesia. Higher numbers found in awake or lightly sedated patients

Advantages of BIS

• Potential to safely minimise anaesthetic dosage, resulting in fewer unwanted anaesthetic dosage, resulting in fewer unwanted anaesthetic drug side-effects, faster wake up time and earlier discharge (B-Aware trial found reduced anaesthetic dosage in BIS targetted group- though this was with both inhalational and TIVA patients)
• BIS monitoring may reduce awareness under anaesthesia, especially high risk patients. B-Aware trial showed a 82% decrease in awareness in high-risk patients, with a NNT was 138.
• Propofol has a variable and broad ED95, making individualised dosing difficult. BIS may help this.

Disadvantages of BIS

• Its measurement can be affected by many factors other than depth of consciousness (eg. muscle activity)
• Not validated in some patient populations (eg. children)
• Not uniform with anaesthetic technique- eg. ketamine elevates, muscle relaxants reduce, opioids do not change (at odds with what we know about anaesthesia)
• Lag time before value displayed
• Expensive- cost of consumables/monitor
• Does not replace vigilance with lines/infusions
• No marked transition in value between awake and asleep (unlike evoked auditory potentials)
• Significant inter-patient variability
• Controversy in literature- B-Unaware trial demonstrated no difference in BIS-guided versus ET agent concentration groups.

Because of inherent problems of estimating the plasma (and therefore brain) concentration of propofol concentration in TIVA, BIS is be a useful guide in this technique. I think it should be the standard of care in TIVA, especially in high risk cases. Although there is additional cost, the cost of awareness is also high (monetary and psychological). The cost in the B-Aware trial of preventing one case of awareness was found to be $2200. This is arguably cost-effective.
May 2004 Question 10
Digital nerve blockade is simple and easy to perform and provides useful analgesia for a variety of minor surgical procedures. The technique is essentially the same for fingers and toes.
Anatomy
In the hand, the common digital nerves are derived from the median and ulnar nerves and divide in the distal palm into paired volar (or palmar) branches. These run with the digital vessels on either side of the flexor tendon sheath of each finger and supply the lateral and palmar aspect of each finger together with the tip and nail bed area. The smaller dorsal digital nerves, derived from the radial and ulnar nerves, run on the dorsolateral aspect of each finger and supply sensation to the back of the finger, only as far as the PIP joint.
In the foot, the digital nerves are the terminal branches of the tibial and peroneal nerves which, in their turn, are branches of the sciatic nerve.


Method
The patient's hand is placed palm down and the skin cleaned. A 25g needle is inserted into the dorsal aspect of the web space between the fingers as proximally as possible and close to the phalanx (figure). The needle is advanced through the tissues until just below the skin on the palmar side.

After aspirating to ensure that a vessel has not been entered, 1-2ml of local anaesthetic is injected to block the volar branch and, as the needle is withdrawn, a further 0.5-1 ml is injected to block the dorsal branch. The nerves on the radial aspect of the thumb are best blocked by a subcutaneous wheal of local anaesthetic injected at its base.
Either 1% plain lignocaine or O.5% plain bupivacaine (or other equivalent agent) can be used. On no account should adrenaline-containing solutions be used. The digital arteries are end arteries and ischaemia or necrosis can occur if adrenaline is injected. Limit the injection volume to 2mL on each side. The mechanical pressure effects of injecting solution into a potentially confined space should always be borne in mind, particularly in blocks at the base of the digit. In patients with small vessel disease, perhaps an alternative method should be sought in addition to avoidance of digital tourniquet
The anatomy of the digital nerves in the foot is similar and a similar technique can be used to block them.



Choice of Local Anaesthetic Agent
The choice of the type and concentration of local anesthetic for a digital block is based on the desired duration of blockade. The onset times and duration of anesthesia for some commonly used local anesthetics mixtures
HK 2011

Discuss the pros and cons of total intravenous anaesthesia with propofol (TIVA-P).

List six surgical procedures where TIVA-P may be advantageous.
ADVANTAGES
• Smooth induction with minimal coughing and hiccupping
• Easy and quick to titrate depth
• Rapid recovery with minimal hangover
• ↓PONV
• ↓ operating room pollutio
• Improved recovery time
• Specifc benefit in laryngoscopy and bronchoscopy where difficult to admin volatile (Jet ventilation)
• Inhibits hypoxic vasoconstriction (Thoracic surgery)
• Neurosurgery
o Mainenance of cerebral autoregulation,
o ↓ cerebral blood flow
o ↓ metabolic rate
o Allow intraoperative monitoring
• Safe to use in pts with MH

DISADVANTAGES
• Expensive pump
• Remifentanil is expensive
• More expensive than volatiles
• Lack of suitable alternatives (that do not cause resp depression or bradycardia)
• Inter individual variation in clinical response
• Does not take into account other drugs, co-morbidities, temp, children or elderly
• Unable to monitor effect site concentration
• Not predicable as volatile
• No muscle relaxation, no inhibition of spinal reflexes
• Interruption or disconection takes longer to recognize


INDICATIONS
• Non triggering anaethetic (MH)
• Significant PONV
• Airway endoscopy, laryngeal and tracheal surgery
• Neurosurgery
• Thoracic surgery
• Short day surgery (endoscopy)
• Remote location
• During transport
HK 2011
What are the risk factors for laryngospasm? How would you minimize and manage laryngospasm after tracheal extubation in a 6 months old infant after general anaesthesia?
LARYNOSPASM
• Protective reflex closure of glottis
• Incidence 0..04-14%
• 40% post exubation
RISK FACTORS Patient
• Young age
• Passive smoking
• Recent URTI
• Hyperactive airway : asthma
• ASA 4, Ex-prem under 1 yr
• Electrolyte disturbances low Mg or Ca
Anaesthetic
• Light plane of anesthesia
• Airway instrumentation ETT>LMA
• Junior anaesthetist
• Ketamine, thiopentone, desflurane
Surgical
• ENT surgery: T&A’s : 20-25%
PREVENTION
• Risk factor modification
• IV induction
• Lignocaine IV or topical
• Exubate deep
MANAGEMENT
• Remove offending stimulus
• 100% O2 with CPAP
• Optimize airway with gentle jaw thrust
• Morgani maneuver
• Drugs
o Propfol 0.5mg/kg
o Sux 0.5mg/kg w atrop 0.2mic/kg
• Refractory latngospasm
o SLN block
o Transtracheal lig through cric mem
COMPLICATIONS
• Oxygen saturation 60%
• Bradycardia 6%
• Aspiration 3%
• Negative pres pulmonary edema 4%
• Cardiac arrest 0.5%
• Hypoxia
HK 2011

Describe the anatomy of the antecubital fossa. What structures can potentially be damaged during venous cannulation in this area?
DESCRIPTION
This is the triangular hollowed area on the anterior aspect of the elbow. It contains a number of vessels and nerves that enter and exit the forearm, and which can easily be damaged.
RELATIONS
Inferomedial – pronator teres
Inferolateral – brachioradialis
Superior – a line joining the two epicondyles of the humerus (medial and lateral)
Roof – deep fascia (reinforced by bicipital aponeurosis). Lying superficial to the fascia are the median cubitalvein and the medial cutaneous nerve of the forearm, and the basilic vein (medial) and cephalic vein (lateral)
Floor – supinator (laterally) and brachialis (medially)
CONTENTS
From medial to lateral:
● Median nerve
● Brachial artery – considerable variations may occur. The artery may bifurcate high in the upper arm, and superficial radial and ulnar branches may also be found. The superficial ulnar artery variation (found in 2%) is at particular risk during antecubital venepuncture attempts
● Biceps tendon
● Radial nerve (with posterior interosseous branch)
SUPERFICIAL VEINS
These also show considerable variation:
1. Cephalic vein – drains the radial forearm
2. Basilic vein – drains the ulnar forearm
3. Medial cubital vein – joins the basilic and cephalic vein to form an ‘H’
arrangement. It frequently receives the median vein of the forearm, and
may bifurcate to form an ‘M’ arrangement
HK 2011

Succinylcholine has no place in modern anaesthetic practice. Discuss this statement.
Suxamthonium
-depolarising muscle relaxant
-chemical structure is 2 ACh molecules joined together

USES
-facilitate endotracheal intubation during rapid sequence induction (for anticipated or known difficult airway, full stomach,)
-short term muscle relaxation eg ETC

DOSE
-1 mg/kg in adults
-2mg/kg in children
-3 mg/kg in neonates

Pharmacokinetics
Distribution: unknown due to rapid redistribution and metabolism
Metabolism: hydrolysis by plasma cholinesterases
Excretion: 2-10% extred unchanged, 1/2 life 5 min

ADVERSE EFFECTS
1. CVS effects : bradycardia esp in children/infants/neonates
2. RESP: apnoea
3. CNS: muscle paralysis, phase 2 block (preated doses, large doses, infusions), raised ICP, OCP
4. GIT: raised intragastric pressure, with decrease LOS tone
5. Metabolic: raised K 0.5 mmol, risk of hyperkalaemia and torsades esp in renal failure
6. MH: 1:5000
7. Sux apnoea: 4%
8. Allergy: leading cause, 1:5000
9. Limited use in neuromusclar disorders

Sux's use in difficult intubation is being replaced by Rocuonium- Sugammadex (main resistence is unfamiliarity and cost)

SUGAMMADEX (Bridion 100mg/ml)
• Modified cyclodextrin
• Chelate or encapsulate rocuronium (and to a lesser extent vecuronium)
• Reducing the amount of NMBD available to bind nicotinic receptors at the neuromuscular junction
• True reversal of NMB i
• reversal to T4:T1 of 0.9 is possible in approximately 90 seconds if sugammadex 16mg/kg is given 3 minutes after rocuronium 1.2mg/kg. This is faster than spontaneous recovery after suxamethonium

DOSE
• Dose dependent speed of onset
• Dose depends on level of neuromuscular blockade:
• PTC 1 – 2 = 4mg/kg
• TOF2 = 2mg/kg
• Immediate reversal dose of 16mg/kg recommended
• Base on actual body weight in obese

ADVANTAGES
• Sugammadex can cater for routine reversal of NMB
• In the rare care of a ‘can’t intubate can’t ventilate’ profound NMB can be reversed by high dose sugammadex 16mg/kg. This reversal when sugammadex is given 3 mins after a dose of rocuronium 1.2mg/kg was swifter than the degree of spontaneous recovery after sux
• High dose rocuronium offers as swift an onset of NMB than sux and a cleaner side effect profile
HK 2011

Outline the adverse effects of tourniquet use in orthopaedic surgery and the strategies to minimize them.
SPECIFICATIONS
• Cuff >40% circumfrence of limb
• Minimal inflation pressure to prevent arterial flow
o Upper limb = systolic +50mmHg
o Lower limb = 2 x systolic
• Limit 2 hours in healthy
• Deflate every 2 hrs for 10 min to allow distal reperfusion of muscles

CONSEQUENCES – LOCAL
Nerve: UL>LL (Radial UL Sciatic LL)
• Mechanical pressure > Ischemic inj
Muscle
• Greatest just below tourniquet
• Ischamic inj>mechan pressure
• Post Tourniquet syndrome: combo of muscle ischemia, odema and microvascular congestion
Vascular
• Arterial inj is uncommon
• Mechanical pres can traumatize atheromatous vessels causing plaque fracture/rupture in pts with PVD
Skin
• Pressure necrosis and friction burns
• Chemical burns from skin prep soaked beneath tourniquet
Tourniquet pain
• Difficult to treat, unresponsive to systemic analgesia or deep anaesthesia
• Transmitted by unmyelinated C fibers
• Can occur with spinal or epidural
SYSTEMIC
Cardiovascular
• Exsanguination: ↑VR and CO, lower limb can return 400ml transfusion
• Response to pain: tachy and HT
• Release of ischemic mediators
o Negative inotropy
o Vasodilatation
o Arrhythmia
• ↓VR Shift blood volume back to limb
• Not well tolerated in pts with poor cardiac function
Respiratory
• Tourniquet pain: tachypnoea
• ↑ETCO2 release of tourniquet requiring ↑ MV
Cerebral
• rapid ↑PaCO2 assed with 50%↑MCA blood flow, peaks 2-4minutes
Hematological
• Tourniquet and surgical stress
o Systemic hypercoagulabilty
o Venous stasis resulting in thrombus
o Deflation may result in PTE
• Risk of precipitating sickle cell crisis
Temperature
• Reduced heat distribution from central to peripheral compartment
• ↓heat loss from tourniquet limb
• Tourniquet deflation results redistributive loss as limb is reperfused
Metabolic
• Production of ischaemic mediators: CO2, lactate, K+
o ↑K+ by 0.2mmol/L peak 3 min
o ↑lactate 2mmol/L, elevated 30 min
o ↓pH (CO2/lactate) peak at 4 min
• ↑O2 consumption
o Replace O2 debt
o Sympathetic resp to CO2 load
HK 2011

Discuss the risk factors and prevention strategies for chronic post surgical pain.
Pain persisting for more than 3 months post operatively is chronic pain. It is one of the most common complications of surgery. Pain is the a psychological sensory experience.
1. Genetic susceptibility
• Possible heritable response although no gene responsible have been identified
• Polymorphisms of catecholamine-O-methyltranferase
• Red hair and fair skin
2. Preceding pain
• Past history of chronic pain, eg phantom limb pain have more intense and enduring pain to subsequent operations
• History of backache, IBS or headache
3. Psychological factors
• Expectation of pain, fear, past memories, social environment, work, and levels of physical activity
• Catastrophising
• Perceived level of social support
• Preoperative anxiety
4. Age and sex
• Older patients have reduced risk of developing chronic pain
• Women have higher postoperative pain than men
5. Surgical operation
• Not correlated with size of incision, but instead type and duration of surgery (>3 hours) and experience of surgeon
• Repeat surgery
• Amputation : 30-50% incidence of chronic pain
• Thoracotomy : 20-30%
• Mastectomy : 30-40%
• Inguinal hernia : 10%
• Coronary bypass : 30-50%
• Caesarian section : 10%
6. Post operative factors
• Radiotherapy
• Neurotoxic chemotherapy




PART B
1. Surgical technique : opt for less invasive techniques, more careful incision and dissection
2. Optimizing pre operative pain management
3. Preoperative psycho-social support and optimization, counseling as required
4. Aggressive and multimodal pain management
5. Pre-emptive analgesia
• Regional analgesia commenced before surgical incision
i. Little benefit for prevent CP, but beneficial in acute pain outcome measures in thoracotomy
ii. Benefits have been found with spinals for caesarian section and epidural analgesia in abdominal surgery
iii. Reduction in phantom limb pain NNT6
iv. Paravertabal block for mastectomy patients reduced incidence of pain
• Local anaesthetic would infiltration : positive-premptive effects especially in the setting of amputation
• Systemic NMDA antagonsists
• Systemic opiods : not shown to be benefitial
• Systemic NSAIDs : positive pre-emptive effect

6. Protective and preventive analgesia
• Protective : technique that reduces measures of sensitization such as hyperalgesia
• Preventative : persistence of analgesic treatment effeicacy beyond expected duration
• NMDA receptor antagonists : ketamine and dextromethorphan produced significant preventive analgesia
• Ketamine bolus followed by infusion for 72 hours reduced the incidence of severe phantom limb pain
• Low dose ketamine 3mg/hour shown to reduce pain in thoracotomy
HK 2011

8. Outline your management of a patient presenting intraoperatively with ST segment depression on ECG that is consistent with acute myocardial ischaemia.
• 50% due to plaque rupture
• Common at day 3-4
• Silent infarct

Risk factors
• Plaque rupture
o Procoagulation (continue aspirin)
o Shear stress (alpha and beta blockade)
o Infammation (statins and b blockers)
• Supply and demand
o Demand: HR, afterload, preload and contractility
o Supply: oxygen flux Hb, O2 saturation, HR, DBP, flow during diastole
• Vasospasm: hypercarbia is primary cause

Diagnosis
TOE
• New regional wall motion abnormality
• Occurs before ECG changes
• Specific area involved indicates vessel
• Limitations: specialist training, difficult study, risks with TOE

ECG
• Sensitivity of individual leads
o II 33%
o V5 75%
o II & V5 80%
o II & V5 & V6 93%
o II & V2-V5 100%
• Criteria
o ST dep > 2mm 80msec from J point
o Upsloping ST dep >1mm 60sec from J point
o ST elevation
o T wave inversion
• Filter mode: ST filter mode or diagnostic mode (0.05-100Hz) better than monitoring mode (0.5-40Hz)

PAC
• Ischemia lead to LV wall stiffness causing increase PAWP (poor specificity)
• DDx of increased PAWAP
o Ischemia
o MR
o Increased afterload
o Systemic hypertension
o Decreased pulmonary compliance

MANAGEMENT
Immediate
• Ensure ABC
• Optimize O2 supply and delivery
• Increase O2 supply
o 100% FiO2
o BP to high normal for pt
o Reduce HR, treat tachycardia
o Hb, Hct>30%
• Decrease O2 demand
o Analgesia
o GTN to reduce preload 10mic/kg/hr
• Inform surgeon, defer or expedite surgery
• NG aspirin and statin
• Consider invasive monitoring
• Reassess for
o Haemodynamic changes
o Evolving ECG changes
o
Post op
• Serial ECG and troponins
• ECG monitoring
• HDU/CCU admission with cardiology referral
• TIMI risk stratification
• Further investigations
o Stress testing
o Angiography
HK 2011

9. A 40 years old woman with BMI 41 undergoes bariatric surgery for reduction of body weight. What are the perioperative issues?
Airway
• Incidence of difficult intubation ~ 13%
• Neck circumference at level of cricoid
o Most sensitive
o >60cm = 35% risk
• Ramp position (external auditory meatus in line with sternum)
• Adequate pre-oxygenation (↓ FRC)
• Anticipate difficult mask ventilation (beard, OSA, no teeth, large breasts, central fat distribution)
• ↑GORD (↑ intra-abdominal pressure and hiatus hernia)
• ↑ residual gastric volum with ↓ pH

Respiratory
o PEEP 10cmH2O: ↓atelectasis & hypoxia
o More significant reduction in FRC in obese anaesthetised patents
o ↓50% cf 20% for normal
o Tidal volume calculations based on IBW
o Extubate sitting
Cardiovascular
o AHA/ACC Risk stratification

Positioning
• Supine position in obese patients:
o IVC compression & ↓ VR
o ↓pulmonary compliance
o ↓FRC
• Lithotomy/Prone/Lateral
• Obesity & Diabetes are risk factors for neuropraxia

Equipment
o Need for invasive BP
o small cuff overestimates blood pressure
o IV access
o Suitable for day surgery

Regional Anaesthesia
• Epidural: ↓LA requirements,] ↑cephalad spread (engorged extradural ), higher block,
• Advantages of epidural analgesia:
o reduced incidence DVT
o improved analgesia and opioid sparing
o earlier recovery intestinal motility
Other
• High risk of PE – Double risk of thromboembolism compared to non-obese.

Drugs
• ↑ Vd, esp lipid soluble drugs
o Thiopentone, BDZ
o NMBD same Vd as non-obese
• Clearance time same however elimination half life prolonged
• Calculate ideal body weight
IBW = Ht (cm) – 100 (males)
• Calculate Lean body weight
LBW=(1.1x weight)–(128x(weight/height)2)
Females use 1.07 and 148
• For most drugs, calculate dose based on IBW
• Remifentanyl, although highly lipophilic, has little change in Vd, hence calculate dose on IBW
• Propofol –induction use IBW, maintenance use TBW
• NMBD (hydrophilic) should be calculated on IBW, possible exception with atracurium – dose on TBW
• Paracetamol dose based on IBW
• Suxamethonium: calculate dose on TBW, to a maximum of 120 – 140mg
• Thiopentone (dose based on TBW) due to higher CO
• Midazolam use TBW for initial dose, IBW for subsequent dosing
• Fentanyl use TBW
• Desflurane
o inhalational Anaesthetic of choice
o Lowest solubility in fat tissue.
o Fastest recovery profile
HK Paper 18th March 2011

1. A 70 kg adult presents on your list for a routine laparotomy. Following induction of anaesthesia and the administration of a full dose of a non-depolarising muscle
relaxant, you discover that you cannot intubate him. A laryngeal mask airway (LMA) is not helpful. Give a detailed account of how you would proceed to achieve oxygenation in this patient.
This is an anaesthetic emergency
Priorities are to maintain oxygenation
Call for Senior Anaesthetic Help

DAS Guideline : Unanticipated difficult airway: PLAN C
PLAN A: Direct laryngoscopy
• Optimise: neck flex, head ext, RAMP, BURP, bougie, alternative laryngoscope
• Max 3 attempts, maintain oxygenation and anaesthesia
PLAN B: ILMA or cLMA
• No more than 2 attempts
• Fibreoptic placement, confirmation
• 3 ILMA 7.0 ETT
• 4/5 ILMA 7.5 ETT
• 3/4 cLMA 6.0 ETT
PLAN C maintain vent and oxygenation
• Mask ventilation or cLMA
• Reverse and awaken
• Postpone surgery
PLAN D CICV
• Rescue technique for CICV
• Needle cricothyroidotomy
• Surgical cricothyroidotomy

PLAN C
Mask ventilation:
-100% FIO2
-Optimise airway position (RAMP, jaw thrust, chin lift)
-Two hand technique with assistence if required,
-Use of adjuncts (nasopharyngeal, oropharyngeal)
-If able to oxygenate, options available include
1. Waking the patient up, recuronium and sugammadex
Dose 16mg/kg for immediate reversal within 3 minutes
-If different NDMR used then continue BMV ventilation, use reversing agent (neostig/glyco)
-Discuss what happened with patient, quality assurance
2. I would consider different method of intubation with advanced techniques (although I would likely wake pt up)
-consider macintosh blade, miller blade
-asleep fiberoptic intubation
-retrograde intubation

PLAN D if BMV is unsuccessful
HEARD IN WEST AUST CICV
• Needle crico failure or success
• Palpable neck anatomy
o Yes: Scalp bougie
o No: Scalp finger

NEEDLE CRICOTHYROIDOTOMY
1. NDH: Cricothyroid membrane in the midline and stabilize
2. DH: 5ml syringe with 14G cannula, aim 45° caudally, aspirate-advance
3. End point: aspiration of air
4. Stabilize against patient and advance cannula into trachea
5. Repeat air aspiration to confirm pos
6. Attach to Jet ventilator
a. Insp 1 sec, Exp 4 sec
b. RR 8-10
c. Observe rise & fall of chest
7. Arrange for surgical or definitive airway as soon as possible

SCALPEL FINGER NEEDLE
• NDH stabilize neck
• DH vertical midline incision at least 6cm through SC tissue
• Insert fingers of both hands to separate strap muscles and identify structures
• Insert 14G with DH, aspirate and advance
• Secure, insert and confirm as per needle cricothyroidotomy
• Attach jet ventilation
• Convert using Melker 5.0 tube

COMPLICATIONS OF TRACHEOSTOMY
EARLY
Minor
• Haemorrhage
• Difficult tube placement
• False passage
• Subcutenous emphysema
Intermediate
• Desaturation
• Hypotension
• Cannular misplacement
• Convert to open
• Aspiration
• Damage to local structures
o Tracheal rings
o Oesphagus
o Thyroid
o Carotid
o RLN/SLN/phrenic
Serious
• Death
• Arrest
• Pneumonthorax
• Pneumomediastinum

INTERMEDIATE
• Stoma infection
• Disconnection
• Loss of airway

LATE
• Tracheal stenosis
• Tracheomalacia
• Tracheosophageal fistula
• Pneumonia
• Aspiration
• Innominate artery erosion